You are on page 1of 417

ORTAÖĞRETİM

FEN LİSESİ

DERS KİTABI
9

ORTAÖĞRETİM FEN LİSESİ MATEMATİK 9 DERS KİTABI


ORTAÖĞRETİM

FEN LİSESİ

Matematik
9
DERS KİTABI

YAZARLAR

Ali UÇAK
Emin EMİR
Firdevs UÇKUN
Güler KUTLU
Seçil KAHRAMAN

Matematik 9 | 1
HAZIRLAYANLAR

Editör
Prof. Dr. Erdal ULUALAN

Dil Uzmanı
Gülçin GÜRPINAR

Program Geliştirme Uzmanı


Yrd. Doç. Dr. Nihal TUNCA

Ölçme ve Değerlendirme Uzmanı


Yrd. Doç. Dr. Recep Serkan ARIK

Rehberlik ve Gelişim Uzmanı


Mevlüt DURAN

Görsel ve Grafik Tasarım Uzmanı


Hacı Mehmet ÖZTÜRKOĞLU

2 | Matematik 9
İSTİKLÂL MARŞI

Korkma, sönmez bu şafaklarda yüzen al sancak; Bastığın yerleri toprak diyerek geçme, tanı:
Sönmeden yurdumun üstünde tüten en son ocak. Düşün altındaki binlerce kefensiz yatanı.
O benim milletimin yıldızıdır, parlayacak; Sen şehit oğlusun, incitme, yazıktır, atanı:
O benimdir, o benim milletimindir ancak. Verme, dünyaları alsan da bu cennet vatanı.

Çatma, kurban olayım, çehreni ey nazlı hilâl! Kim bu cennet vatanın uğruna olmaz ki feda?
Kahraman ırkıma bir gül! Ne bu şiddet, bu celâl? Şüheda fışkıracak toprağı sıksan, şüheda!
Sana olmaz dökülen kanlarımız sonra helâl. Cânı, cânânı, bütün varımı alsın da Huda,
Hakkıdır Hakk’a tapan milletimin istiklâl. Etmesin tek vatanımdan beni dünyada cüda.

Ben ezelden beridir hür yaşadım, hür yaşarım. Ruhumun senden İlâhî, şudur ancak emeli:
Hangi çılgın bana zincir vuracakmış? Şaşarım! Değmesin mabedimin göğsüne nâmahrem eli.
Kükremiş sel gibiyim, bendimi çiğner, aşarım. Bu ezanlar -ki şehadetleri dinin temeli-
Yırtarım dağları, enginlere sığmam, taşarım. Ebedî yurdumun üstünde benim inlemeli.

Garbın âfâkını sarmışsa çelik zırhlı duvar, O zaman vecd ile bin secde eder -varsa- taşım,
Benim iman dolu göğsüm gibi serhaddim var. Her cerîhamdan İlâhî, boşanıp kanlı yaşım,
Ulusun, korkma! Nasıl böyle bir imanı boğar, Fışkırır ruh-ı mücerret gibi yerden na’şım;
Medeniyyet dediğin tek dişi kalmış canavar? O zaman yükselerek arşa değer belki başım.

Arkadaş, yurduma alçakları uğratma sakın; Dalgalan sen de şafaklar gibi ey şanlı hilâl!
Siper et gövdeni, dursun bu hayâsızca akın. Olsun artık dökülen kanlarımın hepsi helâl.
Doğacaktır sana va’dettiği günler Hakk’ın; Ebediyyen sana yok, ırkıma yok izmihlâl;
Kim bilir, belki yarın, belki yarından da yakın. Hakkıdır hür yaşamış bayrağımın hürriyet;
Hakkıdır Hakk’a tapan milletimin istiklâl!

Mehmet Âkif Ersoy

Matematik 9 | 3
GENÇLİĞE HİTABE

Ey Türk gençliği! Birinci vazifen, Türk istiklâlini, Türk Cumhuriyetini,


ilelebet muhafaza ve müdafaa etmektir.

Mevcudiyetinin ve istikbalinin yegâne temeli budur. Bu temel, senin


en kıymetli hazinendir. İstikbalde dahi, seni bu hazineden mahrum
etmek isteyecek dâhilî ve hâricî bedhahların olacaktır. Bir gün, istiklâl
ve cumhuriyeti müdafaa mecburiyetine düşersen, vazifeye atılmak için,
içinde bulunacağın vaziyetin imkân ve şeraitini düşünmeyeceksin! Bu
imkân ve şerait, çok namüsait bir mahiyette tezahür edebilir. İstiklâl
ve cumhuriyetine kastedecek düşmanlar, bütün dünyada emsali
görülmemiş bir galibiyetin mümessili olabilirler. Cebren ve hile ile aziz
vatanın bütün kaleleri zapt edilmiş, bütün tersanelerine girilmiş, bütün
orduları dağıtılmış ve memleketin her köşesi bilfiil işgal edilmiş olabilir.
Bütün bu şeraitten daha elîm ve daha vahim olmak üzere, memleketin
dâhilinde iktidara sahip olanlar gaflet ve dalâlet ve hattâ hıyanet içinde
bulunabilirler. Hattâ bu iktidar sahipleri şahsî menfaatlerini, müstevlîlerin
siyasî emelleriyle tevhit edebilirler. Millet, fakr u zaruret içinde harap ve
bîtap düşmüş olabilir.

Ey Türk istikbalinin evlâdı! İşte, bu ahval ve şerait içinde dahi vazifen,


Türk istiklâl ve cumhuriyetini kurtarmaktır. Muhtaç olduğun kudret,
damarlarındaki asil kanda mevcuttur.

Mustafa Kemal Atatürk

4 | Matematik 9
MUSTAFA KEMAL ATATÜRK

Matematik 9 | 5
6 | Matematik 9
İÇİNDEKİLER

ORGANİZASYON ŞEMASI...................................................................................................... 10

9.1. MANTIK
.9.1.1. ÖNERMELER VE BİLEŞİK ÖNERMELER....................................................................... 13
1. Önermelerde Temel Kavramlar.................................................................................. 14
2. Bileşik Önerme........................................................................................................... 17
3. Koşullu Önerme ve İki Yönlü Koşullu Önerme........................................................... 23
4. Sözel ya da Sembolik Mantık Dilinde Verilen Bileşik Önermelerin
Birbirine Dönüştürülmesi.......................................................................................... 28
5. Totoloji ve Çelişki....................................................................................................... 28
ALIŞTIRMALAR..................................................................................................................... 30
.9.1.2. AÇIK ÖNERMELER VE İSPAT YÖNTEMLERİ................................................................. 32
1. Her ( 6 ) ve Bazı ( 7 ) Niceleyicileri............................................................................ 32
2. Açık Önerme............................................................................................................... 32
3. Tanım, Aksiyom, Teorem ve İspat Kavramları........................................................... 35
4. Mantık Kuralları ile Yapılan İspatlamalar.................................................................. 36
5. Tümevarım Yöntemi................................................................................................... 39
ALIŞTIRMALAR..................................................................................................................... 42
ÖLÇME VE DEĞERLENDİRME................................................................................................ 44

9.2. KÜMELER
.9.2.1. KÜMELERDE TEMEL KAVRAMLAR.............................................................................. 49
1. Temel Kavramlar........................................................................................................ 50
2. Alt Küme..................................................................................................................... 54
3. Eşit Küme................................................................................................................... 60
ALIŞTIRMALAR..................................................................................................................... 61
9.2.2. KÜMELERDE İŞLEMLER VE BAĞINTI.......................................................................... 62
1. Kümelerde İşlemler.................................................................................................... 63
ALIŞTIRMALAR..................................................................................................................... 77
2. Kümelerin Kartezyen Çarpımı.................................................................................... 78
3. Bağıntı........................................................................................................................ 84
ALIŞTIRMALAR..................................................................................................................... 87
ÖLÇME VE DEĞERLENDİRME -1 ........................................................................................ 88
ÖLÇME VE DEĞERLENDİRME -2 ......................................................................................... 90

Matematik 9 | 7
9.3. DENKLEM VE EŞİTSİZLİKLER
.9.3.1. SAYI KÜMELERİ.......................................................................................................... 95
1. Sayı Kümeleri Arasındaki İlişki................................................................................. 96
ALIŞTIRMALAR..................................................................................................................... 105
.9.3.2. BİRİNCİ DERECEDEN DENKLEMLER VE EŞİTSİZLİKLER............................................. 106
1. Gerçek Sayılarda Aralık Kavramı.............................................................................. 107
2. Birinci Dereceden Denklemler ve Eşitsizliklerin Çözümü........................................ 110
ALIŞTIRMALAR..................................................................................................................... 121
3. Birinci Dereceden Mutlak Değerli Denklem ve Eşitsizlikler..................................... 122
4. Birinci Dereceden İki Bilinmeyenli Denklem ve Eşitsizlik Sistemleri...................... 131
ALIŞTIRMALAR..................................................................................................................... 136
9.3.3. ÜSLÜ İFADELER VE DENKLEMLER.............................................................................. 138
1. Üslü İfade İçeren Denklemler.................................................................................... 139
ALIŞTIRMALAR..................................................................................................................... 145
2. Köklü İfadeler İçeren Denklemler............................................................................. 146
ALIŞTIRMALAR..................................................................................................................... 154
.9.3.4. DENKLEM VE EŞİTSİZLİKLERLE İLGİLİ UYGULAMALAR............................................. 155
1. Oran ve Orantı ........................................................................................................... 156
ALIŞTIRMALAR..................................................................................................................... 164
2. Problemler.................................................................................................................. 165
ALIŞTIRMALAR..................................................................................................................... 172
ALIŞTIRMALAR..................................................................................................................... 191
ÖLÇME VE DEĞERLENDİRME -1........................................................................................... 195
ÖLÇME VE DEĞERLENDİRME -2........................................................................................... 197
ÖLÇME VE DEĞERLENDİRME -3........................................................................................... 199

9.4. BÖLÜNEBİLME
.9.4. BÖLÜNEBİLME KURALLARI........................................................................................... 202
1. Tam Sayılarda Bölünebilme....................................................................................... 203
ALIŞTIRMALAR..................................................................................................................... 207
ALIŞTIRMALAR..................................................................................................................... 215
2. Tam Sayılarda EBOB ve EKOK.................................................................................... 219
ALIŞTIRMALAR..................................................................................................................... 229
ÖLÇME VE DEĞERLENDİRME................................................................................................ 230

8 | Matematik 9
9.5. ÜÇGENLER
9. .5.1 ÜÇGENLERDE TEMEL KAVRAMLAR............................................................................. 237
1. Üçgende Açı Özellikleri.............................................................................................. 237
2. Üçgende Açı Kenar Bağıntıları................................................................................... 255
3. Üçgen Eşitsizliği......................................................................................................... 258
ALIŞTIRMALAR..................................................................................................................... 263
9. .5.2. ÜÇGENİN YARDIMCI ELEMANLARI............................................................................. 264
1. Üçgende Açıortay....................................................................................................... 265
2. Üçgende Kenarortay.................................................................................................. 276
3. Üçgenin Kenar Orta Dikmeleri................................................................................... 286
4. Üçgende Yükseklik.................................................................................................... 291
ALIŞTIRMALAR..................................................................................................................... 301
9. .5.3. ÜÇGENLERDE EŞLİK VE BENZERLİK........................................................................... 303
1. Üçgenlerin Eşliği........................................................................................................ 303
2. Üçgenlerin Benzerliği................................................................................................. 307
3. Thales Teoremi........................................................................................................... 319
4. Benzerlik Uygulamaları............................................................................................. 320
ALIŞTIRMALAR..................................................................................................................... 324
9. .5.4 DİK ÜÇGENDE TRİGONOMETRİ.................................................................................... 326
1. Dik Üçgende Pisagor Teoremi.................................................................................... 326
2. Öklid Teoremi............................................................................................................. 329
ALIŞTIRMALAR..................................................................................................................... 334
3. Dik Üçgende Dar Açıların Trigonometrik Oranları..................................................... 335
ALIŞTIRMALAR..................................................................................................................... 342
9.5.5 ÜÇGENİN ALANI........................................................................................................... 344
1. Üçgenlerde Alan Uygulamaları.................................................................................. 345
ALIŞTIRMALAR..................................................................................................................... 361
ÖLÇME VE DEĞERLENDİRME -1........................................................................................... 363
ÖLÇME VE DEĞERLENDİRME -2........................................................................................... 366

9.6. VERİ
.9.6.1. MERKEZİ EĞİLİM VE YAYILIM ÖLÇÜLERİ.................................................................... 372
1. Merkezi Eğilim Ölçüleri.............................................................................................. 372
2. Merkezi Yayılım Ölçüleri............................................................................................ 376
ALIŞTIRMALAR..................................................................................................................... 382
.9.6.2. VERİLERİN GRAFİKLE GÖSTERİLMESİ........................................................................ 384
1. Histogram Oluşturma................................................................................................. 386
2. Grafik Türleri.............................................................................................................. 387
ALIŞTIRMALAR..................................................................................................................... 397
ÖLÇME VE DEĞERLENDİRME ............................................................................................... 400
SEMBOLLER VE OKUNUŞLARI.............................................................................................. 405
ALIŞTIRMA ÇÖZÜMLERİ....................................................................................................... 406
CEVAP ANAHTARI................................................................................................................. 410
SÖZLÜK................................................................................................................................. 411
KAYNAKÇA............................................................................................................................ 414
GÖRSEL KAYNAKÇA............................................................................................................. 415

Matematik 9 | 9
ORGANİZASYON ŞEMASI Karekod uygulamasıyla
alt öğrenme alanlarına
interaktif olarak
ulaşabilirsiniz. Alt Öğrenme Alanı.

Alt öğrenme alanına


yönelik dikkat çeken
çalışmaların ve
uygulamaların bulunduğu
bölümdür.

Öğrenme Alanı.

Alt Öğrenme Alanı.

Alt öğrenme alanında neler öğrenileceğini, alt Alt öğrenme alanı ile
öğrenme alanını öğrenmenin önemini, alt öğrenme ilgili tanım, bilgi ve
alanının günlük hayatla ilişkisini tanıtan bölümdür. özelliklerin bulunduğu
bölümdür.

Alt öğrenme alanındaki işlenişe yönelik Alt öğrenme


kısa tarihi bilgilerin bulunduğu bölümdür. alanındaki dikkat
edilmesi gereken
uyarıların bulunduğu
bölümdür.
TARİHÇE

Alt öğrenme alanıyla


ilgili işlenişleri
açıklayan örneklerin
ve çözümlerinin
bulunduğu bölümdür.

Alt öğrenme alanının tarihsel gelişimini içeren bölümdür.

Alt öğrenme alanının


pekiştirilmesi amacıyla
verilen uygulama
sorularının bulunduğu
bölümdür.

Alt öğrenme alanıyla


ilgili düşünmeye,
tartışmaya ve
araştırmaya yönelik
soruların bulunduğu
bölümdür.

Bir veya birkaç işlenişi


içeren pekiştirme sorularının
bulunduğu bölümdür.

Her alt öğrenme alanının sonunda


verilen değerlendirme sorularının
bulunduğu bölümdür.

10 | Matematik 9
SAYILAR VE CEBİR
9.1. MANTIK
Neler Öğreneceksiniz? Mantık
Öğrenmek Neden Önemlidir?

• Önerme kavramı ile bir önermenin • İnsanlar günlük yaşantılarında duygu


doğruluk değerini, değilini (olumsuzunu) ve düşüncelerini ifade etmek için
ve denk önermeleri, farklı cümleler kurar. Kişi, kendine ve
• “ve, veya, ya da, ise, ancak ve ancak” sosyal çevresine zarar verecek cümleler
bağlaçları ile oluşturulan bileşik kurmaktan sakınır. Bu durum mantıklı
önermeleri ve bu önermelerin doğruluk konuşma, düşünme ve hareket etme
değerlerini, şeklinde de ifade edilir.
• Elektrik devrelerindeki paralel-seri • Matematik, doğru ve sistemli düşünme
bağlama ile ve/veya/ya da bağlaçları sanatı olarak da ifade edilebilir. Mantık
arasındaki ilişkiyi, ise doğru düşünebilme bilimidir. Bu
• De Morgan kurallarını, açıdan bakıldığında mantık, matematiğin
ayrılmaz bir parçasıdır. Bilinenleri
• Tek yönlü ve iki yönlü koşullu önermeleri,
kullanarak yeni gerçeklere ulaşmak, yeni
bu önermeler arasındaki ilişkileri,
buluşlar yapmak sistemli akıl yürütme ile
• Bir koşullu önermenin tersini, karşıtını ve mümkündür.
karşıt tersini bulmayı,
• Mantık kuralları, matematikte ve günlük
• Sözel veya sembolik mantık dilinde yaşantıda problemlerin anlaşılmasında
verilen önermeleri birbirine önemli bir yer tutar. Problemler, mantık
dönüştürmeyi, ilke ve sembolleri kullanılarak formüle
• Totoloji ve çelişki kavramlarını, edilir. Çözümlemeler sonucu elde edilen
• Açık önermeleri ve açık önermelerin sonuçlar yorumlanarak kesin hükümlere
doğruluk değerlerini bulabilmeyi, ulaşılır.
• Tanımlı tanımsız terim, aksiyom, teorem • Sembolik mantık, matematiksel zekânın
ve ispatlama kavramlarını, alt yapısını oluşturur. Alt yapısı güçlü
• Bir hükmün doğruluğunu ya da olan bireyler, matematiksel kavram ve
yanlışlığını, bir hükmü farklı ispat problemleri çözmede üstünlük elde eder.
yöntemlerini kullanarak ispatlayabilmeyi
öğreneceksiniz.

Matematik 9 | 11
MANTIK

Gottfried Wilhelm Leibniz

G
ottfried Wilhelm Leibniz [Gottfried Wilhelm Leybnits (1646-
1716)], matematik ve mantık alanında önemli çalışmalar yapmış
bir bilim insanıdır. Leibniz, matematiksel simge mucitlerinin
en önemlilerinden biridir. Matematik dışında hukuk, siyaset, tarih,
metafizik, mantık, edebiyat ve felsefe alanlarında da yaptığı çalışmalarla
insanlığa büyük hizmetlerde bulunmuştur.
George Boole [Corç Buul (1815-1864)], matematik ve mantık
alanında çalışmaları olan ünlü İngiliz bilim insanıdır. Matematiksel
mantık kuramına ilişkin Boolean cebrini geliştirmiştir. Boole cebri
olarak adlandırılan mantık cebri, sayısal bilgisayar devreleri tasarımının
matematiksel temelini oluşturur. Bilgi teknolojilerinin ulaştığı bugünkü
aşamada onun çalışmalarının katkısı övgüye değerdir. 1848 yılında
“Mantığın Matematiksel Analizi” adlı eserini yayımlayarak matematik
biliminde yeni bir sayfa açmıştır.
Bertrand Russell [Bertrat Rassıl (1872-1970)]; “Matematiğin İlkeleri”
adlı kitabında önermelerin “ve, veya, ise, ancak ve ancak" gibi mantıksal
bağlaçlarla ilişkisini kurup mantık bilimini tanıtmıştır. Matematiği p & q
şeklindeki önermeler bütünü olarak ifade ederek farklı bir yaklaşım
ortaya koymuştur.

12 | Matematik 9
MANTIK

9.1.1. ÖNERMELER VE BİLEŞİK ÖNERMELER

Etkinlik

Yasin ve Hilal, amaçlarına ulaşmak için sayısal derslerdeki eksikliklerini gidermek istemektedir. Bu
amaçla okul rehber öğretmeni ile görüşürler. Görüşme sonucunda aşağıdaki önerileri not ederler.
• Sayısal derslerde başarılı olmak için dersi derste anlamalısınız.
• Derste anlamadığınız her konuyu zamanında öğretmeninize sorarak öğrenmelisiniz.
• Günlük tekrarlarınızı yazarak yapmalısınız.
• Uzun vadeli değil amaca dönük kısa vadeli planlar yapmalısınız.
Aşağıdaki cümlelerde boş bırakılan yerlere “ve, veya, ise, ancak ve ancak” kelime ya da kelime
gruplarından uygun olanı yukarıdaki bilgilere göre yazınız.
-Yasin ve Hilal'in amaca ulaşmaları ............... yapılan plana uymaları ile mümkündür.
-Yasin ve Hilal amaca ulaşmak istiyor ............... kısa vadeli amaca dönük planlar yapmalıdır.
-Yasin ve Hilal, öğrenilen kavram ............... konuları yazarak tekrar etmelidir.
-Yasin ve Hilal derste anlamadığı her konuyu zamanında öğretmenine sorup öğrenmeli ...............
çok tekrar yaparak anlamalıdır.
Siz de yukarıdaki kelime ya da kelime gruplarını kullanarak olumlu olumsuz cümleler kurunuz.
Kurduğunuz cümlelerin hükmünün doğru yanlış olma durumlarını tartışınız.
“Mantıklı düşün!” veya “Mantıklı ol!” sözlerinden ne anladığınızı tartışınız.

Matematik 9 | 13
MANTIK

1. Önermelerde Temel Kavramlar


İnsanı diğer canlılardan ayıran en temel özellik, akıl sahibi bir varlık olarak düşünen bir canlı olmasıdır.
Düşüncelerin anlamlı ve tutarlı olduğu zamandaki akıl yürütme yoluna mantık denir.
İnsanlar günlük yaşantılarında, sosyal ve kültürel ilişkilerinde farklı cümleler kurarak kendilerini ifade eder.
Bu ifadeler duygu ve düşüncelerin açıklanmasının doğal bir sonucudur. Örneğin “Kızılırmak, uzun bir ırmaktır.”
cümlesi bazılarına göre doğru olabileceği gibi bazılarına göre de yanlış olabilir. Çünkü ırmakların hangi ölçüden
sonra “uzun” olarak niteleneceğinin kriteri belli değildir. Bu cümle “Ülkemizin Karadeniz'e dökülen en uzun
ırmağı Kızılırmaktır.” şeklinde söylenseydi doğru bir ifade olurdu. “Bir üçgenin iç açılarının ölçüleri toplamı
180c dir.” cümlesi ise matematikte doğruluğu gösterilmiş olduğundan doğru bir hüküm bildirir. “Nasılsın?” ve
“Koşma!” gibi cümleler, soru ve ünlem cümlesi olduğu için bir hüküm bildirmez.


Doğru ya da yanlış, kesin hüküm bildiren ifadelere önerme adı verilir.
Önermeler genellikle “p, q, r, s, t” gibi küçük harflerle gösterilir.

ÖRNEK 1:
Aşağıdaki ifadelerin önerme olup olmadığını inceleyiniz.
a) p: “Çift ve asal olan doğal sayı sadece 2 dir.”
b) q: “Bugün tiyatroya gidelim.”
c) r: “Sakarya, Karadeniz Bölgesi'ne ait bir ilimizdir.”
ç) s: “Kiraz, lezzetli bir meyvedir.”
d) t: “Onur, başarılı bir öğrencidir.”
e) v: “ 1 - 5 =- 6 tir.”

ÇÖZÜM:
a) “Çift ve asal olan doğal sayı sadece 2 dir.” ifadesi kesin hüküm bildirdiğinden bir önermedir.
b) “Bugün tiyatroya gidelim.” cümlesi bir istek cümlesi olduğundan önerme değildir.
c) “Sakarya, Karadeniz Bölgesi'ne ait bir ilimizdir.” ifadesi kesin hüküm bildirdiğinden önermedir.
ç) Kirazın lezzetli bir meyve olması, kişiden kişiye farklılık gösterebilir. Bu cümle, kesin bir yargı
bildirmemektedir. Bu sebeple bu ifade bir önerme değildir.
d) Başarı kriteri belli olmadığından bu ifade, kesin bir hüküm bildirmez. Bu sebeple önerme değildir.
e)“ 1 - 5 =- 6 tir.” ifadesi kesin hüküm bildirdiğinden önermedir.

Bir Önermenin Doğruluk Değeri


Doğru ya da yanlış kesin hüküm bildiren ifadeler önerme olarak tanımlandığından bir önermenin iki
farklı doğruluk değeri olur. Eğer p önermesi doğru ise D veya 1, yanlış ise Y veya 0 ile gösterilir.
p önermesi doğru ise p / 1 , yanlış ise p / 0 şeklinde yazılır.

Bir p önermesinin doğruluk değeri,


tabloda p q p ve q gibi iki önermenin doğruluk
değerleri, tablo biçiminde yandaki gibi
p p 1 1 gösterilebilir.
D veya 1 1 0
Y 0 0 1
biçiminde gösterilir.
0 0
Bir önerme için iki farklı durum vardır. Bu Birbirinden bağımsız iki önerme için dört farklı
durum 2 1 = 2 olarak yazılır. durum vardır. Bu durum 2 2 = 4 olarak yazılır.

14 | Matematik 9
MANTIK

p q r
1 1 1
1 1 0
1 0 1
1 0 0 Birbirinden bağımsız herhangi üç önerme için sekiz farklı durum vardır. Bu
durum 2 3 = 8 olarak yazılır.
0 1 1
0 1 0
0 0 1
0 0 0

n ! N + olmak üzere birbirinden bağımsız n tane önermenin 2 n tane doğruluk değeri vardır.

ÖRNEK 2:
Birbirinden bağımsız p, q, r, s ve t önermelerinin kaç farklı doğruluk değeri olduğunu bulunuz.

ÇÖZÜM:
5 tane önerme olduğundan bu önermelerin 2 5 = 32 farklı doğruluk değeri vardır.

İki Önermenin Denkliği


Doğruluk değerleri aynı olan önermelere denk önermeler denir. p ve q önermelerinin doğruluk
değerleri aynı ise bu durum p / q şeklinde gösterilir. Doğruluk değerleri aynı olmayan p ve q önermeleri
p _ q biçiminde gösterilir.

ÖRNEK 3:
Aşağıdaki önermelerin doğruluk değerlerini inceleyiniz. Bu önermelerden denk olanları belirtiniz.
p: “7 asal sayıdır.”
q: “Bir hafta 7 gündür.”
r: “En küçük asal sayı 3 tür.”

ÇÖZÜM:
p önermesi doğru bir önermedir. p / 1
3p/q
q önermesi doğru bir önermedir. q / 1
r önermesi yanlış bir önermedir. ( r / 0 ) Bu durumda p _ r ve q _ r yazılır.

Bir Önermenin Değili (Olumsuzu)

Bir önermenin hükmünün değiştirilmesiyle elde edilen yeni önermeye bu önermenin değili
(olumsuzu) denir. p önermesinin değili pl ya da + p ile gösterilir.
p önermesi doğru ise değili yanlış, p önermesi yanlış ise değili doğru önerme olur.

p pl ^plhl
1 in değili $ 1l / 0
1 0 1 0 ın değili $ 0l / 1 tir.
0 1 0 Bir önermenin değilinin değili kendisidir. ^plhl / p tir.

Matematik 9 | 15
MANTIK

ÖRNEK 4 :
Aşağıdaki önermelerin değillerini yazınız. Önermelerin ve değillerinin doğruluk değerlerini bulunuz.
a) p: “Tek doğal sayıların pozitif tam sayı kuvvetleri de tektir.”
b) q: “23 sayısı, 2 ile tam bölünür.”
c) r: “Karenin dış açıları toplamı 360c dir. ”
ç) s: “Doğrusal olmayan üç noktanın ikişer ikişer birleştirilmesiyle elde edilen şekil üçgendir.”

ÇÖZÜM:
a) p: “Tek doğal sayıların pozitif tam sayı kuvvetleri de tektir.” önermesi doğru bir önerme olur. Çünkü iki
tek sayı çarpıldığında sonuç yine bir tek sayı olur. Buna göre p / 1 dir. Bu önermenin değili:
pl : “Tek doğal sayıların pozitif tam sayı kuvvetleri tek değildir.” şeklindedir. pl önermesi yanlış önerme
olduğundan pl / 0 olur.

b) q: “23 sayısı, 2 ile tam bölünür.” önermesi yanlış bir önermedir. Çünkü 23 = 2 $ 11 + 1 dir. Buna göre
q / 0 dır. Bu önermenin değili:
ql : “23 sayısı, 2 ile tam bölünmez.” şeklindedir. ql önermesi doğru önerme olduğundan ql / 1 olur.

c) r: “Karenin dış açıları toplamı 360c dir.” önermesi doğru bir önermedir. Çünkü, dış açıların her biri 90c
dir. Buna göre r / 1 dir. Bu önermenin değili:
rl : “Karenin dış açıları toplamı 360c değildir.” şeklindedir. rl önermesi yanlış bir önerme olduğundan
rl / 0 olur.

ç) s: “Doğrusal olmayan üç noktanın ikişer ikişer birleştirilmesiyle elde edilen şekle üçgen denir.” önermesi
doğru bir önerme olduğundan s / 1 dir. Bu önermenin değili:
sl : “Doğrusal olmayan üç noktanın ikişer ikişer birleştirilmesiyle elde edilen şekil, üçgen değildir.”
şeklindedir. sl önermesi yanlış bir önerme olduğundan sl / 0 olur.

ÖRNEK 5:
p : “İki tek tam sayının çarpımı tektir. ” önermesinin değilinin değilini yazıp p ile denkliklerini inceleyiniz.

ÇÖZÜM:
p: “İki tek tam sayının çarpımı tektir.” önermesi doğru önerme olduğundan p / 1 dir. p önermesinin değili:
pl : “İki tek tam sayının çarpımı tek değildir.” şeklindedir. Bu önerme yanlış önerme olduğundan pl / 0 olur.
pl önermesinin tekrar değili alınırsa
^plhl : “İki tek tam sayının çarpımı tektir.” şeklinde olur. Bu önerme, doğru önerme olduğundan ^plhl / 1
olur.
p / 1 ve ^plhl / 1 olduğundan p / ^plhl elde edilir.

16 | Matematik 9
MANTIK

2. Bileşik Önerme

İki veya daha fazla önermenin ve ( / ), veya ( 0 ), ya da ( Q ), ise ( & ), ancak ve ancak ( + ) bağlaçları
ile birleştirilmesiyle elde edilen yeni önermelere bileşik önerme denir.

a) "Ve" Bağlacı ile Yapılan Bileşik Önermeler


p: “Mehmet kiraz aldı.”
q: “Mehmet vişne aldı.”
p, q ve pl , ql önermeleri, “ve ( / )” bağlacı ile birleştirilerek bileşik önerme hâlinde yazılmıştır. İnceleyiniz.
p / q : “Mehmet, kiraz ve vişne aldı.”
pl / q : “Mehmet, kiraz almadı ve vişne aldı.”
p / ql : “Mehmet, kiraz aldı ve vişne almadı.”
pl / ql : “Mehmet, kiraz almadı ve vişne almadı.”

“Ve ( / )” bağlacı ile bağlanan bileşik önerme, bileşenlerden her p q p/q


ikisi doğru ise doğru, en az biri yanlış ise yanlıştır. Doğruluk tablosu
yanda verilmiştir. 1 1 1
1 0 0
0 1 0
0 0 0

b) "Veya" Bağlacı ile Yapılan Bileşik Önermeler


p: “Nazlı, sarışın bir öğrencidir.”
q: “Nazlı, mavi gözlü bir öğrencidir.”
p, q, pl ve ql önermeleri “veya( 0 )” bağlacı ile birleştirilerek bileşik önerme hâlinde yazılmıştır. İnceleyiniz.
p 0 q : “Nazlı, sarışın bir öğrencidir veya mavi gözlü bir öğrencidir.”
pl 0 q : “Nazlı, sarışın bir öğrenci değil veya mavi gözlü bir öğrencidir.”
p 0 ql : “Nazlı, sarışın bir öğrencidir veya mavi gözlü bir öğrenci değildir.”
pl 0 ql : “Nazlı, sarışın bir öğrenci değildir veya mavi gözlü bir öğrenci değildir."

“Veya( 0 )” bağlacı ile bağlanan bileşik önerme; bileşenlerden en p0q


p q
az biri doğru ise doğru, her iki bileşen de yanlış ise yanlıştır. Doğruluk
tablosu yanda verilmiştir. 1 1 1
1 0 1
0 1 1
0 0 0

Matematik 9 | 17
MANTIK

“Ve” ve “Veya” Bağlacıyla Yapılan Bileşik Önermelerin Özellikleri

1. Her p önermesi için p / p / p ve p 0 p / p tir. (Tek kuvvet özelliği)


p p p/p / p p p p0p / p
1 1 1 1 1 1
0 0 0 0 0 0

2. p ve q önermeleri için p / q / q / p ve p 0 q / q 0 p tir. (Değişme özelliği)

p q p/q q/p p q p0q q0p


1 1 1 1 1 1 1 1
1 0 0 0 1 0 1 1
0 1 0 0 0 1 1 1
0 0 0 0 0 0 0 0

3. Her p, q ve r önermeleri için p / ^q / rh / ^p / qh / r ve p 0 ^q 0 rh / ^p 0 qh 0 r tir. (Birleşme özelliği)

p q r q/r p / ^q / rh p/q ^p / qh / r
1 1 1 1 1 1 1
1 1 0 0 0 1 0
1 0 1 0 0 0 0
1 0 0 0 0 0 0
0 1 1 1 0 0 0
0 1 0 0 0 0 0
0 0 1 0 0 0 0
0 0 0 0 0 0 0
Benzer şekilde p 0 ^q 0 r h / ^p 0 q h 0 r denkliğinin doğruluğunu tablo yaparak gösteriniz.

4. Her p, q ve r önermeleri için


a) p / ^q 0 rh / ^p / qh 0 ^p / rh tir. (“ve” bağlacının “veya” bağlacı üzerine soldan dağılma özelliği)

p q r q 0 r p / ^q 0 rh p / q p/r ^p / qh 0 ^p / rh
1 1 1 1 1 1 1 1

1 1 0 1 1 1 0 1

1 0 1 1 1 0 1 1

1 0 0 0 0 0 0 0

0 1 1 1 0 0 0 0

0 1 0 1 0 0 0 0

0 0 1 1 0 0 0 0

0 0 0 0 0 0 0 0

18 | Matematik 9
MANTIK

b) p 0 ^q / rh / ^p 0 q h / ^p 0 rh tir. (“veya” bağlacının “ve” bağlacı üzerine soldan dağılma özelliği)


Siz de “veya” bağlacının “ve” bağlacı üzerine soldan dağılma özelliği olduğunu tablo yaparak gösteriniz.
5. Her p, q önermesi için ^p / qhl / pl 0 ql ve ^p 0 qhl / pl / ql tir. (De Morgan kuralları)
Bu denkliklerin doğruluk tablosu aşağıda verilmiştir. İnceleyiniz.

p q pl ql p 0 q p / q ^p 0 q hl pl / ql ^p / q hl pl 0 ql
1 1 0 0 1 1 0 0 0 0
1 0 0 1 1 0 0 0 1 1
0 1 1 0 1 0 0 0 1 1
0 0 1 1 0 0 1 1 1 1

p ve q önermeleri için
1. p / 1 / p, p / 0 / 0, p 0 1 / 1, p00 / p
2. p / pl / 0, p 0 pl / 1

ÖRNEK 1:

6]1 0 0g 0 ]0 / 0gl@l bileşik önermesinin doğruluk değerini bulunuz.

ÇÖZÜM:
6]1 0 0g 0 ]0 / 0gl@l / 51 0 0l?l / 51 0 1?l / 51?l / 0 olur.

ÖRNEK 2:

^p / qlhl / ^pl 0 qlh / pl olduğunu doğruluk tablosu yaparak gösteriniz.

ÇÖZÜM:
p q pl ql p / ql ^p / qlhl pl 0 ql ^p / qlhl / ^pl 0 qlh
1 1 0 0 0 1 0 0
1 0 0 1 1 0 1 0
0 1 1 0 0 1 1 1
0 0 1 1 0 1 1 1

Doğruluk tablosundaki ^p / qlhl / ^pl 0 qlh önermesi ile pl önermesinin doğruluk değerleri aynı
olduğundan ^p / qlhl / ^pl 0 qlh / pl tir.

ÖRNEK 3:
p / 6^pl 0 q hl 0 ^p / q h@ bileşik önermesinin en sade şeklini yazınız.

ÇÖZÜM:
p / 6^pl 0 q hl 0 ^p / q h@ / p / 6^p / qlh 0 ^p / q h@ (De Morgan kuralı)
/ p / 6p / ^ql 0 q h@ ( / nin 0 üzerine dağılma özelliği)
/ p / 6p / 1@
/ p/p
/p

Matematik 9 | 19
MANTIK

ÖRNEK 4:
^p / q hl 0 r / 0 olduğuna göre p, q ve r önermelerinin doğruluk değerlerini bulunuz.

ÇÖZÜM:
^p / q hl 0 r / 0 ise ^p / q hl / 0 ve r / 0 olmalıdır.
^p / q hl / 0 ise p / q / 1 olur. Buradan p / 1, q / 1 elde edilir.
O hâlde p, q ve r önermelerinin doğruluk değerleri p / 1, q / 1, r / 0 bulunur.

ÖRNEK 5:
^pl 0 q hl / ]r / slg bileşik önermesinin p / 0, q / 1, r / 0 ve s / 1 için doğruluk değerini bulunuz.

ÇÖZÜM:
Verilen doğruluk değerleri bileşik önermede yerine yazılırsa
^pl 0 q hl / ]r / slg / ]0l 0 1gl / ]0 / 1lg
/ ]1 0 1gl / ]0 / 0g
/ 1l / 0
/ 0/0
/ 0 bulunur.

ÖRNEK 6:
p / rl / 1 ve pl 0 ql / 0 ise 6p / ^ql 0 r h@ 0 6^pl 0 q h / rl@ bileşik önermesinin doğruluk değerini bulunuz.

ÇÖZÜM:
p / rl / 1 ise p / 1 ve rl / 1 olmalıdır. Buradan r / 0 olur.
pl 0 ql / 0 ise 1l 0 ql / 0
0 0 ql / 0
ql / 0
q / 1 tir.
p / 1, q / 1 ve r / 0 doğruluk değerleri 6p / ^ql 0 r h@ 0 6^pl 0 q h / rl@ bileşik önermesinde yerine
yazılırsa
6p / ^ql 0 r h@ 0 6^pl 0 q h / rl@ / 61 / ]1l 0 0g@ 0 6]1l 0 1g / 0l@
/ 61 / ]0 0 0g@ 0 6]0 0 1g / 1@
/ ]1 / 0 g 0 ]1 / 1 g
/ 001
/ 1 bulunur.

Sıra Sizde

SORU:
6p / ^pl 0 q h@l / 6ql 0 ^p / q h@ / ql olduğunu gösteriniz.
ÇÖZÜM:

20 | Matematik 9
MANTIK

Ve / Veya Bağlaçlarının Elektrik Devrelerinde Kullanılışı

Sembolik mantığın matematik dışında elektrik devrelerinde de kullanım alanı vardır. Elektrik
devrelerinde akımın geçmesi 1, geçmemesi 0 ile gösterilirse tüm elektrik devreleri sembolik mantık ile
ifade edilebilir.
• Açık anahtar (akım geçirmeyen anahtar): şeklinde gösterilir. p anahtarı açık ise doğruluk
değeri p / 0 tir.
• Kapalı anahtar (akım geçiren anahtar): şeklinde gösterilir. p anahtarı kapalı ise doğruluk
değeri p / 1 tir.

Aşağıdaki elektrik devreleri, akım geçiren ve geçirmeyen durumları göstermektedir. İnceleyiniz.

Şekil: 1 Şekil: 2

Şekil 1: Akım geçmediğinden lamba yanmaz. Şekil 2: Akım geçtiğinden lamba yanar.
Bu durumda seri ve paralel bağlama aşağıdaki gibi gösterilir.
p q r
Elektrik devrelerinde seri bağlama şeklinde çizilir. p / q / r ile gösterilir.
p
Elektrik devrelerinde paralel bağlama q şeklinde çizilir. p 0 q ile gösterilir.

Bir elektrik devresine karşılık gelen bileşik önerme yazılırken anahtarların açık ya da kapalı olması
dikkate alınmaz. Ancak devre çizilirken anahtarların açık ya da kapalı olması dikkate alınır.

ÖRNEK 7:
Aşağıdaki şekilde verilen elektrik devresine karşı gelen bileşik önermeyi yazınız. Lambanın yanıp
yanmayacağını belirtiniz.

p
q r

ÇÖZÜM:
Şekilde q ve r anahtarları seri bağlanmış, p anahtarı ise bu anahtarlara paralel bağlanmıştır. Bu durumda
elektrik devresine karşı gelen bileşik önerme p 0 ^q / r h olur.
p anahtarı açık olduğundan akım geçirmez. p / 0 tir.
q ve r anahtarları kapalı olduğundan akım geçirir. q / 1 ve r / 1 tir.
Bulunan doğruluk değerleri bileşik önermede yerine yazılırsa

p 0 ^ q / r h / 0 0 ]1 / 1g
/ 001
/ 1 bulunur. Bu durumda devreden akım geçer ve lamba yanar.

Matematik 9 | 21
MANTIK

ÖRNEK 8:
p / 1, q / 1, r / 0, s / 1, t / 0 olduğuna göre
a) ^pl / s h 0 6q 0 ]r / tlg@ bileşik önermesine karşı gelen elektrik
devresini çiziniz.
b) Lambanın yanıp yanmayacağını belirtiniz.

ÇÖZÜM:
pl s
a) p / 1, q / 1, r / 0, s / 1, t / 0 olmak üzere
p, q, r, s, t anahtarları için p, q, s kapalı konumu; r, t açık q
konumu gösterir. ^pl / s h 0 6q 0 ]r / tlg@ bileşik önermesine ait
elektrik devresi yanda çizilmiştir. r tl
b) p / 1, q / 1, r / 0, s / 1, t / 0 değerleri bileşik önermede
yerine yazılırsa
^pl / s h 0 6q 0 ^r / tlh@ / ^0 / 1 h 0 61 0 ^0 / 1 h@ / 0 0 ^1 0 0 h
/ / 0 0 1 / 1 olduğundan lamba yanar.

c) “Ya da ( Q )” Bağlacı ile Yapılan Bileşik Önermeler


p: “ Her x, y ! R için x 2 - y 2 = ^x - y h^x + y h tir. ”
q: “61, asal sayıdır.”
p, q, pl ve ql önermeleri “ya da ( Q )” bağlacı ile birleştirilerek bileşik önerme hâlinde yazılmıştır. İnceleyiniz.
p Q q : “ Her x, y ! R için x 2 - y 2 = ^x - y h^x + y h ya da 61 asal sayıdır.”
pl Q q : “ Her x, y ! R için x 2 - y 2 ! ^x - y h^x + y h ya da 61 asal sayıdır.”
p Q ql : “ Her x, y ! R için x 2 - y 2 = ^x - y h^x + y h ya da 61 asal sayı değildir.”
pl Q ql : “ Her x, y ! R için x 2 - y 2 ! ^x - y h^x + y h ya da 61 asal sayı değildir.”

p q pQq

“Ya da ( Q )” bağlacı ile bağlanan bileşik önerme; bileşenlerinin 1 1 0


doğruluk değerleri aynı ise yanlış, bileşenlerinin doğruluk değerleri 1 0 1
farklı ise doğrudur. 0 1 1
0 0 0

“Ya da” Bağlacının Özellikleri


1. p, q önermeleri için 2. Her p, q ve r önermesi için
p Q q / q Q p (Değişme özelliği) p Q ^q Q r h / ^p Q q h Q r tir. (Birleşme özelliği)

p q pQq qQp p q r qQr p Q ^q Q rh pQq ^p Q qh Q r


1 1 0 0 1 1 1 0 1 0 1
1 0 1 1 1 1 0 1 0 0 0
1 0 1 1 0 1 0
0 1 1 1
1 0 0 0 1 1 1
0 0 0 0
0 1 1 0 0 1 0
0 1 0 1 1 1 1
0 0 1 1 1 0 1
0 0 0 0 0 0 0
3. ^p Q qhl / pl Q q / p Q ql
4. p Q p / 0, p Q 1 / pl , p Q 0 / p, p Q pl / 1

22 | Matematik 9
MANTIK

ÖRNEK 9:
ql / 0 ve pl Q q / 0 olduğuna göre 6^pl / q h Q ^p 0 qlh@l Q p bileşik önermesinin doğruluk değerini
bulunuz.

ÇÖZÜM:
ql / 0 & q / 1 olur. q / 1 değeri pl Q q / 0 ifadesinde yerine yazılırsa
pl Q 1 / 0 ise pl / 1
ise p / 0 bulunur. Buradan
6^pl / q h Q ^p 0 qlh@l Q p / 6]0l / 1g Q ]0 0 1g@l Q 0 / 6]1 / 1g Q ]0 0 0g@l Q 0
/ ]1 Q 0gl Q 0
/ 1l Q 0 / 0 Q 0 / 0 sonucuna ulaşılır.

ÖRNEK 10:
^p Q plh / ^q Q q h / ^p Q q h bileşik önermesini en sade biçimde yazınız.

ÇÖZÜM:
^p Q plh / ^q Q q h / ^p Q q h / 1 / 0 / ^p Q q h
/ 0 / ^p Q qh
/ 0 bulunur.

3. Koşullu Önerme ve İki Yönlü Koşullu Önerme


a) Koşullu Önerme
p ve q iki önerme olmak üzere p ve q önermelerinin ise ]&g bağlacı ile birleştirilmesiyle elde edilen
p & q önermesine koşullu önerme denir. p & q önermesi p ise q şeklinde okunur.

Köyde yaşayan Ahmet ÇELİK "Eğer muhtar seçilirsem köy yolu asfalt yaptırılacaktır." şeklinde bir cümle
kuruyor. Kişinin kurduğu bu cümle,
p: “Ahmet ÇELİK muhtar seçilir.”
q: “Köy yolu asfalt yaptırılacaktır.”
şeklinde iki önermenin birleşimi ile kurulmuş bir bileşik önermedir. Kullanılan bağlaç ise ]&g bağlacıdır. Cümle,
koşula bağlı olduğundan koşullu önerme olur.
p, q ve pl , ql önermeleri, ise ]&g bağlacı ile birleştirilerek bileşik önerme hâlinde yazılmıştır. İnceleyiniz.

p & q : “Eğer muhtar seçilirsem köy yolu asfalt yaptırılacaktır.”


pl & q : “Eğer muhtar seçilmezsem köy yolu asfalt yaptırılacaktır.”
p & ql : “Eğer muhtar seçilirsem köy yolu asfalt yaptırılmayacaktır.”
pl & ql : “Eğer muhtar seçilmezsem köy yolu asfalt yaptırılmayacaktır.”

p & q koşullu önermesinde


p q p&q
p: Koşullu önermenin hipotezidir.
q: Koşullu önermenin hükmüdür. 1 1 1
1 0 0
p & q koşullu önermesinin doğruluk değeri 1 ise bu koşullu 0 1 1
önermeye gerektirme denir. 0 0 1

Matematik 9 | 23
MANTIK

ÖRNEK 1:

p & ^ql 0 r h / 0 olduğuna göre p 0 ^q Q r h bileşik önermesinin doğruluk değerini bulunuz.

ÇÖZÜM:
p & ^ql 0 r h / 0 ise p / 1 ve ql 0 r / 0 tir.
ql 0 r / 0 ise ql / 0 ve r / 0 tir.
Buradan q / 1 olur.
p / 1, q / 1, r / 0 için p 0 ^q Q r h / 1 0 ]1 Q 0g
/ 101
/ 1 bulunur.

Bir Koşullu Önermenin Karşıtı, Tersi, Karşıt Tersi

p ve q önermeleri ile oluşturulan p & q koşullu önermesi verilmiş olsun. Bu durumda


a) p & q önermesinin karşıtı q & p ,
b) p & q önermesinin tersi pl & ql ,
c) p & q önermesinin karşıt tersi ql & pl
şeklinde tanımlanır.

p & q önermesinin tersi pl & ql ile p & q önermesinin değili ^p & q hl aynı olmadığı aşağıdaki tabloda
gösterilmiştir.

p q pl ql pl & ql p&q ^p & q hl


1 1 0 0 1 1 0
1 0 0 1 1 0 1
0 1 1 0 0 1 0
0 0 1 1 1 1 0

ÖRNEK 2:
“ x = 3 & x 2 = 9 dur.” koşullu önermesinin karşıtı, tersi ve karşıt tersini yazınız.

ÇÖZÜM:
p & q :“ x = 3 & x 2 = 9 dur. ” koşullu önermesinde
2
p & q önermesinin karşıtı q & p : “ x = 9 & x = 3 tir. ”
p & q önermesinin tersi pl & ql : “ x Y= 3 & x2 Y
= 9 dir. ”
2
p & q önermesinin karşıt tersi ql & pl : “ x Y=9 &xY = 3 dir. ”

24 | Matematik 9
MANTIK

“İse ^&h ” Bağlacının Özellikleri


p ve q önermeleri için
1. p & p / 1
2. p & 1 / 1, p & 0 / pl , 1 & p / p
0 & p / 1, p & pl / pl , pl & p / p
3. p & q / pl 0 q tir. 4. p & q / ql & pl tir.

p q pl p & q pl 0 q p q pl ql p & q ql & pl


1 1 0 1 1 1 1 0 0 1 1
1 0 0 0 0 1 0 0 1 0 0
0 1 1 1 1 0 1 1 0 1 1
0 0 1 1 1 0 0 1 1 1 1

ÖRNEK 3:
“Hava bulutlu ise yağmur yağar.” önermesinin olumsuzunu yazınız.

ÇÖZÜM:
p: “Hava bulutludur.”
q: “Yağmur yağar.”
p & q : “Hava bulutlu ise yağmur yağar.” önermesinin olumsuzu,
^p & q hl / ^pl 0 q hl / p / ql olduğundan
^p & q hl : “Hava bulutlu ve yağmur yağmaz.” olur.

ÖRNEK 4:
^pl & q h & p önermesine denk olan önermeyi bulunuz.

ÇÖZÜM:
^pl & q h & p / ^p 0 q h & p / ^p 0 q hl 0 p / ^pl / qlh 0 p / ^pl 0 p h / ^ql 0 p h
/ 1 / ^ql 0 p h
/ ql 0 p olur.

ÖRNEK 5:
^p / q h & ^r 0 qlh / 0 olduğuna göre ^p Q q h & r önermesinin doğruluk değerini bulunuz.

ÇÖZÜM:
^p / q h & ^r 0 qlh / 0 ise ^p / q h / 1 ve r 0 ql / 0 tir.
p / q / 1 ise p / 1 ve q / 1 dir. r 0 ql / 0 ise r / 0 ve ql / 0, q / 1 bulunur.
p / q / 1 ve r / 0 için ^p Q q h & r / ]1 Q 1g & 0
/0&0
/ 1 bulunur.

Matematik 9 | 25
MANTIK

ÖRNEK 6:
^p & qlhl & ^p & plh önermesine denk olan önermeyi bulunuz.

ÇÖZÜM:
^p & qlh & ^p & plh / ^pl 0 qlh 0 pl
l ^ p & ql / pl 0 ql , p & pl / plh
/ ^ql 0 plh 0 pl (Değişme özelliği)
/ ql 0 ^pl 0 plh (Birleşme özelliği)
/ ql 0 pl / q & pl bulunur.

b) İki Yönlü Koşullu Önerme


p ve q iki önerme olmak üzere p ve q önermelerinin ancak ve ancak bağlacı ile birleştirilmesiyle elde
edilen p + q önermesine iki yönlü koşullu önerme denir. p + q önermesi “p ancak ve ancak q” şeklinde
okunur.

p: “ABC üçgeni bir eşkenar üçgendir.”


q: “Her bir açısı 60c dir.”
p, q ve pl , ql önermeleri “ancak ve ancak ( + )” bağlacı ile birleştirilerek bileşik önerme hâlinde yazılmıştır.
İnceleyiniz.
p + q : “ABC üçgeni eşkenar üçgendir ancak ve ancak her bir açısı 60c dir.”
pl + q : “ABC üçgeni eşkenar üçgen değildir ancak ve ancak her bir açısı 60c dir.”
p + ql : “ABC üçgeni eşkenar üçgendir ancak ve ancak her bir açısı 60c değildir.”
pl + ql : “ABC üçgeni eşkenar üçgen değildir ancak ve ancak her bir açısı 60c değildir.”

p + q iki yönlü koşullu önermesi p ve q nun doğruluk değerleri


p q p+q
aynı iken doğru, farklı iken yanlıştır.
1 1 1
1 0 0
0 1 0
0 0 1

ÖRNEK 7:
^pl & q h & ^p 0 rlh / 0 olduğuna göre ^p + qlh / ^q & r h önermesinin doğruluk değerini bulunuz.

ÇÖZÜM:
^pl & q h & ^p 0 rlh / 0 ise pl & q / 1 ve p 0 rl / 0 tir.
p 0 rl / 0 ise p / 0 ve rl / 0, r / 1 olur.
p / 0 bulunduğunda
pl & q / 1 ise 0l & q / 1
1 & q / 1, q / 1 bulunur.
p / 0, q / 1 ve r / 1 için
^p + qlh / ^q & r h / ]0 + 1lg / ]1 & 1g
/ ]0 + 0g / 1
/ 1/1
/ 1 tir.

26 | Matematik 9
MANTIK

“Ancak ve Ancak ^+h " Bağlacının Özellikleri


p ve q önermeleri için
1. p + q / ^p & qh / ^q & ph
p q p & q q & p ^p & qh / ^q & ph p+q
1 1 1 1 1 1

1 0 0 1 0 0

0 1 1 0 0 0

0 0 1 1 1 1

2. p + q / pl + ql
3. p + p / 1, p + pl / 0, p + 1 / p, p + 0 / pl
4. ^p + q hl / pl + q / p + ql
5. ^p + q hl / p Q q

ÖRNEK 8:
6^p & qlh / p@l + ^p & qh önermesinin en sade şeklini bulunuz.

ÇÖZÜM:
6^p & qlh / p@l + ^p & qh / 6^pl 0 qlh / p@l + ^pl 0 qh
/ 6^pl / p h 0 ^ql / p h@l + ^pl 0 q h
/ 60 0 ^ql / p h@l + ^pl 0 q h
/ ^ql / p hl + ^pl 0 q h
/ ^pl 0 q h + ^pl 0 q h
/ 1 bulunur.

ÖRNEK 9:
^p & q h + p önermesinin en sade şeklini bulunuz.

ÇÖZÜM:
^p & q h + p / 6^p & q h & p@ / 6p & ^p & q h@
/ 6^pl 0 q h & p@ / 6p & ^pl 0 q h@
/ 6^pl 0 q hl 0 p@ / 6pl 0 ^pl 0 q h@
/ 6^p / qlh 0 p@ / 6^pl 0 plh 0 q@
/ p / ^pl 0 q h
/ ^p / plh 0 ^p / q h
/ 0 0 ^p / qh
/ ^p / q h bulunur.

Matematik 9 | 27
MANTIK

4. Sözel ya da Sembolik Mantık Dilinde Verilen Bileşik Önermelerin Birbirine


Dönüştürülmesi

Sözel olarak verilen bir önerme; sembolik mantık dilinde yazılırken sözel ifadede bulunan
matematiksel sembol, kavram ve bağlaçlar belirlenir. Daha sonra sözel olarak verilen önerme, sembolik
mantık dilinde yazılır.
Önerme sembolik mantık dilinde verilmişse kullanılan sembol, bağlaç ve diğer matematiksel
kavramlar sözel ifadeye dönüştürülerek önermenin sözel ifadesi yazılır.

ÖRNEK 1:
Aşağıda verilen önermeleri sözel sembolik mantık dilinde yazınız.
a) “Bütün tam sayılar aynı zamanda rasyonel sayıdır.”
b) “Bir gerçek sayının 5 katının 3 eksiği 7 ise bu sayı 2 dir.”
c) “ ^x 2 = 25 h & ]x = 5 0 x =- 5g tir.”

ÇÖZÜM:
p ve q önermeleri sözel olarak yazılmış birer önerme iken r önermesi sembolik mantık dilinde yazılmış bir
önermedir.
a) p(x) : “ x ! Z & x ! Q ” şeklinde yazılır.
b) q önermesi, sembolik mantık dilinde “Bir gerçek sayı (x), gerçek sayının 5 katının 3 eksiği ]5x - 3g , eşitlik
( = )” sembol ve kavramları kullanılarak q(x) : “ ^x ! R, 5x - 3 = 7 h & ]x = 2g ” şeklinde yazılır.
c) r(x): “Bir x gerçek sayısının karesi 25 ise bu sayılar - 5 veya 5 tir.”

5. Totoloji ve Çelişki
Bir bileşik önerme, bileşenlerinin bütün doğruluk değerleri için doğru (1) oluyorsa totoloji, yanlış (0)
oluyorsa çelişki olarak tanımlanır.

ÖRNEK 1:
p 0 pl ve p / pl önermelerinin doğruluk değerlerini tablo yaparak inceleyiniz.

ÇÖZÜM:

p pl p 0 pl

1 0 1
0 1 1
Totoloji

p pl p / pl
1 0 0
0 1 0
Çelişki
p 0 pl bileşik önermesi, p nin tüm doğruluk değerleri için doğru (1) olduğundan bir totolojidir.
p / pl bileşik önermesi, p nin tüm doğruluk değerleri için yanlış (0) olduğundan bir çelişkidir.

28 | Matematik 9
MANTIK

ÖRNEK 2:
6^p 0 qlh & ^pl 0 q h@ + ^pl 0 q h bileşik önermesinin totoloji olduğunu gösteriniz.

ÇÖZÜM:
6^p 0 qlh & ^pl 0 q h@ + ^pl 0 q h / 6^p 0 qlhl 0 ^pl 0 q h@ + ^pl 0 q h
/ 6^pl / q h 0 ^pl 0 q h@ + ^pl 0 q h
/ "6^pl / q h 0 pl@ 0 q , + ^pl 0 q h (pl / pl / 1)
/ "6^pl / q h 0 ^pl / 1 h@ 0 q , + ^pl 0 q h
/ "6pl / ^q 0 1 h@ 0 q , + ^pl 0 q h
/ "6pl / 1@ 0 q , + ^pl 0 q h
/ ^pl 0 q h + ^pl 0 q h
/1
Bileşik önerme, p ve q önermelerinin tüm doğruluk değerleri için doğru olduğundan totolojidir.

ÖRNEK 3:
6p + ^p + 0h@ + 6^p & qh 0 p@ bileşik önermesinin çelişki olduğunu gösteriniz.

ÇÖZÜM:
6p + ^p + 0h@ + 6^p & qh 0 p@ / ^p + plh + 6^pl 0 q h 0 p@ / 0 + 6^pl 0 ph 0 q@ / 0 + ^1 0 qh
/0+1
/ 0 olduğundan çelişkidir.

ÖRNEK 4:
6^p & qlh & ^p / qlh@ & 6^ql / ph & p@ bileşik önermesinin totoloji olduğunu gösteriniz.

ÇÖZÜM:
6^p & qlh & ^p / qlh@ & 6^ql / ph & p@ / 6^pl 0 qlhl 0 ^p / qlh@ & 6^ql / phl 0 p@
/ 6^p / q h 0 ^p / qlh@ & 6q 0 pl 0 p@
/ 6p / ^q 0 qlh@ & 6q 0 ^pl 0 p h@
/ ^p / 1h & ^q 0 1h / p & 1 / 1 olduğundan totolojidir.

Sıra Sizde

SORU:
6^p / q hl & ^p & q h@ 0 p bileşik önermesini en sade şekilde yazınız.
ÇÖZÜM:

Matematik 9 | 29
MANTIK

ALIŞTIRMALAR

1 Aşağıdaki cümlelerden önerme olanların doğruluk değerlerini karşılarındaki sütunlara yazınız.

Önerme Doğruluk Değeri


a) Ordu-Giresun Havaalanı, denize dolgu yapılarak inşa edilmiştir.
b) En büyük negatif tam sayı - 2 dir.
c) Trafik kurallarına uyunuz.
ç) 3 katının 4 eksiği 17 olan doğal sayı - 7 dir.
d) Hangi spor dalı ile ilgileniyorsun?
e) Negatif tam sayıların tek kuvvetleri negatiftir.

2
Aşağıda verilen önermelerden denk olanların arasına “ / ” sembolünü, denk olmayanların arasına “ _ ”
sembolünü yazınız.
p: “Tümler iki açının ölçüleri toplamı 90c dir.” p...q q...r
q: “Doğrusal olan üç nokta bir üçgen belirtir.” q...s s...r
r: “ 3x + 4 =- 8 ise x =- 4 tür. ” p...r p...s
s: “Doğal sayılar kümesi, tam sayılar kümesini kapsar.”

3 Aşağıda verilen önermelerin her birinin değilini (olumsuzunu) yazınız.

Önerme Önermenin Değili


p: “ 5 - 3 = 2 ” pl :
q: “316 sayısı, 5 ile tam bölünmez.” ql :
r: “ 2 4 = 4 2 ” rl :
s: “ 16 - 3 4 = 2 ” sl :

4 ^pl 0 q hl / rl / 1 olduğuna göre p, q ve r önermelerinin doğruluk değerlerini bulunuz.

5 pl 0 q / 0 ve q 0 rl / 1 ise 6p 0 ^ql / r h@ 0 6^p 0 q hl / rl@ bileşik önermesinin doğruluk değerini


bulunuz.

6 ^pl / q h & ^p 0 rlh / 0 olduğuna göre p & ^ql & r h önermesinin doğruluk değerini bulunuz.

30 | Matematik 9
MANTIK

7 ^rl / p h 0 6p / ^r 0 q h@ bileşik önermesinin en 12 ql & ^q & p h bileşik önermesinin doğruluk


sade şeklini bulunuz. değerini, doğruluk değerleri tablosu yaparak
bulunuz.

13 Aşağıda sembolik mantık dilinde yazılmış


8 p 0 ^p / q h / p olduğunu doğruluk değerleri önermelerin değilini bulunuz.
tablosu yaparak gösteriniz. a) p(x): “ 7x, 3x 2 - 5 $ 2 ”
b) q(x): “ ^7x, x 3 1 0 h / ^6x, x 2 $ 0 h ”

Aşağıdaki şekildeki elektrik devresine karşılık 14 a) ^pl 0 qlh / ^p / q h bileşik önermesinin


9
gelen bileşik önermeyi yazarak ampulün yanma çelişki olduğunu tablo yapmadan
durumunu belirtiniz. gösteriniz.

b) 6^p & qlh & pl@ 0 ql bileşik önermesinin


p totoloji olduğunu tablo yapmadan
q gösteriniz.
r s

10 Birbirinden bağımsız 3n-1 tane önermeden 2


tanesinin doğruluk değeri bilinmektedir. Bu
önermelerin doğruluk tablosunda 16 15 tane
farklı durum var ise n kaçtır? 15 p: x = y q: x + y = 0
2 2
r: x = y s: x $ y # 0
Yukarıda verilenlere göre aşağıdaki
önermelerin doğruluk değerlerini bulunuz.
a) ^p & q h 0 ^q & p h
b) p + r
c) s & ^p / q h
11 a) “Bir hayvan tavuksa 2 ayaklıdır.” Koşullu ç) ^ql / r h & sl
önermesinin karşıtı, tersi ve karşıt tersini
yazınız.

b) “ ^x 3 =- 1 h & ]x =- 1g ” koşullu
önermesinin karşıt tersini yazınız.

Matematik 9 | 31
MANTIK

9.1.2. AÇIK ÖNERMELER VE İSPAT YÖNTEMLERİ


1. Her ( 6 ) ve Bazı ( 7 ) Niceleyicileri
Günlük konuşma dilinde ve matematikte "her, bütün, hemen hemen hepsi, bazı, en az bir, hiçbiri" gibi
sözcük ya da sözcük grupları kullanılır. Örneğin
“Her tek sayının karesi tektir.”
“Her gün ders çalıştı.”
“Bazı doğal sayılar 3 ile tam bölünür.”
“15 Temmuz Şehitleri konulu şiir yarışmasında her sınıftan en az bir öğrenci ödül aldı.”
cümleleri incelendiğinde

“Her” sözcüğü bütün, hepsi, tamamı anlamına gelir. Her sözcüğü “ 6 ” sembolü ile gösterilir.
“Bazı” sözcüğü ile “en az bir” sözcüğü aynı anlama gelmektedir. “Bazı” sözcüğü “ 7 ” sembolü ile
gösterilir.
“Her” sözcüğü evrensel niceleyici, “bazı” sözcüğü varlıksal niceleyici olarak isimlendirilir.

2. Açık Önerme

p : “Çift olan asal sayı yalnız 2 dir.”


q(x) : “ x ! Z, 3x + 9 = 0 ”
p önermesinde değişken bulunmazken q(x) önermesinde x değişkeni bulunmaktadır. q(x), x in bazı
değerleri için sağlanırken bazı değeri için sağlanmaz.
İçinde en az bir değişken bulunduran ve bu değişkenlere verilen değerler sonucunda kesin olarak
doğru ya da yanlış yargı bildiren ifadelere açık önerme denir. Açık önermeyi sağlayan değerler kümesine,
açık önermenin doğruluk kümesi (çözüm kümesi) denir.
Örneğin “ ....., Ege Bölgesi'ndedir.” ifadesi, içinde bilinmeyen bulundurduğu için bir açık önermedir.
Boş bırakılan yere Kütahya yazılırsa “Kütahya, Ege Bölgesi'ndedir.” önermesi doğru bir önerme olur.
Eğer Bursa yazılırsa “Bursa, Ege Bölgesi'ndedir.” önermesi yanlış bir önerme olur.
A kümesinde tanımlı p önermesi “her” ve “bazı” sözcükleri kullanılarak “ 6x ! A, p ] x g ” ya da
“ 7x ! A, p ] x g ” şeklinde yazılabilir.

ÖRNEK 1:
p(x): “ x 2 1 9, x ! Z ” açık önermesinin doğruluk kümesini bulunuz.

ÇÖZÜM:
x 2 1 9, x ! Z
x 1 3, x ! Z
- 3 1 x 1 3, x ! Z

p(x) önermesinde x yerine -2, -1, 0, 1, 2 yazılırsa elde edilen önermelerin doğruluk değeri 1 olur. Bunun
dışındaki değerlerde önerme yanlış olur. Öyleyse bu durumda p(x) açık önermesinin doğruluk kümesi
" - 2, - 1, 0, 1, 2 , bulunur.

32 | Matematik 9
MANTIK

ÖRNEK 2:
Aşağıda sözel olarak verilen önermeleri, sembolik mantık diliyle yazarak doğruluk değerlerini inceleyiniz.
a) p: “Her x tam sayısı için x 2 $ 0 tir.”
b) q: “Bütün tam sayılar aynı zamanda birer doğal sayıdır.”
c) r: “En az bir x doğal sayısı için x 3 - 1 = 0 tir.”
ç) s: “Bazı n doğal sayıları için n 2 = 5 tir.
d) t: “3 katının 10 eksiği, 5 ten küçük olan en az bir doğal sayı vardır.”

ÇÖZÜM:

a) p: “Her x tam sayısı için x 2 $ 0 tir.” önermesi sembolik mantık diliyle p(x): “ 6x ! Z, x 2 $ 0 ” şeklinde
yazılır. Önerme, her x tam sayısı için doğru olduğundan doğruluk değeri 1 dir.

b) q: “Bütün tam sayılar aynı zamanda doğal sayıdır.” önermesi sembolik mantık diliyle
q(n):“ 6n ! Z, n ! N ” şeklinde yazılır. Bu önerme, bütün tam sayılar için doğru
olmadığından yanlış bir önermedir. Örneğin - 2 ! Z ve - 2 z N dir.

c) r: “En az bir x doğal sayısı için x 3 - 1 = 0 tir.” önermesi sembolik mantık diliyle
r(x): “ 7x ! N, x 3 - 1 = 0 ” şeklinde yazılır. Önerme bazı doğal sayılar için sağlandığından doğru bir
önermedir. 1 ! N için 1 3 - 1 = 0 olduğundan en az bir doğal sayı için x 3 - 1 = 0 tir.

ç) s: “Bazı n doğal sayıları için n 2 = 5 tir. ” önermesi sembolik mantık diliyle s(x): “ 7n ! N, n 2 = 5 ”
şeklinde yazılır. Bu önerme hiçbir doğal sayı için sağlanamadığından yanlış bir önermedir.

d) t: “3 katının 10 eksiği, 5 ten küçük olan en az bir doğal sayı vardır.” önermesi sembolik mantık diliyle
t(x): “ 7x ! N, 3x - 10 1 5 tür. ” şeklinde yazılır. Bu önerme bazı doğal sayılar için sağlandığından
doğru bir önermedir. Örneğin, x = 0 için 3.0 - 10 =- 10 < 5 olup sağlanır.

ÖRNEK 3:
Aşağıda sembolik mantık diliyle verilen önermeleri sözel olarak yazınız.
a) p(x): “ 7x ! R, 2x - 3 = 0 ”
b) q(x): “ 6x ! N, x 2 1 7 ”
c) r(x): “ 6x ! R, x 2 - 1 = ]x - 1g]x + 1g ”

ÇÖZÜM:
p, q ve r önermelerinin sözel olarak yazılışı,
a) p(x): “ 7x ! R, 2x - 3 = 0 ” önermesi için p: “Bazı gerçek sayıların iki katının üç eksiği sıfırdır.”
b) q(x): “ 6x ! N, x 2 1 7 ” önermesi için q: “Her doğal sayının karesi 7 den küçüktür”
c) r(x):“ 6x ! R, x 2 - 1 = ]x - 1g]x + 1g ” önermesi için r: “Her gerçek sayının karesinin bir eksiği, bu
gerçek sayının bir eksiği ile bir fazlasının çarpımına eşittir.”

Matematik 9 | 33
MANTIK

Her ( 6 ) ve Bazı ( 7 ) Niceleyicilerinin Değili (Olumsuzu)

p(x), x değişkenine bağlı bir önerme olsun. pl] x g , p önermesinin değili olmak üzere
a)“ 7x, p ] x g tir.” önermesinin değili “ 6x, pl] x g tir.”
b)“ 6x, p ] x g tir.” önermesinin değili “ 7x, pl] x g tir.”
şeklinde tanımlanır. Bu durum,
67x, p ] x g@l / 6x, pl ] x g ve 66x, p ] x g@l / 7x, pl ] x g şeklinde de yazılır.
Aşağıda bazı sembol ve niceleyicilerin değili tablo hâlinde verilmiştir.
Sembol 7 6 0 / 1 # 2 $ = !
Değili 6 7 / 0 $ 2 # 1 ! =

ÖRNEK 4:

Aşağıda sembolik mantık diliyle verilen önermelerin değillerini yazınız.


a) p(x): “ 7x ! Z, 3x - 6 = 0 tir.”
b) q(n): “ 6n ! N, 2n çift doğal sayıdır.”

ÇÖZÜM:
a) p(x): “ 7x ! Z, 3x - 6 = 0 tir.” ise
pl ] x g / 6x ! Z, 3x - 6 Y
= 0 olur.
b) q(n): “ 6n ! N, 2n çift doğal sayıdır.” ise
ql]ng | “ 7n ! N, 2n çift doğal sayı değildir”.

ÖRNEK 5:

^ 7x ! N, 4x - 12 = 0 h 0 ^ 6x ! Z, x 2 $ 0 h bileşik önermesinin değilini bulunuz.

ÇÖZÜM:
p ve q önermeleri için ^p 0 q hl / pl / ql tir. (De Morgan kuralı)
p(x):“ 7x ! N, 4x - 12 = 0 ” ve q(x): “ 6x ! Z, x 2 $ 0 ” alınırsa

7^7x ! N, 4x - 12 = 0 h 0 ^6x ! Z, x 2 $ 0 hAl / ^7x ! N, 4x - 12 = 0 hl / ^ 6x ! Z, x 2 $ 0 hl


/ ^6x ! N, 4x - 12 Y= 0h / ^7x ! Z, x 2 1 0 h olur.

ÖRNEK 6:

^6x ! R, x 2 + 1 1 0h & ^7x, y ! Z, 2x - 3y = 0h bileşik önermesinin değilini bulunuz.

ÇÖZÜM:
p ve q önermeleri için ^p & q hl / ^pl 0 q hl / p / ql olduğundan
7^6x ! R, x 2 + 1 1 0 h & ^7x, y ! Z, 2x - 3y = 0 hAl
/ ^6x ! R, x 2 + 1 1 0h / ^6x, y ! Z, 2x - 3y Y

= 0 h olur.

34 | Matematik 9
MANTIK

3. Tanım, Aksiyom, Teorem ve İspat Kavramları


İnsanlar günlük hayatlarında, sosyal ve kültürel ilişkilerinde farklı cümleler kurarak iletişimini
gerçekleştirir. Bu iletişimin amacına ulaşması, kullanılan cümleler kadar seçilen kelimelerle de yakından
ilişkilidir. Bazen satırlar ya da paragraflar alabilen anlatımlar, uygun birkaç cümle ile daha kısa ve öz
şekilde aktarılabilir. Avukatlar ise uzun ve karmaşık sorunları, mesleki kelime ve cümlelerle daha kısa ve
öz olarak anlatabilirler.
Bilim dallarının günlük konuşma dilinden farklı, kendine özgü terimleri vardır. Bu terimler, bir bilim
dalına özgü sözcük ya da sözcük gruplarıdır. Örneğin roman, hikâye, sıfat, dörtlük vb. terimler edebî
terimlerdir. Denklem, eşitsizlik, rakam, açı, üçgen vb. terimler ise matematiksel terimlerdir.
Matematikte bir kavram ve özellik ifade edilirken belli terimler kullanılır. Bu terimler iki grupta
toplanır:
1) Tanımsız terimler
2) Tanımlı terimler
Tanımsız Terimler: Başka bir terim ya da tanıma ihtiyaç duyulmadan anlaşılabilen terimlerdir. Örneğin
nokta, doğru, düzlem tanımsız terimlerdir.
Tanımlı Terimler: Tanımlı ya da tanımsız terimler kullanılarak tanımlanmaya ihtiyaç duyulan
terimlerdir. Örneğin denklem terimi, tanımlı bir terimdir. Çünkü denklem “İçinde değişken bulunan ve
değişkene verilen bazı değerler için sağlanan eşitliktir.” şeklinde tanımlanır. Dikkat edilirse denklem
tanımı yapılırken “değişken, eşitlik” gibi farklı terimler de kullanılmıştır.
Tanım: Bir kavram ya da terimi, tanımlı tanımsız terimler kullanmak suretiyle özelliklerini
belirterek açıklamaya tanım (tanımlama) adı verilir. Üçgen “Doğrusal olmayan üç noktanın ikişer ikişer
birleştirilmesiyle elde edilen geometrik bir şekildir.” şeklinde tanımlanır. Bu tanımda tanımsız terimler
(nokta, doğru) kullanıldığına dikkat ediniz.
Aksiyom: İspata gerek duyulmaksızın doğruluğu kabul edilen önermelere aksiyom denir. Örneğin
“İki noktadan bir doğru geçer.” önermesi bir aksiyomdur.
Teorem: Doğruluğu ispatsız kabul görmeyen önermelere teorem denir. Örneğin
“Bir üçgenin dış açılarının ölçüleri toplamı 360c dir.” önermesi doğrudan kabul edilebilecek bir
önerme değildir. Belli terim, kavram ve tanımlamalar kullanılarak ispatlanma zorunluluğu vardır.
p ve q iki önerme olmak üzere
p önermesi doğru iken p & q koşullu önermesi doğru ise p & q önermesi bir teoremdir.
p & q teoreminde
p: Teoremin hipotezi (varsayım),
q: Teoremin hükmü (yargı) dır.

“Akıllı insan düşündüğü her şeyi söylemez


fakat söylediği her şeyi düşünür.”
Aristoteles

ÖRNEK 1:
“ABC üçgen ise iç açılarının ölçüleri toplamı 180c dir.” teoreminin hipotez ve hükmünü yazınız.

ÇÖZÜM:
Hipotez: “ABC üçgendir.”
Hüküm: “ABC üçgeninin iç açılarının ölçüleri toplamı 180c dir.”

Matematik 9 | 35
MANTIK

4. Mantık Kuralları ile Yapılan İspatlamalar

Bir teoremin hipotezi doğru iken tanım, aksiyom, teorem vb. matematiksel işlemler kullanarak
hükmünün de doğru olduğunu göstermek için yapılan işlemler bütününe teoremin ispatlanması denir.
Matematikte kullanılan ispat yöntemleri şema hâlinde Şekil: 9.1.1 de verilmiştir. İnceleyiniz.

İSPAT YÖNTEMLERİ

Tümdengelim Tümevarım

Doğrudan İspat Dolaylı İspat

Karşıt Ters Yöntemi Çelişki Aksine Örnek Verme


ile İspat Yöntemi ile İspat Yöntemi ile İspat

Şekil: 9.1.1

1. Doğrudan İspat
Bu yöntemde p & q önermesinin teorem olduğunu göstermek için p & q bileşik önermesinin hipotezi (p)
doğru kabul edilerek hükmün (q) de doğru olduğu gösterilir. Bu yönteme doğrudan ispat adı verilir.

ÖRNEK 1:
“m tek, n çift tam sayı ise m $ n çift tam sayıdır.”

ÇÖZÜM:
Hipotez(p): “m tek, n çift tam sayıdır.”
Hüküm(q): “ m $ n çift tam sayıdır.”
p & q : “(m tek, n çift tam sayıdır) & ( m $ n çift tam sayıdır.)” şeklinde yazılabilir.
İspat : m tek, n çift tam sayı olsun. O zaman k, t ! Z olmak üzere m = 2k + 1, n = 2t dir.
m $ n = ]2k + 1g $ 2t

m $ n = 4kt + 2t
= 2 ]2kt + tg k, t ! Z iken 2kt + t = h ! Z ve 2h çift olduğundan
14444244443
h
= 2h ]h ! Zg m $ n bir çift tam sayıdır.

36 | Matematik 9
MANTIK

ÖRNEK 2:
“ABC bir üçgen ise iç açılarının ölçüleri toplamı 180c dir.” teoremini doğrudan ispat yöntemi ile ispatlayınız.

ÇÖZÜM:
Hipotez (p):“ABC bir üçgendir.”
Hüküm (q): “ m ^W B h + m ^V
A h + m ^V C h = 180c dir. ”
W B h + m ^V
p & q : “(ABC bir üçgen) & ( m ^ A h + m ^V C h = 180c dir. )” şeklinde yazılabilir.
İspat:
ABC bir üçgen olsun. O zaman şekildeki gibi 5BC? kenarına A köşesinden d paraleli çizildiğinde

5BC? ' d


B h = m]EABg ve m ^V
m ^V C h = m]DACg (iç ters açılar)
% %

DAE doğru açı olduğundan m]DAEg = x + y + z = 180c olur. Buradan
%

m ^W B h + m ^V
A h + m ^V C h = x + y + z = 180c olduğu görülmüş olur.

2. Dolaylı İspat
a) Karşıt Ters Yöntemi ile İspat

Bu yöntemde p & q / ql & pl olduğundan p & q yerine ql & pl ispatlanır. Bu şekilde yapılan ispat
yöntemine karşıt ters yöntemi ile ispat denir.

ÖRNEK 3:
“ 6x ! R, ]3x - 5 = 4g & ]x = 3g tür. ” teoremini karşıt ters yöntemi ile ispatlayınız.

ÇÖZÜM:
Hipotez(p): “ 6x d R, 3x - 5 = 4 tür. ”
Hüküm(q): “ x = 3 tür. ” olmak üzere
“ p & q / “ ]3x - 5 = 4g & ]x = 3g tür. ”” yazılabilir.
İspat :
Karşıt ters yönteminde p & q / ql & pl olduğundan p & q yerine ql & pl ispatlanmalıdır.
ql & pl : ]x Y
= 3g & ]3x - 5 Y = 4g olduğu gösterilmelidir.
q' : x Y= 3 şeklindedir.
3x Y= 3$3 (3 ile genişletme)
= 9 - 5 ( - 5 ekleme)
3x - 5 Y
3x - 5 Y
=4

p' ifadesi elde edilir. Öyleyse ql & pl doğrudur. Bu durumda p & q önermesi ispatlanmış olur.

Matematik 9 | 37
MANTIK

ÖRNEK 4:
“Bir tam sayının karesi çift ise kendisi de çifttir.” teoremini karşıt ters yöntemi ile ispatlayınız.

ÇÖZÜM:
Hipotez(p): “ x 2 çifttir.”
Hüküm(q): “x çifttir.” olmak üzere
2
p & q : “( x çifttir.) & (x çifttir.)” yazılabilir.
İspat :
ql & pl : “x çift değilse x 2 çift değildir.” gösterilmelidir.
ql : “x çift değildir.” O zaman x tektir. k ! Z olmak üzere x = 2k + 1 yazılır.
pl : x 2 = ]2k + 1g2
= 4k 2 + 4k + 1
= 2 ^2k 2 + 2k h + 1
144444424444443 5
çift + tek $ tek & pl olur. ql & pl doğrudur.
Bu durumda p & q önermesinin doğruluğu ispatlanmış olur.

b) Çelişki Yöntemi ile İspat

Bu yöntemde verilen teoremin hükmünün değili alınarak çelişki elde edilmeye çalışılır.
p & q önermesinin doğru olduğunu ispatlamak yerine p & q önermesinin değili olan
^p & q hl / ^pl 0 q hl / p / ql önermesinin yanlış olduğu gösterilir. Bu yönteme çelişki yöntemi ile ispat
denir.

ÖRNEK 5:
“ 5 rasyonel sayı değildir.” önermesini çelişki yöntemi ile ispatlayınız.

ÇÖZÜM:
m
Hipotez: p: “ m, n ! Z ; m ve n aralarında asal sayılar, n Y
= 0, n ! Q ”
m
Hüküm: q: “ 5 Y = n ”
m m
p & q : “ m, n ! Z ; m ve n aralarında asal sayılar, n Y
= 0 olmak üzere n ! Q, = n dir. ” yazılır.
5Y

İspat : m, n ! Z ; m ve n aralarında asal sayılar, n Y


= 0 olmak üzere 5 rasyonel sayı olsun. O zaman
m şeklinde yazılabilir.
5= n

5 = n & ^ 5h = b n l & 5 = 2
m 2 m 2 m2

n
2 2 2
5n = m bulunur. Bu durumda m ve dolayısıyla m, 5 ile tam bölünür.
k ! Z olmak üzere m = 5k alınabilir. Bu değer yerine yazılırsa

5n 2 = ]5kg2 & 5n 2 = 25k 2


n 2 = 5k 2 bulunur. Benzer düşünce ile n 2 ve dolayısıyla n, 5 ile tam bölünür.

Sonuç olarak hem m hem de n, 5 ile bölünebildiğinden aralarında asal olamaz. Bu durum, m ve n nin
aralarında asal seçilmesiyle çelişir. O hâlde “ 5 rasyonel sayı değildir.” önermesi doğrudur.

38 | Matematik 9
MANTIK

ÖRNEK 6:
“ 6x ! R, ]x = 2g & ]3x - 5 Y
= 10g ” önermesini çelişki yöntemi ile ispatlayınız.

ÇÖZÜM:
p & q : “ x ! R, ]x = 2g & ]3x - 5 Y
= 10g ” olduğundan
^p & q hl / p / ql önermesinin yanlış olduğu gösterilmelidir.
p / ql : x = 2 ve 3x - 5 = 10 olur.
ql : 3x - 5 = 10 & 3x = 15
& x = 5 olur. Bu değer x = 2 ile çelişir. O hâlde p & q önermesi doğrudur.

c) Aksine Örnek Verme Yoluyla İspat

Bir önermenin ya da teoremin hükmünü sağlamayan en az bir karşı örnek vererek hükmün yanlış
olduğu söylendiğinde mantık ve cebirsel işleme ihtiyaç duyulmadan ispat yapılmış olur. Bu yöntemle
yapılan ispata aksine örnek verme yoluyla ispat denir.

ÖRNEK 7:
“2 ve 6 ile bölünebilen her doğal sayı, 12 ile de bölünür.” önermesinin yanlış olduğunu aksine örnek verme
yoluyla ispatlayınız.

ÇÖZÜM:
2 ve 6 ile bölünebilen doğal sayılar: 0, 6, 12, 18, 24, ... şeklinde yazılabilir. Bu sayılardan örneğin 18, 2 ve 6
ile bölünebildiği hâlde 12 ile bölünmez. O hâlde verilen önerme yanlıştır.

ÖRNEK 8:
“Bütün asal sayılar tektir.” önermesinin yanlış olduğunu aksine örnek verme yoluyla ispatlayınız.

ÇÖZÜM:
1 ve kendisinden başka böleni olmayan doğal sayıların asal sayı olduğu bilinmektedir. Bu sayılardan bazıları
2, 3, 5, 7, ... olarak yazılabilir. 2 asal sayı olduğu hâlde çift olup 1 ve kendisinden başka böleni yoktur. O hâlde
önerme yanlış bir önermedir. Bütün asal sayılar tek değildir.

5. Tümevarım Yöntemi

Matematiksel tümevarım, ardışık doğal sayılar ile ilgili “her” niceleyicisini içeren önermelerin ispatı
için kullanılır. Bunun için verilen önerme,
“ 6n d N +, p ^n h ” ise
1. p ^1h in doğru olduğu gösterilir.
2. k d N + olmak üzere p ^ k h & p ^k + 1 h önermesinin doğru olacağı gösterilir. Bunun için doğrudan
ispat yöntemi ile p ^ k h doğru kabul edilerek p ^k + 1h in de doğru olacağı gösterilir.
3. p ^n h önermesinin bütün n d N + için doğru olacağı gösterilmiş olur.
Yukarıda verilen işlemler kullanılarak yapılan ispat yöntemine tümevarım yöntemi ile ispat denir.

Matematik 9 | 39
MANTIK

ÖRNEK 1:
n $ ]n + 1 g
6n d N + için p ]ng: 1 + 2 + 3 + ... + n = 2 önermesini tümevarım yöntemiyle gösteriniz.

ÇÖZÜM:
1 $ ]1 + 1 g
n = 1 için p ]1g = 1 = 2 = 1 olduğundan p(1) doğrudur.
k $ ]k + 1 g
n = k için p ] k g: 1 + 2 + 3 + ... + k = 2 olsun.
n = k + 1 için p ]k + 1g önermesinin doğru olduğu gösterilmelidir.
k $ ]k + 1 g
1 + 2 + 3 + ... + k = 2 eşitliğinin her iki tarafına (k+1) eklenirse
k $ ]k + 1 g ]k + 1 g
1 + 2 + 3 + ... + k + ]k + 1g = 2 +
k $ ]k + 1 g + 2 ]k + 1 g
= 2
]k + 1 g $ ]k + 2 g
= 2 bulunur.

]k + 1g $ 6]k + 1g + 1@
p ]k + 1g = 1 + 2 + 3 + ... + k + ]k + 1g = 2 olur.
Buna göre p(k) doğru kabul edildiğinden p(k+1) de doğru olur.
n ]n + 1 g
O hâlde 6n d N + içinp ]ng: 1 + 2 + 3 + ... + n = 2 sonucuna ulaşılır.

ÖRNEK 2:
n $ ]n + 1g l2
6n d N + için p ]ng: 1 3 + 3 3 + 5 3 + ... + n 3 = b 2 önermesini tümevarım yöntemiyle gösteriniz.

ÇÖZÜM:
1 $ ]1 + 1 g l
n = 1 için p ]1g = 1 2 = 1 = b 2 = 1 olduğundan p(1) doğrudur.
k $ ]k + 1g l2
n = k içinp ]k g: 1 3 + 3 3 + 5 3 + ... + k 3 = b 2 doğru olsun.
n=k+1 için p(k+1) önermesinin doğru olduğu gösterilmelidir.
k $ ]k + 1g l2
1 3 + 3 3 + 5 3 + ... + k 3 = b 2 eşitliğinde n = k + 1 için ]k + 1g3 terimi eşitliğin her iki tarafına
eklenirse
k $ ]k + 1g l2 ]k + 1g3
1 3 + 3 3 + 5 3 + ... + k 3 + ]k + 1g3 = b 2 +

k 2 ]k + 1g2 ]k + 1g3
= 4 +
k 2 ]k + 1g2 + 4 ]k + 1g3
= 4
k k + 1 + 4 ]k + 1g2 ]k + 1g
2
] g2
= 4
]k + 1g 6k + 4 ]k + 1g@ ]k + 1g2 ^k 2 + 4k + 4 h ]k + 1g2 ]k + 2g2
2 2
= 4 = 4 = 4
]k + 1g]k + 2g
=b l bulunur. Bu durumda
2

2
]k + 1g]k + 2g 2
p ]k + 1g = 1 3 + 2 3 + 3 3 + ... + k 3 + ]k + 1g3 = b 2
l ve p ]k g doğru kabul edildiğinden p(k+1)
de doğru olur.
n ]n + 1g l2
O hâlde 6n d N + içinp ]ng: 1 3 + 2 3 + 3 3 + ... + n 3 = b 2 önermesi doğrudur.

40 | Matematik 9
MANTIK

ÖRNEK 3:

6n d N + için p ]ng: 1 + 3 + 5 + ... + ]2n - 1g = n 2 önermesini tümevarım yöntemiyle gösteriniz.

ÇÖZÜM:
n = 1 için p ]1g = ]2 $ 1 - 1g = 1 = 1 2 = 1 olduğundan p(1) doğrudur.
n = k için p ] k g: 1 + 3 + 5 + ... + ]2k - 1g = k 2 doğru olsun.
n = k + 1 için p ]k + 1g önermesinin doğru olduğu gösterilmelidir.
1 + 3 + 5 + ... ]2k - 1g = k 2 eşitliğinde n = k + 1 için 2n - 1 = 2 ]k + 1g - 1 = 2k + 1 terimi eşitliğin her iki
tarafına eklenirse
1 + 3 + 5 + ... ]2k - 1g + ]2k + 1g = k 2 + ]2k + 1g

= ]k + 1g2 bulunur.
Bu durumda 1 + 3 + 5 + ... ]2k - 1g + ]2k + 1g = ]k + 1g2 ve p ]k g doğru kabul edildiğinden p(k+1) de
doğru olur.
O hâlde 6n d N + için p ]ng: 1 + 3 + 5 + ... ]2n - 1g = n 2 önermesi doğrudur.

ÖRNEK 4:

6n d N + için p ]ng: 1 $ 2 + 2 $ 3 + 3 $ 4 + ... + ]n + 1g = n + 1 önermesini tümevarım yöntemiyle


1 1 1 1 n
n$
gösteriniz.

ÇÖZÜM:

n = 1 için p ]1g =
1 1 1 1
1 $ ]1 + 1 g 2 1 + 1 2
= = = olduğundan p(1) doğrudur.

n = k için p ] k g: 1 $ 2 + 2 $ 3 + 3 $ 4 + ... + ]k + 1g = k + 1 doğru olsun.


1 1 1 1 k

k$

n = k + 1 için p ]k + 1g önermesinin doğru olduğu gösterilmelidir.

1 1 1 1 k
1 $ 2 + 2 $ 3 + 3 $ 4 + ... + k $ ]k + 1g = k + 1 eşitliğinde

1 1
]k + 1g6]k + 1g + 1@ ]k + 1g $ ]k + 2g
n = k + 1 için = terimi eşitliğin her iki tarafına eklenirse

1 1 1 1 1 k 1
1 $ 2 + 2 $ 3 + 3 $ 4 + ... + k $ ]k + 1g + ]k + 1g $ ]k + 2g = k + 1 + ]k + 1g $ ]k + 2g

k ]k + 2 g + 1 k 2 + 2k + 1 ]k + 1g2 k+1 ]k + 1 g
= ]k + 1 g ]k + 2g = ]k + 1 g ]k + 2 g = ]k + 1 g ]k + 2 g = k + 2 = ]k + 1 g bulunur. Bu durumda
$ $ $ +1
]k + 1 g
p ]k + 1g = 1 $ 2 + 2 $ 3 + 3 $ 4 + ... + ]k + 1g + ]k + 1g ]k + 2g = ]k + 1g ve p ]k g
1 1 1 1 1

k$ + +1
doğru kabul edildiğinden p(k+1) de doğru olur.

O hâlde 6n d N + içinp ]ng: 1 $ 2 + 2 $ 3 + 3 $ 4 + ... + ]n + 1g = n + 1 önermesi doğrudur.


1 1 1 1 n

n$

Matematik 9 | 41
MANTIK

ALIŞTIRMALAR

1 Aşağıda verilen önermeleri sembolik mantık diliyle ifade ederek önermelerin doğruluk değerlerini
bulunuz.
a) p: “Bazı x tam sayıları için 3x - 7 = 11 tir.”
b) q: “Bütün x tam sayıları için x 2 = 36 tir.”
c) r: “Her rasyonel sayı kesirli yazılamaz.”

2 Aşağıda sembolik mantık diliyle verilen önermeleri sözel olarak ifade ediniz.

Sembolik İfade Sözel İfade


p(x): “ 7x ! R, x 1 4 & x 2 2 16 ”

q(x): “ 6x ! R, ]x - 2g2 $ 0 ”

r(x): “ 7x ! Z, x 2 1 8 & x - 1 2 0 ”

s(x): “ 6x ! R, 2 -x 2 0 ”

3 Aşağıdaki önermelerin değillerini yanlarındaki boşluğa yazınız.

Önerme Değili
p(x): “ 7x ! R, x - 1 $ 0 ”
q(x): “ ^7x ! R, 4x + 3 =- 5h & ]x =- 2g ”
r(x): “ ^7x ! R, x 3 1 0h / ^6x ! R, x 2 $ 0h ”
s(x): “ ^6x ! N, x $ 0h 0 ^7x ! N, 2 - x 1 0h ”

4 Aşağıdaki açık önermelerin doğruluk kümelerini bulunuz.


a) p(x): “ 2x + 3 1 11, x ! N ”
b) q(x): “ x 2 - 1 = 0, x ! Z - ”
c) r(x,y): “ x 2 + y 2 = 1, x ! Z, y ! Z ”

5
Aşağıdaki önermelerin hipotezini ve hükmünü yazınız.
a) “ 5x - 7 = 18 ise x - 1 = 4 tir. ”
b) “ABCD dörtgeni bir kare ise her bir açısının ölçüsü 90c dir.”

6 “Bir ABC üçgeninin dış açılarının ölçüleri toplamı 360c dir.” teoremini doğrudan ispat yöntemi ile
ispatlayınız.

42 | Matematik 9
MANTIK

7 “ 6x ! R, ]x Y
= 5g & ]- 4x + 13 Y=- 7g ” +
10 6n ! N için
önermesini çelişki yöntemi ile ispatlayınız.
a) p ]ng: 2 + 4 + 6 + ... + 2n = n $ ]n + 1g

b) p ]ng: 1 2 + 2 2 + 3 2 + ... + n 2
n $ ]n + 1g]2n + 1g
= 6
önermelerini tümevarım yöntemiyle ayrı ayrı
ispatlayınız.

8 “ 3 rasyonel sayı değildir.” önermesini çelişki 11 2 - 3 sayısı rasyonel değildir." önermesini


yöntemi ile ispatlayınız. çelişki yöntemi ile ispatlayınız.

9 “3 ve 15 ile bölünebilen her doğal sayı, 45 ile de 12 n ! N olmak üzere


+

p ]ng: 1 + x + x 2 + x 3 + ... + x n - 1 = 1 - x
bölünür.” önermesinin yanlış olduğunu aksine 1 - xn
örnek verme yöntemi ile ispatlayınız.
önermesini tümevarım yöntemi ile ispatlayınız.

13 Ayşe, Burak, Canan, Emre, Filiz ve Derya isimli altı öğrencinin üniversite sınav sonuçlarına göre yaptıkları
tercih listelerinde tıp, mühendislik, mimarlık, eczacılık ve hukuk bölümleri bulunmaktadır.
• Her iki öğrenci iki farklı bölüm tercih etmiştir.
• 4 kişi tıp, 3 kişi mühendislik, 2 kişi mimarlık, 2 kişi eczacılık ve 1 kişi hukuk bölümünü tercih etmiştir.
• Ayşe, tıp ve eczacılık bölümlerini tercih etmiştir.
• Filiz ve Emre, farklı bölümleri tercih etmiş ancak Derya ve Canan aynı bölümleri tercih etmişlerdir.
• Hukuk bölümünü tercih eden kızlardan bir kişidir ve bu kişi aynı zamanda mimarlık bölümünü de
tercih etmiştir.
Yukarıda verilen bilgilere göre aşağıdaki önermelerin doğruluk değerlerini tabloya yazınız.

Önerme Doğruluk Değerleri


Filiz, hukuk bölümünü tercih etmiştir.

Canan, tıp ve mühendislik bölümlerini tercih etmiştir.

Burak, mimarlık bölümünü tercih etmemiştir.

Burak ve Filiz, eczacılık bölümünü tercih etmiştir.

Emre veya Burak, tıp bölümünü tercih etmiştir.


Burak, mühendislik bölümünü tercih etmiş ise Emre, eczacılık bölümünü
tercih etmiştir.
Emre, hukuk bölümünü tercih etmiştir veya Canan, mimarlık bölümünü
tercih etmiştir.

Matematik 9 | 43
MANTIK

ÖLÇME VE DEĞERLENDİRME

1
Aşağıdaki terimlerden kaç tanesi tanımlı 4 6^0 & p h & ^p & plh@ 0 p önermesinin en sade
terimdir? şekli aşağıdakilerden hangisidir?
I. Nokta A) p B) p & pl C) 1
II. Doğru
D) pl 0 q E) 0
III. Açı
IV. Uzay
V. Küme
A) 1 B) 2 C) 3 D) 4 E) 5

2
Aşağıdakilerden hangisi önermedir? 5
p / 0, q / 1, r / 1 olduğuna göre
aşağıdakilerden hangisi yanlıştır?
A) Kapıyı kapat!
B) İki doğal sayı alıp toplayınız. A) pl / ^q & r h / 1
C) 12 sayısı, 5 ile tam bölünür mü? B) p / ^q 0 r h / 0
D) En küçük asal sayı 3 tür. C) p & ^ql & r h / 1
E) Günaydın! D) p Q ^q & rlh / 0
E) ^pl & q h 0 rl / 0

3
“20 sayısı, 6 ile tam bölünemez.” önermesinin
6 ^p + q hl önermesi aşağıdaki önermelerden
değili aşağıdakilerden hangisidir? hangisine denk değildir?
A) 20 sayısı, 6 ile tam bölünür.
A) ^pl Q q hl
B) 20 sayısı, 6 ya kalanlı bölünür.
B) p + ql
C) 6 sayısı, 20 yi bölmez.
C) p Q q
D) 20 sayısı, 5 ile tam bölünür.
D) ^p & q h / ql
E) ^p / qlh 0 ^q / plh
E) 20 sayısı, 6 nın katı değildir.

44 | Matematik 9
MANTIK

7 pl & q / 1 ve q 0 r / 0 olduğuna göre 10 Aşağıdakilerden hangisi ^pl / q h 0 ql bileşik


sırasıyla p, q ve r nin doğruluk değerleri önermesinin değilidir?
aşağıdakilerden hangisidir?
A) ^p 0 qlh 0 q
A) (1, 0, 0) B) (1, 1, 0) C) (0, 1, 1) B) ^p / qlh / q
D) (1, 1, 1) E) (0, 1, 0) C) ^p 0 qlh / q
D) ^pl 0 q h 0 ql
E) ^pl / q h 0 q

8 ^p & q h 0 ^rl & q h / 0 olduğuna göre 11 ^pl & q h & r önermesinin karşıt tersi
aşağıdakilerden hangisinin doğruluk değeri 1 aşağıdakilerden hangisidir?
dir?
A) r & ^pl & q hl
A) pl / r B) ^pl & q hl & rl
B) ^p & qlh + r C) r 0 ^p / qlh
C) ^p 0 q h / r D) r 0 ^pl / qlh
D) ^p + q h & r E) r / ^p 0 qlh
E) ^p 0 r h / ^p & q h

9
Aşağıdaki önermelerden hangisi totolojidir? “Bir hayvan, aslansa dört ayaklıdır.” koşullu
12
önermesinin tersi aşağıdakilerden hangisidir?
A) p + pl B) p Q pl C) p / 0
D) 1 & p E) p & pl A) “Bir hayvan dört ayaklı değilse aslan
değildir.”
B) “Bir hayvan dört ayaklı ise aslandır.”
C) “Bir hayvan dört ayaklı değilse aslandır.”
D) “Bir hayvan aslansa dört ayaklı değildir.”
E) “Bir hayvan aslan değilse dört ayaklı
değildir.”

Matematik 9 | 45
MANTIK

13
p 16
p(x): “4 ten küçük bazı reel sayıların karesi,
q 16 dan büyüktür.” açık önermesinin sembolik
mantık dilinde yazılışı aşağıdakilerden
r hangisidir?
m s
A) p(x): “ 6x ! R, x 1 4 & x 2 2 16 ”
n
B) p(x): “ 7x ! R, x 1 4 & x 2 2 16 ”
C) p(x): “ 6x ! R, x 2 4 & x 2 1 16 ”
Yukarıda verilen elektrik devresine karşı gelen
bileşik önerme aşağıdakilerden hangisidir? D) p(x): “ 6x ! R, x 2 4 & x 2 2 16 ”

A) m / 6^p / r / s hl 0 q@ / n
E) p(x): “ 7x ! R, x 2 4 & x 2 2 16 ”

B) m / 6^pl / q / rlh 0 sl@ / n


C) m 0 6^pl 0 q 0 rlh / sl@ / n
D) m 0 6^pl 0 q 0 rlh 0 sl@ 0 n
E) m / 6^pl 0 q 0 r h / sl@ / n

14
x bir tam sayı olmak üzere aşağıdaki açık 17 ^6x ! R, x 2 + 2 2 0h 0 ^7x, y ! R, x + y $ 3h
önermelerden hangisi doğrudur? açık önermesinin değili aşağıdakilerden
A) 6x, 1 - x 2 x 2 hangisidir?
B) 6x, 1 + x 1 0 A) ^7x ! R, x 2 + 2 2 0 h / ^6x, y ! R, x + y 1 3 h
C) 7x, x - x 2 # 0 B) ^6x ! R, x 2 + 2 1 0 h / ^7x, y ! R, x + y $ 3 h
C) ^6x ! R, x 2 + 2 # 0 h 0 ^6x, y ! R, x + y 1 3 h
x
D) 7x, x - 1 = 1
E) 6x, ]- xg3 = x 3 D) ^7x ! R, x 2 + 2 1 0 h 0 ^7x, y ! R, x + y $ 3 h
E) ^7x ! R, x 2 + 2 # 0 h / ^6x, y ! R, x + y 1 3 h

15 p:“ x, y ! Z için x $ y = 10 ” açık önermesinin 18 x, y ! Z olmak üzere x + y # 6 açık


doğruluk kümesinin eleman sayısı kaçtır? önermesi için kaç farklı ^x, y h ikilisi vardır?
A)6 B)8 C)10 D)12 E)16 A) 85 B) 80 C) 72 D) 63 E) 56

46 | Matematik 9
KÜMELER

SAYILAR VE CEBİR
9.2. KÜMELER
Neler Öğreneceksiniz? Küme Kavramları ve
Uygulamalarını Öğrenmek Neden
Önemlidir?
• Küme kavramını ve kümenin farklı • İnsanoğlu yaşamının her anında hayatını
gösterimlerini, kümeler ve elemanlarının kolaylaştırmak ve yaptığı işlerde zaman
sembollerle nasıl ifade edildiğini, kazanmak ister. Bu amaçla sahip olduğu
• Boş küme, evrensel küme, sonlu sonsuz bilgileri veya etrafındaki varlıkları sayma,
küme, alt küme ile eşit ve denk küme sıralama, listeleme, gruplandırma ve
kavramlarını, düzenleme ihtiyacı duyar. Kısacası
insanoğlu bilerek veya farkına varmadan
• Kümelerde birleşim, kesişim, fark ve
nesneleri zihninde kümeleştirmektedir.
tümleme işlemlerini; ayrık kümeyi, De
Morgan kurallarını ve uygulamalarını, • Kümeler, günlük hayatın her aşamasında
görülebilir. Nesnelerle ilgili yapılan
• Kümelerde yapılan işlemler ile sembolik
gruplandırma ya da sınıflandırmalara
mantıkta kullanılan sembol ve bunları
market, mağaza vb. reyonları, evin
ifade eden işlemler arasındaki ilişkiyi,
odaları ve eşyaları buna örnek olarak
• Küme işlemlerini kullanarak günlük gösterilebilir.
hayata ilişkin problemlerin nasıl
• Matematikte de birçok konu küme
çözülebildiğini,
kavramı ve işlemleri üzerine kurulmuştur.
• Sıralı ikili ya da sıralı n li kavramlarını ve Bu sayede eldeki bilgi ve veriler
uygulamalarını, düzenlenerek bunlar üzerinde istatistiksel
• İki ya da daha fazla kümenin kartezyen işlemler yapılmaktadır. Örneğin
çarpımı ve özelliklerini, kartezyen çarpım istatistikte verileri gruplara ayrıldıktan
kümesinin düzlemde nasıl gösterildiğini, sonra veriler üzerinde çalışma yapılır ve
• Kartezyen çarpım kümesinin alt kümesi grup hakkında tutarlı sonuçlara ulaşılır.
olarak bağıntı ve ters bağıntı kavramlarını • Kümelerde işlemler bölümü, cebirsel bir
ve uygulamalarını öğreneceksiniz. yapı oluşturulmasını sağlar. Bu işlemler
ve onların özellikleri sayesinde, kümeler
konusu birçok uygulama alanına sahip
olup günlük yaşam problemlerinin
çözümlerinde etkin bir biçimde kullanılır.

Matematik 9 | 47
KÜMELER

(Georg Kantor 1845-1918)

O
n dokuzuncu yüzyıla kadar matematikteki işlemlerde bir
birliktelik yoktu. Bu durum, işlemlerin daha düzenli ve kolay
yapılmasını engellemekteydi.
Matematikte birliktelik sağlama ihtiyacı 19. yüzyılın sonlarına doğru
ortaya çıkmıştır. Böylece bu birlikteliği sağlamaya yarayan, doğada her
zaman var olan ve uzun yıllar boyunca kullanılmakta olan kümelerin;
matematik terimi olarak tanımı 19. yüzyılda yapılmıştır.
Dönemin Alman matematikçisi Georg Cantor (Georg Kantor 1845-
1918); bütün matematik araştırmalarında ve problemlerinde ele
alınan nesnelerin kendi aralarında belirli birtakım özelliklere göre
gruplanabileceğini, bu durumda anlaşılırlığın ve çözüme yönelik işlem
yapmanın daha da kolay olacağını fark etmiştir.
1847’de Bolzeno’nun (Bulzano) oluşturduğu küme ve sonsuz küme
kavramları, 1867-1871 yılları arasında Georg Cantor’un (Georg Kantor)
çalışmalarıyla geliştirildi. 1874’te yayımlanan Cantor’un makalesi,
kümeler konusunun ortaya çıkışında önemli bir adım olarak görülür.
Cantor’un, sonsuz kavramı üzerine gerçekleştirdiği çalışmalar, sayılar
kuramında karşılaşılan sorunlarla ilişkilendirilerek kümelerin temelinin
atılmasını sağlamıştır.
Günümüze kadar geliştirilen kümeler konusu; mühendislikte,
iktisatta, yapay zekâ çalışmalarında ve bilim felsefesi gibi dallarda geniş
bir uygulama alanı bulmaktadır.
Evde, okulda, sokakta düşünülenlerin ve yaşananların her biri birer
küme oluşturur. Bir mağazaya gidildiğinde eşyaların kümelenmiş olarak
yerleştirilmesi istenilen eşyaya ulaşma açısından kolaylık sağlamaktadır.
Kümeleri bir karmaşıklığın içerisindeki düzen olarak düşünebilirsiniz.

48 | Matematik 9
KÜMELER

9.2.1. KÜMELERDE TEMEL KAVRAMLAR

Etkinlik

Sınıflar arası bilgi yarışması için 9/A sınıf öğretmeni; matematik, edebiyat ve fizik
öğretmenlerinden en az üç başarılı öğrenci ismi vermelerini istiyor.
• Matematik öğretmeni : Duru, Ahmet, Umut, Arzu
• Edebiyat öğretmeni : Mustafa, Duru, Umut, Serap
• Fizik öğretmeni : Batu, Arzu, Umut, Duru, Hülya’yı seçiyor.
Sizce matematik, edebiyat ve fizik öğretmenlerinin farklı isimler vermelerinin nedeni ne olabilir?
Başarı kriteri verilmiş olsaydı, önerilen isimler yine farklı mı olurdu?
Bir gruba ait olan ya da olmayan nesnelerin iyi tanımlanmış olmasından ne anlarsınız?
Matematiksel açıdan kümeyi oluşturan ögelerin iyi tanımlanması gerektiği akla gelen ilk husustur.
İyi tanımlama topluluğa neyin dâhil edilip edilmeyeceğinin açık olarak belirtilmesi, kişilere göre
seçeneklerin değişiklik göstermemesidir.

Matematik 9 | 49
KÜMELER

1. Temel Kavramlar
Küme Kavramı
İyi tanımlanmış, birbirinden farklı nesneler topluluğuna (yığınına) küme denir. Buradaki nesne
soyut ya da somut olabilir.

ÖRNEK 1:
Aşağıdaki ifadelerden hangisi veya hangileri bir küme belirtir? Nedeniyle açıklayınız.
I. Bazı çiçekler
II. 5 ten küçük doğal sayılar
III. En güzel aylar
IV. Okulumuzdaki yeşil gözlü öğrenciler
V. Türkiye’nin coğrafi bölgeleri

ÇÖZÜM:
I ve III. maddelerde verilen ifadeler, kişiye göre farklı gruplar oluşturabileceği için küme belirtmez. II, IV ve
V. maddeler iyi tanımlandığı için birer kümedir.

Kümeyi oluşturan nesnelerin veya sembollerin her birine kümenin elemanları adı verilir. Kümeler
genellikle büyük harflerle gösterilir.
a, A kümesinin elemanı ise bu durum a ! A ile gösterilir ve a elemanıdır A diye okunur. a, A
kümesinin elemanı değil ise a z A şeklinde gösterilir ve bu durum a elemanı değildir A diye okunur.

ÖRNEK 2:
A kümesi, 3 ten küçük doğal sayılar olsun. A kümesinin elemanlarını ve bu kümeye ait olmayan üç doğal
sayıyı belirtiniz.

ÇÖZÜM:
A kümesi, 3 ten küçük doğal sayılar şeklinde verildiğinden 0, 1, 2 A kümesinin elemanlarıdır. Bu durum
0 ! A, 1 ! A, 2 ! A şeklinde gösterilir. 3 ve 3 ten büyük doğal sayılar, A kümesinin elemanı olmadığı için
örneğin 3 z A, 4 z A, 5 z A şeklinde gösterilir.

Kümelerin Gösterilişi
Kümeler, yaygın olarak üç farklı yolla gösterilir:
I) Liste Yöntemi ile Gösterim
Kümeyi oluşturan bütün elemanların " , parantezinin içerisinde aralarına virgül konularak
gösterilmesidir.
II) Venn Şeması Yöntemi ile Gösterim
Kümeyi oluşturan bütün elemanların kapalı bir eğri içerisinde önüne konularak gösterilmesidir.
III) Ortak Özellik Yöntemi ile Gösterim
Genellikle eleman sayıları çok olan kümelerin gösterilmesinde kullanılır. Kümenin bütün
elemanlarının sahip olduğu ortak özelliğin matematiksel veya sözel bir ifade ile gösterilmesidir.
A = " x ; x in özelliği ,
Burada x ; ifadesi x lerden oluşuyor öyle ki diye okunur.

50 | Matematik 9
KÜMELER

ÖRNEK 3:
2, 4, 6, 8 elemanlarından oluşan kümeyi belirtilen üç farklı yöntemi kullanarak gösteriniz.

ÇÖZÜM:

I. Verilen küme C ise liste yöntemi ile gösterimi C = " 2, 4, 6, 8 , olur.


II. C
2

4 C kümesinin venn şeması yöntemi ile gösterimi olur.


6

III. C kümesinin ortak özellik yöntemiyle gösterimi:


C = " x ; 1 ile 10 arasındaki bir çift doğal sayıdır , = " x ; 1 1 x 1 10, x = 2k, k ! N + , şeklinde yazılır.

ÖRNEK 4:
D = " x ; x # 15 ve x bir asal sayıdır , kümesini liste yöntemiyle yazınız.

ÇÖZÜM:
1 ve kendisi dışında böleni olmayan 1 den büyük sayılar, asal sayı olarak tanımlandığından D kümesinin
liste şeklinde yazılışı D = " 2, 3, 5, 7, 11, 13 , olur.

ÖRNEK 5:
" 0, 4, 8,12,16, 20 , kümesinin ortak özellik yöntemiyle gösteriminin aşağıdakilerden hangisi olduğunu
belirtiniz.
A) " x ; 0 # x # 22, x = 2k,k ! N ,
B) " x ; 4 # x 1 22, x = 4k,k ! N ,
C) " x ; 0 # x # 21, x = 3k,k ! N ,
D) " x ; - 2 # x # 23, x = 4k,k ! N ,
E) " x ; 0 # x # 30, x = 5k,k ! N ,

ÇÖZÜM:
" 0, 4, 8, 12, 16, 20 , kümesinin elemanları 4 ün katı olan doğal sayılardır. Bu sayıların en küçüğü 0, en
büyüğü 20 dir. Bu şartları sağlayan seçenek D dir.

n ! N olmak üzere bir A kümesinin eleman sayısı s ^A h = n olarak ifade edilir.

ÖRNEK 6:
B = " 2, a,1, " 3,4 ,, ! c +,b , kümesinin eleman sayısını bulunuz.

ÇÖZÜM:
B = " 2, a,1, " 3, 4 ,, ! c +,b ,_bb
bb olduğundan s ]Bg = 6 dır. Görüldüğü gibi bir küme başka bir kümenin
elemanı olabilir. Örneğin; " 3, 4 ,, " c ,
`b
bb
1 23 4 5 6 b
a

Matematik 9 | 51
KÜMELER

ÖRNEK 7:
A = # k ; 9 1 x 1 65, x = k 2,k ! N - kümesinin eleman sayısını bulunuz.

ÇÖZÜM:
Verilen ifadeye göre kümenin elemanları, karesi 9 ile 65 arasındaki doğal sayılardır.
Bu elemanlar aşağıdaki sayılar olduğundan A kümesinin elemanları:
4 2 = 16 _bb
bb
5 2 = 25 bb
bb
6 2 = 36 `b A = " 4, 5, 6, 7, 8 , olur. s ^A h = 5 bulunur.
bb
7 2 = 49 bb
bb
8 2 = 64 b
a

ÖRNEK 8:
D = # x ; x # 99, x = 3k,k ! Z + - kümesinin eleman sayısını bulunuz.

ÇÖZÜM:
D kümesinin elemanları 3 ün katı ve en çok 99 olan pozitif tam sayılardır. Buna göre, D kümesinin
elemanları D = " 3, 6, 9,... , 96, 99 , olur.
Ardışık terimleri arasındaki farkı sabit olan bu tür kümelerin eleman sayısı:

s ^Dh =
son terim - ilk terim 99 - 3 96
+ 1 = 3 + 1 = 3 + 1 = 33 olur.
artış miktarı

Boş Küme
Elemanı olmayan kümeye boş küme denir. Boş kümenin gösterimi Q olup liste yöntemi ile
gösterimi " , biçimindedir. Boş kümenin eleman sayısı sıfırdır.

ÖRNEK 9:
Aşağıdaki kümelerin eleman sayılarını bulunuz.
“ A = " x ; x , karesi kendisinden küçük olan bir doğal sayıdır , ”
“ B = " x ; x , negatif bir asal sayıdır , ”
“ C = " x ; x 2 + 4 = 0, x ! R , ”

ÇÖZÜM:
• x 2 1 x eşitsizliğini sağlayan herhangi bir doğal sayı yoktur. Bu nedenle A = " , tir.
• Bir ve kendisinden başka böleni olmayan pozitif doğal sayılara asal sayı denir. Bu nedenle negatif asal
sayı yoktur. B = " ,
• x 2 + 4 = 0
x 2 =- 4 denklemini sağlayan gerçek sayı yoktur. 6x d R ise x 2 $ 0 dır. Bu nedenle C = " ,

" Q , ve "" ,, kümeleri, birer elemanlı kümeler olduğu için boş küme değildir.

52 | Matematik 9
KÜMELER

ÖRNEK 10:
Aşağıdaki seçeneklerde gösterilen kümelerden hangisinin boş küme olmadığını nedeni ile açıklayınız.

• A = " x ; x 1 - 8, x bir doğal sayıdır ,


• B = " x ; x 2 + 1 = 0 ve x bir gerçek sayıdır ,
• C = " x ; 1 1 x 1 3, x bir gerçek sayıdır ,
• D = " x ; 4 1 x 1 5, x bir tam sayıdır ,

ÇÖZÜM:
A kümesinde - 8 den küçük doğal sayı, B kümesinde karesi negatif olan gerçek sayı, D kümesinde 4 ile 5
arasında olan bir tam sayı olmadığından bu kümelerin elemanı yoktur. Bu yüzden A, B, D kümeleri birer boş
kümedir.
1 ile 3 arasında en az bir gerçek sayı bulunduğundan C kümesi boş küme değildir.

Sonlu ve Sonsuz Kümeler


Eleman sayıları bir doğal sayı ile gösterilebilen kümelere sonlu küme, sonlu olmayan kümelere de
sonsuz küme denir.
A = " x ; 0 1 x 1 10, x ! Z , kümesinin elemanları 0 ile 10 arasındaki tam sayılardır.
Bu sayılar " 1, 2, 3, 4, 5, 6, 7, 8, 9 , şeklinde ifade edilebilir ve s (A) = 9 olur. Bu durumda A kümesi sonlu
sayıda elemana sahiptir denir.
Doğal sayılar kümesi incelendiğinde 6n ! N için n + 1 ! N olduğundan kümenin eleman sayısı
bir doğal sayı ile ifade edilemez. Bu durumda doğal sayılar kümesi sonsuz kümedir.

ÖRNEK 11:
Aşağıdaki ifadeleri sonlu ve sonsuz küme olarak sınıflandırınız.

A = " x ; x 1 15, x pozitif tek tam sayıdır ,


B = " x ; x, çift doğal sayılar,
C = " x ; x, asal rakamlar ,
D = " x ; - 2 < x 1 11, x rasyonel sayı ,

ÇÖZÜM:
A = " 1, 3, 5, 7, 9, 11, 13 ,
B = " 0, 2, 4, ... ,
C = " 2, 3, 5, 7 ,
D kümesinde - 2 ile 11 arasında sonsuz sayıda rasyonel sayı bulunmaktadır.
Buna göre A ve C kümeleri, sonlu sayıda eleman bulundurduğundan sonlu kümeler olur. B ve D kümeleri
ise sonsuz sayıda eleman bulundurduğundan sonsuz kümeler olur.

Matematik 9 | 53
KÜMELER

2. Alt Küme
İnsanoğlunun çevresinde bulunan nesneler gruplara ayrılarak sınıflandırılır. Oluşan gruplardan
bazıları diğer grupları kapsar ya da onların içinde yer alır. Örneğin balıklar, omurgalıların içinde yer alırken
omurgalılar da hayvanlar topluluğunun içinde yer alır. Hayvanlar da canlılar âleminin bir parçasıdır.
Bu duruma benzer olarak bir alışveriş merkezinin market, ayakkabı, giyim ve yiyecek gibi farklı
bölümlerden oluştuğu hatta bu bölümlerin de daha küçük alt bölümlere ayrılarak gruplandığı da görülebilir.
Yukarıdaki bölümleri ve bunların alt bölümlerini birbirinin içinde yer alma ve kapsama açısından nasıl ifade
edebilirsiniz?

A ve B herhangi iki küme olmak üzere A kümesinin her elemanı


aynı zamanda B kümesinin de elemanı oluyor ise A kümesine, B
kümesinin alt kümesi denir. A 3 B biçiminde gösterilir. B
Bu ifade, B kümesi A kümesini kapsar şeklinde de söylenir. B 4 A A
biçiminde gösterilir.
Alt kümenin venn şemasıyla gösterilişi yandaki gibidir.
A kümesinin en az bir elemanı, B kümesinin elemanı değil ise A
kümesi B kümesinin alt kümesi değildir denir ve A M B biçiminde
gösterilir.

ÖRNEK 1:
A = " 1, 2 ,, B = " 1, 2, 3 ,, C = " 1, 2, 3, 5, 7 , kümeleri arasındaki kapsama ilişkisini inceleyiniz.

ÇÖZÜM:
A kümesinin elemanlarının tamamı hem B hem de C kümesinde yer almaktadır. B kümesinin elemanları
da C kümesinde yer almaktadır.
Bu bilgiler doğrultusunda örnekteki üç küme şu şekilde ilişkilendirilebilir:
A 3 B , A 3 C ve B 3 C dir ya da B 4 A, C 4 A ve C 4 B yazılabilir.
Başka bir ifadeyle: A 3 B 3 C veya C 4 B 4 A yazılabilir.

ÖRNEK 2:
A = " x ; 0 1 x 1 6, x bir doğal sayıdır ,
B = " 1, 3, 5 ,
Kümeleri arasındaki alt küme ilişkisini belirtiniz.

ÇÖZÜM:
A
A = " 1, 2, 3, 4, 5 , B
4 2 1

B = " 1, 3, 5 , 3
Şekilde gösterildiği gibi B kümesinin bütün elemanları A kümesinin 4
5
de elemanı olduğu için B kümesi, A kümesinin alt kümesidir. Bu durum,
B 3 A veya A 4 B şeklinde gösterilir.

54 | Matematik 9
KÜMELER

Alt Kümenin Özellikleri


1. Her küme, kendisinin alt kümesidir. ^A 3 A h
2. Boş küme, her kümenin alt kümesidir. ^Q 3 A h
3. ^A 3 Bh / ^B 3 C h & A 3 C dir.

ÖRNEK 3:
A = " ,, B = ! a +, C = " a,b ,, D = " a,b, c , kümelerinin alt kümelerini yazınız.

ÇÖZÜM:
Alt küme ilişkisini daha iyi görebilmek için aşağıdaki tabloyu inceleyiniz.

Küme Kümenin Eleman Sayısı Alt Kümeleri Alt Küme Sayısı

A=" , s ^Ah = 0 Q 1 = 2 0 = 2 s(A)


B = !a + s ^Bh = 1 Q, ! a + 2 = 2 1 = 2 s(B)

C = " a, b , s ^Ch = 2 Q, ! a +, ! b +, " a, b , 4 = 2 2 = 2 s(C)

D = " a, b, c , s ^ D h = 3 Q, ! a +, ! b +, ! c +, " a, b ,, " a, c ,, " b, c ,, " a, b, c , 8 = 2 3 = 2 s(D)

Tablodaki kümeler, eleman sayıları ve alt küme sayıları arasındaki ilişki dikkate alınarak incelendiğinde alt
küme sayısının, her zaman kümenin eleman sayısı kadar 2 nin kuvveti alınarak bulunduğu görülür.

n elemanlı bir kümenin alt kümelerinin sayısı 2 n şeklinde hesaplanır.

ÖRNEK 4:
A = " x ; 0 1 x 1 18, x bir asal sayıdır , kümesinin alt küme sayısını bulunuz.

ÇÖZÜM:

A = " 2, 3, 5, 7,11,13,17 , olduğundan s (A) = 7 dir.


Alt küme sayısı = 2 7 = 128 olur.

ÖRNEK 5:
Alt küme sayısı 512 olan bir kümenin eleman sayısını bulunuz.

ÇÖZÜM:
Kümenin eleman sayısı n ise alt küme sayısı 2 n dir. 512 = 2 9 olduğundan 2 n = 2 9, n = 9 olur.

Bir kümenin kendisi dışındaki alt kümelerinin sayısı 2 n - 1 ile hesaplanır.

ÖRNEK 6:
Kümenin kendisi dışındaki alt kümelerinin sayısı 63 olan bir kümenin eleman sayısını bulunuz.

ÇÖZÜM:
s ^A h = n olsun. Kümenin kendisi dışındaki alt kümelerinin sayısı 63 olduğundan 2 n - 1 = 63 yazılır.
2 n = 64 = 2 6 & s ^A h = n = 6 bulunur.

Matematik 9 | 55
KÜMELER

ÖRNEK 7:
A kümesinin alt küme sayısı 32, B kümesinin kümenin kendisi dışındaki alt kümelerinin sayısı 63 ise bu iki
kümenin eleman sayılarının toplamını bulunuz.

ÇÖZÜM:
s (A) = n ise alt küme sayısı 2 n ve s (B) = m ise kümenin kendisi dışındaki alt kümelerinin sayısı 2 m - 1
olur.
2 n = 32 2 m - 1 = 63 & 2 m = 64
2n = 25 2m = 26
n = 5 & s ^Ah = 5 m = 6 & s ^Bh = 6 bulunur.
Buradan
s (A) + s (B) = n + m = 5 + 6 = 11 elde edilir.

ÖRNEK 8:
A = " 1, 2, 3, 4 , kümesinin bütün alt kümelerindeki elemanlarının sayı değerlerinin toplamını bulunuz.

ÇÖZÜM:
1 sayısının içinde bulunduğu A nın alt kümelerinin sayısını bulmak için A kümesinden 1 elemanı çıkarılır.
Geriye kalan 3 elemanın alt küme sayısı hesaplanır. Bu alt kümelerin her birine 1 elemanı tekrar eklenir. Bu
durumda içerisinde 1 i bulunduran alt küme sayısı 2 3 = 8 tane olur.
Benzer şekilde 2, 3, 4 sayılarından her birini içinde bulunduran sekizer tane alt küme vardır.
Bu durumda 1, 2, 3, 4 sayıları sekiz defa toplanacağından
Alt kümelerdeki sayıların toplamı: 8 $ ^1 + 2 + 3 + 4 h = 8 $ 10 = 80 olur.

ÖRNEK 9:
Bir kümenin eleman sayısı 4 artırılırsa alt küme sayısında nasıl bir değişiklik olacağını bulunuz.

ÇÖZÜM:
s (A) = n olsun. Bu durumda alt küme sayısı 2 n olur.
A kümesinin eleman sayısı 4 artırıldığında s (A) = n + 4 olur. Bu durumda alt küme sayısı,
2 n + 4 = 2 n $ 2 4 = 2 n $ 16 = 16 $ 2 n olur. Sonuç olarak alt küme sayısı 16 katına çıkmış olur.

ÖRNEK 10:
A = " a,b ,, B = " a,b, c, d, e , kümeleri veriliyor.
A 3 C 3 B olacak şekilde kaç farklı C kümesi yazılabileceğini bulunuz.

ÇÖZÜM:
A 3 C 3 B & " a,b , 3 C 3 " a,b, c, d, e , yazılabilir.
C kümesinde a ve b elemanları olduğundan bunların dışındaki c, d ve e elemanlarının yardımıyla
oluşturulan alt kümeler, yazılacak olan farklı C kümelerinin sayısını verir.
O hâlde 2 3 = 8 tane C kümesi yazılabilir.

56 | Matematik 9
KÜMELER

ÖRNEK 11:
İki kümenin alt küme sayılarının toplamı 68 ise bu kümelerin eleman sayılarının toplamını bulunuz.

ÇÖZÜM:
İki küme A, B ve eleman sayıları s ^A h = a ve s ^B h = b olsun.
2 a + 2 b = 68 yazılır.
Alt kümelerinin sayısı 1, 2, 4, 8, 16, 32, 64, 128,... olabilir. Bu sayılardan toplamı 68 olabilecek iki tanesi
sadece 4 ve 64 tür. Bu durumda
2 a + 2 b = 4 + 64 = 2 2 + 2 6 olur. Buradan a ve b nin 2 ve 6 olması gerekir.
s ^A h + s ^B h = 2 + 6 = 8 olur.

ÖRNEK 12:
Bir öğretmen 5 öğrencisi arasından
a) En çok dört öğrencinin katılacağı,
b) En az iki öğrencinin katılacağı gezi grubunu kaç farklı şekilde oluşturulabileceğini bulunuz.

ÇÖZÜM:

Öğrencilerden oluşan küme A olsun.

a) s (A) = 5 olduğundan en çok dört elemanlı alt kümelerinin sayısı, kendisi dışındaki bütün alt
kümelerdir. Toplam alt küme sayısı: 2 5 = 32 dir. O hâlde en çok dört elemanlı alt küme sayısı:
32 - 1 = 31 olur.

b) Bu kümenin en az iki elemanlı alt kümelerinin sayısı tespit edileceği için A kümesinin bir elemanlı alt
kümeleri ve boş küme toplama dâhil edilmez.
Boş olan alt küme sayısı: 1
A kümesi 5 elemanlı olduğundan bir elemanlı alt küme sayısı 5 tir.
En az iki elemanlı alt kümelerinin sayısı: 2 5 - ]1 + 5g = 32 - 6 = 26 olur.

ÖRNEK 13:

A = " a,b, c, d, e, f, i , kümesinin alt kümelerinin kaç tanesinde en az bir sesli harf olduğunu bulunuz.

ÇÖZÜM:
7
A kümesinin tüm alt kümelerinin sayısı: 2 = 128 dir. A kümesinden a, e, i sesli harfleri çıkarılırsa
" b, c, d, f, , kümesi elde edilir. Bu kümenin alt küme sayısı 2 4 = 16 olur. Bu alt kümelerin hiçbirinde sesli harf
yoktur.
Tüm alt küme sayısından içinde sesli harf olmayan alt küme sayısı çıkarılırsa içinde en az bir sesli harf
bulunduran alt küme sayısı bulunmuş olur. Bu değer 128 - 16 = 112 olur.

Matematik 9 | 57
KÜMELER

ÖRNEK 14:
A = " 1, 2, 3, 4, 5 , kümesinin alt kümelerinin kaç tanesinde
a) 3 bulunur?
b) 1 ve 5 bulunur?
c) 1 veya 5 bulunur?
ç) 1 ve 2 birlikte bulunmaz?
d) Ne 1 ne de 2 bulunur?

ÇÖZÜM:

a) A kümesinden 3 elemanı çıkarılırsa " 1, 2, 4, 5 , kümesi bulunur. Bu kümenin alt küme sayısı 2 4 = 16
olur. Her bir alt kümeye 3, eleman olarak eklenirse 3 ün de içinde bulunduğu 16 tane alt küme elde
edilir.
Bu kümeler
" , $ "3 ,
" 1 ,, " 2 ,, " 4 , " 5 , " " 3, 1 ,, " 3, 2 ,, " 3, 4 , " 3, 5 ,
" 1, 2 ,, " 1, 4 ,, " 1, 5 ,, " 2, 4 ,, " 2, 5 ,, " 4, 5 , " " 3, 1, 2 ,, " 3, 1, 4 ,, " 3, 1, 5 ,, " 3, 2, 4 ,, " 3, 2, 5 ,, " 3, 4, 5 ,
" 1, 2, 4 ,, " 1, 2, 5 ,, " 1, 4, 5 , $ " 3, 1, 2, 4 ,, " 3, 1, 2, 5 ,, " 3, 1, 4, 5 ,
" 1, 2, 4, 5 , $ " 3, 1, 2, 4, 5 , şeklinde yazılabilir.
b) A kümesinden 1 ve 5 çıkarılırsa " 2, 3, 4 , kümesi bulunur. Bu kümenin alt küme sayısı 2 3 = 8 olur. Her
bir alt kümeye 1 ve 5 elemanları eklenirse 8 tane alt kümede 1 ve 5 elemanı bulunur.
c) 1. Yol:
A kümesinden 1 ve 5 çıkarılırsa " 2, 3, 4 , kümesi bulunur. Bu kümenin 2 3 = 8 tane alt kümesi vardır.

Alt kümelere 1 ve 5 eklenirse 8 tane


Alt kümelere 1 eklenirse 8 tane
Alt kümelere 5 eklenirse 8 tane
24 tane alt kümede 1 veya 5 eleman olarak bulunur.

2. Yol:
A kümesinin tüm alt küme sayısı: 2 5 = 32 dir.
İçinde 1 veya 5 in bulunmadığı alt küme sayısı: 2 3 = 8 olduğundan
içinde 1 veya 5 in bulunduğu alt küme sayısı: 32 - 8 = 24 olur.

ç) A kümesinden 1 ve 2 elemanları çıkarılırsa " 3, 4, 5 , kümesi elde edilir. Bu kümenin alt küme sayısı
2 3 = 8 dir.
Bu alt kümelerin her birine 1 eklenirse içinde 1 bulunan alt küme sayısı 8, buna benzer olarak da içinde
2 bulunan alt küme sayısı yine 8 olur. Sonuç olarak, içinde 1 ve 2 elemanlarının birlikte bulunmadığı alt
küme sayısı
8 + 8 + 8 = 24 olur.

d) A kümesinden 1 ve 2 elemanları çıkarılırsa " 3, 4, 5 , kümesi elde edilir.


Bu kümenin alt küme sayısı 2 3 = 8 dir. Bu alt kümelerde 1 veya 2 eleman olarak bulunmadığından
sonuç 8 olarak bulunur.

58 | Matematik 9
KÜMELER

ÖRNEK 15:
A = " 1, 2, 3, 4 , kümesinin ardışık herhangi iki sayı içermeyen kaç tane alt kümesi olduğunu bulunuz.

ÇÖZÜM:
I. Yol:
Eleman sayısı az olduğundan istenen alt kümeler kolayca yazılabilir.
Bu alt kümeler: Q, " 1 ,, " 2 ,, " 3 ,, " 4 ,, " 1, 3 ,, " 1, 4 ,, " 2, 4 , olup 8 tanedir.
II. Yol:
Eleman sayısı fazla olduğunda istenen alt kümeleri yazmak zor olabilir. Bu durumda Alt küme sayısını
bulmak için bir genelleştirme yapılabilir mi?
A = " 1, 2, 3, 4, ..., n , olsun. ^n ! N h
A kümesinin ardışık herhangi iki sayı içermeyen alt kümelerinin sayısı a n ile gösterildiğinde

Küme İstenen Alt Kümeler an


n = 0 için A = Q Q a0 = 1
n = 1 için A = " 1 , Q, " 1 , a1 = 2
n = 2 için A = " 1, 2 , Q, " 1 ,, " 2 , a2 = a0 + a1 = 3
n = 3 için A = " 1, 2, 3 , Q, " 1 ,, " 2 ,, " 3 ,, " 1, 3 , a3 = a2 + a1 = 5

n = 4 için A = " 1, 2, 3, 4 , Q, " 1 ,, " 2 ,, " 3 ,, " 4 ,, " 1, 3 ,, " 1, 4 ,, " 2, 4 , a4 = a3 + a2 = 8


... ... ...

n ! N için A = " 1, 2, 3, 4, ..., n , Q, " 1 ,, " 2 ,, " 3 ,, " 4 , ... an = an-1 + an-2
Tabloda görüldüğü gibi istenen alt kümelerin sayıları arasında a n = a n - 1 + a n - 2 ^n $ 2h bağıntısı vardır. Bu
bağıntının ispatını tümevarım ispat yöntemini kullanarak yapabilirsiniz.

Sıra Sizde
SORU :
A = " -5, - 3, 0, 1, 2, 3, 6 , kümesi veriliyor. Bu kümenin alt kümelerinin kaç tanesinde
a) Pozitif sayı bulunur?
b) Asal sayı bulunmaz?
c) Elemanları çarpımı sıfırdır?
d) Elemanları çarpımı negatiftir?

ÇÖZÜM:

A kümesiyle ilgili yukarıdaki soruların dışında farklı bir soru yazarak çözümünü bulunuz.

Matematik 9 | 59
KÜMELER

3. Eşit Küme
A ve B iki küme olsun. A ve B kümelerinin tüm elemanları aynı ise bu kümelere birbirine eşit
kümeler denir. Bu durum A = B şeklinde gösterilir.
Eğer bu kümeler birbirine eşit değilse bu durum A ! B şeklinde gösterilir.
A ve B kümelerinin birbirine eşit kümeler olması ancak ve ancak bu kümelerin birbirlerinin alt
kümesi olmasıyla mümkündür. Bu durum ^A 3 Bh / ^B 3 A h + A = B şeklinde ifade edilir.

ÖRNEK 1:

D = # x ; x 2 1 18, x ! N -
B = " x ; x 1 5, x ! N , kümelerinin elemanlarını karşılaştırınız.

ÇÖZÜM:
D kümesi, karesi 18 den küçük doğal sayılardan oluşur. D = " 0, 1, 2, 3, 4 ,
B kümesi, 5 ten küçük olan doğal sayılardır. B = " 0, 1, 2, 3, 4 ,
Bulunan D ve B kümelerinin elemanlarının aynı olduğu görülür. Bu durumda D = B elde edilir.

A ve B gibi herhangi iki kümenin sadece eleman sayıları eşitse bu kümelere birbirine denk kümeler
denir ve A / B şeklinde gösterilir. Birbirine eşit olan kümeler birbirine denktir ancak birbirine denk
olan kümeler birbirine eşit olmak zorunda değildir.

ÖRNEK 2:

Y = " a,b, c ,
G = " x ; 1 1 x 1 5, x ! N ,
S = " x ; x,15 in pozitif tam bölenleri ,
L = # x ; x 1 10, x = 3k,k ! Z + - kümelerinden denk olanları bulunuz.

ÇÖZÜM:

Y = " a,b, c , s^Yh = 3


Kümeler incelendiğinde Y, G ve L kümelerinin eleman sayılarının
G = " 2, 3, 4 , s ^Gh = 3 eşit olduğu görülür. O hâlde bu üç küme birbirlerine denk
S = " 1, 3, 5,15 , s^S h = 4 kümelerdir. Bu durumda Y / G / L yazılır.
L = " 3, 6, 9 , s ^ L h = 3 elde edilir.

Sıra Sizde

SORU :
A ve B kümeleri için m, n ! A ve p ! B dir. A kümesinin alt kümelerinin 32 tanesinde m ve n birlikte
bulunur. B kümesinin alt kümelerinin 16 tanesinde p bulunmaz.
Buna göre A ve B kümelerinin birbirine denk olup olmadığını inceleyiniz.

ÇÖZÜM:

60 | Matematik 9
KÜMELER

ALIŞTIRMALAR

1 C = " 1, 2, " 1, 2 ,, 3, ! 4 +, 5 , 7 D = " a,b, c, d, e , kümesinin alt kümelerinin kaç


kümesi için aşağıdakilerden hangisinin doğru tanesinde;
olduğunu belirtiniz. a) a bulunur?
A) s (C) = 7 b) a ve b bulunur?
B) 4 ! C c) a veya b bulunur?
ç) a ve b bulunur, c bulunmaz?
C) 5 z C
d) a ve b birlikte bulunmaz?
D) " 1, 2 , ! C e) Ne a ne de b bulunur, hesaplayınız.
E) s (C) = 5

2 A = # x ; x 1 100, x = 5k,k ! Z + -
kümesinin eleman sayısını bulunuz. 8
Alt küme sayısı ile kendisi dışındaki alt
kümelerinin sayısı toplamı 1023 olan bir
kümenin eleman sayısını bulunuz.
3 A = " x ; 0 1 x 1 19, x ! N , kümesi veriliyor.
Buna göre aşağıdaki kümelerden hangileri A
kümesinin alt kümesidir?
B = " x ; 0 1 x 1 20, x = 3k, k ! Z ,
C = " x ; 2x - 5 1 15, x ! N , A kümesinin kendisi dışındaki alt kümelerinin
9
D = " x ; - 7 1 x 1 20, x = 5k, k ! Z , sayısı 15 ve B kümesinin alt küme sayısı 256
E = " x ; 3x - 7 = 13, x ! N , olarak veriliyor. Buna göre bu kümelerin
eleman sayılarının çarpımını bulunuz.

4
Aşağıdakilerden hangileri boş küme belirtir,
bulunuz.
I. Negatif doğal sayılar
II. Haftanın c harfi ile başlayan günleri 10 B = " 2, 3, 4 , kümesi veriliyor. Buna göre
III. A = "" ,, aşağıdakilerden hangilerinin B kümesine eşit
IV. 3 katının 5 fazlası 3 olan doğal sayı olduğunu bulunuz.
V. " x ; x + 3 = 0, x ! Z , C = " x ; 1 1 x 1 5, x ! N ,
VI. A = # x ; x 2 + 2 = 0, x ! N - D = " x ; x 1 5, x ! N ,
E = " x ; 2 # x # 4, x ! N ,
F = " x ; x 1 6, x = 2k,k ! N ,

5 " x ; x 1 40, x = 5k,k ! N , kümesinin alt


küme sayısını bulunuz.

11 " 0,1, 2, 3, 4, 5, 6, 7 , kümesinin alt kümelerinin


kaç tanesinde en az bir tane tek sayı
6
Kümenin kendisi dışındaki alt kümelerinin
bulunduğunu hesaplayınız.
sayısı 127 olan bir kümenin en çok 1 elemanlı
alt küme sayısını bulunuz.

Matematik 9 | 61
KÜMELER

9.2.2. KÜMELERDE İŞLEMLER VE BAĞINTI

Etkinlik

Ada ve Ebru Türkiye Bayan Voleybol Millî Takımı Yılın Altısı seçmeleri için Bahar, Çağla, Eda, Gözde,
Naz, Neslihan, Polen, Seda, Ezgi, Güldeniz arasından seçim yapacaktır.
Antrenörlerden Ada Yılın Altısı takımına Çağla, Gözde, Neslihan, Polen, Ezgi, Güldeniz,
Ebru ise Yılın Altısı takımına Gözde, Polen, Çağla, Eda, Seda, Bahar oyuncularını seçiyor.
Ada veya Ebru’dan en az birisinin Yılın Altısı için oy verdikleri oyuncuların oluşturduğu küme:
K = " ......................................................................................... ,
Ada ve Ebru’nun her ikisinin de Yılın Altısı için ortak oy verdikleri oyuncuların oluşturduğu küme:
L = " .......................................................................................... ,
Ada’nın Yılın Altısına alıp Ebru’nun almadığı oyuncuların oluşturduğu küme:
M = " ........................................................................................ ,
Ada’nın Yılın Altısı için seçmediği oyuncuların oluşturduğu küme:
N = " ......................................................................................... ,
• K
kümesinde Ada veya Ebru’dan en az birisinin Yılın Altısı için oy verdikleri tüm
voleybolcuların bulunduğunu ve her voleybolcunun bir kez yazıldığını,
• L kümesinde Ada ve Ebru’nun her ikisinin de Yılın Altısı için oy verdikleri ortak voleybolcuların
bulunduğunu,
• M kümesinde sadece Ada’nın Yılın Altısına aldığı voleybolcuların bulunduğunu,
• N
kümesinde Ada’nın Yılın Altısı için seçtiği oyuncular dışında kalan voleybolcuların
bulunduğunu fark ettiniz mi?
Oluşturulan kümeler, hangi küme işlemleri ile ifade edilir? Düşününüz.

62 | Matematik 9
KÜMELER

1. Kümelerde İşlemler
Kümelerde Birleşim İşlemi
A ile B iki küme olsun. A ile B kümelerinin tüm elemanlarının oluşturduğu kümeye, A ve B kümelerinin
birleşim kümesi adı verilir. A , B şeklinde gösterilir.
A , B = " x ; x ! A veya x ! B , = " x ; x ! A 0 x ! B , olarak ifade edilir.

A B A B A B

A,B A,B A,B = A

Sembolik mantıktaki 0 (veya) sembolü kümelerde , (birleşim) işlemine karşılık gelir.

ÖRNEK 1:
A = " a, b, c, d, e ,, B = " 1, 2, 3, a, b , kümelerinin tüm elemanları ile oluşturulan kümeyi liste yöntemiyle
yazınız.

ÇÖZÜM:
A ve B kümelerinin tüm elemanlarının oluşturduğu küme A , B dir.
A , B = " a, b, c, d, e, 1, 2, 3 , elde edilir.
Burada her iki kümede bulunan elemanların bir kez yazılmış olduğuna dikkat ediniz.
Birleşim İşleminin Özellikleri
1. A , A = " x ; x ! A 0 x ! A , = A olduğundan A , A = A olur.
Bir kümenin kendisi ile birleşimi yine kendisidir. Buna tek kuvvet özelliği adı verilir.

2. A , Q = " x ; x ! A 0 x d Q , = A olduğundan A , Q = Q , A = A olur.


Bir kümenin boş küme ile birleşimi yine kendisidir. Bu durumda boş küme, birleşim işleminin etkisiz
elemanıdır.
3. A , B = " x ; x ! A 0 x ! B , = " x ; x ! B 0 x ! A , = B , A olduğundan A , B = B , A olur.
Kümelerde birleşim işleminin değişme özelliği vardır.

4. A , ^B , Ch = " x ; x ! A 0 ^x ! B 0 x ! C h, = " x ; ^x ! A 0 x ! Bh 0 x ! C ,
= " x ; x ! ^A , B h, 0 " x ; x ! C , = ^A , Bh , C = A , B , C olur.
Kümelerde birleşim işleminin birleşme özelliği vardır.

ÖRNEK 2:
A = " 1, 2, 3 ,, B = " 3, 5, 7 , ve C = " 6, 7, 8 , kümeleri için A , ^B , C h ve ^A , B h , C kümelerini yazınız.

ÇÖZÜM:
B , C = " 3, 5, 7 , , " 6, 7, 8 , = " 3, 5, 6, 7, 8 , olduğundan
A , ^B , C h = " 1, 2, 3 , , " 3, 5, 6, 7, 8 , = " 1, 2, 3, 5, 6, 7, 8 , ...(1)
A , B = " 1, 2, 3 , , " 3, 5, 7 , = " 1, 2, 3, 5, 7 , olduğundan
^A , B h , C = " 1, 2, 3, 5, 7 , , " 6, 7, 8 , = " 1, 2, 3, 5, 6, 7, 8 , ...(2)
(1) ve (2) den A , ^B , C h = ^A , Bh , C elde edilir.

Matematik 9 | 63
KÜMELER

5. A , B = Q olduğunda A ve B kümeleri boş küme olur. Bu durumun tersi de doğrudur. Kümeler boş
küme ise birleşim kümesi de boş küme olur.
A , B = Q + A = Q ve B = Q olur.

6. B 3 A & A , B = " x ; x ! A 0 x ! B , = " x ; x ! A 0 x ! A , = " x ; x ! A , = A olduğundan


B 3 A & A , B = A olur.

ÖRNEK 3:
s ^A h = 10 ve s ^Bh = 7 olsun. Bu durumda
a) s ^A , B h nin en büyük değerini bulunuz.
b) s ^A , B h nin en küçük değerini bulunuz.
c) B M A ise s ^A , B h nin en küçük değerini bulunuz.

ÇÖZÜM:
a) İki kümenin birleşiminin eleman sayısının en büyük olması için kümeler, ortak eleman
bulundurmamalıdır. Bu durum venn şemasında gösterilirse
s ^A h = 10 ve s ^B h = 7 & s ^A , B h = s ^A h + s ^Bh = 10 + 7 = 17 bulunur.
A
B

b) İki kümenin birleşiminin eleman sayısının en küçük olması için kümelerden biri, diğerinin alt kümesi
olmalıdır.

A
Bu durumda B 3 C olur. s ^A , B h nin en küçük değeri s ^A h ya eşit
B olacağından s ^A , B h = s ^A h = 10 olarak bulunur.

c) B M A olduğundan B kümesinin A kümesinde olmayan en az bir elemanı vardır.


A B

4 1
6

s ^A , B h = 4 + 6 + 1 = 11 olur.

64 | Matematik 9
KÜMELER

Kümelerde Kesişim İşlemi


A ile B iki küme olsun. A ile B kümelerinin ortak elemanlarının alınması ile oluşturulan kümeye, A
ve B kümelerinin kesişim kümesi adı verilir. A + B şeklinde gösterilir.
A + B = " x ; x ! A ve x ! B , = " x ; x ! A / x ! B , olarak ifade edilir.

A B A B A
B

A+B A+B = Q A+B = B

Sembolik mantıktaki / (ve) sembolü kümelerde + (kesişim) işlemine karşılık gelir.

ÖRNEK 4:

A + B = " x ; x 1 20, x bir asal sayıdır ,, B = " y ; y < 10, y ! N , kümelerinin ortak elemanları ile
oluşturulan kümeyi liste yöntemiyle yazınız.

ÇÖZÜM:
A kümesi, 20 den küçük asal sayılar ise A = " 2, 3, 5, 7, 11, 13, 17, 19 , ve
B kümesi, 10 dan küçük doğal sayılar ise B = " 0, 1, 2, 3, 4, 5, 6, 7, 8, 9 , olur.
A ve B kümelerinin ortak elemanlarının alınması ile oluşacak küme D ile gösterilirse
D = " 2, 3, 5, 7 , elde edilir.

Kesişim İşleminin Özellikleri


1. A + A = " x ; x ! A / x ! A , = " x ; x ! A , = A olduğundan A + A = A olur.
Bir kümenin kendisi ile kesişimi yine kendisidir. Buna tek kuvvet özelliği adı verilir.

2. A + B = " x ; x ! A / x ! B , = ! x ! B / x ! A + = B + A olduğundan A + B = B + A olur.


Kümelerde kesişim işleminin değişme özelliği vardır.

3. A + ]B + Cg = " x ; x ! A / ]x ! B / x ! Cg, = " x ; ]x ! A / x ! Bg / x ! C ,


= " x ; x ! A / x ! B , + " x ; x ! C , = ]A + Bg + C olur.
Kümelerde kesişim işleminin birleşme özelliği vardır.
4. A + Q = " x ; x ! A / x ! Q , = " x ; x ! Q , = Q olduğundan A + Q = Q + A = Q olur.
Bir kümenin boş küme ile kesişimi boş kümedir. Bu durumda boş küme kesişim işleminin yutan elemanıdır.

A + B = Q & A ve B kümelerinin ortak elemanı yoktur. Bu durumda A ve B kümeleri ayrık kümelerdir


denir.

5. B 3 A & A + B = " x ; x ! A / x ! B , = " x ; x ! B , = B olduğundan B 3 A & A + B = B olur.


Bu durumda kesişim kümesi, kapsanan kümeye (alt kümeye) eşit olur.

Matematik 9 | 65
KÜMELER

6. A + ]B , Cg = " x ; x ! A / ]x ! B 0 x ! Cg,
= " x ; ] x ! A / x ! B g 0 ] x ! A / x ! C g,
= " x ; x ! ^ A + B h 0 x ! ^ A + C h,
= ^A + B h , ^A + C h olduğundan
A + ]B , Cg = ]A + Bg , ]A + Cg elde edilir.
Bu durum, kesişim işleminin birleşim işlemi üzerine soldan dağılma özelliği olarak ifade edilir.
Benzer işlemlerle ]B , Cg + A = ]B + Ag , ]C + Ag olduğunu gösterebilirsiniz.
Kesişim işleminin birleşim işlemi üzerine hem soldan hem de sağdan dağılma özelliği olduğundan
kesişim işleminin birleşim işlemi üzerine dağılma özelliği vardır.

7. A , ]B + Cg = " x ; x ! A 0 x ! ^B + C h,
= " x ; x ! A 0 ] x ! B / x ! C g,
= " x ; ] x ! A 0 x ! B g / ] x ! A 0 x ! C g,
= " x ; x ! ^ A , B h / x ! ^ A , C h,
= ]A , Bg + ]A , Cg olduğundan
A , ]B + Cg = ]A , Bg + ]A , Cg elde edilir.

Bu durum birleşim işleminin kesişim işlemi üzerine soldan dağılma özelliği olarak ifade edilir.
Benzer işlemlerle ]B + Cg , A = ]B , Ag + ]C , Ag olduğunu gösteriniz.
Birleşim işleminin kesişim işlemi üzerine hem soldan hem de sağdan dağılma özelliği olduğundan birleşim
işleminin kesişim işlemi üzerine dağılma özelliği vardır.

ÖRNEK 5:
A + B = " x ; - 2 < x # 4, x ! Z , ve
A + C = " y ; - 1 # y < 4, y ! N , olduğuna göre A + ]B , Cg kümesini liste yöntemiyle yazınız.

ÇÖZÜM:
Verilenlerden A + B = " - 1, 0, 1, 2, 3, 4 , ve A + C = " 0, 1, 2, 3, 4 , olduğu görülür.
A + ]B , Cg = ]A + Bg , ]A + Cg olduğundan (Dağılma Özelliği)
A + ] B , C g = " - 1, 0, 1, 2, 3, 4 , , " 0, 1, 2, 3 ,

= " - 1, 0, 1, 2, 3, 4 , olarak bulunur.
8.
A B
a) A ve B ayrık kümeler ise A , B kümesinin eleman sayısı,
s ]A , Bg = s ]Ag + s ]Bg şeklinde olur.

b) A ve B ayrık kümeler değilse A , B kümesinin eleman sayısı


s ^ A h + s ^ B h - s ]A + B g = ^ x + y h + ^ y + z h - y

= x + y + z = s ]A , Bg bulunur.
s ] A , B g = s ^A h + s ^Bh - s ]A + Bg tir. A B

x y z

66 | Matematik 9
KÜMELER

ÖRNEK 6:
Bir seyahat acentesi Trabzon ve Rize illerine Karadeniz Gezisi
düzenliyor. Geziye katılan öğrencilerin bu illerden en az birisini
gezeceği bilinmektedir. 24 öğrenci Rize’yi, 13 öğrenci hem Rize hem
de Trabzon’u gezmek istiyor. Gezi grubunda 43 öğrenci olduğuna göre
Trabzon’u gezmek isteyen kişi sayısını bulunuz.

ÇÖZÜM:
Rize’yi gezmek isteyenlerin kümesi R ise s ^R h = 24 tir.
Trabzon’u gezmek isteyenlerin kümesi T ile gösterilsin. Hem Trabzon hem de Rize’yi gezmek isteyenlerin sayısı
s ]T + Rg = 13 tir. Gezi grubu 43 kişi ise s ]T , Rg = 43 olur. s ]T , Rg = s ^ T h + s ^R h - s ]T + Rg olduğundan
verilen değerler yerine yazıldığında 43 = s ^ T h + 24 - 13, s ^ T h = 32 bulunur.

ÖRNEK 7:
A ve B kümeleri için s ^A + Bh = 3, s ^A h = 2 $ s ^Bh, s ^A , Bh = 24 olduğuna göre B kümesinin eleman
sayısını bulunuz.

ÇÖZÜM:
A B
s ]Bg = x + 3 olduğundan s ]Ag = 2s ]Bg = 2 ]x + 3g = 2x + 6 olur.
24 = 2x + 6 + x + 3 - 3 denkleminden x = 6 bulunur. 2x+3 3 x
Bu durumda s ^Bh = 6 + 3 = 9 olur.

8-c)
A, B ve C herhangi üç küme olsun. Bu kümelerin birleşiminin eleman sayısı aşağıdaki şekilde hesaplanır.
(Şekildeki harfler bulundukları bölgenin eleman sayısını göstermektedir.)

A B

a x
b
d
y z s ]A , B , Cg = a + b + c + d + x + y + z dir.
c C

s^ A h = a + d + x + y s^ A + B h = d + x s ^ A + B + C h = d ... (3)
s^ B h = b + d + x + z s] A + C g = d + y
s^ C h = c + d + y + z s] B + C g = d + z
+ +
= a + b + c + 2x + 2y + 2z + 3d... (1) = x + y + z + 3d... (2)

(1) toplamından (2) toplamı çıkarılıp bulunan değere (3) ilave edilirse
]1g - ]2g + ]3g = ^a + b + c + 2x + 2y + 2z + 3d h - ^x + y + z + 3d h + d elde edilir.
Gerekli sadeleştirmeler yapılırsa
= a + b + c + d + x + y + z = s ^A , B , C h değeri elde edilir.
s ^A , B , C h = s ]Ag + s ]Bg + s ]Cg - 6s ]A + Bg + s ]A + Cg + s ]B + Cg@ + s ]A + B + Cg

Matematik 9 | 67
KÜMELER

ÖRNEK 8:
T+S Y = Q olmak üzere s ]Tg = 7 ve s ]S g = 4 olduğuna göre s ]T , Sg nın en çok kaç elemanlı olduğunu
bulunuz.
ÇÖZÜM: T S

T+S Y = Q olduğundan birleşim kümesinin en çok olabilmesi için


kesişim kümesinde en az sayıda eleman bulunmalıdır. s ]T + Sg = 1 6 1 3
alınmalıdır.
s ]T , Sg = s ]Tg + s ]S g - s ]T + Sg = 7 + 4 - 1 = 10 olur.

ÖRNEK 9:
K M L olmak üzere ve s ] K g = 6 , s ^ L h = 8 olduğuna göre s ] K , L g nin en az kaç elemanlı olduğunu
bulunuz.
ÇÖZÜM:
K kümesi, L nin alt kümesi olmadığından birleşim kümesinin eleman sayısının en az olması için kesişim
kümesinde en çok sayıda eleman bulunmalıdır. K M L olduğuna göre s ^K + L h 6 dan küçük olmalıdır. Bu
durumda s ^K + L h = 5 olur.

K L

s^ K , L h = s^ K h + s^ L h - s^ K + L h
1 5 3
= 6+8-5
=9
ÖRNEK 10:
A = " 1, 2, 3, 4 , kümesinin elemanlarının boş kümeden farklı birleşimleri A kümesini veren ayrık iki
kümeye kaç farklı şekilde ayrılabileceğini bulunuz.

ÇÖZÜM:
I. Yol:
Bu kümeler X ve Y olsun. A nın her elemanı için bu eleman X veya Y kümesinde olur. Bu durumda her
eleman için iki farklı durum oluşur.
s ]Ag = 4 olduğundan 2 $ 2 $ 2 $ 2 = 2 4 = 16 farklı şekilde ayırma işlemi yapılır.
XY = Q ve Y Y = Q olduğundan 16 - 2 = 14 şekilde ayırma işlemi gerçekleşir.
X ve Y kümeleri için aynı ayırma işlemi iki kez oluştuğu için 14, ikiye bölünmelidir.
24 - 2
O hâlde sonuç = 2 = 7 dir.

II. Yol:

"1, " 2, 3, 4 ,
Bu kümelerden birine 1 elemanını yerleştiriniz. 1 in yanına gelecek
her bir eleman ya da elemanlardan kalanlar, diğer kümeyi oluşturur. Bu
" 1, 2 , " 3, 4 , durumda bütün elemanlar 1 in yanına gelirse diğer küme boş küme
" 1, 4 , " 2, 3 , olur. Boş kümeden farklı ayrık küme oluşturulması istendiği için sonuç
2 3 - 1 = 7 olarak bulunur.
" 1, 3 , " 2, 4 ,
" 1, 3, 4 , "2,
" 1, 2, 4 , "3,
" 1, 2, 3 , "4,

68 | Matematik 9
KÜMELER

Evrensel Küme ve Bir Kümenin Tümleyeni


Belirli bir konuda üzerinde işlem yapılan, bütün kümeleri içine alan, boş kümeden farklı en geniş
kümeye evrensel küme denir. Evrensel küme genel olarak E sembolü ile gösterilir.

E
A B

A, B 3 E olmak üzere A + E = A, A , E = E olur.

Ege Bölgesi’ne ait illerle ilgili bir işlem yapıldığında bu illerin oluşturduğu en büyük küme Ege Bölgesi illeri
olur.
Bu durumda evrensel küme Ege Bölgesindeki illerin kümesi olur.

A 3 E olmak üzere, evrensel kümede olup da A da olmayan elemanların kümesine, A kümesinin


tümleyeni denir. A' ile gösterilir.
A' = " x ; x ! E / x z A , şeklinde ifade edilir.

Örneğin aşağıdaki şekilde


E E = " a, b, c, d, e , evrensel kümesinin bir alt kümesi " d, e , dir.
Örnekteki " d, e , kümesi, A kümesinin tümleyeni olarak bulunur.
A
a
d
A' = " d, e ,, s ^A' h = 2 , A = " a, b, c , , s ^A h = 3 ,
b c A , Al = E ve A + Al = Q olduğundan
e s ^A , Alh = s ^ E h = s ^A h + s ^Alh = 3 + 2 = 5 olur.

Özellikler:
A 3 E olmak üzere s ^ E h = s ^A h + s ^A' h
1 5 A 3 B & B' 3 A'
2 ^ A' h ' = A 6 A , A' = E, A + A' = Q
3 Q' = E, E' = Q 7 E , A' = E, E + A' = A'
4 De Morgan Kuralları
^A , B h ' = A' + B'
^A + B h ' = A' , B'
Tablo: 9.2.1
ÖRNEK 11:
E, A ve B nin evrensel kümesi olmak üzere s ]Ag + s ]B'g = 23 ve s ]Bg + s ]A'g = 17 olduğuna göre s ]Eg
sayısını bulunuz.

ÇÖZÜM:
s ]Ag + s ]B'g = 23
+ s ]Bg + s ]A'g = 17 eşitlikleri taraf tarafa toplanırsa
s ^A h + s ^B' h + s ^B h + s ^A' h = ^s ^A h + s ^A' hh + ^s ^B h + s ^B' hh
23 + 17 = s ^ E h + s ^ E h
40 = 2 $ s ^ E h
20 = s ^ E h

Matematik 9 | 69
KÜMELER

ÖRNEK 12:
E, A ile B nin evrensel kümesi olmak üzere 6]A + Bg , ]A + B'g@' ifadesini en sade şekilde yazınız.

ÇÖZÜM:
]A + Bg , ]A + B'g = A + ]B , B'g = A + E = A olduğundan
6]A + Bg , ]A + B'g@' = 5A?' = A' olur.

Kümelerde Fark İşlemi


A ve B iki küme olsun.
A kümesinde olup B kümesinde olmayan elemanların oluşturduğu kümeye (yalnız A da olan
elemanların oluşturduğu kümeye) A fark B kümesi adı verilir.
A - B veya A = B şeklinde gösterilir. Buna göre
A - B = A = B = " x ; x ! A / x z B , dir.

Aşağıdaki taralı alanlar incelendiğinde s ]A , Bg = s ]A = Bg + s ]B = Ag + s ]A + Bg olduğu görülür.

A B

A=B A+B B=A

ÖRNEK 13:

A = " 2, 3, 5, 7, 9, 11 ,, B = " 0, 1, 3, 5, 10, 12 , kümeleri veriliyor.


a) Bu iki kümedeki elemanlar içinden yalnız A da olan elemanların kümesini,
b) Bu iki kümedeki elemanlar içindeki yalnız B de olan elemanların kümesini,
c) A + B kümesini yazıp Venn şeması yöntemiyle gösteriniz.

ÇÖZÜM:

A B a) Yalnız A da olan elemanların kümesi:


2 7 0 1 A - B = " 2, 7, 9, 11 ,
3
b) Yalnız B de olan elemanların kümesi:
9 11 5 10 12 B - A = " 0, 1, 10, 12 ,
c) A + B = " 3, 5 ,

70 | Matematik 9
KÜMELER

Özellikler

1 A = B = " x ; x ! A / x z B , = " x ; x ! A / x ! B' , = A + B' olduğundan A = B = A + B' elde edilir.


2 A=A = Q 3 A=Q = A
4 E = A = A' 5 A=E = Q

6 Q=A = Q 7 ^A = B hl = ^A + Blhl = Al , ^Blhl = Al , B

A 3 B & A = B = " x ; x ! A / x z B , demektir. Ancak


8 A 3 B durumunda x z B olamaz.
Sonuç olarak A 3 B & A = B = Q olur.
Tablo: 9.2.2

ÖRNEK 14:

A M B ve B M A olmak üzere s ]A , Bg = 20, s ]A + Bg = 5 olduğuna göre s ]B = Ag en çok kaç olacağını


bulunuz.

ÇÖZÜM:
B = A kümesinin eleman sayısının en çok olabilmesi için
A B
A = B kümesinin eleman sayısının en az olması gerekir.
1 5 x
s ]A = Bg = 1 ve s ]B = Ag = x alınırsa
s ]A , B g = s ]A = B g + s ]B = A g + s ]A + B g
20 = 1 + x + 5 ise x = 14 bulunur.

ÖRNEK 15:
E, A ile B nin evrensel kümesi olmak üzere s ]A = Bg = 9, s ]Bg = 5 ve s ]A' + B'g = 2 olduğuna göre s ]Eg
sayısını bulunuz.

ÇÖZÜM:

A B E s ^A + B h = x alınırsa
s ]A' + B'g = s 6]A , Bg '@ = 2 olur.
9 x 5-x s ]B = Ag + s ]A + Bg = s ]Bg = 5 olduğundan
s ] E g = s ] A = B g + s ] B = A g + s ] A + B g + s ] A ' + B' g
2 s ]Eg = 9 + 5 + 2 = 16 bulunur.

Sıra Sizde
SORU :
A ve B, E evrensel kümesinin alt kümeleri olmak üzere ^ B = A h + ^ A , B ' h
ifadesinin en sade şeklini bulunuz.

ÇÖZÜM:

Matematik 9 | 71
KÜMELER

Küme İşlemleri ve Sembolik Mantık Kuralları Arasındaki İlişki


Kümelerle sembolik mantık arasında gösterim, sembol ve bunları ifade eden işlemler arasında ilişki vardır.
Bu ilişki aşağıdaki tabloda verilmiştir.

Sembolik Mantık 0 1 0 / Değil (') /


Kümeler Q E , + Tümleme (') =
Tablo: 9.2.3

Sembolik Mantık Kümeler


p 0 p' / 1 A , A' = E
p / p' / 0 A + A' = Q
p01 / 1 A,E = E
p 0 p / p (Tek Kuvvet Özelli ̇ği ̇) A , A = A (Tek Kuvvet Özelli ̇ği ̇)
p / p / p (Tek Kuvvet Özelli ̇ği ̇) A + A = A (Tek Kuvvet Özelli ̇ği ̇)
p 0 q / q 0 p (Değişme Özelli ̇ği ̇) A , B = B , A (Değişme Özelli ̇ği ̇)
^p 0 q h ' / p' / q' (De Morgan Kuralı ) ]A , Bg ' = A' + B' (Tümleme İşlemi )
p 0 ^q / r h / ^p 0 q h / ^p 0 r h (Dağılma Özelliği) A , ]B + Cg = ]A , Bg + ]A , Cg (Dağılma Özelliği)
Tablo:9.2.4

ÖRNEK 16:
A ve B boş olmayan iki küme olsun. ]A + Bg ' = A' , B' eşitliğini sembolik mantık kurallarından
yararlanarak gösteriniz.

ÇÖZÜM:

]A + B g ' = " x ; x ! ]A + B g ' ,


= " x ; ]x ! A / x ! Bg ' ,
= " x ; x ! A' 0 x ! B' ,
= " x ; x ! ]A' , B'g,
= A' , B' bulunur.

ÖRNEK 17:
p ve q önermeleri sırasıyla A ve B kümeleriyle ilişkilidir. ^p / q' h ' 0 p / 0 olduğuna göre A ve B kümelerini
bulunuz.

ÇÖZÜM:
^p / q' h ' 0 p' / 0 & p' / 0 / ^p / q' h ' / 0
& p / 1 / p / q' / 1
p / 1 olduğundan 1 / q' / 1 & q' / 1 & q / 0 olur.
p, A kümesi ile q, B kümesi ile ilişkili olduğundan
p/1&A=E
q / 0 & B = Q bulunur.

72 | Matematik 9
KÜMELER

ÖRNEK 18:
p ve q önermeleri sırasıyla A ve B kümeleriyle ilişkilidir. 6^p 0 q' h ' / ^p' 0 q h '@ ' 0 q' önermesinin en sade
şeklinin Tablo 9.2.4 e göre karşılık geldiği kümeyi bulunuz.

ÇÖZÜM:
p, A kümesi ve q, B kümesi ile ilişkili olduğundan
6^p 0 q' h ' / ^p' 0 qh '@ ' 0 q' " 6]A , B'g ' + ]A' , Bg '@ ' , B'
= 6]A' + Bg + ]A + B'g@ ' , B'
= 6]A + A'g + ]B + B'g@ ' , B'
= Q' , Bl
= E , Bl
=E bulunur.

Kümelerle İlgili Uygulamalar


Küme problemlerinde aşağıdaki şekli incelemeniz çözüm aşamasında yararınıza olacaktır.

B E
A
A : arpa ekilen alan
x a y B : buğday ekilen alan
d Y : yulaf ekilen alan
b c E : evrensel küme
Y Şekildeki harfler bulundukları bölgenin eleman sayısını ifade
z
etmektedir.
e

1 Arpa ekilen alan a+b+d+x


2 Buğday ekilen alan a+c+d+y
3 Yulaf ekilen alan b+d+c+z
4 Yalnız buğday ekilen alan y
5 Arpa ve buğday ekilen alan a+d
6 Her üç ürünün de ekildiği alan d
7 Buğday ekilip yulaf ekilmeyen alan a+y
8 Arpa veya buğday ekilen alan a+b+c+d+x+y
9 Yalnız bir ürün ekilen alan x+y+z
10 Yalnız iki ürün ekilen alan a+b+c
11 En çok bir ürün ekilen alan x+y+z+e
12 En az bir ürün ekilen alan a+b+c+d+x+y+z
13 Hiç ürün ekilmeyen (nadasa bırakılan alan) e
14 En az iki ürün ekilen alan a+b+c+d
15 En çok iki ürün ekilen alan a+b+c+e+x+y+z
16 Yulaf ekilmeyen alan a+e+x+y
Tablo: 9.2.5

Matematik 9 | 73
KÜMELER

ÖRNEK 19:
Kapadokya ve Safranbolu’ya yapılacak geziye katılan bir sınıftaki
öğrencilerin %35 i Kapadokya’yı, %40 ı Safranbolu’yu, %15 i hem
Kapadokya hem de Safranbolu’yu gezmek istiyor. Bu yerlere yapılacak
olan gezilerin hiçbirine katılmak istemeyen 8 öğrenci olduğuna göre bu
sınıfın mevcudunu bulunuz.
ÇÖZÜM:
E Sınıf mevcudu 100x kabul edilirse
K S s ^K = S h = 20x, s ^K + S h = 15x, s ^S = Kh = 25x olur.
20x + 15x + 25x = 60x
20x 15x 25x 100x - 60x = 40x ı geziye katılmamıştır.
40x = 8 olur.
8 1
Bu durumda x = 40 = 5 = 0, 2 bulunur.
s ^ E h = ^0, 2h $ 100 = 20 olur.
8

ÖRNEK 20:
24 kişilik bir topluluktaki herkes Türkçe bilmektedir. Bu toplulukta
Arapça bilmeyen 10 kişi, Japonca bilmeyen 15 kişi ve bu üç dili de bilen
7 kişi olduğuna göre bu toplulukta sadece Arapça bilenlerin sayısını
bulunuz.
ÇÖZÜM:
T s ^A = J h = x, s ^A + J h = y, s ^J = A h = z, s ^A , J hl = t olsun
x + y + z + t = 24 ...(1)
A J
z + t = 10 ...(2)
x + t = 15 ...(3)
x y z
y = 7 (4) denklemleri elde edilir. (2) deki değer (1) de yerine
yazıldığında x + y + 10 = 24 elde edilir.
t Buradan x + y = 14 bulunur.
y = 7 değeri yerine yazılırsa sadece Arapça bilen kişi sayısı x = 7
bulunur.
ÖRNEK 21:
Bir sınıftaki öğrencilerin %20 si erkektir. Bu öğrencilerden matematik dersinden geçen kızlar; bu dersten
1
kalan erkek öğrencilerin iki katı, tüm öğrencilerin 3 üdür. Matematik dersinden kalan 14 kız olduğuna göre
bu sınıfın mevcudunu bulunuz.
ÇÖZÜM:
Dersten kalan erkek öğrenci sayısı x olsun. Bu durumda matematik dersinden geçen kız öğrenci sayısı
2x tir. Bu sayı, sınıfın üçte biri olduğundan sınıfın tamamı 6x olur.
20 6x
Sınıftaki erkek öğrencilerin sayısı: 6x $ 100 = 5 tir.
6x 24x
Sınıftaki kız öğrencilerin sayısı : 6x - 5 = 5 olur. Bulunan değerler tabloda gösterilirse
Kız Erkek
Geçenler 2x 24x
2x + 14 = 5 & 10x + 70 = 24x & x = 5
Kalanlar 14 x
O hâlde sınıf mevcudu: 6 $ 5 = 30 olarak bulunur.
24x 6x 6x
Toplam
5 5

74 | Matematik 9
KÜMELER

ÖRNEK 22:

Bir grubun %70 i İngilizce, %75 i Türkçe, %60 ı Almanca bilmektedir. Bu grubun en az % kaçının her üç dili
de bildiğini bulunuz.

ÇÖZÜM:
E grubun kümesi, T Türkçe bilenlerin kümesi, İ İngilizce bilenlerin kümesi ve A Almanca bilenlerin kümesi
olmak üzere
s ^ E h = 100 denirse s ]Tg = 75, s ] i g = 70, s ]Ag = 60 olur.

s ]T , i , Ag = s (T) + s (i) + s (A) - s (T + i) - s (T + A) - s (i + A) + s (T + i + A)


100 - e = 75 + 70 + 60 - ]a + dg - ]b + dg - ]c + dg + d
T İ

100 - e = 205 - ]a + b + c + 3dg - d


x a z

d d = 105 - ]a + b + c + dg + e
d nin en az değeri için e nin en küçük, ^a + b + c + dh nin en büyük değeri
b c

y alması gerekir.
e A O hâlde a + b + c + d = 100 ve e = 0 olmalıdır. Dolayısıyla d nin en
E küçük değeri 5 tir.

ÖRNEK 23:
Sinema, tiyatro ve konser etkinliklerinden en az birini sevenler ile hiçbirini sevmeyenlerden oluşan 25
kişilik bir arkadaş grubunda
1. En az birini seven 24 kişi,
2. Tiyatro veya konser seven 20 kişi,
3. En çok birini seven 10 kişi,
4. Üç etkinliği de seven 3 kişi,
5. Sinema ve tiyatro seven 5 kişi,
6. Sadece konser seven 2 kişi vardır.
Buna göre tiyatro sevip konser sevmeyen kişi sayısını bulunuz.

ÇÖZÜM:
Verilenler yandaki şekle göre yazıldığında
S T x + y + z + a + b + c + d + e = 25 ...(1)
x a z x + y + z + a + b + c + d = 24 ...(2)
d y + z + a + b + c + d = 20 ...(3)
b c x + y + z + e = 10 ...(4)
y d = 3 ...(5)
K
e a + d = 5 ...(6)
E y = 2 ...(7) ise a + z = ?

(1) eşitliğinden (2) eşitliği çıkarılırsa e = 1 bulunur.


(2) eşitliğinden (3) eşitliği çıkarılırsa x = 4 bulunur.
(6) eşitliğinden (5) eşitliği çıkarılırsa a = 2 bulunur.
(4) eşitliğinde bilinen değerler yerine yazılırsa 4 + 2 + z + 1 = 10 olur. Buradan z = 3 bulunur.
Buna göre tiyatro sevip konser sevmeyen kişi sayısı: a + z = 2 + 3 = 5 olarak bulunur.

Matematik 9 | 75
KÜMELER

ÖRNEK 24:
A = # x ; x < 500, x ! Z + - kümesinin kaç tane elemanı 4 veya 6 ya bölünebilir, 5 e bölünemez? Bulunuz.

ÇÖZÜM:
M: 4 e bölünenler kümesi, N: 6 ya bölünenler kümesi, P: 5 e bölünenler kümesi olsun.
4 e bölünenlerin sayısı x = 4k ^k ! Z + h için
4k < 500 olup k < 4 = 125 dir. Dolayısıyla s ]Mg = 125 - 1 = 124
500

6 ya bölünenlerin sayısı x = 6k ^k ! Z + h için
6k < 500 ise k < 6 olup k < 83, 3 s ]Ng = 83 olur.
500

Benzer şekilde
500
4 ve 6 ya bölünebilen en küçük doğal sayı 12 olacağından 4 ve 6 ya bölünenlerin sayısı 12 nin tam
kısmı 41 olduğundan s ]M + Ng = 41 dir.
500
4 ve 5 e bölünebilen en küçük doğal sayı 20 olacağından 4 ve 5 e bölünebilenlerin sayısı 20 = 25 olup
s ]M + Pg = 25 - 1 = 24
500
6 ve 5 e bölünebilen en küçük doğal sayı 30 olacağından 6 ve 5 e bölünebilenlerin sayısı 30 tam kısmı
16 olduğundan s ]N + Pg = 16 dır.

4, 5 ve 6 ya bölünenlerin sayısı 60 nın tam kısmı 8 dir. O hâlde s ]M + N + Pg = 8 dir.


500

s ]M , Ng = s ]Mg + s ]Ng - s ]M + Ng = 124 + 83 - 41 = 166
Bulunan eleman sayıları kullanılarak venn şeması oluşturulduğunda

M N
67 33 34
4 veya 6 ya bölünüp 5 e bölünmeyenlerin sayısı
8
16 8 166 - ]16 + 8 + 8g = 166 - 32 = 134 tür.

ÖRNEK 25: A B

Şekilde verilen taralı bölgeyi, küme işlemlerini kullanarak ifade ediniz.

ÇÖZÜM:
C

A B
(1) (1) numaralı bölge ]A + Bg = C
(2) numaralı bölge ]A + Cg = B
(2) (3) (3) numaralı bölge ]B + Cg = A
şeklinde ifade edilebilir.

İstenen taralı bölge bu üç bölgenin birleşim kümesi olduğundan taralı bölgenin tamamı:
6]A + Bg = C@ , 6]A + Cg = B@ , 6]B + Cg = A@ = 6]A + Bg , ]A + Cg , ]B + Cg@ = ]A + B + Cg
işlemi ile ifade edilmiş olur.

76 | Matematik 9
KÜMELER

ALIŞTIRMALAR

1 A + B = " 1, 3, 5, 7 , ve A + C = " 1, 2, 3, 4, 5, 6 , 6 s ^K h = 12 ve s ^ L h = 5 olmak üzere


olduğuna göre A + ^B , C h kümesinin eleman a) K + L ! Q olduğuna göre s ^K , L h nin en
sayısını bulunuz. çok kaç ve en az kaç olduğunu bulunuz.
b) L M K olduğuna göre s ^K , L h nin en çok
kaç ve en az kaç olduğunu bulunuz.

2 A = " y ; 10 1 y 1 160, y = 2k, k ! N , 7


A, B 3 E olmak üzere
B = " x ; 30 1 x 1 200, x = 5k, k ! N , A + B ! Q, s ^A h = 3 $ s ^Bh ve s ^A , Bh = 23
kümeleri veriliyor. s ^A , B h değerini bulunuz. olduğuna göre s ^B = A h en büyük değerini
bulunuz.

3 s ^A h = 10, s ^B h = s ^C h = 9 8 6^A , Q h + ^B , Qh@ , A ifadesinin en sade


s ^A + B h = s ^A + C h = 3, s ^B + C h = 8 şeklini bulunuz.
s ^A + B + C h = 1 olduğuna göre
s ^A , B , C h değerini bulunuz.

9
Bir sınıftaki öğrencilerin %55 i matematik,
%75 i biyoloji dersinden başarılıdır. Sınıfta
4 A ve B kümeleri için s ^A h = 5, s ^Bh = 13 ve her iki dersten başarısız olan öğrenci
s ^B = A h = 3 $ s ^A = B h olduğuna göre s ^A , B h bulunmamaktadır. Bu sınıfta biyolojiden
değerini bulunuz. başarılı olup matematikten başarısız olan 9
öğrenci bulunduğuna göre her iki dersten
başarılı olan öğrenci sayısı kaçtır? Bulunuz.

5 A, B 3 E olmak üzere s ^Al h + s ^Bh = 26 10


Boş olmayan A ve B kümeleri için
s ^A h + s ^Blh = 14 olduğuna göre s ^ E h 3 $ s ^A - Bh = 8 $ s ^A + Bh = 6 $ s ^B - Ah
değerini bulunuz. olduğuna göre s ^A , B h nin en küçük değerini
bulunuz.

Matematik 9 | 77
KÜMELER

2. Kümelerin Kartezyen Çarpımı


Günümüzde birçok insan rutin hayatında değişiklik yapmak için spor müsabakaları, sinema, tiyatro gibi
sosyal etkinliklere gitmektedir. Etkinliğe katılan insanların gösterinin yapıldığı salonlardaki biletlerinde
belirtilen yerleri bulmalarına yönelik sorununun nasıl çözülebileceğini düşününüz.

Sıralı İkili
a ile b birer nesne olmak üzere ^a, b h şeklindeki ifadeye bir sıralı ikili veya kısaca ikili denir.
Buradaki a ile b birer sayı olmak zorunda değildir. a ya bu sıralı ikilinin birinci bileşeni, b ye bu sıralı
ikilinin ikinci bileşeni denir.
Benzer şekilde a, b ve c nesneleri kullanılarak oluşturulan ^a, b, c h ifadesine de sıralı üçlü denir.
Buradan yola çıkılarak bir genelleme yapıldığında a 1, a 2, a 3, ..., a n - 1 , a n elemanları kullanılarak
oluşturulan ^a 1, a 2, a 3, ..., a n h ifadesine sıralı n li denir.

Örneğin sinema salonlarındaki koltuk numaraları birer sıralı ikilidir.


^B, 7 h
sıra koltuk no.
Benzer şekilde bir öğrencinin okulu, sınıfı ve okul numarası birlikte kullanılarak bir üçlü oluşturulabilir.
Böylece A lisesinin 9/A sınıfındaki 88 numaralı öğrenciyi üçlü oluşturarak gösterilebilir.
A Lisesi 9/A 88

Siz de benzer şekilde günlük hayatta karşılaştığımız sıralı ikili, sıralı üçlü, ... , sıralı n li örnekleri bulabilir
misiniz?

Sıralı İkililerin Eşitliği


İki sıralı ikili birbirine eşit ise bu sıralı ikililerin aynı sıradaki bileşenleri birbirine eşittir.
^a, b h = ^ c, d h + ^a = c ve b = d h dir.

ÖRNEK 1:
^3x + 5, y + 2 h = ^- 1, 7 h ise ^x, y h sıralı ikilisini bulunuz.

ÇÖZÜM:
3x + 5 =- 1 ve y + 2 = 7 ise
3x =- 6 ve y = 5 ise ^x, y h = ^- 2, 5 h olarak bulunur.
x =- 2

ÖRNEK 2:

^2 3a + b, 3 a - b h = b 4 a + 3, 19 l ise a $ b değerini bulunuz.


ÇÖZÜM:
2
3a + b
= 2 2a + 6 3 a - b = 3 -2 a+b = 6
ve
3a + b = 2a + 6 a - b =- 2 a - b =- 2
a+b = 6 a=2
b = 4 olup a $ b = 8 dir.

78 | Matematik 9
KÜMELER

Kartezyen Çarpım
Boş kümeden farklı A ve B kümeleri verilsin. Birinci bileşeni A kümesinden, ikinci bileşeni B
kümesinden alınarak oluşturulan tüm ikililerin kümesine A ve B kümelerinin kartezyen çarpım kümesi
veya A kartezyen B denir.
A # B ortak özellik yöntemiyle A # B = "^x, y h ; x ! A ve y ! B , şeklinde yazılabilir.
A = B durumunda A # A elde edilir ve buküme A 2 şeklinde gösterilir

ÖRNEK 3:
A = " 1, 2, 3 ,, B = " a, b , kümeleri verilsin. A # B, B # A ve A # A kartezyen çarpım kümelerini liste
yöntemiyle yazınız.

ÇÖZÜM:
A # B = "^1, a h, ^1, b h, ^2, a h, ^2, b h, ^3, a h, ^3, b h,
B # A = "^a, 1h, ^b, 1h, ^a, 2 h, ^b, 2 h, ^a, 3 h, ^b, 3 h,
A # A = "^1, 1 h, ^1, 2 h, ^1, 3 h, ^2, 1 h, ^2, 2 h, ^2, 3 h, ^3, 1 h, ^3, 2h, ^3, 3 h,
ÖRNEK 4:
M # N = "^ 0, 1h, ^- 1, 3h, ^ 3, 5h, ^ 0, 3h, ^ 0, 5h, ^- 1, 5h, ^ 3, 1h, ^ 3, 3h, ^- 1, 1h,
N # P = "^1, 0h, ^5, 0h, ^3, 0 h, kartezyen çarpım kümeleri veriliyor.
Buna göre N , ]M + Pg kümesini bulunuz.

ÇÖZÜM:
M # N deki sıralı ikililerin birinci bileşenlerinin kümesi M yi, ikinci bileşenlerinin kümesi N yi verir. Benzer
biçimde M # P deki sıralı ikililerin ikinci bileşenlerinin kümesi P yi verir. Buna göre
M = " 0, - 1, 3 ,
N = " 1, 3, 5 ,
P = ! 0 + kümeleri bulunur.
Buradan M + P = " 0 , ve N , ^M + P h = " 0, 1, 3, 5 , elde edilir.
Kartezyen Çarpımın Özellikleri
1. A = " a 1, a 2, a 3, ..., a k , , B = " b 1, b 2, b 3, ..., b n , kümeleri verilsin. Bu kümelerin kartezyen çarpımı
aşağıdaki tabloda görülmektedir.
B
b1 b2 b3 ... bn
a1 ^ a 1, b 1 h ^ a 1, b 2 h ^ a 1, b 3 h ... ^ a 1, b n h
a2 ^ a 2, b 1 h ^ a 2, b 2 h ^ a 2, b 3 h ... ^ a 2, b n h
� a3 ^ a 3, b 1 h ^ a 3, b 2 h ^ a 3, b 3 h ... ^ a 3, b n h
...

...

...

...

...

ak ^ a k, b 1 h ^ a k, b 2 h ^ a k, b 3 h ... ^ a k, b n h
Tablo: 9.2.6

Tablodaki tüm ikililer kartezyen çarpım kümesinin elemanlarıdır.


A # B = "^a 1, b 1 h, ^a 1, b 2 h, ^a 1, b 3 h, ..., ^a k, b n h,
O hâlde kartezyen çarpım kümesinin eleman sayısı s ]A # Bg = s ]Ag $ s ]Bg = k $ n olarak bulunur.
Boş kümeden farklı A ve B kümeleri için s ]A # Bg = s ]B # Ag = s ]Ag $ s ]Bg dir.

Matematik 9 | 79
KÜMELER

2. A # B = "^x, y h ; x ! A ve y ! B ,
B # A = "^y, x h ; y ! B ve x ! A ,
Elde edilen ikililerde A # B kümesinin elemanlarının birinci bileşeni ile ikinci bileşeninin yer değiştirmesiyle
B # A nın elemanları elde edilir.
x ! y için ^x, y h ! ^y, x h olduğuna göre kartezyen çarpım işleminin değişme özelliği yoktur.
A ! B için A # B ! B # A dır.

3. A # ]B , Cg = "^x, yh ; x ! A / y ! ]B , Cg,
= "^x, y h ; x ! A / ^y ! B 0 y ! C h,
= "^x, y h ; ^x ! A / y ! Bh 0 ^x ! A / y ! C h,
= "^x, y h ; ^x, y h ! A # B 0 ^x, y h ! A # C ,
= ]A # B g , ]A # C g
olduğundan kartezyen çarpım işleminin birleşim işlemi üzerine dağılma özelliği vardır.
A # ]B , Cg = ]A # Bg , ^A # C h Bu eşitlik, kartezyen çarpım işleminin birleşim işlemi üzerine soldan
dağılma özelliği olarak ifade edilir. Buradan yola çıkıldığında kartezyen çarpım işleminin soldan ve sağdan
birleşim, kesişim ve fark işlemleri üzerine dağılma özelliği olduğu görülmektedir.
I. A # ]B , Cg = ]A # Bg , ]A # Cg
II. A # ]B + Cg = ]A # Bg + ]A # Cg
III. A # ]B = Cg = ]A # Bg = ]A # Cg

ÖRNEK 5:

Bir okuldaki A sınıfında 10 öğrenci, B sınıfında 8 öğrenci vardır. Birinci öğrenci A sınıfından, ikinci öğrenci B
sınıfından olacak şekilde 2 öğrenci seçiliyor. Bu ikilinin kaç farklı şekilde oluşturulabileceğini bulunuz.

ÇÖZÜM:

Verilenlere göre s ^A h = 10 ve s ^B h = 8 olur.


s ]A # Bg = s ]Ag $ s ]Bg = 10 $ 8 = 80 elde edilir.

ÖRNEK 6:

A = " x ; - 2 1 x 1 4, x ! Z ,
B = # x ; 0 1 x 2 1 5, x ! Z -
C = " 1, 2, 3, 4, 5 ,
s ^]A # Bg , ]A # Cgh değerini bulunuz.

ÇÖZÜM:

A = " - 1, 0, 1, 2, 3 , B = " - 1, 1, 2, - 2 , C = " 1, 2, 3, 4, 5 ,


]A # Bg , ]A # Cg = A # ]B , Cg (Özellik 4)
B , C = " - 2, - 1, 1, 2, 3, 4, 5 , , s ]B , Cg = 7 ve s ]Ag = 5 değerleri yerine yazıldığında
s ^A # ]B , Cgh = s ]Ag $ s ]B , Cg = 5 $ 7 = 35 bulunur.

80 | Matematik 9
KÜMELER

ÖRNEK 7:
5 $ s ]A g = 3 $ s ]B g
s ]A + B g = 2 $ s ]A - B g
s ]A , Bg = 24 olduğuna göre s 6]B # Ag - ]A # Ag@ değerini bulunuz.

ÇÖZÜM:
A B 5 $ s ^A h = 3 $ s ^B h & s ^A h = 3x için s ^B h = 5x olur.
s ^A + B h = 2 $ s ^A - B h & s ^A - B h = x için s ^A + B h = 2x
olduğundan s ^B - A h = 3x olur.
x 2x 3x

s ^A , B h = x + 2x + 3x = 6x = 24 ise x = 4 bulunur.

s 6]B # Ag - ]A # Ag@ = s 6]B - Ag # A@ = s ]B - Ag $ s ]Ag = 12 $ 12 = 144 olarak bulunur.

ÖRNEK 8:
s ]A # Bg = s ]Ag + s ^B h + 10 olduğuna göre s 6A # ]A , Bg@ değeri en çok kaçtır?

ÇÖZÜM:
s ^A h = a, s ^B h = b olmak üzere
a $ b = a + b + 10
a $ b - a = b + 10
a ^b - 1 h = b + 10
b + 10 b - 1 + 11 11
a = b - 1 = b - 1 = 1 + b - 1 olduğundan b - 1 = 1 ise b = 2 ve a = 12 olur.
b - 1 = 11 ise b = 12 ve a = 2 olur.
s 6A # ^A , Bh@ nin en çok olması için A kümesinin eleman sayısının en fazla ve A ile B kümelerinin ayrık
kümeler olması gerekir.
Bu durumda s ^A h = 12, s ^B h = 2 ve s ^A , B h = 14 olur. Buna göre s 6A # ^A , Bh@ = 12 $ 14 = 168 dir.

Sıra Sizde

SORU :
A ve B kümeleri için
s ^]A , Bg # Bh = 132
s ^]A + Bg # Bh = 77 olduğuna göre s ]A - Bg + s ]B - Ag toplamını bulunuz.

ÇÖZÜM:

Matematik 9 | 81
KÜMELER

Kartezyen Çarpımın Grafiği

R 2 = R # R = "^x, y h ; x ! R ve y ! R , kümesinin belirttiği noktaların oluşturduğu düzleme


kartezyen koordinat sistemi (analitik düzlem) denir.
^x, y h ! R 2 ise bu noktaların birinci bileşenine noktanın apsisi, ikinci bileşenine noktanın ordinatı
denir. Oluşan ^x, y h ikilisine A nın koordinatları adı verilir.

y (x,y)
A noktasına karşılık gelen sıralı ikili ^x, y h ise x e A nın
apsisi, y ye A nın ordinatı denir. Oluşan
A # B = "^x, y h ; x ! A ve y ! B , kümesinin
elemanlarının analitik düzlemde işaretlenmesiyle elde
edilen görüntüye A # B nin grafiği denir.
O(0,0) x

Grafik: 9.2.1

ÖRNEK 9:

A = " 1, 2, 3 , , B = " a, b , olarak veriliyor. A # B yi koordinat sisteminde gösteriniz.


ÇÖZÜM:
B

A#B A # B = " ^ 1, a h, ^ 1, b h, ^ 2, a h, ^ 2, b h, ^ 3, a h, ^ 3, b h ,
kümesindeki sıralı ikililerinin birinci bileşenleri noktanın apsisi,
b ikinci bileşenleri noktanın ordinatları olacak biçimde noktalar
koordinat sisteminde gösterilir.
a

A
1 2 3

ÖRNEK 10:

A = " a, b, c, d , B = " 0, 1, 2, 3, 4 , kümeleri veriliyor. Buna göre A # B nin alt kümelerinin kaç tanesinde
a) ^a, 0 h bulunur?
b) ^b, 1 h ve ^d, 1 h bulunur?
c) ^c, 0h veya ^a, 4 h bulunur?

ÇÖZÜM:
s ]A # Bg = 4 $ 5 = 20 dir.
a) A # B kümesinin ^a, 0 h dışında 19 tane elemanı olduğu için bu kümenin 2 19 tane alt kümesi vardır. Her
bir alt kümeye ^a, 0 h eleman olarak eklenirse ^a, 0 h ın da içinde bulunduğu 2 19 tane alt küme elde
edilir.
b) A # B kümesinin ^b, 1 h ve ^d, 1 h elemanları dışında 18 tane elemanı vardır. Bu elemanlardan oluşan
kümenin, 2 18 tane alt kümesi vardır. Bu alt kümelerin her birine ^b, 1 h ve ^d, 1 h eleman olarak
eklenirse ^b, 1 h ve ^d, 1 h in de içinde bulunduğu 2 18 tane alt küme elde edilir.

c) Tüm alt kümelerinin sayısı: 2 20 dir.


A # B kümesinin ^c, 0h ve ^a, 4 h elemanları dışında 18 tane elemanı olduğu için içerisinde ^c, 0h ve ^a, 4 h
elemanları bulunmayan alt küme sayısı 2 18 dir.
Bu durumda ^c, 0h veya ^a, 4 h elemanlarının bulunduğu alt küme sayısı,
2 20 - 2 18 = 2 18 $ ^2 2 - 1 h = 3 $ 2 18 olur.

82 | Matematik 9
KÜMELER

ÖRNEK 11:

A = " 1, 2, 3 , kümesi veriliyor. A # A kümesinin grafiğini çiziniz. A # A nın tüm elemanlarını içeren en
küçük çemberin çapının kaç birim olduğunu bulunuz.
A
ÇÖZÜM:
A#A
A # A nın tüm elemanlarını içeren çember yandaki şekilde 3
verilmiştir. Şekilde görüldüğü gibi A # A yı içine alan en küçük
çemberin çapı Pisagor bağıntısından 2

1
R2 = 22 + 22
R 2 R2 = 8 A
1 2 3
R = 2 2 br olarak bulunur.
2

ÖRNEK 12:
A ve B kümeleri için s ]A + Bg = 2, s ]Ag = 5 ve s ^]A , Bg # B h = 54 olduğuna göre B kümesinin eleman
sayısını bulunuz.

ÇÖZÜM:
A B Şekle göre s ]Bg = x + 2 s ]A , Bg = x + 5 olur.
Buradan s ^]A , Bg # Bh = s ]A , Bg $ s ]Bg
s ^]A , Bg # Bh = ]x + 2g]x + 5g = 54
x 2 + 7x + 10 = 54 & x 2 + 7x - 44 = 0 & ]x + 11g]x - 4g = 0
3 2 x

olup x =- 11 veya x = 4 olarak bulunur. Kümelerin eleman sayıları


negatif değer alamayacağından x = 4 olur. Bu durumda s ]Bg = 6 tir.

ÖRNEK 13:

A = " x ; x ! 6a, 10@, x ! Z , , B = " x ; x ! 63, a@, x ! Z , ve a bir tam sayı olmak üzere A # B nin tüm
elemanlarını içeren en küçük dairenin çevresi 5r olduğuna göre a nın alabileceği değerlerin toplamını
bulunuz.

ÇÖZÜM:

a bir tam sayı olduğuna göre A # B nin tüm elemanlarını içeren en


küçük dikdörtgenin boyutları ]10 - ag ve ]a - 3g birimdir.
(a - 3)

O Bu dikdörtgeni kapsayan en küçük çember, dikdörtgenin çevrel


çemberidir. Çemberin yarıçapı r olsun.
2rr = 5r olup 2r = R = 5 br dir.
( 10 - a )

Pisagor teoreminden
]10 - ag2 + ]a - 3g2 = R 2 = 5 2 = 25
100 - 20a + a 2 + a 2 - 6a + 9 = 25
2a 2 - 26a + 84 = 0
a 2 - 13a + 42 = 0
]a - 6 g $ ]a - 7 g = 0
a = 7 veya a = 6 dır. Bu durumda a nın alabileceği değerler toplamı 6 + 7 = 13 olur.

Matematik 9 | 83
KÜMELER

3. Bağıntı
İki kavram arasında kurulan ilişki ya da bağ bağıntı olarak tanımlanır. Bir önceki ünitede bahsedildiği gibi
veriler arasındaki ilişkiler (üretim ve tüketim arasındaki ilişki, yıllara göre işsizlik oranı vs.) birer bağıntı örneği
olarak verilebilir.

A ve B herhangi iki küme olsun. A # B nin her alt kümesine A dan B ye bağıntı denir. Çoğunlukla
b, b 1, b 2, ... ile gösterilir. b 3 A # B dir.
Bu tanıma göre s ]Ag = m, s ]Bg = n olarak verildiğinde s ]A # Bg = m $ n olduğu için A dan B ye
tanımlanabilecek bütün bağıntıların sayısı, 2 m.n olarak hesaplanır.
^x, y h ! b ise bu durum y b x şeklinde yazılır ve y elemanı, b bağıntısı ile x elemanına bağlıdır diye
okunur.
A # A nın her alt kümesine A da bir bağıntı denir.

ÖRNEK 1:
A = " a, b, c, d ,, B = " 5, 7 , kümeleri veriliyor. Aşağıda tanımlanan kümelerin A dan B ye bağıntı olup
olmadığını inceleyiniz.
b 1 = "^a, 5h, ^c, 5h, ^ d, 5h, ^ d, 7h,
b 2 = "^c, 5h, ^a, 7h, ^d, 1h,
b3 = ! +
b 4 = "^b, 5 h, ^a, 7 h, ^5, c h, ^c, 7 h, ^a, 5 h,
b5 = A # B

ÇÖZÜM:
A # B = "^ a, 5h, ^ a, 7h, ^b, 5h, ^ b, 7h, ^ c, 5h, ^ c, 7h, ^ d, 5h, ^d, 7h, dir.
b 1 3 A # B olduğundan b 1 , A dan B ye bir bağıntıdır.
^d, 1 h z A # B olduğundan b 2 M A # B dir. b 2 , A dan B ye bir bağıntı değildir.
" , 3 A # B olduğundan b 3 A dan B ye bir bağıntıdır.
^ 5, c h z A # B olduğundan b 4 M A # B dir. b 4 , A dan B ye bir bağıntı değildir.
A # B 3 A # B olduğundan b 5 A dan B ye bir bağıntıdır.
ÖRNEK 2:
A = " a, b , B = " 1, 2, 3 , kümeleri veriliyor. Buna göre
a) A dan B ye tanımlanan bağıntı sayısını bulunuz.
b) A dan B ye tanımlanan kaç bağıntıda ^a, 1 h veya ^b, 1 h elemanlarının bulunduğunu hesaplayınız.
c) B de tanımlanan kaç bağıntıda ^1, 3 h elemanının bulunduğunu, ^3, 1 h elemanının bulunmadığını
hesaplayınız.
ç) b = "^x, y h ; x $ y, ^x, y h ! B # B , bağıntısının elemanlarını yazınız.

ÇÖZÜM:
a) s ]A # Bg = 2 $ 3 = 6 olduğundan A dan B ye bağıntı sayısı = 2 2 $ 3 = 2 6 = 64 olur.
b) Tüm bağıntıların sayısı: 2 6 = 64 tür.
A # B kümesinin ^a, 1 h ve ^b, 1 h elemanları dışında 4 tane elemanı olduğu için içerisinde ^a, 1 h ve
^b, 1 h elemanlarının bulunmadığı bağıntı sayısı: 2 4 = 16 dır.
Bu durumda ^a, 1 h veya ^b, 1 h elemanlarının bulunduğu bağıntı sayısı: 64 - 16 = 48 olur.
c) B # B de s ^B # Bh = s ^Bh $ s ^Bh = 3 $ 3 = 9 olduğundan ^1, 3 h ve ^3, 1 h elemanları dışında 7 eleman
vardır. Dolayısıyla oluşan 2 7 = 128 tane bağıntıya ^1, 3 h elemanı eklenir, ^3, 1 h eklenmezse 128 tane
bağıntıda ^1, 3 h elemanı bulunur, ^3, 1 h elemanı bulunmaz.
ç) b = "^ 1, 1h, ^ 2, 1h, ^ 2, 2h, ^ 3, 1h, ^ 3, 2h, ^ 3, 3h,

84 | Matematik 9
KÜMELER

Bağıntının Grafiği
Bir bağıntının grafiği bu bağıntının elamanlarının analitik düzlemde işaretlenmesiyle veya Venn şeması
gösterimiyle elde edilir.
ÖRNEK 3:
A = " 1, 3, 5, 7 , ve B = " - 1, 0, 1, 2 , kümeleri verilsin.
b = "^1, - 1h, ^3, 0h, ^1, 1h, ^ 7, 2h, ^ 7, - 1h, bağıntısının grafiğini çiziniz.

ÇÖZÜM:
1) Venn Şeması ile Gösterim 2) Analitik Düzlemde Gösterim
B
b

A B
1 -1 2

3 0 1

5 1 A
0 1 2 3 4 5 6 7

7 2 -1

Bağıntının Tersi
A dan B ye tanımlı b = "^x, y h ; x ! A ve y ! B , bağıntısının tersi b -1 = "^y, x h ; y ! B ve x ! A ,
bağıntısıdır.
^x, y h ! b + ^y, x h ! b -1 ve
b 3 A # B + b -1 3 B # A dir.

ÖRNEK 4:
A = " - 3, - 2, - 1, 0, 1, 2, 3, 4 ,, B = " 1, 3, 4, 5 , kümeleri için A dan B ye
b = "^- 3, 3h, ^- 1, 4h, ^0, 1h, ^- 1, 5h, ^1, 4h, ^ 3, 5h, ^ 4, 4h, ^ 2, 3h, bağıntısı veriliyor.
Buna göre b ile b -1 bağıntılarının grafiklerini analitik düzlemde çiziniz.

ÇÖZÜM:

b = "^- 3, 3h, ^- 1, 4h, ^0, 1h, ^- 1, 5h, ^1, 4h, ^ 3, 5h, ^ 4, 4h, ^ 2, 3h,
b -1 = "^3, - 3h, ^4, - 1h, ^1, 0h, ^5, - 1h, ^4, 1h, ^5, 3h, ^4, 4h, ^3, 2 h, olur.
B
y=x
5
b
4

3 Yandaki şekilde de görüldüğü gibi b ile b -1 bağıntılarının


grafikleri y = x (1. açıortay doğrusu ) doğrusuna göre
2
simetriktir.
1
A
-3 -2 -1 0 1 2 3 4 5
-1

-2
b -1
-3

Matematik 9 | 85
KÜMELER

ÖRNEK 5:
A = " x ; - 3 < x # 4, x ! Z , olmak üzere A da tanımlanan b = #^x, y h ; y x = 4, ^x, y h ! A # A -
bağıntısı veriliyor. b ve b -1 bağıntılarının elemanlarını yazınız ve b bağıntısını venn şeması yöntemiyle
gösteriniz.

ÇÖZÜM: b

A = " - 2, - 1, 0, 1, 2, 3, 4 , kümesine göre istenen bağıntının koşulu


3

göz önünde bulundurulduğunda -2


4
y x = 4 & y = 4 için x = 1
= y = 2 için x = 2 1
-1

= y =- 2 için x = 2 dir. Bu durumda


b = "^1, 4h, ^2, 2h, ^2, - 2 h, ve b -1 = "^4, 1h, ^2, 2h, ^- 2, 2h, olur. 0
2

A dan A ya tanımlı b bağıntısına ait grafik yanda verilmiştir.


ÖRNEK 6:
C = " 1, 2, 3, 4 , ve D = " 0, 1, 2, 5 , kümeleri veriliyor.
a) b 1 = #^x, yh ; x 2 + y 2 # 13, ^x, y h ! C # D - bağıntısını bulunuz.
b) b 2 = "^x, yh ; y = x + 1, ^x, y h ! C # C , ise b 2-1 bağıntısını bulunuz.

ÇÖZÜM:

C ve D kümelerinin elemanları b 1 ve b 2 bağıntılarında belirtilen özelliklere göre yazılırsa


b 1 = "^ 1, 0h, ^ 1, 1h, ^ 1, 2h, ^ 2, 0h, ^ 2, 1h, ^ 2, 2h, ^ 3, 0h, ^ 3, 1h, ^ 3, 2h,
b 2 = "^1, 2h, ^2, 3h, ^3, 4h, ise b 2-1 = "^2, 1h, ^3, 2h, ^4, 3h, olur.

Sıra Sizde

SORU :
A = " x ; - 20 1 x # 18, x = 2k, k ! Z , kümesinde tanımlı
b = "^x, y h ; y = 3x, ^x, y h ! A # A , bağıntısı veriliyor.
b -1 ters bağıntısının grafiğini çiziniz.

ÇÖZÜM:

Yukarıdaki A kümesinde tanımlı bir b bağıntısı yazınız. Bağıntınızın tersinin grafiğini


çiziniz.

86 | Matematik 9
KÜMELER

ALIŞTIRMALAR

1 A # B = "^a, 8 h, ^b, 0 h, ^a, - 1 h, ^c, 0 h, ^c, 8 h, ^b, - 1 h, ^a, 0 h, ^b, 8 h, ^c, - 1 h,


C # D = "^- 1, c h, ^0, c h, ^3, d h, ^3, c h, ^- 1, d h, ^0, d h,
s 6]D - Ag # ]B , Cg@ değerini bulunuz.

2 ^2a + b - 3, a + 3b - 5 h = ^a - 5b, 7a + 3 h ise 6 a ! Z olmak üzere


a $ b değerini bulunuz. M = " x ; a # x # 8, x ! Z ,
N = " y ; 2 # y # a, y ! Z ,
M # N nin tüm elemanlarını içeren en küçük
dikdörtgenin alanı 9 br 2 olduğuna göre a
kaçtır? Bulunuz.

3 b x + 7 , 1 l = ^3, x^y + 5hh ise x - y değerini


8
bulunuz.

7 A = " - 2, 1, 3, 5 , ve B = " 3, 8 , kümeleri


veriliyor. Buna göre
a) A dan B ye kaç farklı bağıntı yazılabileceğini
bulunuz.
4 A = " x ; x 1 4, x ! N ,
b) A dan B ye tanımlanan kaç bağıntıda ^1, 8 h
B = " - 2, - 1, 0, 1 , kümeleri veriliyor.
ve ^5, 8 h bulunur?
a) s ]A # Bg değerini bulunuz. c) A dan B ye tanımlanan kaç bağıntıda ^5, 8 h
b) ]A # Bg yi analitik düzlemde gösteriniz. bulunur ancak ^3, 3 h bulunmaz?
c) ]A # Bg nin kaç tane alt kümesinde ^0, 0 h ç) A dan B ye tanımlanan en çok bir elemanlı
bulunmaz? kaç tane bağıntı vardır?
ç) ]A # Bg nin kaç alt kümesinde ^1, - 1 h
veya ^3, 1 h bulunur?

5 K = " x ; x 1 20, x = 3k, k ! N , 8 A = " - 2, 1, 3, 5 , B = " 3, 8 , kümeleri veriliyor.


L = # y ; 10 1 y 2 1 145, y ! Z - Buna göre aşağıdaki bağıntıları koordinat
M = " z ; 8 1 z # 20, z ! Z , ise
sisteminde gösteriniz.
s ^^K # M h + ^K # L hh değerini bulunuz. a) b 1 = "^x, y h ; x 1 y, ^x, y h ! A # B ,
b) b 2 = #^x, y h ; x 2 + y 2 2 15 ^x, y h ! A # B -

Matematik 9 | 87
KÜMELER

ÖLÇME VE DEĞERLENDİRME -1

1
Aşağıdaki ifadelerden hangisi iyi tanımlanmış 6 I. A = " x ; - 1 1 x 1 7, x ! N ,
değildir? II. B = " y ; y 2 - 1, y ! Z ,
A) Kış mevsiminin ayları III. C = # z ; z 2 + 5 = 0, z ! N -
B) Apartmanımızdaki insanlar IV. D = " t ; 1 1 t 1 2, t ! R ,
C) Güzel çocuklar V. E = " m ; m haftanın bir günüdür ,
D) Ülkemizin coğrafi bölgeleri Yukarıda verilen kümelerden hangisi ya da
E) İki basamaklı doğal sayılar hangileri sonlu kümedir?
A) Yalnız V B) I ve III C) III ve IV

2 A = " x ; - 6 # x # 25, x = 2k, x ! Z , D) I, II ve IV E) I, III ve V


kümesinin eleman sayısı kaçtır?
A) 16 B) 17 C) 18 D) 19 E) 20

3 A = " 1, 2, ! 1 +, " 1, 2 ,, 3, " 1, 2, 3 ,, kümesi için 7


A, B ve C nin evrensel kümesi E olmak üzere
aşağıdakilerden hangisi yanlıştır? I. A 3 E
II. Q 3 A
" 1, 2 , ! A
III. ]A 3 Bg / ]B 3 Cg & A 3 C
A)
" 1, 2 , 3 A
IV. ]A 3 Bg / ]B 3 Ag + A = B
B)
C) !1 + 3 A
V. A 3 A
D) " 1, 2, 3 , ! A
Yukarıdaki önermelerden kaç tanesi doğrudur?
E) !2 + ! A
A) 1 B) 2 C) 3 D) 4 E) 5

4 I. Negatif doğal sayılar


II. Rakamlar
III. Türkçede P harfi ile başlayan aylar
IV. x 3 = 27 denklemini sağlayan tam sayılar Bir kümenin eleman sayısı 4 arttırıldığında
8
V. A = "" ,, alt küme sayısı 60 arttığına göre bu kümenin
kendisi hariç alt küme sayısı kaçtır?
Yukarıdakilerden hangisi ya da hangileri boş
küme belirtir? A) 2 B) 3 C) 4 D) 5 E) 6
A) Yalnız I B) I ve II C) I ve III
D) III ve IV E) III ve V

A, B ve C kümelerinin evrensel kümesi E olmak 9 s ]A + B g s ]A = B g s ]B = Ag


5 = = ve
üzere s ]Ag + s ]B'g = 15 2 3 4
s ]A'g + s ]Bg = 21 , s ]C'g = 7 olduğuna göre s ]A = Bg + s ]B = Ag = 28
s ]Cg kaçtır? olduğuna göre s ]A + Bg kaçtır?

A) 7 B) 8 C) 9 D) 10 E) 11 A) 8 B) 10 C) 12 D) 14 E) 16

88 | Matematik 9
KÜMELER

10 ^a + 3b, 5a - 2b + 4 h = ^3, 2 h olduğuna göre 14 A B


a + b toplamı kaçtır?
A) 0 B) 1 C) 2 D) 3 E) 4

C
Şekilde verilen taralı bölge hangi seçenekte
doğru verilmiştir?
A) ]A , B , Cg - ]A , Cg
11 T = " x ; 25 1 x # 180, x = 5k, k ! N , B) ]A - Bg , ]C - Ag
R = " y ; 60 # y 1 200, y = 3t, t ! N , C) ]A , Bg - ]A + B + Cg
kümeleri veriliyor. Buna göre, R = T kümesinin D) ^A - ]B , Cgh , ]B - Ag
eleman sayısı aşağıdakilerden hangisi olur? E) ]C - Ag , ]B , A'g

A) 32 B) 34 C) 36 D) 38 E) 40
15
Futbol veya voleybol oynayanlardan
oluşan bir grupta hem futbol hem voleybol
oynayanlar, sadece futbol oynayanların 3
katının 5 eksiğidir. Voleybol oynayanlar,
futbol oynayanların 2 katıdır. En çok bir oyun
oynayan 13 kişi olduğuna göre grup kaç
kişidir?
12 T = # x ; x 2 # 9, x ! N - veriliyor.
Aşağıdakilerden hangileri T den Z ye bir A) 17 B) 19 C) 21 D) 23 E) 25
bağıntı değildir?

I. b 1 = "^1, 0h, ^- 3, 5h, ^0, - 5h,


II. b 2 = &^3, 3h, b 2, 5 l0
1

III. b 3 = " , A ve B kümeleri E evrensel kümesinin iki alt


kümesi olmak üzere ]B = Ag ' + ]A + Bg '
16
IV. b 4 = T # T
ifadesi aşağıdakilerden hangisine eşittir?
A) I-III B) II-III C) I-II A) B B) E C) Bl D) Q E) Al
D) II-IV E) III-IV

13 A + B = " h, a, m, s, i , ve C = " k, e, f, a, l , 17
Bir sınıftaki öğrencilerin %40 ı matematikten,
olduğuna göre ]A # Cg + ]B # Cg kümesinin %20 si hem matematik hem de fizikten kalmış,
eleman sayısı kaçtır? %70 i ise fizikten geçmiştir. Sadece fizikten
kalan 4 kişi olduğuna göre hem matematik
A) 10 B) 15 C) 20 D) 25 E) 30 hem de fizikten geçen kaç öğrenci vardır?
A) 18 B) 20 C) 24 D) 25 E) 30

Matematik 9 | 89
KÜMELER

ÖLÇME VE DEĞERLENDİRME -2

1 A = " x ; x = 5k, x 1 250, k ! N , 5 A = " 1, 2, 3, 4, 5 , ise


B = " y ; y = 4t, y # 300, t ! N , ise A da tanımlı kaç bağıntıda ^1, 2 h bulunur,
s ]A , Bg kaçtır? ^5, 3 h bulunmaz?
A) 75 B) 84 C) 96 A) 223 B) 224 C) 225 D) 226 E) 227
D) 113 E) 122

2 A = " 0, 1, 2, 3, 4 , kümesinin alt kümelerinin


kaç tanesinde en az bir çift sayı bulunur?
A) 24 B) 26 C) 28 D) 30 E) 32
6 A + B ! Q olmak üzere s ]Ag = 7 ve
s ]Bg = 4 olduğuna göre s ]A , Bg nin
alabileceği en küçük değer ile en büyük
değerin toplamı kaç olur?
A) 16
B) 17
3 A = " x ; - 1 # x 1 3, x ! Z , C) 18
B = " y ; 3 1 y 1 12, y asal , D) 19
C = " z ; z 1 7, z ! N , ise E) 20
s 6]A , Cg # ]B - Cg@ kaçtır?
A) 12 B) 13 C) 14 D) 15 E) 16

4 A = " 1 , 2 , a, b , 7 A , B = " 2, 3, 4, 5 , 7 ,
B = " 1, 2, 3, 4, 5, 6, a, b , kümeleri veriliyor. A # C = "^3, 5h, ^3, 8h, ^4, 5h, ^4, 8h,
A 3 C 3 B koşulunu sağlayan kaç farklı C kümeleri veriliyor. A + B = Q olduğuna göre
kümesi yazılabilir? B + C aşağıdakilerden hangisidir?

A) 16 B) 17 C) 18 D) 19 E) 20 A) " 4, 2 ,
B) " 5, 6 ,
C) !5 +
D) !7 +
E) " 3, 8 ,

90 | Matematik 9
KÜMELER

8
Alt küme sayısı, eleman sayısının 32 katı olan 11 A = " x ; 10 # x 1 80, x ! Z , kümesinde
bir küme kaç elemanlıdır? tanımlı b = "^x, y h ; y = 5x , bağıntısının
eleman sayısı kaçtır?
A) 7
A) 10
B) 8
B) 9
C) 9
C) 8
D) 10
D) 7
E) 11
E) 6

9 A = " 1, 2, 3, 4 , olduğuna göre A # A nın kaç 12 " 0, 1, 2, 3, 4 , kümesinin alt kümelerinin kaç
alt kümesinde ^1, 1 h, ^2, 2 h, ^3, 3 h, ^4, 4 h tanesinde ne 1 ne de 2 bulunur?
birlikte bulunur?
A) 8 B) 12 C) 16 D) 24 E) 32
A) 218
B) 216
C) 214
D) 212
E) 210

13 A = " x ; x 1 650, x ! Z + , kümesinin


elemanlarından kaç tanesi 7 veya 6 ya bölünür,
4 e bölünemez?
A) 115 B) 110 C) 100
D) 96 E) 85

10 A = " 1, 2, 3, 4, 5, 6, 7, 8, 9, 10 , kümesinin
elemanları, boş olmayan iki ayrık kümeye kaç
farklı şekilde ayrılabilir?
A) 185
B) 255
C) 511
D) 678 14
Tam sayılar kümesinde tanımlı
E) 1023 b = "^x, y h ; y = 2x + m , bağıntısı veriliyor.
^3, - 5 h ! b -1 ise m kaçtır?

A) 10 B) 11 C) 12 D) 13 E) 14

Matematik 9 | 91
KÜMELER

15 A = " 1, 2, 3, 4, 5, 6, 7, 8, 9 , kümesinin 19
Bir ilçedeki insanların %80 i A marketinden,
herhangi iki ardışık sayı içermeyen kaç alt %75 i B marketinden, %65 i C marketinden
kümesi vardır? alışveriş yapmaktadır. Bu insanların en az
yüzde kaçı bu üç marketten de alışveriş
A) 34 B) 45 C) 55 D) 78 E) 89 yapmaktadır?
A) 12 B) 14 C) 16 D) 18 E) 20

16 A = " x ; x 1 50, x ! N , kümesinin


elemanlarından kaç tanesi ne 2 ne de 3 ile
bölünür?
A) 40 B) 41 C) 42 D) 43 E) 44
20
Bir sınıftaki öğrencilerden kimya dersinden
geçen herkes edebiyat dersinden de geçmiştir.
Bu sınıfta ne matematik ne de kimya dersinden
geçen kişi bulunmamaktadır.
• En çok bir dersten geçen 15 kişi,
• Edebiyat dersinden geçip kimya dersinden
kalan 7 kişi,
• Kimya dersinden kalan 18 kişi bulunduğuna
göre
17 A + B nin alt küme sayısı; A = B nin alt küme
sayısının 8, B = A nın alt küme sayısının 1 sadece edebiyat dersinden geçen kaç kişi
32 vardır?
katıdır.
s ]A , Bg = 17 olduğuna göre s ]A # Bg
A) 8 B) 7 C) 5 D) 4 E) 3

kaçtır?
A) 105 B) 110 C) 120
D) 135 E) 140

18 A = " 1, 2, 3, 4, 5, 6, 7 , kümesinin alt 21 A = " x x ! 6a, 7@, x ! Z}


kümelerindeki elemanların sayı değerleri B = " y y ! 6-10, a@, y ! Z}
toplamı kaçtır? kümeleri veriliyor.
A) 1578 B) 1680 C) 1792 a bir tam sayı olmak üzere A # B nin tüm
D) 2560 E) 2816 elemanlarını içeren en küçük dairenin alanı,
169r
4 olduğuna göre a nın alabileceği
değerler toplamı kaçtır?
A) -4 B) -3 C) 0 D) 2 E) 5

92 | Matematik 9
SAYILAR VE CEBİR
9.3. DENKLEMLER ve EŞİTSİZLİKLER
Neler Öğreneceksiniz? Denklem ve Eşitsizlikleri
Öğrenmek Neden Önemlidir?

• Sayı kümelerini (doğal sayılar, tam sayılar, • Tam sayılar ve rasyonel sayılar, insanların
rasyonel sayılar, irrasyonel sayılar ve yaşadığı çevrede karşılaştığı olay ve
gerçek sayılar) ve bu kümelerin arasındaki durumları doğru biçimde ölçmesinde ve
ilişkiyi, tanımlayabilmesinde yeterli olmamıştır.
• Gerçek sayılar kümesindeki işlemlerin Daha ayrıntılı ve güvenilir bilgilere
özelliklerini, ulaşabilmek için farklı sayı kümelerine
ihtiyaç duyulmuştur.
• Birinci dereceden bir ve iki bilinmeyenli
denklem ve eşitsizlik sistemlerinin • Ülkemizdeki bölgesel sıcaklık
özelliklerini, bunların çözüm kümelerini değerlerindeki sapmalarda,
analitik düzlemde göstermeyi ve üretim miktarlarındaki hata payı
kartezyen çarpımını, hesaplamalarında mutlak değerli
denklemler ve eşitsizlikler kullanılır.
• Mutlak değer kavramını, mutlak değerli
denklem ve eşitsizliklerin çözüm • Bunun yanında çok büyük veya çok küçük
kümelerini bulmayı, sayıları içeren durumlarla ilgili ifadelerde
üslü denklemler kullanmak, işlemleri
• Üslü ve köklü ifadeleri içeren
kolaylaşmaktadır.
denklemlerle ilgili uygulamaları yapmayı,
• Günlük hayattaki boy kilo
• Oran ve orantı (doğru ve ters orantı)
karşılaştırmalarında, nesnelerin
kavramlarını ve bunların çözüm
maketlerinin yapımında oran ve orantı
yöntemlerini, altın oran kavramını,
kavram ve hesaplamaları kullanılmaktadır.
bunlarla ilgili günlük hayat problemlerini
modellemeyi ve çözmeyi, • Gerçek hayatta karşılaşılan problemlerin
çözümünde denklem veya eşitsizlikleri
• Farklı problem türleriyle ilgili (sayı,
kullanmak, denklemi anlamayı ve
kesir, yaş, yüzde, kâr zarar, faiz,
yorumlamayı kolaylaştıracağından
karışım, hareket, işçi havuz vb.) bilgileri
çözüme ulaşmada fayda sağlayacaktır.
yorumlamayı ve çözmeyi, birimler arası
GSM operatörlerinin tarifelerinden uygun
dönüşümleri öğreneceksiniz.
ve ekonomik olanı seçmek, alışveriş
merkezlerindeki indirimlerden kârlı
olanları belirlemek bunlara örnek olarak
gösterilebilir.

Matematik 9 | 93
DENKLEMLER VE EŞİTSİZLİKLER

Hârizmi’nin Özbekistan’ın Khiva (Hive) şehrinde bulunan heykeli

İ
nsanlar, sayıları bulmadan önce sahip oldukları nesnelerin miktarını
belirlemek için sayma ve eşleme gibi yöntemler kullanmışlardır.
MÖ 3500’lü yıllarda Sümerler tarafından kil tabletlere yazılan bazı
şekiller, bilinen en eski rakamlar olarak kabul edilmektedir.
Hintli Matematikçi Brahmagupta (Bramagupta 598-660); “Siddhanta”
(Sidanta) adlı eserinde, dokuz ayrı sayı işareti ve sıfır ile hesap yapmaya
dair kuralları göstermiştir. Brahmagupta bu eserinde ayrıca borç
anlamına gelen negatif sayılardan da bahsetmiştir.
Hârizm Türklerinden olduğu bilinen Matematikçi Hârizmi'nin (780-
850) en önemli özelliği, İslam dünyasında ilk defa yuvarlak bir şekil
olan sıfırla beraber Hint rakamlarını ve ondalık konumlu sayı sistemini
kullanmış olmasıdır. Hârizmi, “Kitâbü’l-Muhtasar fi hisâbi’l-cebr ve’l-
mukabele” adlı eserinde sıfırın dâhil edildiği yeni sayı sistemi ile nasıl
çok yüksek basamaklı sayıların kolayca gösterilebileceğini anlatır. Bu
kitap, adında “cebir” kelimesini taşıyan ilk matematik kitabıdır.
Hârizmî, “Sekiz, diğer sekizden çıkınca geriye bir şey kalmaz. Bu
takdirde hanenin boş kalmaması için bir dairecik koy! Dairecik, boş
hanenin yerine geçmek zorundadır.” diyerek sıfırı tanımlamıştır.
Hârizmi bilimin gelişmesine sağladığı katkılardan dolayı saygı
duyulması gereken İslam bilginlerindendir. Cebirsel niceliği, açık şekilde
ortaya koyan ve cebirsel denklemleri çözerken analitik çözüm yanında
geometrik çizimi de kullanan ilk matematikçi Hârizmi'dir. Onun bu ilme
yaptığı bir diğer önemli katkı ise cebirsel denklemleri çözerken yapılacak
işlemleri, bir sıra düzenine koymak suretiyle bunları cebre uygulamasıdır.
Bu yönteme daha sonra matematik tarihinde “algoritma” denilmiştir.

94 | Matematik 9
DENKLEMLER VE EŞİTSİZLİKLER

9.3.1. SAYI KÜMELERİ

Etkinlik

Eski uygarlıklar (Sümer, Babil, Mısır, Roma vb.) döneminde sayma


işlemleri için farklı yöntemler ve semboller kullanılmıştır. “Bire bir
eşleme yöntemi” ise en çok kullanılan yöntemlerden biri olmuştur.
Buna göre
1. “Bire bir eşleme yöntemi” sözünden ne anlıyorsunuz?
2. Rakamların ve sayı saymanın bilinmediği bir dönemde yaşadığınızı
düşünün. Bu durumda
a) Sınıfınızdaki öğrenci sayısını nasıl ifade ederdiniz? Bir şehrin ya
da ülkenin nüfusu gibi kalabalık gruplardaki kişi sayısını nasıl
bulurdunuz?
b) Günlerin takibi, alışveriş, miras paylaşımı gibi durumlarda nasıl
bir yol izlerdiniz?
3. Nesneleri saymanın dışında bir bütünü parçalara ayırıp ifade etme,
hava sıcaklığındaki değişiklikleri belirtme gibi farklı konularda
kolaylık sağlanması için ihtiyaç duyulan sayı kümeleri hangileridir?
4. Şu ana kadar öğrenmiş olduğunuz sayı kümelerini belirterek bu
kümeleri birbirleriyle ilişkilendiriniz.

Matematik 9 | 95
DENKLEMLER VE EŞİTSİZLİKLER

1. Sayı Kümeleri Arasındaki İlişki


Doğal Sayılar Kümesi
Sayıları yazmak için kullanılan sembollere rakam adı verilir.
Rakamlar kümesi " 0, 1, 2, 3, 4, 5, 6, 7, 8, 9 , dir.
Pozitif tam sayılar kümesi " 1, 2, 3, 4, 5, 6, 7, ... , dir.
Pozitif tam sayılar kümesine sıfırın eklenmesiyle elde edilen kümeye doğal sayılar kümesi denir.
N ile gösterilir. N = " 0, 1, 2, 3, 4, 5, 6, 7, ... , dir.

a, b, c birer rakam olmak üzere


ab iki basamaklı bir sayı olsun. ab = 10a + b
abc üç basamaklı bir sayı olsun. abc = 100a + 10b + c şeklinde yazılır.

ÖRNEK 1:
İki basamaklı ab doğal sayısı verilsin. ab sayısının rakamları yer değiştirdiğinde bu sayı 45 azaldığına göre
kaç farklı ab sayısı olduğunu bulunuz.

ÇÖZÜM:
ab sayısının rakamları yer değiştirdiğinde oluşan yeni sayı ba olur.
ab - 45 = ba olduğundan
10a + b - 45 = 10b + a & 9a - 9b = 45 & 9 ]a - bg = 45 & a - b = 5 tir.
Bu eşitliği sağlayan ab sayıları 94, 83, 72, 61, 50 olmak üzere 5 farklı sayı vardır.

ÖRNEK 2:
a ve b birer doğal sayı olmak üzere a.b = 35 ise a + b nin alabileceği en büyük ve en küçük değerleri
bulunuz.

ÇÖZÜM:
Toplamın en büyük olması için sayılar birbirine en uzak durumda olmalıdır. a = 35, b = 1 alınırsa
a + b = 36 tir.
Toplamın en küçük olması için sayılar birbirine en yakın durumda olmalıdır. a = 7, b = 5 alınırsa a + b = 12
tir.

Tam Sayılar Kümesi


Doğal sayılar kümesine sayma sayılarının negatiflerinin ilave edilmesiyle oluşan kümeye tam sayılar
kümesi denir. Z ile gösterilir.

Z = " ... - 4, - 3, - 2, - 1, 0, 1, 2, 3, 4, ... ,


Z + = " 1, 2, 3, 4, ... , kümesi pozitif tam sayılar kümesi,
Z - = " ..., - 4, - 3, - 2, - 1 , kümesi negatif tam sayılar kümesi olarak isimlendirilir.
0 pozitif tam sayı ya da negatif tam sayı değildir. Sıfırın işareti yoktur.
Z = Z- , !0 + , Z+

Tam sayılar, sayı doğrusu üzerinde aşağıdaki gibi gösterilir:

..... - 4 -3 -2 -1 0 1 2 3 4 .....
1444444444442444444444443 1444444444442444444444443
Negatif tam sayılar Pozitif tam sayılar

96 | Matematik 9
DENKLEMLER VE EŞİTSİZLİKLER

ÖRNEK 3:
a,b,c birbirinden farklı pozitif tam sayılar olmak üzere 5a + 3b + 7c ifadesinin en küçük tam sayı değerini
bulunuz.

ÇÖZÜM:
5a + 3b + 7c ifadesinin en küçük tam sayı olması için katsayısı büyük olan terimlerdeki bilinmeyenler
yerine sırasıyla en küçük pozitif tam sayılar yazılmalıdır.
Bu durumda c = 1, a = 2, b = 3 alınırsa
5 $ 2 + 3 $ 3 + 7 $ 1 = 10 + 9 + 7 = 26 bulunur.

ÖRNEK 4:
a, b, c birbirinden farklı pozitif tam sayılardır. a + 3b + 7c = 60 olduğuna göre a nın alabileceği en büyük
değeri bulunuz.

ÇÖZÜM:
a = 60 - ]3b + 7cg = 60 - 13 = 47
. .
2 1 (En büyük)

ÖRNEK 5:
x ve y birer pozitif tam sayı olmak üzere 3x + 4y = 45 olduğuna göre y nin alabileceği değerler kümesini
bulunuz.

ÇÖZÜM:
3x + 4y = 45 ise 3x = 45 - 4y
45 - 4y 4y
x= 3 = 15 - 3 2 0 olmalıdır.
y sayısının 3 ün katı olduğu görülür. Bu durumda y nin alabileceği değerler 3, 6 ve 9 dur.

2 ile tam bölünebilen tam sayılara çift tam sayılar denir. k ! Z için 2k ile gösterilir.
Çift tam sayılar kümesi: Ç = " ..., - 6, - 4, - 2, 0, 2, 4, 6, ... ,
2 ile tam bölünemeyen tam sayılara tek tam sayılar denir. k ! Z için 2k + 1 veya 2k - 1 ile gösterilir.
Tek tam sayılar kümesi: T = " ..., - 5, - 3, - 1, 3, 5, ... ,

ÖRNEK 6:
a) “Her çift tam sayının karesi yine bir çift tam sayıdır.”
b) “Her tek tam sayının karesi yine bir tek tam sayıdır.” ifadelerinin doğru olduğunu gösteriniz.

ÇÖZÜM:
a) n bir çift tam sayı olsun. O hâlde n = 2k olacak şekilde bir k ! Z vardır.
n 2 = ]2kg2 = 4k 2 = 2 $ 2k 2 = 2t, ^t ! Z h tir.
7
t
Buradan 2t çift olduğundan n 2 çift tam sayı olur.
b) n bir tek tam sayı olsun. O hâlde n = 2k + 1 olacak şekilde bir k ! Z vardır.
n 2 = ]2k + 1g2 = 4k 2 + 4k + 1 = 2 a 2k + 2k k + 1 = 2m + 1, ]m ! Zg tir.
2
14444244443
m
Buradan 2m + 1 tek olduğundan n 2 de tek tam sayı olur.

Matematik 9 | 97
DENKLEMLER VE EŞİTSİZLİKLER

Ç = Çift ve T = Tek sayılardaki işlemler şu şekilde genelleştirilir:


Ç!Ç = Ç Ç.Ç = Ç T n = T ^n ! N +h
T!T = Ç T$T = T Ç n = Ç ^n ! N +h
T!Ç = T T$Ç = Ç

ÖRNEK 7:
Aşağıdaki ifadelerin tek ya da çift olma durumlarını belirtiniz.
a) 3 5 - 2 $ 7 4 + 1
b) 4 17 - 3 2 $ 5 10
c) 3 8 $ 5 18 + 2016 $ 9 15 - ]- 11g3
ç) ^24 3 + ^- 3 h2 h + ^- 2 h-1
7

ÇÖZÜM:

Kuvvet alma işlemi, sayının çift ya da tek olmasını değiştirmediğinden


a) 3 5 - 2 $ 7 4 + 1 " T - Ç $ T + T = T - Ç + T = T + T = Ç olup sayı çifttir.
b) 4 17 - 3 2 $ 5 10 " Ç - T $ T = Ç - T = T olup sayı tektir.
c) 3 8 $ 5 18 + 2016 $ 9 15 - ]- 11g3 " T $ T + Ç $ T - T = T + Ç - T = T - T = Ç olup sayı çifttir.
ç) ^24 3 + ^- 3 h2 h + ^- 2 h-1 ifadesinde 2 z Z olduğundan toplamın sonucu tam sayı olmaz.
7 -1

Bu nedenle ^24 3 + ^- 3 h2 h + ^- 2 h-1 sayısı tek ya da çift sayı değildir.
7

ÖRNEK 8:
a, b, c ! Z ve c Y a + 4b
= 0 olmak üzere c = 3 ise aşağıdaki ifadelerden hangisi kesinlikle doğrudur?
a) a çift ise b tektir.
b) b çift ise c tektir.
c) a çift ise c tektir.
ç) c tek ise a tektir.
d) b tek ise a tektir.

ÇÖZÜM:
a + 4b &
c = 3 a+6
4b = 3c
çift
4b çift tam sayı olduğundan a tek tam sayı ise c de tek tam sayıdır, a çift tam sayı ise c de çift tam sayıdır.
Buna göre ç seçeneğindeki bilgiler kesin doğrudur.

98 | Matematik 9
DENKLEMLER VE EŞİTSİZLİKLER

Rasyonel Sayılar Kümesi


Tam sayılar kümesi, karşılaşılan tüm durumları ifade etmek için yeterli değildir. Öneğin 2x = 3 biçimindeki
3
denklemlerin çözümü, 8 gibi iki tam sayının birbirine bölünmesi ile oluşan bazı sayılar, tam sayılar kümesine
dâhil değildir. O hâlde bu tür sayıları da içine alan yeni bir sayı kümesine ihtiyaç duyulur.

a
a, b ! Z, b Y = 0 ve a ile b aralarında asal olmak üzere b şeklindeki sayılara rasyonel sayılar denir.
Rasyonel sayılar kümesi Q ile gösterilir.
Q = & b ; a, b ! Z ve b Y = 00
a
a a
• b ifadesinde her a ! Z için b = 1 alınırsa 1 = a ! Q olduğundan her tam sayı aynı zamanda
bir rasyonel sayıdır. Bu durumda Z 1 Q olur.
a 0 1
• b ifadesinde a = 0, b Y = 0 alınırsa b = 0 $ b = 0 ! Q olur.
0
Örneğin 3 = 0 dır.
a a
• b ifadesinde a Y = 0, b = 0 alınırsa 0 “tanımsız” olur.
2
Örneğin 0 = 0 tanımsızdır.
a 0
• b ifadesinde a = 0, b = 0 olduğunda 0 “belirsiz” olur.
Q + pozitif rasyonel sayılar kümesini; Q - negatif rasyonel sayılar kümesini göstermek üzere
Q = Q - , ! 0 + , Q + yazılır.

Bütün rasyonel sayılar, sayı doğrusu üzerinde işaretlendiğinde herhangi iki rasyonel sayı arasında
sonsuz çoklukta rasyonel sayı bulunur.

a c
Her b , d ! Q; b Y = 0 olsun. İki sayının toplamının yarısı, bu sayıların orta noktasıdır. Bu
= 0, d Y
durumda
a c
b +d a$d+b$c a a$d+b$c c
2 = 2b $ d ! Q değeri için b < 2 $ b $ d < d yazılır.
Bu işlem ne kadar tekrar edilirse edilsin alınan rasyonel sayılar arasında daima başka bir rasyonel

sayı bulunur.
Bu durum Rasyonel sayılar sayı doğrusunda yoğundur. şeklinde ifade edilir.

1
1 +1 3
Örneğin 2 ve 1 rasyonel sayıları dikkate alındığında 2 2 = 4 ! Q vardır.
1 3
2 4
..... - 1 0 1 .....

Bir sayının ondalık açılımında virgülden sonraki bölüm, belli bir kurala göre tekrar ediyorsa bu tür
sayılara devirli ondalık sayılar denir. Virgülden sonraki tekrar eden ilk sayı grubunun (devreden kısım)
üstüne çizgi çizilerek gösterilir.
1
3 = 0, 3333... = 0, 3
56
45 = 1, 24444... = 1, 24
1
2 = 0, 5000... = 0, 50 = 0, 5 (Devreden sayı 0 ise devir belirtilmez.)
Bu sayılar, devirli ondalık sayılardır.

Matematik 9 | 99
DENKLEMLER VE EŞİTSİZLİKLER

ÖRNEK 9:
0, 5 devirli ondalık sayısının rasyonel olup olmadığını inceleyiniz.

ÇÖZÜM:
0, 5 = x olsun. Bu durumda
x = 0, 5555555.... demektir. Her iki taraf 10 ile çarpılırsa
10x = 5, 555555.... olur.
10x = 5, 555...
- x = 0, 555...
5
9x = 5 olduğundan x = 9 olarak bulunur. O hâlde 0, 5 devirli sayısı rasyonel bir sayıdır.

Her devirli ondalık sayı bir rasyonel sayıdır. Devirli ondalık sayının rasyonel sayı karşılığı pratik olarak
Sayının tamamı - Devretmeyen kısım
Devreden basamak sayısı kadar 9, Virgülden sonraki devretmeyen basamak sayısı kadar 0
formülü ile bulunur.

ÖRNEK 10:
2, 0 17 sayısının kesirli karşılığını bulunuz.

ÇÖZÜM:
Sayının tamamı 2017, devretmeyen kısım 20 dir. Virgülden sonra iki tane devreden, bir tane devretmeyen
rakam olduğundan
2017 - 20 1997
2, 0 17 = 990 = 990 bulunur.

Sıra Sizde

SORU:
a ve b birer rakam olmak üzere
a, b + b, a = 10 şartını sağlayan kaç tane ab iki basamaklı sayısı vardır?

ÇÖZÜM:

TARİHÇE

P isagor (MÖ 570-495) tam sayılar dışında sayının olmadığını


düşünmekte ve tüm sayıların 1 sayısından elde edilebileceğine
inanmaktaydı. O dönemde Yunan matematiğinde tüm uzunluklar, tam
sayılar ya da tam sayıların oranı (rasyonel sayılar) kullanılarak ifade
1 2
edilmekteydi. Oysa kenar uzunlukları 1 birim olan karenin köşegen
uzunluğu 2 birimdir ve bu sayı rasyonel değildir. Nihayetinde Pisagor,
düşüncesinde yanıldığını ve rasyonel olmayan sayıların da var olduğunu
kabul etmiştir. 2 sayısının rasyonel olmadığının ispatı,
Aristo (MÖ 384-322) ve Öklid (MÖ 365-300) tarafından yapılmıştır. 1

100 | Matematik 9
DENKLEMLER VE EŞİTSİZLİKLER

ÖRNEK 11:
2 sayısının rasyonel bir sayı olmadığını gösteriniz.

ÇÖZÜM:
2 sayısının rasyonel olduğunu varsayınız.
O hâlde 2 = b ! Q ^a, b ! Z, b Y = 0h olacak şekilde aralarında asal (1 den başka ortak böleni olmayan)
a
a ve b tam sayıları vardır.
Her iki tarafın karesi alınırsa
^ 2 h = b ba l & 2 = a 2 & a 2 = 2b 2 olup a 2 çift sayıdır.
2 2
2

b
a çift sayı ise a da çift sayıdır. a = 2k, ]k ! Zg şeklinde yazılır. Bu değer yerine yazılırsa
2

]2kg2 = 2b 2 & 4k 2 = 2b 2 & b 2 = 2k 2 bulunur. Benzer düşünceyle b 2 çift sayı olduğundan


b de çifttir. b = 2h, ]h ! Zg şeklinde yazılır.
Hem a hem de b nin çift tam sayı olması, aralarında asal olmadığı sonucunu verir. Bu durum varsayımla
çelişir. O hâlde varsayım yanlıştır. 2 rasyonel sayı değildir.
3, 2 7, 2 + 5 gibi sayıların da rasyonel sayı olmadıkları gösterilebilir.

İrrasyonel Sayılar Kümesi


a
a, b ! Z ve b Y = 0 olmak üzere b şeklinde yazılamayan, başka bir ifadeyle ondalık açılımı sınırsız ve
devirsiz olan sayılara irrasyonel sayılar denir. İrrasyonel sayılar kümesi Q' ile gösterilir.

Bir önceki örnekte 2 sayısının rasyonel bir sayı olmadığı gösterilmişti. Bu tanıma göre 2 sayısı bir
irrasyonel sayıdır. Aynı şekilde r, e ve karekök dışına çıkarılamayan 3 , 5 , 6 gibi sayılar da birer irrasyonel
sayıdır.

ÖRNEK 12:
2 sayısının sayı doğrusu üzerindeki yerini gösteriniz.

ÇÖZÜM:

2
1
O 2
0 1 B P 2

Yukarıdaki şekilde görüldüğü gibi, bir köşesi 0 noktasında bulunan ve dik kenar uzunlukları birer birim olan
ikizkenar dik üçgen oluşturunuz. Bu durumda, OA = 2 olur. O merkezli OA yarıçaplı bir çember çizilir ve
çemberin sayı doğrusunu kestiği noktaya P denirse
OP = OA = 2 br olup P noktası 2 ye karşılık gelir.
1 < 2 < 2 olduğu şekilden görülür. 2 = 1, 41421...

Matematik 9 | 101
DENKLEMLER VE EŞİTSİZLİKLER

Doğal sayıların kareköklerini pergel ve cetvel kullanarak


çizen ve sayı doğrusu üzerinde gösteren ilk bilim insanı Yunan
matematikçisi Archytas’tır [(Arhitas) (MÖ 428-342)]. Kullandığı
yönteme “karekök sarmalı” denir.

ÖRNEK 13:
r sayısının yerini, sayı doğrusu üzerinde gösteriniz.

ÇÖZÜM:
Çapı 1 br olan bir çember alınız. Bu çemberi, sayı doğrusuna 0 noktasına teğet olacak şekilde yerleştiriniz.
Bu çemberin çevresi r birimdir.
O
1 br

O O'

0 r
Çember; sayı doğrusu üzerinde döndürüldüğünde çember üzerindeki O noktasının, 1. tur sonunda sayı
doğrusunu kestiği nokta O' olsun. Çemberin çevresi r birim olduğundan 1. tur sonunda çemberin aldığı yol
r birimdir. Bu durumda O' noktası r sayısına karşılık gelir.

TARİHÇE

sayısı irrasyonel sayıların çeşitliliğine bir örnektir. Bir dairenin çevresinin


çapına oranı daima sabit bir sayıdır. Bu sayı π sembolü ile gösterilir. Arşimet
sabiti veya Ludolph sayısı olarak da bilinir. Babillilerden (MÖ 2000) bu
yana birçok matematikçi, bu sayıyı rasyonel şekilde yazmaya çalışmış ve
bunun için birçok farklı yaklaşık değer kullanmıştır. Son olarak 1761 yılında Lambert
(Lambırt), π sayısının irrasyonelliğini ispatlayarak bu tartışmalara nokta koymuştur.
Pi sayısı, birçok iki boyutlu şeklin ve üç boyutlu cismin alan ve hacmini hesaplarken
kullanılır. Pi sayısının virgülden sonraki 2 trilyon 700 bin civarı basamağı bilgisayar
yardımı ile hesaplanabilmektedir.14 Mart “Dünya π Günü” olarak kutlanmaktadır.
r = 3, 14159265358979323846...

Üst sınıflarda karşılaşacağınız ve matematikte çok önemli bir yere sahip e sayısı (Euler sayısı) da irrasyonel
bir sayıdır. Bu sayının irrasyonelliği Leonhard Euler [Leonırd Oyler (1707-1783)] tarafından kanıtlanmıştır.
e = 2, 71828182845904523536...

Sıra Sizde

SORU:
Herhangi iki irrasyonel sayı arasında daima başka bir irrasyonel sayının bulunduğunu gösteriniz.

ÇÖZÜM:

102 | Matematik 9
DENKLEMLER VE EŞİTSİZLİKLER

Gerçek Sayılar Kümesi


Rasyonel sayılar kümesi ile irrasyonel sayılar kümesinin birleşim kümesine gerçek sayılar (reel sayılar
ya da gerçel sayılar) kümesi denir. R ile gösterilir.
R = Q , Q'

Sayı doğrusundaki rasyonel sayılar arasındaki boşluklar, irrasyonel sayılar tarafından doldurulur. Böylece
oluşturulan doğruya gerçek sayı doğrusu denir. Rasyonel sayılar ile irrasyonel sayılar, gerçek sayı doğrusunun
tamamını hiç boşluk kalmayacak şekilde doldurur. Bunun tersi de doğrudur. Sayı doğrusundaki her nokta, bir
gerçek sayıya karşılık gelir.

Gerçek sayılar kümesi, şu ana kadar tanımlanan bütün sayı kümelerini Q Q'
kapsar. 1
-2
Z
3 olup R = Q , Q' tir.
N1Z1Q1R 2
0 N
Q' 1 R
-5
3
3
2
-2

Gerçek Sayılar Kümesinde Toplama İşleminin Özellikleri


1. Kapalılık özelliği: 6 a, b ! R için a + b ! R dır. (Herhangi iki gerçek sayının toplamı, yine bir gerçek
sayıdır.)
2. Değişme özelliği: 6 a, b ! R için a + b = b + a tir.
3. Birleşme özelliği: 6 a, b, c ! R için a + ]b + cg = ]a + bg + c tir.
4. Etkisiz eleman özelliği: 6 a ! R için a + 0 = 0 + a = a olduğundan toplama işleminin etkisiz elemanı 0
dır.
5. Ters eleman özelliği: 6 a ! R için a + ]- ag = ]- ag + a = 0 olacak şekilde - a ! R sayısı vardır.
a sayısının toplama işlemine göre tersi - a sayısıdır.

Gerçek Sayılar Kümesinde Çarpma İşleminin Özellikleri


1. Kapalılık özelliği: 6 a, b ! R için a $ b ! R dır. (Herhangi iki gerçek sayının çarpımı yine bir gerçek
sayıdır.)
2. Değişme özelliği: 6 a, b ! R için a $ b = b $ a tir.
3. Birleşme özelliği: 6 a, b, c ! R için a $ ]b $ cg = ]a $ bg $ c tir.
4. Etkisiz eleman özelliği: 6 a ! R için a $ 1 = 1 $ a = a olduğundan çarpma işleminin etkisiz elemanı
1 dir.

5. Ters eleman özelliği: 6 a ! R - ! 0 + için a $ a = a $ a = 1 olacak şekilde a ! R sayısı vardır.


1 1 1

a ! R - ! 0 + sayısının çarpma işlemine göre tersi a dır.


1

6. Yutan eleman özelliği: 6 a ! R için a $ 0 = 0 $ a = 0 olduğundan gerçek sayılar kümesinin çarpma


işlemine göre yutan elemanı 0 dır.
7. Dağılma özelliği: 6 a, b, c ! R için a $ ]b + cg = a $ b + a $ c
a $ ]b - cg = a $ b - a $ c tir.
Çarpma işleminin toplama ve çıkarma işlemi üzerine dağılma özelliği vardır.

Matematik 9 | 103
DENKLEMLER VE EŞİTSİZLİKLER

İki farklı reel sayı doğrusunun 0 noktasında dik kesişmesiyle oluşan düzlemin analitik düzlem olarak
isimlendirildiğini biliyorsunuz. R de tanımlı
R 2 = R # R = "^x, y h ; x ! R ve y ! R ,
kümesinin elemanları, sıralı ikililerdir. Bu sıralı ikililerden oluşan kümenin geometrik temsili, analitik
düzlemdeki tüm noktalardır. Bu yüzden R 2 ,kartezyen koordinat sistemine ya da dik koordinat sistemine
karşılık gelir.

Yatay olan eksene x ekseni (apsisler ekseni), düşey olan eksene y


y
ekseni (ordinatlar ekseni) denir.
II. Bölge I. Bölge Analitik düzlemdeki bir A(a,b) ikilisinde a ya A noktasının apsisi,
Ordinat b A(a,b) b ye A noktasının ordinatı denir.
• x ekseni üzerindeki noktalar (a,0) şeklinde olduğundan ordinatları
x
O(0,0) a 0 dır.
Orijin Apsis • y ekseni üzerindeki noktalar (0,b) şeklinde olduğundan apsisleri
III. Bölge IV. Bölge 0 dır.
• O(0,0) noktası, orijin noktası olarak adlandırılır.

ÖRNEK 14:
^ - 1, 1 h, ^ 2, 3 h, ^ 5, 0 h, ^ - 2, 0 h, ^ 0, 4 h, ^ - 2, - 5 h noktalarını dik koordinat sisteminde gösteriniz.

ÇÖZÜM:
y

4 (0,4)
3 (2,3)

(-1,1)
1
(-2,0) (5,0) x
-1 0 2 5

(-2,-5) -5

TARİHÇE

K oordinat kavramını ilk kez tarif eden Pierre de Fermat’tır [Piyer Dö Ferma
(1601-1665)]. Modern koordinat kavramı ise Fransız Matematikçi ve
Filozof Rene Descartes [Rene Dekart (1596-1650)] tarafından verilmiştir.
Descartes, cebrin geometriye uygulanması ile ilgili yaptığı çalışmaları
ilerleterek kartezyen koordinat sistemini “La Géometrie” adlı eserinde
tanıtmıştır. Bu bakımdan Descartes, analitik geometrinin kurucusu olarak
kabul edilmektedir.

Rene Descartes (1596-1650)

104 | Matematik 9
DENKLEMLER VE EŞİTSİZLİKLER

ALIŞTIRMALAR

1
Aşağıdaki tabloda verilen sayılardan hangileri rasyonel, hangileri irrasyoneldir?

-3 2 1, 976 3r 36 2 5-1 2
7
Rasyonel
İrrasyonel

2 Aşağıdaki tam sayıların tek mi çift mi 8


Aşağıdaki sayıların rasyonel sayı olmadıklarını
olduklarını belirtiniz. gösteriniz.
a) 2017.3 24 + 27.84 a) 5 b) 3 - 1 c) 2 3
b) ^2 15 + 9 2 h27 - ]- 1g4 + ]- 6g .7 45
c) ^3 24 - 4 2 h- + 1923 124 - 401
2

6- 1 - 2 - ]- 3 - 4g + ]- 1g@-2 - b - 1
2 + 2 l: 7
3 3 9
Aşağıdaki irrasyonel sayıları, gerçek sayı
doğrusu üzerinde gösteriniz.
işleminin sonucunu bulunuz. a) 7 b) - 34
c) 3 2 ç) 5+1

4
2 - 5 20
3 , 8 , 7 rasyonel sayılarını, gerçek sayı
doğrusu üzerinde gösteriniz.

b 3 - 0, 2 l:
5 1 10
İrrasyonel sayılar kümesinin toplama ve
5 9 çarpma işlemine göre kapalılık özelliği var
2+
3, 9 mıdır? Açıklayınız.
işleminin sonucunu bulunuz.

6
x ile y pozitif tam sayılar olmak üzere 11
Bir rasyonel sayı ile irrasyonel sayının
2x + 5y = 75 olduğuna göre y nin alabileceği toplamının irrasyonel olduğunu gösteriniz.
en büyük ve en küçük değerleri bulunuz.

7
ab iki basamaklı sayısının rakamlarının yerleri 12
Bir rasyonel sayı ile irrasyonel sayının
değiştirildiğinde elde edilen ba iki basamaklı çarpımının irrasyonel olduğunu gösteriniz.
sayısı, ab sayısından 27 fazla olduğuna göre ab
sayısının alabileceği değerleri bulunuz.

Matematik 9 | 105
DENKLEMLER VE EŞİTSİZLİKLER

9.3.2. BİRİNCİ DERECEDEN DENKLEMLER VE EŞİTSİZLİKLER


TARİHÇE
Diophantus (Diyofantus) 3. yy.da yaşadığı tahmin edilen ünlü bir
matematikçidir. Çözümleri tam sayı olması istenen cebirsel denklemler
üzerinde çalışmalar yapmıştır. Bu yüzden çözüm kümesi tam sayı olan
cebirsel denklemlere “Diophantus Denklemleri” adı verilmektedir. En
önemli yapıtı, 13 ciltten oluşan “Arithmetica (Aritmetik)� isimli eseridir.
Bu eserde MÖ 1650 yılında yazılmış olan “Rhind Papirüsü”nde geçen tek
bilinmeyenli cebir problemlerinin çözümlerini ve o güne kadar bilinen
cebir üzerine yapılmış çalışmaları bir araya toplamıştır. Matematiksel
ifadelerdeki sembolleri ilk kez Diophantus kullanmıştır.

Etkinlik
Bir araç kiralama
firmasının farklı iki tipte
araca ait ücretlendirme
durumları aşağıdaki grafikte
gösterilmiştir.
I. aracın günlük kirası
87 TL olup 250 km den
sonraki her 1 km için 28
kr. daha ödenmektedir. II.
aracın günlük kirası ise 127
TL olup 400 km den sonraki
her 1 km için 30 kr. daha
ödenmektedir.

Günlük II Sabit Ücrete


Ücret (TL) I Günlük Sabit Her 1 km Aşımında
Araç Dâhil Edilen
Ücret km Başına Düşen Ücret
km
I. Araç 87 TL 250 km 28 kr.
II. Araç 127 TL 400 km 30 kr.
127
87
Alınan
250 400 Yol (km)

Verilen bilgilerden yararlanarak aşağıdaki soruları yanıtlayınız.


1. Bir günde 450 km gidilirse her bir araca ait kiralama ücreti ne kadar olur?
2. Günlük 550 km gidecek olan biri için hangi aracı kiralamak daha ekonomik olur?
3. Bir günde kaç km gidilirse kiralama ücreti her iki araç için eşit olur?
4. Günlük kiralama için en fazla 185 TL ayıran bir kişi I. aracı kiraladığına göre en fazla kaç km gidebilir?
5. İkinci aracı kiralayan birinin ödediği günlük kiralama ücreti, 151 TL ile 175 TL arasında olduğuna göre
aracın gittiği yolun km si hangi aralıktadır.

106 | Matematik 9
DENKLEMLER VE EŞİTSİZLİKLER

1. Gerçek Sayılarda Aralık Kavramı


Aralık Kavramı
a, b ! R için a < b olmak üzere
1. Kapalı aralık: a ve b ile birlikte, a ile b arasındaki bütün gerçek sayıları kapsayan kümeye kapalı aralık
denir. 6a, b@ ile gösterilir.

6a, b@ = " x ; a # x # b, x ! R ,
a b

2. Açık aralık: a ve b hâriç, a ile b arasındaki bütün reel sayıları kapsayan kümeye açık aralık denir. ^a, b h
ile gösterilir.

^a, b h = " x ; a < x < b, x ! R ,


a b

3. Yarı açık (yarı kapalı) aralık: ^a, b h açık aralığına a ya da b den herhangi birinin dâhil edildiği kümeye
yarı açık aralık denir. 6a, b h veya ^a, b@ ile gösterilir.

6a, b h = " x ; a # x < b, x ! R ,


a b
^a, b@ = " x ; a < x # b, x ! R ,
a b

4. Üstten sınırsız aralıklar: a ! R olmak üzere a dan büyük tüm gerçek sayıların kümesidir.

a dâhil ise 6a, 3 h = " x ; a # x, x ! R ,


a

a dâhil değil ise ^a, 3h = " x ; a < x, x ! R ,


a

5. Alttan sınırsız aralıklar: a ! R olmak üzere a dan büyük tüm gerçek sayıların kümesidir.

a dâhil ise ^- 3, a@ = " x ; x # a, x ! R ,


a
a dâhil değil ise ^- 3, a h = " x ; x < a, x ! R ,
a

6. R nin kendisi de aralıktır.


R = ^- 3, 3h = " x ; - 3 < x < 3, x ! R , R

ÖRNEK 1:
A = 6- 1, 4 h, B = ^3, 5 h ise A - B ve A + B kümelerini bulunuz.

ÇÖZÜM:
B
A-B
_
b
b
b
b
`
b
b
b
b
a

-1 3 4 5
a
bbb
bb
bb
bb
bb
`
bb
bb
bb
bb
bb
b_

A
A - B = 6- 1, 3@ ve A + B = (3, 4) olur.

Matematik 9 | 107
DENKLEMLER VE EŞİTSİZLİKLER

ÖRNEK 2:
x ve y gerçek sayıları için
-4 # x < 6
2 olduğuna göre
-7 < y < 2
a) 2x + 5y ifadesinin değer aralığını bulunuz.
b) 2x - 5y ifadesinin değer aralığını bulunuz.
c) x 3 - y 2 ifadesinin değer aralığını bulunuz.
ç) xy ifadesinin değer aralığını bulunuz.

ÇÖZÜM:

2 5
4
a) -4 # x < 6 -7 < y < 2 - 8 # 2x < 12
- 8 # 2x < 12 - 35 < 5y < 10 + - 35 < 5y < 10
- 43 < 2x + 5y < 22 ise 2x + 5y ! ^- 43, 22 h dir.

b) 2 -5
4
-4 # x < 6 -7 < y < 2 - 8 # 2x < 12
- 8 # 2x < 12 - 10 < - 5y < 35 + - 10 < - 5y < 35
- 18 < 2x - 5y < 47 ise 2x - 5y ! ^- 18, 47h dir.

c) -4 # x < 6 -7 < y < 2


3
- 64 # x 3 < 216
- 64 # x 3 < 216 0 # y 2 < 49 + - 49 < - y 2 # 0
- 113 < x 3 - y 2 < 216 ise x 3 - y 2 ! ^- 113, 216h dır.

ç) -4 # x < 6
-7 < y < 2
Sınır değerleri x ve y nin karşılıklı ikişer ikişer çarpımı sonucunda bulunan değerler,
]- 4g $ ]- 7g = 28 b_b
b
]- 4g $ 2 =- 8 bbb
` bu değerlerin en küçüğü - 42 , en büyüğü 28 olduğundan
6 $ ]- 7g =- 42bbb
6 $ 2 = 12 b - 42 < xy < 28 dir. xy ! ^- 42, 28h
bb
a

ÖRNEK 3:
4x + 5y
- 2 < x < 6, 2 < y # 8 ise y ifadesinin değer aralığını bulunuz.

ÇÖZÜM:

4x + 5y 4x x 1
y = y + 5 olup y aralığını bulmak için x ve y nin sınırları karşılıklı ikişer ikişer çarpılır.
1 1 1
2 1 y # 8 & 8 # y < 2 ve - 2 < x < 6 olduğuna göre

8 ]- 2g = 4 bb
1 - 1 _bb
b
1 3 bbb
8 $ 6 = 4 bb 4x + 5y
`b - 1 1 yx 1 3 & - 4 < 4yx < 12 & 1 < 4yx + 5 < 17 & ! ^1, 17 h olur.
1 ]- 2 g bb y
2 =- 1 bb
bb
1 bb
2 $ 6 = 3 b
a

108 | Matematik 9
DENKLEMLER VE EŞİTSİZLİKLER

ÖRNEK 4:
C = " - 2, 0, 1, 3 , ve D = ^2, 4@ aralıkları verilsin. C # D kümesini koordinat düzleminde gösteriniz.

ÇÖZÜM:
C kümesi, 4 elemanlı sonlu bir kümedir. D kümesi ise ^2, 4@ aralığındaki bütün gerçek sayıları içeren sonsuz
bir kümedir. Bu durumda C # D kümesi analitik düzlemde yarı doğru parçalarından oluşur.
D

C
-2 0 1 3

ÖRNEK 5:

M = ^3, 5@ ve N = 6- 1, 2 h ise M # N kümesini koordinat düzleminde gösteriniz.

ÇÖZÜM:
M # N , düzlemde bir bölge oluşturur. Bu bölgede y = 2 ve x = 3 doğruları aralıklara dâhil olmadığından
bunlar kesikli çizgi ile gösterilir.

M
0 3 5
-1

Sıra Sizde

SORU :

A = 62, 4@ ve B = 61, 5@ aralıkları verilsin. A # B kümesini düzlemde gösteriniz.

ÇÖZÜM:

Matematik 9 | 109
DENKLEMLER VE EŞİTSİZLİKLER

2. Birinci Dereceden Denklemler ve Eşitsizliklerin Çözümü


Birinci Dereceden Bir Bilinmeyenli Denklemler
a, b ! R ve a Y = 0 için ax + b = 0 şeklindeki ifadelere, x değişkenine (bilinmeyen) bağlı birinci dereceden
bir bilinmeyenli denklem denir.
Denklemi sağlayan x değerini bulmaya denklemi çözmek, denklemi sağlayan x ! R değerine denklemin
kökü denir. Denklemin köklerinden oluşan kümeye, denklemin çözüm kümesi adı verilir. Çözüm kümesi
genelde Ç harfi ile gösterilir.
ax + b = 0 ifadesindeki terimlerde x değişkeninin en büyük üssü 1 olduğundan denklem, birinci
dereceden bir denklemdir.
x
Örneğin 2x + 3 = 0, - 5x + 1 = 0, 2 + 4 = 0 denklemleri birinci dereceden bir bilinmeyenli
denklemlerdir. 3x 2 - 4 = 0, x - 1 = 0 denklemleri birinci dereceden denklem değildir.

= 0 ise denklemin R de tek kökü vardır. Ç = & a 0 dır.


-b
ax + b = 0 denkleminde a Y

ÖRNEK 1:

x m - 2 + ]n + 4g x 2 - 6 = 0 ifadesi x değişkenine bağlı birinci dereceden bir bilinmeyenli denklem olduğuna


göre m ve n değerlerini bulunuz.

ÇÖZÜM:
Verilen ifadede ^n + 4 h x 2 teriminde x 2 ikinci dereceden denklem olduğundan x 2 nin katsayısı 0 olmalıdır.
O hâlde n + 4 = 0 olup n =- 4 bulunur. Verilen denklemin birinci dereceden bir denklem olması
istendiğinden x in en büyük kuvveti 1 olmalıdır. Bu durumda m - 2 = 1 ise m = 3 olur.

ÖRNEK 2:

2x - 3 ^x + 5 h - 5 ^2x + 1 h = 0 denkleminin çözüm kümesini bulunuz.

ÇÖZÜM:

2x - 3 ^x + 5 h - 5 ^2x + 1 h = 0
2x - 3x - 15 - 10x - 5 = 0
- 11x = 20
x = 11 olup denklemin çözüm kümesi Ç = & 11 0
- 20 - 20

ÖRNEK 3:
x x+1
2 + 2x + k = 3x - 5 denkleminin bir kökü x = 2 olduğuna göre k sayısını bulunuz.
ÇÖZÜM:
x = 2 denklemi sağladığından x yerine 2 yazıldığında
2 2+1
2 +2$2+k = 3$2- 5

3 27 27 2
1 + 4 + k = 6 - 5 & 5 + k = 5 & k = 5 - 5 = 5 tir.

110 | Matematik 9
DENKLEMLER VE EŞİTSİZLİKLER

ÖRNEK 4:
9
4+ 5 = 7 denkleminin çözüm kümesini bulunuz.
2+ 3
1- x

ÇÖZÜM:
9 3 3 -3
4+ 5 = 7 & 1 - x = 5 & x =- 4 & x = 4
2+ 3
3 1- x
5 1

ÖRNEK 5:
3x + 2
x + 2 ifadesinin tam sayı olması için x in alabileceği tam sayı değerlerinin toplamını bulunuz.
ÇÖZÜM:

3x + 2 3 ]x + 2g - 4 4
x+2 = x+2 = 3 - x + 2 şeklinde yazılırsa x + 2 sayısının 4 ün böleni olması gerektiği görülür.

4 ün bölenleri 1, 2, 4, - 1, - 2, - 4 olduğundan x in alabileceği değerler:


x + 2 = 1 ise x =- 1 x + 2 =- 1 ise x =- 3

x + 2 = 2 ise x = 0 x + 2 =- 2 ise x =- 4
x + 2 = 4 ise x = 2 x + 2 =- 4 ise x =- 6 olur.
x in değerleri toplamı - 1 + 0 + 2 - 3 - 4 - 6 =- 12 bulunur.

ax + b = 0 denkleminde çözüm kümesi, boş küme ise a = 0 ve b Y


= 0 dır.

ÖRNEK 6:

3x - 5 ]x + 1g = 2 ]4 - xg + 7 denkleminin çözüm kümesini bulunuz.

ÇÖZÜM:

3x - 5 ]x + 1g = 2 ]4 - xg + 7
3x - 5x - 5 = 8 - 2x + 7
- 2x - 5 = 15 - 2x
- 5 =- 15 yanlış olduğundan denklemin çözümü Ç = Q dir.

ÖRNEK 7:
x değişkenine bağlı ]m + 1g x + n - 3 = 5x denkleminin R de çözüm kümesi, boş küme ise m + n hangi
değeri alamaz?

ÇÖZÜM:

mx + x + n - 3 - 5x = 0 denklemi düzenlendiğinde ]m - 4g x + n - 3 = 0 bulunur. Denklemin Ç = Q


olduğundan m - 4 = 0 ve n - 3 Y
= 0 olmalıdır. Buna göre m = 4 ve n Y= 3 ise m + n Y= 7 bulunur.

Matematik 9 | 111
DENKLEMLER VE EŞİTSİZLİKLER

ax + b = 0 denkleminde çözüm kümesi R ise a = 0 ve b = 0 dır.

ÖRNEK 8:

5 ]x - 2g + 3x + 1 = 8x - 9 denkleminin çözüm kümesini bulunuz.

ÇÖZÜM:
Denklemde dağılma özelliği kullanılıp x e göre düzenleme yapılırsa
5x - 10 + 3x + 1 = 8x - 9
8x - 9 = 8x - 9
0 = 0 bulunur. Bu durumda, her x ! R için denklem sağlanır.

ÖRNEK 9:

]m - 1g x - n + 3 = 3 ]2x - 7g - 5 eşitliği x in bütün gerçek sayı değerleri için sağlanıyorsa m + n değerini


bulunuz.

ÇÖZÜM:
]m - 1g x - n + 3 = 6x - 21 - 5
]m - 1g x - n + 3 = 6x - 26 bulunur. Buradan m - 1 = 6 ve - n + 3 =- 26
m=7 n = 29 olur.
Bu durumda m + n = 36 elde edilir.

ÖRNEK 10:
4
x + 1 ifadesini R de tanımsız yapan x değerlerinin çarpımı kaçtır?
2- x-2

ÇÖZÜM:
x - 2 = 0 için verilen ifade tanımsızdır. Buradan x = 2 bulunur.
4 4 4 4 ]x - 2 g
2 x + 1 = 2x - 4 - x - 1 = x - 5 = x - 5 olduğundan son ifade x = 5 için de tanımsız olur.
1 - x-2 x-2 x-2 O hâlde verilen ifadeyi tanımsız yapan x değerlerinin
]x - 2 g ]1 g çarpımı 2 $ 5 = 10 olur.

ÖRNEK 11:
a < b < c < d olmak üzere a, b, c, d tam sayılarının üçerli toplamları 29, 34, 43, 71 olduğuna göre c
değerini bulunuz.

ÇÖZÜM:
Bu sayıların toplamı x olsun. a + b + c + d = x ise bu sayılar,
a = x - 71, b = x - 43, c = x - 34, d = x - 29 olduğundan
]x - 71g + ]x - 43g + ]x - 34g + ]x - 29g = x & 4x - 177 = x & 3x = 177 & x = 59 olarak bulunur.
O hâlde bu sayılar - 12, 16, 25, 30 olup c = 25 tir.

112 | Matematik 9
DENKLEMLER VE EŞİTSİZLİKLER

ÖRNEK 12:
x+1
y = 2x - 3 eşitliği veriliyor. Buna göre
a) Hangi x değeri için y hesaplanamaz?
b) Hangi y değeri için x hesaplanamaz?

ÇÖZÜM:
x+1 3
a) y = 2x - 3 ifadesinde 2x - 3 = 0 & x = 2 için y hesaplanamaz.
b) Verilen ifadede x değeri yalnız bırakılırsa
y ]2x - 3g = x + 1 & 2xy - 3y = x + 1 & 2xy - x = 3y + 1
& x ^2y - 1h = 3y + 1

3y + 1 1
& x = 2y - 1 bulunur. Buifadede 2y - 1 = 0 & y = 2 için x hesaplanamaz.

ÖRNEK 13:
15 ardışık tam sayının toplamı biçiminde yazılan pozitif tam sayılar kümesinin elemanları, artan sırada
dizildiğinde baştan 50. terim kaçtır?

ÇÖZÜM:
x 15 ardışık tam sayının toplamı olsun. a ! Z olmak üzere x pozitif tam sayısı,

x = a + ]a + 1g + ]a + 2g + ]a + 3g + ... + ]a + 14g

x = 15a + 1 + 2 + 3 + ... + 14
14 $ 15
x = 15a + 2 = 15a + 7 $ 15
x = 15 ` ;
a + 7j bulunur.
1, 2, 3, 4, ...
50. terim: x = 15 $ 50 = 750 olur.

n ^n + 1h
Ardışık doğal sayıların toplam formülü 1 + 2 + 3 + 4 + ..... + n = 2 dir.
Bu bağıntı Gauss formülü olarak bilinir.

Sıra Sizde

SORU:
a-1
2+ 3
1+ 4
2a - 1 =- 1 denkleminin çözüm kümesini bulunuz.

ÇÖZÜM:

Matematik 9 | 113
DENKLEMLER VE EŞİTSİZLİKLER

Birinci Dereceden İki Bilinmeyenli Denklemler


a, b, c ! R ve a Y = 0 için ax + by + c = 0 şeklindeki ifadelere x ve y değişkenlerine bağlı birinci
= 0, b Y
dereceden iki bilinmeyenli denklem denir. Denklemin kökleri (x,y) ikililerinden oluşur. Birinci dereceden iki
bilinmeyenli bir denklemi gerçek sayılar kümesinde sağlayan sonsuz sayıda (x,y) ikilisi vardır. Bu ikililerin
kümesine denklemin çözüm kümesi denir. Çözüm kümesinin koordinat düzlemindeki görüntüsü, bir doğru
belirtir.

ÖRNEK 14:

2x + y = 4 denklemini sağlayan ^x, y h ikililerinin görüntü kümesini koordinat düzleminde gösteriniz.


(Grafiğini çiziniz.)

ÇÖZÜM: y

“Düzlemdeki herhangi iki noktadan tek doğru geçer.”


4
Burada bu iki nokta, doğrunun eksenlerinin kestiği noktalar olarak
alınırsa grafik çizimi kolaylaşır.
x = 0 için y = 4
3 2x + y = 4
y = 0 için x = 2
2x + y = 4 doğrusu (0,4) ve (2,0) noktalarından geçer. x
Bu doğru şekildeki gibi çizilir. 0 2

ÖRNEK 15:

b 3m + 2 l x + my =- 2 denklemini sağlayan ikililerden birisi ^- 1, 2 h ise m sayısı kaç olmalıdır?


5
ÇÖZÜM:

x =- 1 ve y = 2 değerleri, denklemde yerine yazılırsa


b 3m + 2 l]- 1g + 2m =- 2 & - 3m - 2 + 2m =- 2
5 5
- 3m - 2 + 10m
& 5 =- 2
& - 3m - 2 + 10m =- 10
& 7m =- 8
8
& m =- 7 bulunur.

ÖRNEK 16:

]3x - 6g2 + ^y + 5 h4 = 0 ise x $ y değerini bulunuz.

ÇÖZÜM:

]3x - 6g2 $ 0 ve ^y + 5 h4 $ 0 olduğuna göre bu eşitlik sadece 0 + 0 = 0 olması durumunda sağlanır.


3x - 6 = 0 & x = 2
3 x $ y =- 10 olarak bulunur.
y + 5 = 0 & y =- 5

114 | Matematik 9
DENKLEMLER VE EŞİTSİZLİKLER

ax + by + c = 0 denkleminde her x, y ! R için sağlanıyorsa ^Ç = R 2 h a = 0, b = 0, c = 0 dır.

ÖRNEK 17:

]m - 3g x + ]2n + 1g y + p - 4 = 0 denklemi her x, y ! R için sağlanıyorsa m + n + p değerini bulunuz.


ÇÖZÜM:
Denklemin her x, y ! R için sağlanıyorsa x ve y nin katsayıları 0 olmalıdır.
m - 3 = 0 & m = 3 _bb
b
- 1 bb
2n + 1 = 0 & n = 2 `b m + n + p = 3 + b 2 l + 4 = 13
-1
bb 2
p - 4 = 0 & p = 4 bb
a
ax + by + c = 0 denkleminin çözüm kümesi boş küme ise a = 0, b = 0, c Y
= 0 dır.

ÖRNEK 18:

]a + 1g x + ]b - 8g y = c + 3 denkleminin çözüm kümesi boş küme ise a + b + c toplamının alamayacağı


değeri bulunuz.

ÇÖZÜM:
a + 1 = 0 & a =- 1_bb
bb
b - 8 = 0 & b = 8 `b a + b + c ! 4 dir.
b
c+3Y = 0 &c Y=- 3 bb
a

Basit Eşitsizlikler
a, b ! R olmak üzere a < b, a # b, a > b ya da b $ a şeklindeki ifadeler, basit eşitsizlikler olarak
adlandırılır. Basit eşitsizliklerin özellikleri şunlardır:

x, y, a, b ! R olmak üzere

Bir eşitsizliğin her iki tarafına herhangi bir reel sayı eklenir ya da
a) x < y + x + a < y + a çıkarılırsa eşitsizlik değişmez.

xa < ya, a > 0


b) x < y + ) Bir eşitsizliğin her iki tarafı pozitif bir sayı ile çarpılırsa eşitsizlik
değişmez. Negatif bir sayı ile çarpılırsa eşitsizlik yön değiştirir.
xa > ya, a < 0

]Z] x y
]] a < a , a > 0
c) x < y + [] Bir eşitsizliğin her iki tarafı pozitif bir sayı ile bölünürse eşitsizlik
]] x y değişmez. Negatif bir sayı ile bölünürse eşitsizlik yön değiştirir.
]] a > a , a < 0
\

ç) x < y ve y < z ise x < z dir. “<” işleminin geçişme özelliği vardır.

Matematik 9 | 115
DENKLEMLER VE EŞİTSİZLİKLER

Aynı yönlü eşitsizlikler, taraf tarafa toplanabilir.


Örneğin 3 < 7 ve 5 < 8 & 3 + 5 < 7 + 8
x<y
8 < 15 tür.
d) + a < b Not: Eşitsizliklerde taraf tarafa çıkarma, çarpma ve bölme
x+a < y+b işlemleri yapılamaz.

Her iki tarafı da aynı işaretli olan eşitsizliklerde iki tarafın


e) x ile y aynı işaretli ise çarpmaya göre tersi alınırsa eşitsizlik yön değiştirir.
1 1 1 1
x<y + x > y Örneğin 2 < 3 ise 2 > 3 ,
^x Y= 0, y Y= 0h 1 1
- 5 < - 4 ise - 5 > - 4 tür.

Pozitif sayılar arasındaki eşitsizliklerde her iki tarafın pozitif


n ! Z + olmak üzere doğal sayı kuvveti alınırsa eşitsizlik değişmez.
f)
0 < a < b & an < bn Örneğin 2 < 3 ise 2 2 < 3 2, 2 3 < 3 3, 2 4 < 3 4 tür.

Negatif sayılar arasındaki eşitsizliklerde her iki tarafın tek doğal


g) n ! Z + olmak üzere sayı kuvveti alınırsa eşitsizlik değişmez. Sıfır dışındaki çift doğal
Z] a 2n - 1 < b 2n - 1 < 0 sayı kuvveti alınırsa eşitsizlik yön değiştirir.
]]
] Örneğin - 3 < - 2 ise ]- 3g2 > ]- 2g2 & 9 > 4
a < b < 0 & []
]] 2n
] a > b 2n > 0 ]- 3g3 < ]- 2g3 & - 27 < - 8 dir.
\

Örneğin a = 3 ise 9 > 3, 27 > 3 tür.

Örneğin a = 1 ise b 1 l = 1 < 1 ,


2

2 2 4 2
n ! Z+ - !1 + b 1 l = 1 < 1 tü r.
3
ğ) 2 8 2
: a > 1 & an > a
: 0 < a < 1 & an < a
Örneğin a =- 1 ise b - 1 l = 1 > - 1 ,
2

: - 1 < a < 0 & an > a 2 2 4 2


b - 1 l =- 1 > - 1 tür.
3
a n > a, n çift
: a <-1 & * n 2 8 2
a < a, n tek
−1 den küçük sayıların çift kuvveti kendisinden büyüktür, tek
kuvveti de kendisinden küçüktür.
Örneğin
a =- 2 ise ]- 2g2 = 4 > - 2, ]- 2g4 = 16 > - 2
]- 2g3 =- 8 < - 2, ]- 2g5 =- 32 < - 2 tür.
0 ile 1 arasındaki sayıların karesi, kendisinden küçüktür. Bu
ifadenin tersi de doğrudur. Karesi kendisinden küçük olan
sayılar 0 ile 1 arasındadır.

Örneğin
h) a 2 < a + 0 < a < 1
a = 2 + ` 2 j < 2 tür.
1 1 2 1

a = 3 + ` 3 j < 3 tür.
2 2 2 2

116 | Matematik 9
DENKLEMLER VE EŞİTSİZLİKLER

ÖRNEK 19:

x 2 < x ise 5x + 7 nin alabileceği tam sayı değerlerinin sayısını bulunuz.

ÇÖZÜM:
x 2 < x ise 0 < x < 1 olduğundan
0 < 5x < 5 & 7 < 5x + 7 < 12 olur.
5x + 7 nin alabileceği değerler 8, 9, 10, 11 olup 4 tanedir.

ÖRNEK 20:

x 4 < x 2 , xz < 0 , ]z - xg y 2 > 0 ise aşağıdakilerden hangisi kesinlikle doğrudur?


A) y > 1 B) - 1 < x < 1 C) 0 < z < 1 D) - 1 < x < 0 E) x + 2z < y

ÇÖZÜM:

]z - x g y 2 > 0 & z - x > 0 & z > x


144424443 5
+ +
xz < 0 ise x ile z zıt işaretlidir. z > x olduğundan z pozitif, x negatif olur.
x4 x2
x4 < x2 & < & x 2 < 1 & - 1 < x < 1 & - 1 < x < 0 elde edilir.
x2 x2
O hâlde D seçeneği kesinlikle doğrudur.

ÖRNEK 21:
x ve y tam sayıları için - 9 < x < 5, - 2 < y # 7 olduğuna göre 3x + 4y ifadesinin en küçük ve en büyük
tam sayı değerlerini bulunuz.

ÇÖZÜM:

x in alabileceği değerler kümesi: " - 8, - 7, - 6, ..., 2, 3, 4 ,
y nin alabileceği değerler kümesi: " - 1, 0, 1, ..., 5, 6, 7 ,
x =- 8 ve y =- 1 için 3x + 4y en küçük değerini alır. 3 ]- 8g + 4 ]- 1g =- 24 - 4 =- 28 olur.
x = 4 ve y = 7 için 3x + 4y en büyük değerini alır. 3 $ 4 + 4 $ 7 = 12 + 28 = 40 olur.

Birinci Dereceden Bir Bilinmeyenli Eşitsizlikler


a, b ! R ve a Y = 0 için ax + b # 0, ax + b < 0, ax + b $ 0, ax + b > 0 şeklindeki ifadelere x değişkenine
(bilinmeyen) bağlı birinci dereceden bir bilinmeyenli eşitsizlik denir.
Bu tür eşitsizliklerin çözüm kümesi, reel sayıların bir alt aralığıdır.

ÖRNEK 22:
3x + 4 < 2 ]x - 1g + 4x eşitsizliğinin çözüm kümesini bulup sayı doğrusu üzerinde gösteriniz.

ÇÖZÜM:

3x + 4 < 2x - 2 + 4x
3x + 4 < 6x - 2
6 < 3x & 2 < x & Ç = ^2, 3 h 2

Matematik 9 | 117
DENKLEMLER VE EŞİTSİZLİKLER

ÖRNEK 23:
5x - 7 < 2x + 5 # 4 ]x - 1g + 5 eşitsizliğini sağlamayan x tam sayılarının toplamını bulunuz.

ÇÖZÜM:

5x - 7 < 2x + 5 2x + 5 # 4 ^x - 1 h + 1
ve
3x < 12 2x + 5 # 4x + 1

x<4 4 # 2x
2#x
Buna göre 2 # x < 4 olarak bulunur. O hâlde eşitsizliğin çözüm kümesi, Ç = 62, 4 h tür.
Sağlamayan tam sayılar: ..., - 5 , - 4 , - 3, - 2, - 1 , 0, 1 , 4 , 5 , ... olur.
Bu sayıların toplamı - 3 - 2 =- 5 tir.

ÖRNEK 24:

a liraya alınan bir ürün 5a - 200 liraya satılmaktadır. Satıcının kâr edebilmesi için a nın alabileceği en küçük
tam sayı değerini bulunuz.

ÇÖZÜM:

Satış fiyatının alış fiyatından büyük olduğu durumlarda kâr elde edilir. Ürün a liraya alındığından
5a - 200 2 a & 5a - 200 - a > 0 yazılır.
4a - 200 > 0 & 4a > 200 & a > 50 dir. a en az 51 lira olmalıdır.

ÖRNEK 25:
a, b, c, d tam sayılarının ikişerli toplamları 40, 54, 56, 62, 64, 78 dir. a 1 b 1 c 1 d ise b nin alabileceği
değerler toplamını bulunuz.
ÇÖZÜM:

b 1 c 1 d & a + b 1 a + c 1 a + d dir.

a 1 b 1 c & a + d 1 b + d 1 c + d dir.
O hâlde a + b 1 a + c 1 a + d 1 b + d 1 c + df ^1 h olarak bulunur.
; ; ; ;
40 54 64 78
a 1 b 1 c & a + c 1 b + c 1 d + c dir.

a 1 c 1 d & a + b 1 b + c 1 b + d dir.

O hâlde a + c 1 b + c 1 b + df ^2h olarak bulunur.


; ;
54 64
I. Durum:
b + c = 56 ise a + d = 62 dir. ]a + bg + ]b + dg = 104 & a + 2b + d = 104
& 62 + 2b = 104
& 2b = 42 & b = 21 olur.
II. Durum:
b + c = 62 ise a + d = 56 dır. ]a + bg + ]b + dg = 104 & a + 2b + d = 104
& 56 + 2b = 104
& 2b = 48 & b = 24 olur.
b nin alabileceği değerlerin toplamı 21 + 24 = 45 tir.

118 | Matematik 9
DENKLEMLER VE EŞİTSİZLİKLER

ÖRNEK 26:
- 4 < x < 9 ise A = x 2 - 4x + 3 ise A nın hangi aralıkta değer alacağını bulunuz.

ÇÖZÜM:
A = x 2 - 4x + 3 = x 2 - 4x + 4 - 1 = ]x - 2g2 - 1 olarak yazılabilir.
-4 < x < 9 & - 4 - 2 < x - 2 < 9 - 2 & -6 < x-2 < 7
& 0 # ]x - 2g2 < 49
& - 1 # ]x - 2g2 - 1 < 48
& - 1 # A < 48
& A ! 6- 1, 48h

ÖRNEK 27:
3x + 2y = 5
2 ise x in en geniş değer aralığını bulunuz.
-2 1 y < 2

ÇÖZÜM:
5 - 3x ^ h
2y = 5 - 3x & y = 2 f 1
- 2 < y < 2f ^2 h
(2) eşitsizliğinde (1) yerine yazılırsa
5 - 3x 1
-2 < 2 < 2 & - 4 < 5 - 3x < 4 & - 9 < - 3x < - 1 & 3 > x > 3 olarak bulunur.

ÖRNEK 28:

x 4 ]x - 3 g
2 x ]x - 1g2
< 0 eşitsizliğinin çözüm kümesini bulunuz.

ÇÖZÜM:

x 4 ]x - 3 g
< 0 eşitsizliğinde 6x ! R için x 4 $ 0, 2 x > 0, ve ]x - 1g2 $ 0 olduğundan x - 3 < 0
2 x ]x - 1g2

olmalıdır.
& x < 3 ve x Y
= 0, x Y
= 1 dir.
O hâlde eşitsizliğin çözüm kümesi Ç = ^- 3, 3 h - " 0, 1 , dir.

ÖRNEK 29:

2 2 2 1
a 1 b 1 c 1 0 olmak üzere a + b + c =- 7 olduğuna göre a nın alabileceği en büyük tam sayı
değerini bulunuz.

ÇÖZÜM:

a, b ve c sayıları ortadaki b değerine eşit olarak alınırsa


2 2 2 1 6 1
b + b + b =- 7 & b =- 7 & b =- 42 dir.
Buna göre a sayısının en büyük değeri - 43 olur.

Matematik 9 | 119
DENKLEMLER VE EŞİTSİZLİKLER

Birinci Dereceden İki Bilinmeyenli Eşitsizlikler


ax + by + c # 0, ax + by + c < 0, ax + by + c $ 0, ax + by + c > 0 şeklindeki ifadelere birinci dereceden
iki bilinmeyenli eşitsizlik denir. Eşitsizliği sağlayan (x,y) ikililerinin kümesine eşitsizliğin çözüm kümesi denir.
Çözüm kümesi, analitik düzlemde taralı bölge şeklinde gösterilir.
ax + by + c = 0 ifadesinde y, x e bağlı olarak yazılırsa
by = - ax - c
a c
y =- b x - b olur.
a c
m = - b , n = - b olarak seçilirse denklem y = mx + n olur.
• y = mx + n denkleminin çözüm kümesi, doğru üzerindeki noktaları gösterir.
• y > mx + n eşitsizliğinin çözüm kümesi, y = mx + n doğrusunun üst bölgesidir.
• y 1 mx + n eşitsizliğinin çözüm kümesi, y = mx + n doğrusunun alt bölgesidir.
# veya $ durumunda doğru üzerindeki noktalar, çözüm kümesine ait olduğundan doğru, düz çizgi
şeklinde çizilir.
< veya > durumunda doğru üzerindeki noktalar çözüm kümesine ait olmadığından doğru kesikli çizgi
şeklinde çizilir.

ÖRNEK 30:
2x + y # 4 eşitsizliğinin çözüm kümesini koordinat düzleminde gösteriniz.

ÇÖZÜM: y

İlk olarak 2x + y = 4 doğrusunun grafiğini çiziniz. 4
x = 0 için y = 4 ve y = 0 için x = 2 olduğundan doğru
^0, 4 h ve ^2, 0 h noktalarından geçer.
2x + y = 4
I. Yol:
^0, 0 h noktası, eşitsizlikte yerine yazılırsa 2
x
2.0 + 0 # 4 & 0 # 4 bulunur. 0

Eşitsizlik doğru olduğundan ^0, 0 h noktasının bulunduğu bölge,


çözüm bölgesidir.
II. Yol: Verilen eşitsizlik düzenlenirse
2x + y # 4 & y # 4 - 2x
olup eşitsizliğinin çözüm kümesi (bölgesi) y = 4 - 2x doğrusu ve bu doğrunun alt bölgesidir.

ÖRNEK 31:
3x - 4y > 12 eşitsizliğinin çözüm kümesini koordinat düzleminde y
4 x
gösteriniz.
0
3x - 4y = 12
ÇÖZÜM:

3x - 4y = 12 doğrusunun grafiği çizilir.
x = 0 için y =- 3 ve y = 0 için x = 4 olduğundan
^0, - 3 h ve ^4, 0 h noktaları bulunur. -3
3x - 4y > 12 & 3x - 12 > 4y
3x - 12 y
& 4 >
olup eşitsizliğin çözüm kümesi, doğrunun alt bölgesidir.

120 | Matematik 9
DENKLEMLER VE EŞİTSİZLİKLER

ALIŞTIRMALAR

1 ]3a - 6g x 3 - ]b + 1g x 2 + ]2a + bg x = 3a - 4 7
x+2 5x + 4
x - 3 + 4 # x - 3 eşitsizliğinin çözüm
ifadesi, x değişkenine bağlı birinci dereceden
bir bilinmeyenli denklem ise x değerini kümesini bulunuz.
bulunuz.

8
a ve b tam sayıları için
-2 < a # 3

2 6x + 2 - 3 ]2x - 5g@ + 5 = 1 - 4 ]x + 1g
-4 < b - 1 # 7
2 olduğuna göre 2a - 3b ifadesinin alabileceği
denkleminin çözüm kümesini bulunuz. en küçük ve en büyük değerleri bulunuz.

1
3
3 9 A = 6- 1, 4 h ve B = 60, 2 h ise A # B kümesini
2- x
1- x+1 koordinat düzleminde gösteriniz.

ifadesinin hangi değerler için tanımsız


olduğunu bulunuz.

10 C = ^- 2, 3 h ve D = " - 3, - 2, 0, 1 , ise C # D
kümesini koordinat düzleminde gösteriniz.

4 8
7 =2
2+ 2
3+ x-1

denkleminin çözüm kümesini bulunuz. 11


x + 2 5x + 8 x
3 - 2 +1 < 2 -5
eşitsizliğinin çözüm kümesini bulunuz.

5 x bilinmeyenine bağlı
]m + 3g x - 7 ]x - 1g = 2 ]x - 2g + 2n + 1 12
1
x, y d R için 2 1 x 1 8 ve 3 # y 1 3
x$y
denkleminin sonsuz çözümü varsa m.n değerini olduğuna göre x + y nin alabileceği tam
bulunuz. sayı değerleri toplamını bulunuz.

6
1 4 1 13 3x - y - 6 # 0 eşitsizliğinin çözüm kümesini
5 < 3x - 7 < 2 koordinat düzleminde gösteriniz.
eşitsizliğinin çözüm kümesini bulunuz.

Matematik 9 | 121
DENKLEMLER VE EŞİTSİZLİKLER

3. Birinci Dereceden Mutlak Değerli Denklem ve Eşitsizlikler


Mutlak Değer
Günlük hayatta uzunluk, ağırlık, hacim gibi değerleri ifade ederken kullanılan sayılar pozitif sayılardır. Bir
canlı için kullanılacak boyu - 150 cm , ağırlığı - 85 kg gibi ifadeler, hiçbir değeri ve anlamı olmayan ifadelerdir.
Ancak hava koşullarına bağlı olarak meteorolojik tahminler sonucunda hava sıcaklıkları için 26 %C, - 5 %C gibi
ifadeler kullanılmaktadır. Burada hava sıcaklıklarının sıfırın üstünde (pozitif) veya sıfırın altında (negatif) olduğu
düşünüldüğünden eksi ve artı ifadeler bir anlam kazanmaktadır.
-x x
644444474444448644444474444448
-x 0 x
İki nokta arasındaki uzaklık en küçük sıfır ya da pozitif bir sayı olur. x pozitif bir gerçek sayı ise sıfıra olan
uzaklığı x olur. Eğer x negatif bir gerçek sayı ise x in sıfıra olan uzaklığı - x olur. Bu durum, yukarıdaki sayı
doğrusunda gösterilmiştir. Mesela 5 gerçek sayısının sıfıra olan uzaklığı 5 br dir. - 5 gerçek sayısının sıfıra
olan uzaklığı ise -^- 5h = 5 br olur.

x bir gerçek sayı olmak üzere x in sıfıra olan uzaklığına x in mutlak değeri denir. Mutlak değer, x
şeklinde gösterilir.
Z] x, x 2 0 ise
]]
]
x = [] 0, x = 0 ise olarak tanımlanır.
]]
] - x, x 1 0 ise
\

ÖRNEK 1:
2
- 7, - 4, 0, 3 ve 4 sayılarının mutlak değerlerini bulunuz.

ÇÖZÜM:
Mutlak değeri alınacak sayı pozitif ise olduğu gibi, negatif ise önüne “ - ” işareti konularak mutlak değer
dışına çıkarılır.
- 7 =-]- 7g = 7, ]- 7 1 0g benzer olarak - 4 = 4; 0 = 0; 2 2
3 = 3; 4 = 4
-7 = 7 7 =7
644444474444448644444474444448
-8 -7 -6 -5 -4 -3 -2 -1 0 1 2 3 4 5 6 7 8
İşareti farklı olan sayıların mutlak değerlerinin eşit (sıfıra olan uzaklıkları aynı) olduğuna dikkat ediniz.

ÖRNEK 2:
3 - 2 + 3 - 3 - - 2 3 işleminin sonucunu bulunuz.

ÇÖZÜM:
Bu tür sorularda önce mutlak değer içindeki ifadenin işareti incelenir. İfadenin işareti pozitif ise ifade
olduğu gibi, negatif ise önüne eksi işareti konularak mutlak değer dışına çıkarılır.
3-2 1 0
3 olduğundan

3 - 2 + 3 - 3 - - 2 3 =-^ 3 - 2 h + ^3 - 3 h - 2 3
3- 3 2 0
=- 3 + 2 + 3 - 3 - 2 3
= - 4 3 + 5 bulunur.

122 | Matematik 9
DENKLEMLER VE EŞİTSİZLİKLER

ÖRNEK 3:
c 1 0 olmak üzere
- c + 2c + - 3c işleminin sonucunu bulunuz.

ÇÖZÜM:
c 1 0 ise - c ve - 3c ifadeleri pozitif olup mutlak değer dışına olduğu gibi, 2c ifadesi ise negatif olduğundan
mutlak değer dışına önüne ^- h yazılarak çıkar. Bu durumda işlemin sonucu:
- c + 2c + - 3c = ]- cg - 2c + ]- 3cg =- 6c bulunur.

ÖRNEK 4:
3 1 x 1 5 olmak üzere
2x - 11 + x - 3 - x - 5 işleminin sonucunu bulunuz.

ÇÖZÜM:
3 1 x 1 5 olduğundan mutlak değer içindeki her bir ifadenin işareti incelenirse
2x - 11 + x - 3 - x - 5 =-]2x - 11g + ]x - 3g - 6-]x - 5g@
. . . =- 2x + 11 + x - 3 + x - 5 = 3 bulunur.
^-h ^+h ^-h

Özellikler
x, y ! R için

1 x $0 4 k 2 0 için k.x = k. x

x = -x x.y = x . y
2 5 x x
x-y = y-x
y = y

3 xn = x n
6 x + y # x + y (Üçgen eşitsizliği)

ÖRNEK 5:
x 1 0 1 y olmak üzere
x - 2y + y - 5x + x - y işleminin sonucunu bulunuz.

ÇÖZÜM:
x 1 0 1 y verildiğinden x negatif, y pozitif sayıdır. Her bir mutlak değerli ifadenin işareti incelendiğinde
x - 2y + y - 5x + x - y =-^x - 2yh + ^y - 5xh - x - y
. . . . =- x + 2y + y - 5x - x - y
_-i _+i _-i _+i =- 7x + 2y olur.

Matematik 9 | 123
DENKLEMLER VE EŞİTSİZLİKLER
ÖRNEK 6:

x 1 0 olmak üzere - 4x - - 3x + x işleminin sonucunu bulunuz.

ÇÖZÜM:
x negatif olduğundan
- 4x - - 3x + ]- xg = - 4x - - 4x = - 4x - ]- 4xg = - 4x + 4x = 0 = 0 olur.

ÖRNEK 7:
- 5 # x - 2 # 5 olmak üzere y = 2x - 3 olduğuna göre y nin alabileceği en büyük ve en küçük tam sayı
değerlerinin toplamını bulunuz.

ÇÖZÜM:
-5 # x - 2 # 5 & - 5 + 2 # x - 2 + 2 # 5 + 2
& - 3 # x # 7 bu eşitsizlik 2x - 3 ifadesine benzetilirse
& - 9 # 2x - 3 # 11 bulunur. Mutlak değer alındığında
& 0 # 2x - 3 # 11 ifadesinin en küçük değeri 0, en büyük değeri 11 olur.
Bu değerlerin toplamı 0 + 11 = 11 bulunur.

ÖRNEK 8:
x ! R için x + 3 + x + 2x - 4 ifadesinin en küçük değerini bulunuz.

ÇÖZÜM:
Mutlak değerli ifadelerin içerisini sıfır yapan değerlere kritik nokta denir.
Verilen ifadenin kritik noktaları - 3, 0 ve 2 dir.
x =- 3 için x + 3 + x + 2x - 4 = - 3 + 3 + - 3 + - 6 - 4 = 13
x = 0 için x + 3 + x + 2x - 4 = 0 + 3 + 0 + 0 - 4 = 7
x = 2 için x + 3 + x + 2x - 4 = 2 + 3 + 2 + 4 - 4 = 7 olduğundan x + 3 + x + 2x - 4
ifadesinin en küçük değeri 7 dir.

Sıra Sizde

SORU:
x ! R için x - 1 + x - 2 + f + x - 21 ifadesinin en küçük değerini bulunuz.

ÇÖZÜM:

124 | Matematik 9
DENKLEMLER VE EŞİTSİZLİKLER

Mutlak Değerli Denklemler


İçinde bilinmeyen bulunduran ifadeler sıfıra eşitse denklem olarak adlandırılır. Eğer bir denklem, mutlak
değerli ifade bulunduruyorsa mutlak değerli denklem olarak adlandırılır.
2x + 3 = 0 ve - 2x + 1 = 0 birer denklem örneğidir. Burada ikinci denklem, mutlak değerli ifade
bulundurduğundan bir mutlak değerli denklemdir.
Mutlak değerli denklemler, mutlak değer tanımı kullanılarak çözülür. İfade, mutlak değerden kurtarıldıktan
sonra denklem çözüm yöntemleri ile denklemin çözüm kümesi bulunur.
Mutlak değerli denklemler, aşağıda verilen yöntemlerle çözülür.

i) a 2 0 olmak üzere x = a ise x = a veya x = - a

ÖRNEK 9:
x = 3 denkleminin çözüm kümesini bulunuz.

ÇÖZÜM:
I. Yol:
x = 3 denklemi, başlangıç noktasına uzaklıkları 3 br olan sayıların bulunması demektir. Bu sayıların - 3
ve 3 olduğu açıktır. Bu düşünceden yola çıkıldığında çözüm kümesi Ç = " - 3, 3 , olur.
II. Yol:
x = 3 & x = 3 veya x =- 3 tür. O hâlde denklemin çözüm kümesi Ç = " - 3, 3 , olur.

ÖRNEK 10:
x + 2 - 3 = 4 denkleminin çözüm kümesini bulunuz.

ÇÖZÜM:
x + 2 - 3 = 4 & x + 2 - 3 = 4 veya x + 2 - 3 =- 4
& x+2 = 7 x + 2 =- 1
Ç=Q
x + 2 = 7 veya x + 2 =- 7
x=5 x =- 9
Bu durumda denklemin çözüm kümesi Ç = " - 9, 5 , olur.

a 1 0 olmak üzere x = a & Ç = Q dir. Neden?

ii) x = y & x = y veya x =- y

ÖRNEK 11:
4x - 9 = 2x + 3 denkleminin çözüm kümesini bulunuz.

ÇÖZÜM:
4x - 9 = 2x + 3 & 4x - 9 = 2x + 3 veya 4x - 9 =-^2x + 3h
2x = 12 6x = 6
x=6 x=1
Bu durumda denklemin çözüm kümesi Ç = "1, 6, olur.

Matematik 9 | 125
DENKLEMLER VE EŞİTSİZLİKLER

iii ) x = y & x = y veya x =- y


x = y veya x =- y denklemleri çözüldüğünde bulunan köklerden y $ 0 koşulunu sağlayan kökler,
verilen mutlak değerli denklemin çözüm kümesi olarak alınır.

ÖRNEK 12:

2x - 7 = 6x + 3 denkleminin çözüm kümesini bulunuz.

ÇÖZÜM:

2x - 7 = 6x + 3 & 2x - 7 = 6x + 3 veya 2x - 7 =-]6x + 3g


= - 10 = 4x 8x = 4
-5 1
= x= 2 x= 2
5
x =- 2 değeri 6x + 3 ifadesinde yerine yazılırsa

6 $ b - 2 l + 3 =- 12 1 0 olduğundan x =- 2 değeri çözüm olarak alınmaz.


5 5

1
x = 2 değeri, 6x + 3 ifadesinde yerine yazılırsa

6 $ b 2 l + 3 = 6 2 0 olduğundan x = 2 değeri çözüm olarak alınır.


1 1

Bu durumda denklemin çözüm kümesi, Ç = & 2 0 olur.


1

ÖRNEK 13:

3x + 6 - 2 = x - 1 denkleminin çözüm kümesini bulunuz.

ÇÖZÜM:
Denklem, mutlak değer tanımı kullanarak çözüldüğünde
3x + 6 = 0 & 3x + 6 = 0 x-1 = 0&x-1 = 0
= x =- 2 = x = 1 kritik noktaları bulunur.
Bu kritik noktalara göre mutlak değerli ifadeler:
]Z] -]3x + 6g, x 1 - 2 ise ]Z] -]x - 1g, x 1 1 ise
]] ]]
3x + 6 = [] 0, x =- 2 ise ve x - 1 = [] 0, x = 1 ise olarak yazılır.
]] ]]
] 3x + 6, x 2 - 2 ise ] x - 1, x 2 1 ise
\ \
-3 -2 1 +3
x 1- 2 için - 2 # x 1 1 için x $ 1 için
-^3x + 6h - 2 =-]x - 1g ^3x + 6h - 2 =-]x - 1g ^3x + 6h - 2 = x - 1
- 3x - 6 - 2 =- x + 1 3x + 6 - 2 =- x + 1 3x + 6 - 2 = x - 1
- 2x = 9 4x =- 3 2x =- 5
9 3 5
x =- 2 x =- 4 x =- 2

- 2 ! ^- 3, - 2@ olduğundan - 4 ! _- 2, 1i olduğundan - 2 z 61, 3h olduğundan


9 3 5

çözüm kümesine alınır. çözüm kümesine alınır. çözüm kümesine alınmaz.

Bu durumda, denklemin çözüm kümesi Ç = &- 2 , - 40


9 3 olarak bulunur.

126 | Matematik 9
DENKLEMLER VE EŞİTSİZLİKLER

ÖRNEK 14:

2x + 4 - 3x + 6 + - 5x - 10 = 4 denkleminin çözüm kümesini bulunuz.

ÇÖZÜM:
2x + 4 - 3x + 6 + - 5x - 10 = 4
2 $ ]x + 2 g - 3 $ ]x + 2g + - 5 $ ]x + 2 g = 4
2 $ x + 2 - 3 $ x + 2 + -5 $ x + 2 = 4
2$ x+2 -3$ x+2 +5$ x+2 = 4
4$ x+2 = 4
x + 2 = 1 ise x + 2 = 1 veya x + 2 =- 1
& x =- 1 x =- 3
Ç = "- 3, 1,

iv) x + y = 0 & x = 0 ve y = 0

ÖRNEK 15:

3x - 9 + 5y + 20 + - 4z - 12 = 0 olduğuna göre x $ y $ z çarpımını bulunuz.

ÇÖZÜM:
Mutlak değerli ifadelerin toplamlarının sıfıra eşit olabilmesi için mutlak değerli ifadelerin ayrı ayrı sıfıra eşit
olması gerekir.
3x - 9 + 5y + 20 + - 4z - 12 = 0 & 3x - 9 = 0, 5y + 20 = 0, - 4z - 12 = 0
= x=3 y =- 4 z =- 3
Buradan x $ y $ z çarpımı 3 $ ^- 4h $ ^- 3h = 36 bulunur.

ÖRNEK 16:
x 2 - 5x + 6 - 4 $ x - 3 = 0 denkleminin çözüm kümesini bulunuz.

ÇÖZÜM:

x 2 - 5x + 6 - 4 $ x - 3 = ]x - 2g $ ]x - 3g - 4 $ x - 3
= x-2 $ x-3 -4$ x-3
= x - 3 $ ^ x - 2 - 4 h bulunur.

x - 3 $ ^ x - 2 - 4 h = 0 & x - 3 = 0 veya x - 2 - 4 = 0

x-3 = 0
x=3 x - 2 = 4 veya x - 2 =- 4
x=6 x =- 2 olur.
Buna göre çözüm kümesi Ç = " - 2, 3, 6 , dır.

Matematik 9 | 127
DENKLEMLER VE EŞİTSİZLİKLER

Mutlak Değerli Eşitsizlikler


Mutlak değerli ifade eşitsizlik şeklinde verilmişse mutlak değerli eşitsizlikler olarak adlandırılır.
Mutlak değerli eşitsizliklerin çözümü, mutlak değer tanımı dikkate alınarak çözülür.

i) a $ 0 olmak üzere x # a & - a # x # a

ÖRNEK 17:
x # 4 eşitsizliğinin çözüm kümesini bularak sayı doğrusunda gösteriniz.

ÇÖZÜM:
x # 4 eşitsizliğinin anlamı, mutlak değeri 4 ve 4 ten küçük sayılar demektir. Başka bir ifadeyle sıfıra olan
uzaklıkları 4 ve 4 ten küçük olan sayılardır.
x # 4 & - 4 # x # 4 Buradan çözüm kümesi Ç = 6- 4, 4@ olur.
Çözüm kümesi, sayı doğrusu üzerinde aşağıdaki gibi gösterilir:

-4 4

a 1 0 olmak üzere x # a & Ç = Q dir.

ÖRNEK 18:

- 3x + 2 1 6 eşitsizliğinin çözüm kümesini bulunuz.

ÇÖZÜM:
- 3x + 2 1 6 & - 6 1 - 3x + 2 1 6
& & - 6 - 2 1 - 3x 1 6 - 2
& & - 8 1 - 3x 1 4
8 4
& & 3 2 x 2 - 3 bulunur.

Bu durumda, eşitsizliğin çözüm kümesi Ç = b - 3 , 3 l olur.


4 8

ÖRNEK 19:
x, y ! R olmak üzere 2x - 3 # 5 ve 3y # 6 koşullarını sağlayan tüm ^x, y h sıralı ikililerini kapsayan en
küçük kapalı bölgenin alanını bulunuz.

ÇÖZÜM:
y
2x - 3 # 5 & - 5 # 2x - 3 # 5
A D
& & - 2 # 2x # 8 2

& & - 1 # x # 4f ]1g

3y # 6 & - 6 # 3y # 6 x
-1 0 1 4
6 6
& &- 3 # y # 3
& & - 2 # y # 2f ]2g B -2 C

Oluşan şekil bir dikdörtgen olduğundan A ]ABCDg = 5 $ 4 = 20 br 2 olarak bulunur.

128 | Matematik 9
DENKLEMLER VE EŞİTSİZLİKLER

ii) a $ 0 olmak üzere x $ a & x $ a veya x #- a

ÖRNEK 20:
x $ 3 eşitsizliğinin çözüm kümesini bularak sayı doğrusu üzerinde gösteriniz.

ÇÖZÜM:
x $ 3 eşitsizliğinin anlamı, mutlak değeri 3 ve 3 ten büyük sayılar demektir. Başka bir ifadeyle sıfıra olan
uzaklıkları 3 ve 3 ten büyük olan sayılardır.
x $ 3 & x $ 3 veya x #- 3 yazılır. Bu durumda eşitsizliğin çözüm kümesi Ç = ^- 3, - 3@ , 63, 3h olur.
Sayı doğrusu üzerinde gösterimi aşağıda verilmiştir

-3 3

ÖRNEK 21:
2x + 1 2 3 ve 4x + 5 1 13 eşitsizliklerini sağlayan x tam sayılarının toplamını bulunuz.

ÇÖZÜM:
2x + 1 2 3 & 2x + 1 2 3 veya 2x + 1 1 - 3
2 2x 2 2 1 2x 1 - 4 olduğundan Ç 1 = ^- 3, - 2h , ^1, 3 h olur.
x21 1 x 1-2
2
Çözüm kümesinin sayı doğrusunda gösterilişi aşağıdaki gibidir.

-2 1

4x + 5 1 13 & - 13 1 4x + 5 1 13

& & - 18 1 4x 1 8
& & - 4 1 x 1 2 buradan - 2 1 x 1 2 dir. Ç 2 = b - 2 , 2 l olur.
18 9 9

Çözüm kümesinin sayı doğrusunda gösterilişi aşağıdaki gibidir.

2
-9
2

Ç = Ç 1 + Ç 2 = 6^- 3, - 2 h , ^1, 3 h@ + b - 2 , 2 l = b - 2 , - 2 l , ^1, 2h
9 9

Genel çözüm kümesinin sayı doğrusunda gösterilişi aşağıdaki gibidir.
-2 1 2
-9
2

Bu aralıktaki tam sayılar - 4 ve - 3 olduğundan toplamları ]- 4g + ]- 3g =- 7 olur.

iii) a $ 0, b $ 0 olmak üzere a 1 x 1 b ise x 2 a ve x 1 b veya a 1 x 1 b, - b 1 x 1 - a

ÖRNEK 22:
2 1 x 1 5 eşitsizliğinin çözüm kümesini araştırınız.

ÇÖZÜM:
2 1 x 1 5 & x 1 5 ve x 2 2 demektir.
x 1 5 & - 5 1 x 1 5f ^1 h ve
x 2 2 & x 2 2 veya x 1- 2f ^2 h yazılır.
Buna göre verilen eşitsizliğin çözüm kümesi (1) ve (2) nin kesişimi alınarak bulunur.
Ç = ^- 5, - 2 h , ^2, 5 h tir.

Matematik 9 | 129
DENKLEMLER VE EŞİTSİZLİKLER

ÖRNEK 23:

4 # 3x + 7 # 8 eşitsizliğini sağlayan x tam sayılarının sayısını bulunuz.

ÇÖZÜM:
4 # 3x + 7 # 8 & 4 # 3x + 7 # 8 veya - 8 # 3x + 7 # - 4
& - 3 # 3x # 1 - 15 # 3x # - 11
1 11
& -1 # x # 3 - 5 # x # - 3 yazılır.
Buradan verilen eşitsizliğin çözüm kümesi Ç = :- 5, - 3 D , :- 1, 3 D olur.
11 1

Bu aralıktaki tam sayıların kümesi " - 5, - 4, - 1, 0 , olduğundan eşitsizliği sağlayan tam sayıların sayısı 4
tanedir.

iv) x 1 y & x 2 1 y 2

ÖRNEK 24:
x + 3 1 x - 1 eşitsizliğinin çözüm kümesini bulunuz.

ÇÖZÜM:

x + 3 1 x - 1 & ]x + 3g2 1 ]x - 1g2


& x 2 + 6x + 9 1 x 2 - 2x + 1
& 8x + 8 1 0
& x 1- 1 bulunur.
Eşitsizliğin çözüm kümesi Ç = ^- 3, - 1h olur.
ÖRNEK 25:
x - 1 # 5 olmak üzere y = 4x - 8 ifadesinin alabileceği en büyük ve en küçük tam sayı değerlerinin
toplamını bulunuz.

ÇÖZÜM:

x - 1 # 5&- 5 # x - 1 # 5
& 4 -4 # x # 6
- 16 # 4x # 24
- 24 # 4x - 8 # 16 ise 0 # 4x - 8 # 24 olur.
Buna göre y = 4x - 8 ifadesinin en büyük tam sayı değeri 24, en küçük tam sayı değeri 0 olur.
Buradan 24 + 0 = 24 olur.

ÖRNEK 26:
y = x + 2 denklemini sağlayan ^x, y h ikililerinin kümesini analitik düzlemde gösteriniz.
y
ÇÖZÜM:

I. Durum x $- 2 için y = x+2 y =-x - 2 y = x+2


2
II. Durum x #- 2 için y =- x - 2

Bu iki durumun birleşimi y = x + 2 denkleminin düzlemdeki görüntüsüdür. -2 0


x

130 | Matematik 9
DENKLEMLER VE EŞİTSİZLİKLER

4. Birinci Dereceden İki Bilinmeyenli Denklem ve Eşitsizlik Sistemleri


Birinci Dereceden İki Bilinmeyenli Denklem Sistemleri
x, y bilinmeyen ve a 1, a 2, b 1, b 2, c 1, c 2 ! R olmak üzere a 1 x + b 1 y + c 1 = 0 ve a 2 x + b 2 y + c 2 = 0
denklemlerinden oluşan
a1 x + b1 y + c1 = 0
2 sistemine birinci dereceden iki bilinmeyenli denklem sistemi denir.
a2 x + b2 y + c2 = 0
Her iki denklemi de sağlayan (x,y) ikililerinin kümesine denklem sisteminin çözüm kümesi denir. Bu tür
denklem sistemleri “yok etme, yerine koyma, grafik” yöntemlerinden herhangi biri kullanılarak çözülebilir.

Birinci dereceden iki bilinmeyenli bir denklemin analitik düzlemde bir doğru belirttiğini biliyorsunuz.
a 1 x + b 1 y + c 1 = 0 denkleminin belirttiği doğru, d 1 olsun.
a 2 x + b 2 y + c 2 = 0 denkleminin belirttiği doğru, d 2 olsun.
Buna göre
I. Durum:
a1 b1 c1
a 2 = b 2 = c 2 ise bu iki doğru çakışıktır. d 1 = d 2 tir. Yani iki doğrunun bütün noktaları ortaktır.
Bu durumda denklem sisteminin çözüm kümesi sonsuz elemanlıdır.
II. Durum:
a1 b1 Y c1
a 2 = b 2 = c 2 ise bu iki doğru paraleldir. d 1 ' d 2 dir.
Bu durumda denklem sisteminin çözüm kümesi boş kümedir. Ç = Q
III. Durum:
a1 Y b1
a 2 = b 2 ise bu iki doğru tek noktada kesişir. Ortak nokta, denklem sisteminin çözümüdür.
Bu durumda denklem sisteminin çözüm kümesi tek elemanlıdır.

ÖRNEK 1:
2x - 3y = m

x + ky = 4 denklem sistemini sağlayan ikili ^- 2, 2 h ise k $ m değerini bulunuz.
ÇÖZÜM:
^ - 2, 2 h ikilisi her iki denklemi de sağlar. 2 $ ]- 2g - 3 $ 2 = m & m =- 10
- 2 + k $ 2 = 4 & 2k = 6 & k = 3 olduğundan k $ m =- 30 olarak bulunur.

ÖRNEK 2:
ax - 3y = 4

2x + y = b sisteminin çözüm kümesi sonsuz elemanlı olduğuna göre a ve b değerlerini bulunuz.
ÇÖZÜM:
Çözüm kümesi, sonsuz elemanlı olduğundan doğrular çakışıktır. Bu durumda
a -3 4 -4
2 = 1 = b & a =- 6, b = 3 tir.

ÖRNEK 3:
5x + ]a + 1g y = 2

2x - 4y = 3 sisteminin çözüm kümesi boş küme olduğuna göre a nın değerini bulunuz.
ÇÖZÜM:
Sistemin çözüm kümesi boş küme ise doğrular paraleldir.
5 a+1 Y 2
2 = - 4 = 3 & 2a + 2 =- 20 & 2a =- 22 & a =- 11 tir.

Matematik 9 | 131
DENKLEMLER VE EŞİTSİZLİKLER

ÖRNEK 4:
x - 2y = 4

3x + 2y =- 12 denklem sisteminin çözüm kümesinin varlığını inceleyiniz. Grafik çizerek yorumlayınız.
ÇÖZÜM:
1 Y -2
3 = 2 olduğundan doğrular tek noktada kesişir. Sistemin çözümü ortak noktayı verir. Bunun için yerine
koyma, yok etme veya grafikle çözüm yöntemlerinden faydalanılır.

I. Yol: Yerine Koyma Yöntemi


x - 2y = 4 & x = 4 + 2y ifadesi diğer denklemde yerine yazılırsa
3x + 2y =- 12 & 3 ^4 + 2yh + 2y =- 12
& 12 + 6y + 2y =- 12
& 8y =- 24 ise y =- 3 olur.
Bulunan y değeri x = 4 + 2y ifadesinde yerine yazılırsa x = 4 + 2 $ ^- 3h & x =- 2 olarak bulunur.
Denklem sisteminin çözüm kümesi Ç = "^- 2, - 3 h, olur.
II. Yol: Yok Etme Yöntemi
Denklem sisteminde taraf tarafa toplama işlemi yapılırsa
x - 2y = 4
+ 3x + 2y =- 12
4x =- 8 & x =- 2 olarak bulunur.
Dolayısıyla x değeri, bu iki denklemden herhangi birinde yerine yazılırsa
3 ]- 2g + 2y =- 12 & 2y =- 6 & y =- 3 olur.
O hâlde denklem sisteminin çözüm kümesi Ç = "^- 2, - 3 h, olur. 3x + 2y = -12
y

III. Yol: Grafik Çizme Yöntemi


x - 2y = 4 için 3x + 2y =- 12 için x

x = 0 için y =- 2 $ ^0, - 2h x = 0 için y =- 6 $ ^0, - 6 h -4 0 4

y = 0 için x = 4 $ ^4, 0 h y = 0 için x =- 4 $ ^- 4, 0 h K(-2,-3)


-2

Yandaki grafikte iki doğrunun tek noktada kesiştiği görülüyor. Bu


noktaya K denir. K noktası, denklem sisteminin çözüm kümesi olur.
-6
Noktanın koordinatları, yok etme ya da yerine koyma yöntemiyle x - 2y = 4
bulunmuştu.

ÖRNEK 5:
13x + 19y = 2

17x - 11y = 3 denklem sistemine göre yx değerini bulunuz.
ÇÖZÜM:
-2 17x - 11y = 3
3 13x + 19y = 2
- 34x + 22y =- 6
+ 39x + 57y = 6
5x + 79y = 0
x 79
5x =- 79y & y =- 5 olarak bulunur.

132 | Matematik 9
DENKLEMLER VE EŞİTSİZLİKLER

ÖRNEK 6:
3x - 4y = 12

6x - 8y = 24 denklem sisteminin çözüm kümelerinin varlığını inceleyiniz. Grafik çizerek yorumlayınız.
ÇÖZÜM:
3 4 12
6 = 8 = 24 olduğundan doğrular çakışıktır. y

Yanda görüldüğü gibi doğrular aynı noktalardan geçer. Doğruların


x
bütün noktaları ortaktır. O hâlde denklem sisteminin sonsuz çözümü
0 4
vardır. Yok etme yöntemiyle de bunu görebilirsiniz.
- 2 3x - 4y = 12
+ 6x - 8y = 24
0=0 -3
olup doğru üzerindeki bütün noktalar ortaktır.
Ç = "^x, yh ; 3x - 4y = 12, x ! R, y ! R , olup sonsuz elemanlıdır.

ÖRNEK 7:
x + 2y = 4

3x + 6y = 3 denklem sisteminin çözüm kümelerinin varlığını inceleyiniz. Grafik çizerek yorumlayınız.
ÇÖZÜM:
y
1 2 4
3 = 6 Y = 3 olup doğrular paraleldir. Doğruların grafiklerini çizmek
için geçtikleri ikişer nokta bulunursa
x + 2y = 4 3x + 6y = 3 2
1 x + 2y = 4
x = 0 için y = 2 x = 0 için y = 2
1/2
y = 0 için x = 4 y = 0 için x = 1 x
değerleri elde edilir. Noktalar yardımıyla grafik çizildiğinde 0 1 4
doğruların paralel olduğu görülür. O hâlde ortak noktaları yoktur. 3x + 6y = 3
Bu durumda denklem sisteminin çözümü boş kümedir.

ÖRNEK 8:
2 3 b_b
x+1 - y+1 = 5 bb
bb
b`b
-1 8 bb
x + 1 + y + 1 =- 2 bb denklem sisteminin çözüm kümesini bulunuz.
a
ÇÖZÜM:
2 3
x+1 - y+1 = 5
-1 8
+ 2 x + 1 + y + 1 =- 2
13 &
y + 1 = 1 y + 1 = 13 & y = 12
Birinci denklemde y = 12 yazılırsa
2 3 2 3 2 68
x + 1 - 13 = 5 & x + 1 = 5 + 13 & x + 1 = 13
x + 1 13
& 2 = 68
x = 34 bulunur. Çözüm kümesi Ç = &b 34 , 12 l0 dir.
- 21 - 21
&

Matematik 9 | 133
DENKLEMLER VE EŞİTSİZLİKLER

Birinci Dereceden İki Bilinmeyenli Eşitsizlik Sistemleri


x, y birer bilinmeyen olmak üzere en az iki tane birinci dereceden iki bilinmeyenli eşitsizliğin oluşturduğu
sisteme birinci dereceden iki bilinmeyenli eşitsizlik sistemi denir.
Eşitsizlik sistemlerinin çözümü iki eşitsizliğin çözümünün kesişimidir.

ÖRNEK 9:
5x - 2y < 10

x + y $ 3 eşitsizlik sisteminin çözümünü koordinat düzleminde gösteriniz.

ÇÖZÜM: y

5x - 2y = 10 için x + y = 3 için 3 5x - 2y = 10
x = 0 ise y =- 5 $ ^0, - 5h x = 0 ise y = 3 $ ^0, 3h

y = 0 ise x = 2 $ ^2, 0h y = 0 ise x = 3 $ ^3, 0h x
5x - 10
5x - 2y 1 10 & 2 < y olup doğrunun üst bölgesi alınır. 0 2 3

x + y $ 3 & y $ 3 - x olup doğrunun üst bölgesi alınır. x+y = 3

Bu iki bölgenin kesişimi çözüm kümesidir.


-5

ÖRNEK 10:
y>x
x + 2y # 6
x $ 1 eşitsizlik sisteminin çözümünü koordinat düzleminde gösteriniz.

ÇÖZÜM:

Doğruların grafikleri düzlemde çizilip bölgeler taranırsa y
istenen çözüm bölgesi bulunur.
y = x
x + 2y = 6 için
x = 0 için y = 3
y = 0 için x = 6
3
6-x
x + 2y # 6 ise y # 2 olup doğrunun alt bölgesi taranır.
x + 2y = 6
y > x olduğundan doğrunun üst bölgesi taranır.
x $ 1 olduğundan x = 1 doğrusunun sağ tarafı taranır.
x
İstenen çözüm bölgesi, bu üç bölgenin kesişimidir. 0 1 6

134 | Matematik 9
DENKLEMLER VE EŞİTSİZLİKLER

ÖRNEK 11:
x - 2 + y + 1 = 4 denklemini sağlayan ^x, y h ikililerinin kümesini analitik düzlemde gösteriniz.

ÇÖZÜM:
x - 2 nin kritik noktası 2 dir. y + 1 in kritik noktası - 1 dir. Bu durumda oluşacak dört farklı durum
aşağıdaki tabloda gösterilmiştir. y

I. Durum x $ 2, y $- 1 x-2+y+1 = 4 x+y = 5 3


II. Durum x $ 2, y #- 1 x-2-y-1 = 4 x-y = 7 –x+y=1 x+y= 5
III. Durum x # 2, y #- 1 -x + 2 - y - 1 = 4 -x - y = 3 1
–2 –1 2 5 6 x
IV. Durum x # 2, y $- 1 -x + 2 + y + 1 = 4 -x + y = 1
–1
Bu durumun birleşimi x - 2 + y + 1 = 4 denkleminin düzlemdeki
görüntüsüdür. 3
–x –y =3
Elde edilen şekil köşegenlerinin kesişim noktası ^2, - 1 h ve bir
x– y=7

kenarının uzunluğu 4 2 birim olan bir karedir. –5

a) x - a + y - b = c, ]c $ 0g şeklindeki iki bilinmeyenli denklemler analitik düzlemde ^a, b h


merkezli bir kenarı c 2 birim olan bir kare belirtir.
b) x - a + y - b < c, ]c $ 0g şeklindeki iki bilinmeyenli eşitsizlik analitik düzlemde ^a, b h merkezli
bir kenarı c 2 birim olan bir karenin iç bölgesini belirtir.
c) x - a + y - b > c, ]c $ 0g şeklindeki iki bilinmeyenli eşitsizlik analitik düzlemde ^a, b h merkezli
bir kenarı c 2 birim olan bir karenin dış bölgesini belirtir.

ÖRNEK 12:
x + y #4
eşitsizlik sistemini sağlayan ^x, y h ikililerinin kümesini analitik düzlemde gösteriniz.
x+1 < 2 y

ÇÖZÜM: 4
x, x$0
1. x + y # 4 eşitsizliğinde x = ) ve -x + y = 4 x+y = 4
y, y$0 - x, x # 0
y =) olduğundan aşağıdaki dört durum oluşur.
- y, y # 0
x
-4 0 4
I. Durum x $ 0, y $ 0 için x+y # 4
II. Durum x # 0, y $ 0 için -x + y # 4
-x - y = 4 x-y = 4
III. Durum x # 0, y # 0 için -x - y # 4
IV. Durum x $ 0, y # 0 için x-y # 4 -4

x + y # 4 eşitsizliğinin çözüm bölgesi, bu dört durumun birleşimi ile elde edilen karenin sınırı ve iç
bölgesidir. y

4
2. x + 1 < 2 & - 2 < x + 1 < 2 & - 3 < x < 1 -x + y = 4 x+y = 4
y
4

(1) ve (2) bölgelerinin analitik x


–4 –3 0 1 4
düzlemdeki kesişim bölgesi
x eşitsizlik sisteminin çözümüdür.
–3 0 1 -x - y = 4 x-y = 4

–4

Matematik 9 | 135
DENKLEMLER VE EŞİTSİZLİKLER

ALIŞTIRMALAR

1 2-1 + 2- 3 - 3 - 2 işleminin 7
- 5x + 3 = - 6 + 2x
sonucunu bulunuz. denkleminin kökler çarpımını bulunuz.

2 - 2 1 x 1 4 olmak üzere 8 x + 3 = 2x + 5
x - 4 - 3x + 7 + 4x ifadesinin değerini denkleminin çözüm kümesini bulunuz.
bulunuz.

3 x 1 0 1 y 1 z olmak üzere aşağıdaki 9 - 2x + 6 + 5 = x + 2


ifadelerin değerini bulunuz denkleminin çözüm kümesini bulunuz.
a) x - y + y - z - y - x

b) x - y - y + z + - z
x

4
x 2 0 olmak üzere 10 x, y ! R olmak üzere 2x + 3 = 1 ve
3x - 2x + 3. - x ifadesinin değerini 4y + 5 =- 2x + 1 olduğuna göre y nin
bulunuz. en küçük değerini bulunuz.

2x - 1 + 3 = 5 28
5 11 x ! R için x + 3
+ x + 2x - 4
denkleminin çözüm kümesini bulunuz.
ifadesinin en büyük değerini bulunuz.

6 2x + 5 + 5y - 3 + 3z + 7 = 0 12 - 2 1 x 1 3 olmak üzere, y = 5 + 3x
olduğuna göre x.y.z çarpımını bulunuz. ifadesinin alabileceği en büyük tam sayı
değerini bulunuz.

136 | Matematik 9
DENKLEMLER VE EŞİTSİZLİKLER

13 x - 2 1 3 ve 5x - y + 2 = 0 koşullarını Çözümü - 2 1 x 1 6 olan mutlak değerli


19
sağlayan en büyük y tam sayı değerini eşitsizlik x - a 1 b olduğuna göre a 2 - b 2
bulunuz. toplamını bulunuz.

2x - 3 3x - 6 + x + 1 # 2
14 5 1 1 eşitsizliğini sağlayan tam sayıların 20
eşitsizliğinin çözüm kümesini bulunuz.
toplamını bulunuz.

x - 2 + 3x - 6 + 4x - 8 # 16 a ve b birer pozitif tam sayıdır.


15 21
eşitsizliğini sağlayan doğal sayıların sayısını ]7a - 3bg]4a - 5bg = 19 olduğuna göre
bulunuz. a 2 + b 2 değerini bulunuz.

16 2x - 7 $ 5 eşitsizliğinin çözüm kümesini 22 ^ x + y h k + ^ 2x - y h m


= 5x - y eşitliği her
bulunuz. x, y ! R için sağlanıyorsa ^k, mh sıralı ikilisini
bulunuz.

17 2 1 4x - 5 1 7 eşitsizliğini sağlayan x tam 23


x + 3y - 6 # 0
sayılarının çarpımını bulunuz. 2x - 5y + 10 > 0
eşitsizlik sisteminin çözümünü koordinat
düzleminde gösteriniz.

18 x+2 # x+1 24 y $ x +1
eşitsizliğinin çözüm kümesini bulunuz. y<3
eşitsizlik sisteminin çözüm kümesini analitik
düzlemde gösteriniz.

Matematik 9 | 137
DENKLEMLER VE EŞİTSİZLİKLER

9.3.3. ÜSLÜ İFADELER VE DENKLEMLER

Etkinlik

Rehber Öğretmen Okul Müdürü Fikret Bey, okul adına “Doğa


0
2 =1 Gönüllüleri Hatıra Ormanı” oluşturmak istemektedir.
(1. Adım) Bu amaçla okulun Çevre Kulübü’nden bu konuyla ilgili
bir proje yapmalarını ister.
Çevre Kulübü rehber öğretmeni başkanlığında
öğrencilerle yapılan toplantıda “Bir Meşe Palamudu da
1 Sen Ek” adında bir proje hazırlanmasına karar verilir.
2 =2 Proje kapsamında, İl Orman Müdürlüğünden temin
(2. Adım)
edilen meşe palamutlarının dağıtımında aşağıdaki
yöntem belirlenir:
2
2 =4 Bir meşe palamudunu eken rehber öğretmen, iki
(3. Adım) öğrenciye iki adet meşe palamudu tohumu verecektir.
Tohumu alan her öğrenci, iki arkadaşının da aynı
şekilde meşe palamudu ekmesini sağlayacaktır. Kelebek
etkisiyle bu işlemin devam etmesi planlanmaktadır.

Şekilde belirtilen adımlara göre “Bir Meşe Palamudu da Sen Ek” projesine yönelik aşağıdaki
soruları cevaplayınız.
1. 5. adımın sonunda toplam kaç palamut dağıtıldığını nasıl ifade edersiniz?
2. 10. adımda kaç palamut tohumu dağıtılmış olur?
3. R
ehber öğretmen, projeyi 3 öğrenci ile başlatıp 5. adıma kadar dağıtım yapsaydı kaç palamut
tohumu daha dağıtılmış olurdu?
4. Siz de belirlediğiniz herhangi bir adımda meşe palamudu dağıtılan kişi sayısını belirtiniz.
5. Daha önce de uygulanmış olan benzer projeleri iki hafta süresince araştırarak sonuçlarını
tartışınız.

138 | Matematik 9
DENKLEMLER VE EŞİTSİZLİKLER

1. Üslü İfade İçeren Denklemler


Üslü İfadeler
x bir gerçek sayı ve n bir doğal sayı olmak üzere n tane x gerçek sayısının çarpımı
n
x.x.x.....x = x n şeklinde gösterilir. x sayısı x in n. kuvveti (üssü) şeklinde okunur.
"
n tane
n
x ifadesinde x sayısına taban, n sayısına üs denir.
Üslü Sayıların Özellikleri
x, y gerçek sayı ve m, n doğal sayı olmak üzere

Tüm gerçek sayıların birinci kuvveti kendisidir.


1 6x ! R için x 1 = x tir.
8 1 = 8, ]- 6g1 =- 6, b - 3 l =- 3 , 0 1 = 0
1 1 1

Sıfırdan farklı sayıların sıfırıncı kuvveti 1 dir.


2 = 0 olmak üzere x 0 = 1 dir.
xY
3 0 = 1, ]- 1g0 = 1, b - 5 l = 1
2 0

1
xY= 0 olmak üzere x -n = n
= 2, ` 7 j = 9
3 x 1 -4 -1 1 3 -2 49
1 4 = 5 , 2
veya x n = -n dir. 5
x

Tabanları aynı olan iki üslü sayının çarpımında üsler


toplanır.
6 3 .6 5 = ]6.6.6g . ]6.6.6.6.6g = 6 3 + 5 = 6 8,
4 x m .x n = x m + n dir.
y 3 .y 2 .y 5 = y 3 + 2 + 5 = y 10

^ x m hn = ^x n hm = x m.n dir. ^ 4 3 h2 = 4 3 $ 4 3 = 4 3 + 3 = 4 3.2 = 4 6 veya


5
Üslü sayıların üssü alınırken üsler çarpılır. ^ 4 2 h3 = 4 2 $ 4 2 $ 4 2 = 4 2 + 2 + 2 = 4 2.3 = 4 6

n bir doğal sayı olmak üzere


6 ]- 1g2n = 1 ve ]- 1g2n + 1 =- 1 dir. ]- 1g4 = 1, ]- 1g7 =- 1
- 1 in çift kuvvetleri 1, tek kuvvetleri - 1 dir.

Tabanları aynı olan iki üslü sayının bölümünde payın


= 0 olmak üzere
xY üssünden paydanın üssü çıkarılır.
7 xn 73
xm
= x n .x -m = x n - m = 7 3 - ] -5 g = 7 3 + 5 = 7 8
7 -5

Pozitif gerçek sayıların tüm kuvvetleri pozitiftir.


a) x 2 0 ve n ! Z olmak üzere 1
x n 2 0 dır. 3 2 2 0, 5 -2 = 2 2 0
5

Bu özellikten negatif gerçek sayıların çift


8 kuvvetlerinin pozitif, tek kuvvetlerinin negatif
b) x 1 0 ve n ! Z olmak üzere
olduğu sonucu elde edilir.
n tek ise x n 1 0
* ]- 3g3 = ]- 3g $ ]- 3g $ ]- 3g =- 27 1 0
n çift ise x n 2 0
]- 2g4 = ]- 2g $ ]- 2g $ ]- 2g $ ]- 2g = 16 2 0

Matematik 9 | 139
DENKLEMLER VE EŞİTSİZLİKLER

Üsleri aynı olan üslü sayıların çarpımında tabanlar


çarpımına ortak üs yazılır.
x n $ y n = ^x $ yh 5 3 $ 3 3 = ]5 $ 3g3 = 15 3
n
9
^ x 2 h5 $ ^ y 3 h = ^x 2 $ y 3 h = x 10 $ y 15
5 5

Üs negatif ise taban ters çevrilir.


10 c x m = x n ve c x m = x -n = y n = b y l dir.
n n -n -n n n

b 4 l = b 5 l , b 1 l = 3 5 tir.
-3 3 -5
y y y y x x
5 4 3

Üsleri aynı olan ifadeler toplanıp çıkarılabilir.


3 $ 5 2 - 2 $ 5 2 + 4 $ 5 2 = 5 2 $ ]3 - 2 + 4 g
a, b ! R olmak üzere
11
a $ x n ! b $ x n = x n $ ]a ! bg dir.
= 5 2 $ 5 = 5 3 = 125

ÖRNEK 1:
^ - 3 3 h $ 3 4 $ 3 -2 $ ]- 3g2 işleminin sonucunu bulunuz.

ÇÖZÜM:
]- 3g2 = 3 2 ve 3 -2 = 12 olarak yazılır. Bu değerler, yerine yazıldığında
3
^ - 3 3 h $ 3 4 $ 3 -2 $ ]- 3g2 =- 3 3 $ 3 4 $ 12 $ 3 2 =- 3 3 + 4 =- 3 7 bulunur.
3

ÖRNEK 2:
^ - x 5 h $ x -2 $ ]- xg3 $ ]- xg6
x 3 $ x 5 $ ^- x 4h
işleminin sonucunu bulunuz.

ÇÖZÜM:
^ - x 5 h $ x -2 $ ]- xg3 $ ]- xg6 - x 5 $ x -2 $ ^ - x 3 h $ x 6 x 5 - 2 + 3 + 6 x 12
x $ x $ ^- x h
= 3+5+4 = 12 = =- 1
3 5 4
-x -x - x 12

ÖRNEK 3:
3 21 + 3 23 + 3 25
işleminin sonucunu bulunuz.
3 15 + 3 17 + 3 19
ÇÖZÜM:
Kesrin payı 3 21 , paydası 3 15 parantezine alınırsa
3 21 + 3 23 + 3 25 3 $ ^1 + 3 + 3 h 3 21
21 2 4
= 3 21 - 15 = 3 6 olur.
3 $ ^1 + 3 + 3 h 3 15
15 17 19 = 15 2 4 =
3 +3 +3

ÖRNEK 4:
2x + 4 + 2x + 3 - 2x + 2
işleminin sonucunu bulunuz.
2x + 2 + 2x + 1
ÇÖZÜM:
Üslü sayı özellikleri kullanılırsa
2 x + 4 + 2 x + 3 - 2 x + 2 2 x $ 2 4 + 2 x $ 2 3 - 2 x $ 2 2 2 $ ^2 + 2 - 2 h 16 + 8 - 4 20 10
x 4 3 2

2 $ ^2 + 2h
x+2 x+1 = x 2 x = x 2 = 4 + 2 = 6 = 3 olur.
2 +2 2 $2 +2 $2

140 | Matematik 9
DENKLEMLER VE EŞİTSİZLİKLER

ÖRNEK 5:

3 x = 4 olduğuna göre 9 2x + 2 işleminin sonucunu bulunuz.

ÇÖZÜM:
9 2x + 2 ifadesi 3 x e göre düzenlenirse
9 2x + 2 = ^3 2 h2x + 2 = 3 4x + 4 = ^3 x h4 $ 3 4 = 4 4 $ 3 4 = ]3 $ 4g4 = 12 4 olur.

ÖRNEK 6:

64 2 $ 125 3 çarpımının basamak sayısını bulunuz.

ÇÖZÜM:
Basamak sayısı hesaplamada, sayıyı 10 un kuvveti şeklinde yazmak işlemi kolaylaştırır.
64 2 $ 125 3 = ^2 6 h2 $ ^5 3 h3 = 2 12 $ 5 9 = 2 9 $ 2 3 $ 5 9 = ^2 9 $ 5 9 h $ 2 3 = 10 9 $ 2 3 = 8000...0
144424443
9 tane
2 3
Çarpımın sondan 9 basamağı sıfır olduğundan 64 $ 125 çarpımı 10 basamaklı olur.

ÖRNEK 7:

2 x = a, 3 x = b, 5 x = c olduğuna göre 90 x in a, b ve c türünden değerini bulunuz.

ÇÖZÜM:
90 sayısı asal çarpanlarına ayrıldığında
90 2 b_b
bb
45 3 bb
bb
15 3 `b & 90 = 2 $ 3 2 $ 5 olur. Buradan 90 x = ^2 $ 3 2 $ 5hx = 2 x $ ^3 x h2 $ 5 x = ab 2 c olur.
bb
5 5 bb
bb
1 b
a

ÖRNEK 8:

24 x = n, 3 x = m olduğuna göre 64 x + 1 in n ve m türünden değerini bulunuz.

ÇÖZÜM:
24 2 b_b
bb
12 2 bb
bb
`b & 24 = 2 3 $ 3 1 olduğundan n = 24 = ^2 $ 3 h = 8 $ 3 , 3 = m değeri yerine yazılırsa
x 3 1 x x x x
6 2
bb x x n
3 3 bb n = 8 $ m & 8 = m bulunur.
bb
1 b
a
64 x + 1 = ^8 2 hx + 1 = ^8 x h2 $ 8 2 = 8 2 $ b m l = 64 $ 2 olur.
n 2 n2
m

Matematik 9 | 141
DENKLEMLER VE EŞİTSİZLİKLER

ÖRNEK 9:

9 2 + 18 2 + 27 2 = a olduğuna göre 36 2 + 72 2 + 108 2 değerini a türünden bulunuz.

ÇÖZÜM:
Verilen ifadeden a türünden yazılırsa
36 2 + 72 2 + 108 2 = ]4 $ 9g2 + ]4 $ 18g2 + ]4 $ 27g2
= 4 2 $ 9 2 + 4 2 $ 18 2 + 4 2 $ 27 2
= 4 2 $ ^9 2 + 18 2 + 27 2 h
= 16 a olur.

Üslü Denklemler
İçinde üslü ifade bulunduran denklemlere üslü denklem adı verilir. Bu tür denklemlerin çözümü belli
özellikler kullanılarak yapılır. Bunlar şunlardır:

1) a ! R - " - 1, 0, 1 , olmak üzere a x = a y & x = y dir. (Tabanları eşit, iki üslü sayının üsleri de eşittir.)

ÖRNEK 10:

3 2x + 4 = 3 3x - 1 olduğuna göre x değerini bulunuz.

ÇÖZÜM:
3 2x + 4 = 3 3x - 1 & 2x + 4 = 3x - 1
& x = 5 olur.

ÖRNEK 11:

5 x + 2 + 5 x + 1 = 150 olduğuna göre x değerini bulunuz.

ÇÖZÜM:
5 x + 2 + 5 x + 1 = 150
5 x $ 5 2 + 5 x $ 5 = 150
5 x $ ^5 2 + 5 h = 150
5 x $ 30 = 30 $ 5 & 5 x = 5 1
& x = 1 olur.

ÖRNEK 12:
m ve n birer doğal sayıdır ve 2 m $ 5 n sayısı 11 basamaklı en küçük doğal sayı olduğuna göre n $ m
çarpımını bulunuz.

ÇÖZÜM:
2 m $ 5 n sayısı, 11 basamaklı en küçük doğal sayı ise bu sayı son on basamağı sıfır olan
10 000 000 000 doğal sayısı olur.
2 n $ 5 m = 10 000 000 000
2 n $ 5 m = 10 10
2 n $ 5 m = ]2 $ 5g10
2 n $ 5 m = 2 10 $ 5 10 & m = 10 ve n = 10 & m $ n = 100 bulunur.

142 | Matematik 9
DENKLEMLER VE EŞİTSİZLİKLER

ÖRNEK 13:
^0, 25 h-x + 2 = 8 2x + 1 olduğuna göre x değerini bulunuz.

ÇÖZÜM:
Verilen değerler, tabanları eşit iki üslü ifadeye dönüştürülmeye çalışılırsa
^0, 25 h-x + 2 = 8 2x + 1 & b 25 l = ^2 3 h2x + 1
-x + 2
100

&b 4 l
1 -x + 2
= 2 6x + 3

& c 22 m
1 -x + 2
= 2 6x + 3
& ^2 -2 h-x + 2 = 2 6x + 3 & 2 2x - 4 = 2 6x + 3
& 2x - 4 = 6x + 3
7
& - 7 = 4x, x =- 4 olur.

a = b , m tek ise
2) a m = b m & )
a = ! b , m çift ise
(Üsleri aynı olan iki üslü ifade eşit ise üs tek olduğunda tabanlar, üs çift olduğunda tabanların mutlak
değeri eşittir.)

ÖRNEK 14:

]3x - 1g3 = ]x + 3g3 denkleminde x değerini bulunuz.

ÇÖZÜM:
İki üslü ifade eşit verilmişse öncelikle üssün tek ya da çift olup olmadığına bakılır. Örnekte, üs tek
olduğundan tabanlar eşit olmalıdır.
3x - 1 = x + 3 & 2x = 4
! x = 2 bulunur.

ÖRNEK 15:
]4x - 5g2 = ]x + 1g2 denkleminin çözüm kümesini bulunuz.

ÇÖZÜM:
Üs, çift ve eşit olduğundan tabanların mutlak değerleri eşit olmalıdır.
4x - 5 = x + 1 & 4x - 5 = x + 1 veya 4x - 5 =-]x + 1g
+ 3x = 6 4x - 5 =- x - 1
1 x=2 5x = 4
4
x= 5

Ç = & 5 , 2 0 olur.
4

Matematik 9 | 143
DENKLEMLER VE EŞİTSİZLİKLER

]]Z x = 0 ve a Y
= 0 ise
]]
x
3) a = 1 eşitliği üç farklı durumda sağlanır. Bunlar: [] a = 1 ise
]]
] a =- 1 ve x çift sayı ise
\

ÖRNEK 16:
]x + 1g3x + 7 = 1 denklemini sağlayan x değerini bulunuz.

ÇÖZÜM:

$ ]x + 1g3x + 7 = 1 & 3x + 7 = 0 ise x =- 3


7

$ x + 1 = 1 ise x = 0
$ x + 1 =- 1 ise x =- 2 Bu durumda üs çift olmalıdır.
x =- 2 için 3. ]- 2g + 7 = 1 tek olduğundan çözüm olarak alınmaz.
Ç = & - 3 , 0 0 olur.
7

ÖRNEK 17:
^ x 2 - 3 hx
2
-4
= 1 denklemini sağlayan x değerininin çarpımını bulunuz.

ÇÖZÜM:
^ x 2 - 3 hx
2
-4
=1
$ x 2 - 4 = 0 ise x 2 = 4, x 3, 4 = " 2 tir. x 1, 2 = " 2 değeri için x 2 - 3 ! 0 dir.
$ x 2 - 3 = 1 ise x 2 = 4, x 1, 2 = " 2
$ x 2 - 3 =- 1 ise x 2 = 2, x 5, 6 = " 2 bu durumda üs çift olmalıdır.
x 5, 6 = " 2 için ^" 2 h - 4 =- 2 çift olduğundan çözüm olarak alınır.
2

Ç = " - 2, - 2 , 2 , 2 , bulunur.
x değerlerinin çarpımı ]- 2g . ^- 2 h . ^ 2 h .2 = ]- 4g . ]- 2g = 8 bulunur.

am = bx
4) 3& m x
n = y dir.
an = by

ÖRNEK 18:

3 x = 2 ve 16 y = 27 olduğuna göre x.y çarpımını bulunuz.


ÇÖZÜM:
I. Yol
16 y = 27 & ^2 4 hy = 3 3
& ]2g4y = 3 3 (2 yerine 3 x yazılırsa)
& ^3 x h4y = 3 3
& 3 4xy = 3 3
3
& xy = 4

II. Yol

3x = 2 x
3 = 1 & x.y = 34
2 4y = 3 3 3 4y

144 | Matematik 9
DENKLEMLER VE EŞİTSİZLİKLER

ÖRNEK 19:
2x = m
olduğuna göre 16 x .125 y ifadesinin m ve n türünden değerini bulunuz.
5y = n

ÇÖZÜM:
16 x .125 y = ^2 4 hx . ^5 3 hy
= ^2 4 hx . ^5 3 hy, 2 x = m ve 5 y = n olduğundan
= m 4 .n 3 bulunur.

ALIŞTIRMALAR

^ - x 3 h .x -5 . ]- xg6 . ]- xg3
c
1 7 0, 027 mx
x 2 .x 4
işleminin sonucunu 0, 003 = 243 ise x değerini bulunuz.
bulunuz.

2 5x + 2 - 5x + 3 8 3 x + 2 = 6 x - 1 olduğuna göre 2 x değerini


işleminin sonucunu bulunuz.
5x + 3 + 5x + 2 bulunuz.

4 4 .25 3 - 2 çarpımında oluşan sayının 2x - 1 = m


3 olduğuna göre
3 9
rakamları toplamını bulunuz. 3x + 2 = n
24 x ifadesinin m ve n cinsinden değerini
bulunuz.

4 5 x = 2 olduğuna göre ]25g2x + 1 değerini


bulunuz.

10 ]2x - 1g
x+3
= 1 denklemini sağlayan x
değerlerinin çarpımını bulunuz.

5 15 x = a, 3 x = b olduğuna göre 75 2x in a ve b
türünden değerini bulunuz.

6
Aşağıda verilen denklemlerdeki x değerini 3 x + 1 + 2.3 x + 2 + 5.3 x = 78 denkleminin çözüm
11
bulunuz. kümesini bulunuz.
a) ]3x - 2g3 = ]x + 2g3
b) 2 x + 2 + 2 x + 3 = 48

Matematik 9 | 145
DENKLEMLER VE EŞİTSİZLİKLER

2. Köklü İfadeler İçeren Denklemler


Köklü İfadeler
n ! Z +, n $ 2 olmak üzere x n = a denklemini sağlayan x gerçek sayılarına a sayısının n. dereceden
kökü denir.
1 m
x n = a & x = n a = a n tir. Benzer şekilde x n = a m & x = n a m = a n gösterilir.

• x n = a ifadesinde n tek tam sayı ise 6a ! R için x = n a bir gerçek sayıdır.


• x n = a ifadesinde n çift tam sayı ise a $ 0 için x = n a bir gerçek sayıdır.
3
- 3 , 2 , 5 - 1 , 7 - 2 , 6 13 sayıları birer gerçek sayıdır. Ancak - 2 , 4 - 1 , 12 - 5 sayıları gerçek sayı değildir.

ÖRNEK 1:

3 - x ifadesini gerçek sayı yapan x değerini bulunuz.


ÇÖZÜM:

3 - x ifadesinde kökün derecesi çift (2) olduğundan kök içi sıfırdan büyük olmalıdır. 3 - x $ 0 ise
x # 3 bulunur.

ÖRNEK 2:
4
8 - x + x - 6 ifadesi reel sayı olduğuna göre x yerine yazılabilecek tam sayıların toplamını bulunuz.

ÇÖZÜM:
4
8 - x + x - 6 ifadesinde
4
8 - x kökün derecesi çift (4) olduğundan kök içi pozitif olmalıdır.
Bu durumda 8 - x $ 0 & x # 8 olur. .......(1)
x - 6 da kökün derecesi çift (2) olduğundan kök içi pozitif olmalıdır.
Bu durumda x - 6 $ 0 & x $ 6 olur. .......(2)
(1) ve (2) den x # 8 ve x $ 6 & 6 # x # 8 bulunur. O hâlde x in alabileceği tam sayı değerleri 6, 7, 8 olup
toplamları 6 + 7 + 8 = 21 bulunur.

a , n tek ise
n
an = *
a , n çift ise

ÖRNEK 3:

4 ^- 3h4 + 2 ]- 1g2 - 3 ]- 2g3 işleminin sonucunu bulunuz.

ÇÖZÜM:

4 ^- 3h4 + 2 ]- 1g2 - 3 ]- 2g3 = - 3 + - 1 - ]- 2g



= 3 + 1 + 2 = 6 bulunur.

146 | Matematik 9
DENKLEMLER VE EŞİTSİZLİKLER

ÖRNEK 4:

a 2 0 2 b olmak üzere
6 ^b - ah6 + 5 a 5 - 4 ^a - bh4 işleminin sonucunu bulunuz.

ÇÖZÜM:

6 ^b - ah6 + 5 a 5 - 4 ^a - bh4 = b - a + a - a - b =-^b - ah + a - ^a - bh


. .
_-i _+i
=- b + a + a - a + b
= a bulunur.

ÖRNEK 5:
5
3 x - 2 = 3 27 x - 1 denkleminin çözüm kümesini bulunuz.

ÇÖZÜM:

5 3 x - 2 = 3 27 x - 1 eşitliği köklü ifade içeren bir denklemdir. Bu ifade, kökten kurtarılıp rasyonel üslü
ifadeye dönüştürülürse
x-2 x-1
5
3 x - 2 = 3 27 x - 1 & 3 5 = 27 3

= ^3 3h
x-2 x-1
&3 5 3
x-2 x-1
& 3 5 = 3 3. 3
x-2
& 5 = x-1
& x - 2 = 5x - 5
& 3 = 4x
& x = 4 bulunur. Ç = & 4 0 olur.
3 3

1
n a = a n eşitliği, köklü sayıların aslında üssü rasyonel olan birer üslü sayı olduğunu gösterir.
Bunun sonucu olarak üslü sayılarla ilgili özellikler kullanılarak köklü sayılara ilişkin aşağıdaki özellikler
yazılabilir:
1. ^n a h = _ a n i = a n .n = a olur.
n 1 n 1

^ 2 h = 2, ^5 - 2 h =- 2, ^3 5 h = 5
2 5 3

2. x ! R + olmak üzere x. n a = n a.x n

3 $ 2 = 2 $ 3 2 = 2 $ 9 = 18 ve 2 $ 3 5 = 3 5 $ 2 3 = 3 5 $ 8 = 3 40

3. ^n a h = _ a n i = a n = n a m olur.
m 1 m m

1 1 5
2 = 2 2 1 = 2 2 , 3
5 = 3 51 = 5 3 , 4
75 = 7 4
4. x n a + y n a - z n a = ^x + y - z h n a (Köklü sayılarla toplama ve çıkarma)
- 3 2 + 5 2 + 2 2 = ]- 3 + 5 + 2g 2 = 4 2
2 3 5 - 5 3 5 + 6 3 5 = ]2 - 5 + 6g 3 5

Matematik 9 | 147
DENKLEMLER VE EŞİTSİZLİKLER

ÖRNEK 6:

18 + 72 - 98 işleminin sonucunu bulunuz.

ÇÖZÜM:

Köklü ifadelerde toplama ve çıkarma işlemlerinin yapılabilmesi için kök içinin ve derecesinin aynı olması
gerekir. Buna göre sayılar düzenlenirse

18 = 2 $ 3 2 = 3 2

72 = 2 $ 6 2 = 6 2

98 = 2 $ 7 2 = 7 2 olduğundan

18 + 72 - 98 = 3 2 + 6 2 - 7 2
= ]3 + 6 - 7g 2 = 2 2 bulunur.

ÖRNEK 7:
3
54 - 3 16 + 3 128 işleminin sonucunu bulunuz.

ÇÖZÜM:

3
54 = 3 2 $ 3 3 = 3 3 2

3
16 = 3 2 $ 2 3 = 2 3 2

3
128 = 3 2 $ 4 3 = 4 3 2 olduğundan
3
54 - 3 16 + 3 128 = 3 3 2 - 2 3 2 + 4 3 2
= ]3 - 2 + 4g 3 2 = 5 3 2 bulunur.

ÖRNEK 8:

5 27 + 2 48 - 12
işleminin sonucunu bulunuz.
75 - 3

ÇÖZÜM:

5 27 + 2 48 - 12 5 3 $ 9 + 2 3 $ 16 - 3 $ 4 5 3 $ 32 + 2 3 $ 42 - 3 $ 22
= =
75 - 3 3 $ 25 - 1 $ 3 3 $ 52 - 12 $ 3
5 $ 3 3 + 2 $ 4 3 - 2 $ 3 ]15 + 8 - 2g $ 3 21
]5 - 1 g $ 3
= = = 4 bulunur.
5$ 3-1$ 3

148 | Matematik 9
DENKLEMLER VE EŞİTSİZLİKLER

a $ n b = a n $ b n = ]a $ b g n = n a $ b
1 1 1
n
5.
3
2 $ 5 = 10 , 2 $ 3 3 $ 3 4 = 3 2 $ 3 $ 4 = 3 24
1 1
6. n = 1 = b b l = b olur. b Y
n
a an a n n a
=0
b bn
7. Köklü ifadede, kökün içerisindeki ifadenin kuvveti ve kök kuvveti pozitif bir tam sayı ile çarpılıp
bölünebilir.
m
a) n
a m = a n (Üs, r ! Z + sayısı ile genişletilirse)
m. r
= a n.r = n.r a m.r olur. (Kökün derecesini genişletme)
3 2 5 = 3 $ 2 2 5 $ 2 = 6 2 10

b) m, n ! Z, a $ 0, r ! Z +
m
n a m = a n pay ve payda r ye bölünürse
n
m|r r m
= a n|r = a r olur. (Kökün derecesini sadeleştirme)

` r çift ise a r $ 0 olmalıdır. h


n m

8 ^- 3h6 ! 8: 2 ^- 3h6: 2

8 ^- 3h6 ! 4 ^- 3h3 dir. Çünkü ^- 3h3 1 0 tür.

ÖRNEK 9:

3 $ 5 7 işleminin sonucunu bulunuz.

ÇÖZÜM:

3 $ 5 7 köklü sayılarının çarpımını yapabilmek için kök derecelerinin eşit olması gerekir.
^2, 5hekok = 10 olduğundan
3 = 2.5 3 1.5 = 10 3 5 ve 5 7 = 5.2 7 1.2 = 10 7 2 yazılır.
Bu durumda
3 $ 5 7 = 10 3 5 $ 10 7 2
= 10 35 $ 72
= 10 243 $ 49
= 10 11907 bulunur.

ÖRNEK 10:

2 $ 3 3 = 6 x ise x değerini bulunuz.

ÇÖZÜM:
2 $ 3 3 = 6 x & 2.3 2 3 $ 3.2 3 2 = 6 x
6
23 $ 6 32 = 6 x
6
23 $ 32 = 6 x
23 $ 32 = x
x = 8 $ 9 = 72 bulunur.

Matematik 9 | 149
DENKLEMLER VE EŞİTSİZLİKLER

ÖRNEK 11:

3 -1 $3 3 +1
6
işleminin sonucunu bulunuz.
3 -1

ÇÖZÜM:

3-1$3 3+1
6
işleminde köklerin dereceleri 6 da eşitlenirse
3-1

^ 3 - 1 h $ 3. 2 ^ 3 + 1 h
2.3 1 .3 1.2
3-1$3 3+1
^ 3 - 1h
6
= 6
3-1
^ 3 - 1h $ ^ 3 + 1h
6 3 6 2

^ 3 - 1h
= 6

^ 3 - 1h $ ^ 3 + 1h
3 2
6

^ 3 - 1h
=

^ 3 - 1h $ ^ 3 + 1h
6 2 2
=
_^ 3 h2 - 1 2 i
6 2
=
^3 - 1 h2
6
=
= 6 22
=3 2

8. Çarpımları rasyonel olan iki irrasyonel sayıdan her biri, diğerinin eşleniği olarak tanımlanır.
Köklü rasyonel ifadelerde, paydayı kökten kurtarmak için paydadaki sayının eşleniği ile pay ve payda
çarpılır.

Aşağıda bazı köklü sayılar ve eşleniği verilmiştir.


Sayı Eşleniği Sayı $ Eşlenik
^ a + bh $ ^ a - bh = ^ ah - ^ bh = a - b
2 2
a+ b a- b
a a a$ a=a
a +b a -b ^ a + bh $ ^ a - bh = a - b 2
n
xa n
xb n
x a $ n x b = n x a + b = x, ]n = a + bg

ÖRNEK 12:

2 2
- işleminin sonucunu bulunuz.
3-1 3+1

ÇÖZÜM:
Paydayı rasyonel yapmak için kesir paydanın eşleniği ile genişletilirse
2 2 2 $ ^ 3 + 1h 2 $ ^ 3 - 1h
- = 3-1 - 3-1
3-1 3+1
^ 3 + 1h ^ 3 - 1h

2$ 3+2-2$ 3+2
= 2
4
= 2 = 2 bulunur.

150 | Matematik 9
DENKLEMLER VE EŞİTSİZLİKLER

ÖRNEK 13:

1 1 3
- + işleminin sonucunu bulunuz.
2-2 3 2+2 3 2
ÇÖZÜM:
1 1 3 2+2 3 2-2 3 3$ 2
2 - ^2 3 h 2 - ^2 3 h
- + = 2 2 - 2 2 + 2
2-2 3 2+2 3 2
^2 + 2 3 h ^2 - 2 3 h ^ 2h

2+2 3-2+2 3 3$ 2 4 3 3$ 2
= 4-4$3 + 2 =- 8 + 2
3 3$ 2 - 3+3 2
=- 2 + 2 = 2 bulunur.

ÖRNEK 14:
x+1
3x + 3x + 3x
y+1 işleminin sonucunu bulunuz.
5y + 5y + 5y + 5y + 5y
ÇÖZÜM:
x+1
3x + 3x + 3x x+1
3 $ 3x x+1
3x + 1 3
y+1 = y+1 = y+1 = 5
5y + 5y + 5y + 5y + 5y 5 $ 5y 5y + 1

ÖRNEK 15:
2003 $ 2005 + 1 işleminin sonucunu bulunuz.

ÇÖZÜM:

2003 $ 2005 + 1 = ]2004 - 1g $ ]2004 + 1g + 1 = 2004 2 - 1 2 + 1 = 2004 2 = 2004 bulunur.

ÖRNEK 16:

355 $ 390 - 354 $ 391 işleminin sonucunu bulunuz.

ÇÖZÜM:
x = 354 ve y = 390 olsun.
355 $ 390 - 354 $ 391 = ]x + 1g $ y - x $ ^y + 1h = xy + y - xy - x
y - x = 390 - 354 = 36 = 6 bulunur.

x ! 2 y Şeklindeki İfadeler
x + 2 y = a + b olsun. Her iki tarafın karesi alınırsa
x + 2 y = ^ a + b h = a + 2 ab + b = ]a + bg + 2 ab olur. Buradan x = a + b ve y = a $ b bulunur.
2

• ]a + b g + 2 a $ b = a + b
• ]a + b g - 2 a $ b = a - b , ]a 2 b g

Örneğin 9 - 2 20 işleminde 9 = 4 + 5 ve 20 = 4 $ 5 olarak yazılırsa


9 - 2 20 = ]4 + 5g - 2 4 $ 5 = 5 - 2 bulunur.

Matematik 9 | 151
DENKLEMLER VE EŞİTSİZLİKLER

Örneğin, 4 + 12 işleminde formülün kullanılabilmesi için içerideki kökün katsayısı 2 olmalıdır.


4 + 12 = 4 + 3 $ 4 = 4 + 2 2 $ 3 = 4 + 2 3 elde edilir.
4 = 1 + 3 ve 3 = 1.3 olarak yazılırsa ]1 + 3g + 2 1 $ 3 = 3 + 1 = 3 + 1 bulunur.

ÖRNEK 17:

9 + 4 5 işleminin sonucunu bulunuz.

ÇÖZÜM:

9 + 4 5 işleminde kuralı uygulayabilmek için içerideki kökün katsayısı 4 yerine 2 olmalıdır.


9+4 5 = 9+2$2 5
9 + 2 5 $ 2 2 = 9 + 2 $ 5 $ 4 = 9 + 2 20 elde edilir.
9 = 4 + 5 ve 20 = 4 $ 5 olarak yazılırsa
9 + 4 5 = 9 + 2 20 = ]4 + 5g + 2 4 $ 5 = 2 + 5 bulunur.

ÖRNEK 18:

4 - 7 - 4 + 7 işleminin sonucunu bulunuz.

ÇÖZÜM:
I. Yol
4 - 7 - 4 + 7 işleminde kuralı uygulayabilmek için içerideki köklerin katsayısı 2 yapılmalı. Bunun
için köklerin içindeki ifadeler 2 ile çarpılıp bölündüğünde
2 ^4 - 7 h 2 ^4 + 7 h 8-2 7 8+2 7 8-2 7 8+2 7
2$ - 2$ = 2 - 2 = - ifadesinde
2 2
8 = 1 + 7 ve 7 = 1.7 olarak yazılırsa

]1 + 7g - 2 1.7 ]1 + 7g + 2 1.7 7- 1 7+ 1 7 - 1 - ^ 7 + 1h
- = - =
2 2 2 2 2
-2
= (Pay ve payda 2 nin eşleniği olan 2 ile genişletilirse)
2
-2 $ 2 -2 $ 2
= = 2 =- 2 bulunur.
2$ 2
II. Yol
4 - 7 - 4 + 7 = x ]x 1 0g olsun. Her iki tarafın karesi alınırsa
_ 4 - 7 - 4 + 7 i = x 2 ise 4 - 7 - 2 $ _ 4 - 7 $ 4 + 7 i + 4 + 7 = x 2
2

= 8 - 2 $ _ 4 - 7 $ 4 + 7 i = x2
= 8 - 2 ^ 16 - 7 h = x 2
= 8 - 2 $ 3 = x2
x 1 0 olduğundan x =- 2 olur.

a b c
• x = a.b.c x
a
• x $ b y $ c z = a.b.c x b.c $ y c $ z

152 | Matematik 9
DENKLEMLER VE EŞİTSİZLİKLER

ÖRNEK 19:

3 işleminin sonucunu bulunuz.

ÇÖZÜM:
1
3 = 2.2.2.2.2.2 3 = 64 3 = 3 64 bulunur.

ÖRNEK 20:

3 3 3 3 işleminin sonucunu bulunuz.

ÇÖZÜM:
15
3 3 3 3 = 2.2.2.2 3 2.2.2 $ 3 2.2 $ 3 2 $ 3 = 16 3 8 $ 3 4 $ 3 2 $ 3 = 16 3 15 = 3 16 bulunur.

ÖRNEK 21:

3
2 4 2 5 32 işleminin sonucunu bulunuz.

ÇÖZÜM:

^2 4 h5 $ 2 5 $ 2 5 = 60 2 30 = 2 bulunur.
3 3$4$5
2 4 2 5 32 =

x $ n x $ n x... = n - 1 x tir. _ 4 27 $ 4 27 $ 4 27... = 4 - 1 27 = 3 3 3 = 3 i


n

x | n x | n x | ... = n + 1 x tir. _ 3 16 | 3 16 | 3 16 | ... = 3 + 1 16 = 4 2 4 = 2 i


n

ÖRNEK 22:

3 5
A= 16 $ 3 16 $ 3 16... olduğuna göre A 3: 5 A 3 5 A 3 ... ifadesinin sonucunu bulunuz.

ÇÖZÜM:
3
1.Yol: Yukarıdaki kurala göre A = 16 $ 3 16 $ 3 16... = 3 - 1 16 = 16 = 4 olur. A = 4 değeri
5
A 3: 5 A 3: 5 A 3: ... ifadesinde yerine yazılırsa

^ 2 2 h3 = 6 2 6 = 2
5
4 3: 5 4 3: 5 4 3: ... =
5+1

3
2. Yol: A = 16 $ 3 16 $ 3 16... ifadesi sonsuza kadar devam ettiğinden, tekrarın başladığı kısım da A olur.
Bu durumda ifade A = 3 16 $ A olur. Her iki tarafın 3. dereceden kuvveti alınırsa
A 3 = ^3 16 $ A h
3
5
4 3: 5 4 3: 5 4 3: ... = x
A 3 = 16 $ A 5 3
4 :x = x
A 2 = 16 & A = 4 bulunur.
^ 5 4 3: x h = x 5
5

43 5
^ 2 h3 6
x =x & 2 =x
! 26 = x6
! x = 2 bulunur.

Matematik 9 | 153
DENKLEMLER VE EŞİTSİZLİKLER

ÖRNEK 23:
3
3$ 9 $ 3 $ 3 9... ifadesinin değerini bulunuz.

ÇÖZÜM:
3 2$3
3$ 9 $ 3 $ 3 9... = 3 2 $ 3 3 $ 2 $ 3 3 2 $ 3 3 $ 2 $ 3 3 2 $ 3 3 ...
6
= 3 5 $ 6 3 5 $ 6 3 5 ...
= 6 - 1 35
= 5 35
=5

ALIŞTIRMALAR

1
6
3 - x + x - 1 ifadesini reel sayı yapan x 5
2 2
+
tam sayılarının toplamını bulunuz. 5-3 2 5+3 2
işleminin sonucunu bulunuz.

2
3
]- 3g3 - 4 ]- 4g4 + 5 ]- 1g5 6
Aşağıdaki işlemlerin sonuçlarını bulunuz.
işleminin sonucunu bulunuz. a) 3$3 5
b) 4
5+1$3 5-1
12
5-1

c) 2016 $ 2018 + 1
ç) 3+ 8
3 a 1 0 1 b olmak üzere
4
]a - bg4 - 3 a 3 - 5 ]a - bg5
işleminin sonucunu bulunuz.

4
7 27 $ 4 27 $ 4 27... + 125: 125: 125...
ifadesinin değerini bulunuz.

4
3
2 2x - 1 = 3 16 x + 1 denkleminin çözüm
kümesini bulunuz.
8 2+ 3 + 2- 3
işleminin sonucunu bulunuz.

154 | Matematik 9
DENKLEMLER VE EŞİTSİZLİKLER

9.3.4. DENKLEM VE EŞİTSİZLİKLERLE İLGİLİ UYGULAMALAR

Etkinlik

3
1. Dünyanın yüz ölçümü 510 milyon km 2 dir. Dünyanın yaklaşık 4 ü suyla kaplı olduğuna göre kara
2
ve suyla kaplı alanlar kaçar km dir?
2. 1/30 000 000 ölçekli bir haritada,
• Türkiye’nin doğusu ile batısı arasındaki kuş uçuşu mesafe 1500 km ise bu mesafe haritada kaç
cm ile gösterilir?
• Türkiye’nin iz düşüm alanı yaklaşık 780 000 km 2 ise Türkiye haritada ne kadar yer kaplar?
• Van Gölü’nün yüz ölçümü yaklaşık 3700 km 2 ise Van Gölü haritada ne kadar yer kaplar?
• Harita üzerinde Asya Kıtası’nın kapladığı alan 1486 cm 2 olduğuna göre Asya Kıtası’nın gerçek
yüz ölçümü kaçtır?
3. Haritada 12 cm ile gösterilen iki nokta arasındaki uzaklığın gerçekte 180 km olduğu bilindiğine
göre bu haritanın ölçeği nedir?
4. Siz de yaşadığınız ilin yüz ölçümünün farklı ölçekli herhangi bir harita kullanarak haritada ne kadar
yer kapladığını hesaplayınız.
5. Yukarıdaki sorulara cevap vermek için kullandığınız yöntemi arkadaşlarınızla tartışınız.

Matematik 9 | 155
DENKLEMLER VE EŞİTSİZLİKLER

1. Oran ve Orantı
Okul hayatınız dışında, tüm hayatınız boyunca sizlere yardımcı olacak
olan matematik konularından birisi de oran orantı konusudur. Günlük
yaşamda, birçok alanda pratik matematik hesapları yapabilmek için bu
konudan yararlanılmaktadır.
Örneğin halk dilinde bir buçuk parmak olarak ifade edilen demir
borular üzerindeki büyüklükleri belirtmek için kullanılan oranlar, yiyecek
içecek ya da ilaçların içeriğini belirten oranlar, Türkiye’deki Genel Ağ
kullanımı veya genç yaşlı nüfus oranları gibi karşılaşılan birçok durum bu
konuyla ilişkilidir.
O hâlde sıklıkla karşılaşılan oran orantı konusunun matematikteki
anlamı nedir incelemekte fayda var.

Oran
Aynı birimdeki iki çokluğun birbirine bölünmesine oran denir.
a
En az biri sıfırdan farklı olan a ve b reel sayıları verildiğinde a nın b ye oranı a|b veya b şeklinde
gösterilir.
• Oranın birimi yoktur.
• Kesirlerde olduğu gibi, verilen oranın payı ve paydası sıfırdan farklı bir sayı ile genişletilip
sadeleştirilebilir.

ÖRNEK 1:
Ada’nın boyu 160 cm, Atlas’ın boyu 80 cm ise Atlas’ın boyunun
Ada’nın boyuna oranını bulunuz.

ÇÖZÜM: 160 cm

Verilen bilgilere göre Atlas’ın boyunun Ada’nın boyuna oranı:


80 cm
80 1
160 = 2 olur.

ÖRNEK 2:
Bir tren vagonunda 24 bay, 28 bayan yolcu bulunmaktadır. İlk durakta
bu vagona 4 evli çift binmiş, 6 bayan ve 8 bay vagondan inmiştir. Buna
göre vagondaki bayan sayısının bay sayısına oranını bulunuz.

ÇÖZÜM:
4 evli çiftin binmesiyle bay ve bayanların sayısı dörder kişi artar. Vagondaki bay ve bayan sayısı: 24 + 4 = 28
bay ve 28 + 4 = 32 bayan olur.
İnen yolculardan sonra:
Bayların sayısı: 28 - 8 = 20
Bayanların sayısı: 32 - 6 = 26 olur.
26 13
Bu durumda, bayanların sayısının bayların sayısına oranı: 20 = 10 olarak bulunur.

156 | Matematik 9
DENKLEMLER VE EŞİTSİZLİKLER

ÖRNEK 3:
Bilal Bey’in kasım ayı doğal gaz faturasının ilk endeksi 260 m 3 , son
endeksi 332 m 3 tür. Bilal Bey’in 86,4 TL faturası geldiğine göre 1 m 3
doğal gaza kaç TL ödediğini bulunuz.

ÇÖZÜM:
Kasım ayında 332 - 260 = 72 m 3 doğal gaz kullanımı gerçekleşmiştir.
86,4 TL fatura 72 m 3 doğal gaz kullanımına eşittir. Bu durumda 1 m 3
86, 4
doğal gaz kullanımına denk gelen ücret: 72 = 1, 2 TL olur.

Orantı
a c a c
İki veya daha fazla oranın eşitliğine orantı denir. b = k ve d = k ise orantı, b = d = k şeklinde
gösterilir.
a c
b = d eşitliği a | c = b | d biçiminde de gösterilir. Eşitlikteki konumlarına göre a ve d değerlerine
dışlar, b ve c değerlerine içler adı verilir.
Benzer şekilde a = c = e = k & a | c | e = b | d | f olur. k, orantı sabitidir.
b d f

k ! R - ! 0 + olmak üzere
a b
• 2 = 3 veya 3a = 2b olduğunda a = 2k, b = 3k dir.
• a | b | c = 2 | 3 | 5 + a = 2k, b = 3k, c = 5k dir.

ÖRNEK 4:
x 1 ax + y
y = 3 ve y - x = 2 olduğuna göre a değerini bulunuz.
ÇÖZÜM:
x = k ve y = 3k olduğundan
ax + y a $ k + 3k k $ ]a + 3 g a + 3
y - x = 3k - k = 2k = 2 =2
Buradan a + 3 = 4 ve a = 1 bulunur.

ÖRNEK 5:
x | y | z = 5 | 3 | 2 ve x - y + z = 16 olduğuna göre x değerini bulunuz.

ÇÖZÜM:
x = 5k, y = 3k, z = 2k olduğundan 5k - 3k + 2k = 16 & 4k = 16 & k = 4
Buradan x = 5k = 5 $ 4 = 20 bulunur.

ÖRNEK 6:
a-2 b+1
3 = 2 ve 3a - b = 21 olduğuna göre a değerini bulunuz.

ÇÖZÜM:
a-2 b+1
3 = 2 = k & a = 3k + 2 ve b = 2k - 1 olur.
9k + 6 - 2k + 1 = 21
7k = 14 ise k = 2 bulunur. O hâlde a = 3 $ 2 + 2 = 8 tir.

Matematik 9 | 157
DENKLEMLER VE EŞİTSİZLİKLER

Orantının Özellikleri
1. Orantıda içler çarpımı, dışlar çarpımına eşittir.
a c
b = d & a $ d = b $ c tir.

ÖRNEK 7:
a+b 2 a
a - b = 3 olduğuna göre b oranının bulunuz.
ÇÖZÜM:

a - b = 3 & 2 ]a - b g = 3 ]a + b g
a+b 2
2a - 2b = 3a + 3b
- 3b - 2b = 3a - 2a
-5 b a a
- 5b = a & b = b olduğundan b =- 5 bulunur.

2. Bir orantıda içler veya dışlar kendi arasında yer değiştirebilir. Bu durumda oranlar değişse bile orantının
eşitliği değişmez.
a c a b d c
b = d & c = d veya b = a olabilir.

3. Paylar ve paydalar kendi aralarında toplanır ya da çıkarılırsa orantı sabiti değişmez.


a c a+c a-c
b = d = k & b + d = b - d = k olur.

ÖRNEK 8:
2a + b 2b + c a + 2c a
b - c = a - c = 3c olduğuna göre c oranını bulunuz.
ÇÖZÜM:
Özellik 3 kullanıldığında
2a + b 2b + c a + 2c
b - c = a - c = 3c = k
2a + b + 2b + c + a + 2c 3a + 3b + 3c 3 ]a + b + cg
b - c + a - c + 3c = a + b + c = a + b + c = k & k = 3 olur.

a + 2c a 7c
Buradan 3c = 3 & a + 2c = 9c & a = 7c & c = c = 7 bulunur.

4. m ve n sıfırdan farklı birer reel sayı olmak üzere


a c m$a n$c m$a"n$c
i) b = d = k & m $ b = n $ d = k ve m $ b " n $ d = k olur.

ÖRNEK 9:
a c e 1
b = d = f = 3 olmak üzere 2a - c + 3e = 14 ve - d + 3f =- 24 olduğuna göre b değerini bulunuz.
ÇÖZÜM:
a c e 1
b = d = f = 3
^2h ] -1 g ]3 g

Özellik 4-i yardımıyla


2a - c + 3e 1 14 1
2b - d + 3f = 3 & 2b - 24 = 3 & 42 = 2b - 24
66 = 2b & b = 33 bulunur.

158 | Matematik 9
DENKLEMLER VE EŞİTSİZLİKLER

a c a c m $ a2 " n $ c2
ii) b = d = k & b $ d = k 2 ve = k 2 dir.
m $ b2 " n $ d2
a c
5. a, b, c sayılarının dördüncü orantılısı x ise b = x olur.
(a: birinci orantılı sayı, b: ikinci orantılı sayı, c: üçüncü orantılı sayıdır.)

ÖRNEK 10:
4, 10 ve 16 sayıları ile dördüncü orantılı olan sayıyı bulunuz.

ÇÖZÜM:
4 16 160
10 = x ise 4x = 160 bulunur. Buradan x = 4 = 40 olur.

Orantı Çeşitleri
a) Doğru Orantı

a ve b çokluklarından a artarken b de aynı oranda artıyorsa veya a b


azalırken b de aynı oranda azalıyorsa a ile b doğru orantılı denir. a = kb
a ile b doğru orantılı ise b = k veya a = kb dir. ^k ! R + h
a

a, b ve c sayıları sırasıyla x, y ve z sayıları ile doğru orantılı ise


a b c
x = y = z = k şeklinde yazılır.

ÖRNEK 11:
a ile b sayıları doğru orantılıdır. a = 6 iken b = 4 ise b = 24 iken a nın değerini bulunuz.

ÇÖZÜM:
a
a ile b sayıları doğru orantılı olduğundan b = k olmalıdır.
6 3
4 = k &k = 2
a 3
24 = 2 & 2a = 72 & a = 36 olur.

ÖRNEK 12:
18, 20 ve 22 yaşlarındaki üç kardeş kestane üreticiliği yapmaktadır.
180 kg kestaneyi yaşlarıyla doğru orantılı olarak paylaştıklarında 20
yaşındaki kardeşin kaç kg kestane alacağını hesaplayınız.

ÇÖZÜM:
x y z
18 = 20 = 22 = k & x = 18k, y = 20k, z = 22k olur.
x + y + z = 180 & 18k + 20k + 22k = 180 & 60k = 180 & k = 3
bulunur.
Buradan y = 20 k = 20 $ 3 = 60 kg kestane alır.

Matematik 9 | 159
DENKLEMLER VE EŞİTSİZLİKLER

ÖRNEK 13:
Çarpımları ile toplamlarının 4 eksiği doğru orantılı olan iki sayıdan birincisi 4 iken ikincisi 12 dir. Buna göre
birinci 15 iken ikincinin kaç olduğunu bulunuz.

ÇÖZÜM:
Birinci sayı x, ikinci sayı y olsun. Bu durumda
x$y 4 $ 12
x + y - 4 = k & 4 + 12 - 4 = k & k = 4 olur.
15 $ y 15 $ y
15 + y - 4 = 4 & 11 + y = 4 & 15y = 44 + 4y
& 11y = 44
& y = 4 bulunur.

ÖRNEK 14:
Ahmet Bey’in aylık mantar üretimine ait bilgiler aşağıdaki grafikte
verilmiştir.
Adet

3600

60
Ay
4 6

İşe ilk başladığında 60 adet mantar almıştır. 6. ayın sonunda 3600 adet üretim yaptığına göre ilk 4 ayda
yapmış olduğu üretim miktarını bulunuz.

ÇÖZÜM:
6. ve 4. ayın üretim miktarları oranlandığında
3600 - 60 6 3540 3
m - 60 = 4 & m - 60 = 2 & m - 60 = 2360 olur.

ÖRNEK 15:
8 inç ekrana sahip olan bir tablet ekranının kaç cm ye eşit olduğunu
bulunuz. ]1 inç = 2, 54 cmh

ÇÖZÜM:
1 inç 2,54 cm

8 inç x cm

D.O
x = 8 $ 2, 54 = 20, 32 cm olur.

160 | Matematik 9
DENKLEMLER VE EŞİTSİZLİKLER

b) Ters Orantı

a ve b çokluklarından, a artarken b aynı oranda azalıyor veya b


a azalırken b aynı oranda artıyorsa a ve b ters orantılıdır denir
a ile b ters orantılı ise b = a veya a $ b = k tir. ^k ! R + h
k
k
b= a

a, b ve c sayıları sırasıyla x, y ve z sayıları ile ters orantılı ise


a b c
a.x = b.y = c.z = k veya 1 = 1 = 1 = k şeklinde yazılır.
x y z

ÖRNEK 16:
y
2x + 1 sayısı, 3 sayısı ile ters orantılıdır. x = 2 iken y = 6 oluyorsa x = 4 iken y sayısını bulunuz.

ÇÖZÜM:
]2x + 1g $ 3 = k olduğundan x = 2 ve y = 6 yazıldığında k = ]2 $ 2 + 1g $ 6
y
3 = 5 $ 2 = 10 bulunur.
x = 4 için ]2 $ 4 + 1g $ 3 = 10 & 9 $ 3 = 10
y y

& 3y = 10
10
& y = 3 olur.

ÖRNEK 17:
111 tane bilye; 4, 5 ve 6 yaşlarındaki üç kardeşe yaşlarıyla ters orantılı
olarak paylaştırılmak isteniyor. Her bir kardeşe kaçar tane bilye
düştüğünü bulunuz.

ÇÖZÜM:
k k k
4a = 5b = 6c = k & a = 4 , b = 5 , c = 6 olur.
k k k 15k + 12k + 10k
4 + 5 + 6 = 111 & 60 = 111
37k
& 60 = 111
111 $ 60
& k = 37 = 180
180 180 180
a = 4 = 45, b = 5 = 36, c = 6 = 30 tane bilye alırlar.

ÖRNEK 18:
1 1 1
a $ x = b $ y = c $ z = 4 ve x + y + z = 12 olduğuna göre a + b + c değerini bulunuz.

ÇÖZÜM:
a b c a+b+c a+b+c
1 = 1 = 1 =4 & 1 1 1 =4 & 12 = 4 & a + b + c = 48 olur.
x y z + +
x y z

Matematik 9 | 161
DENKLEMLER VE EŞİTSİZLİKLER

ÖRNEK 19:
Birbirini çeviren iki çarkın birinde 24, diğerinde 36 diş vardır. Küçük
çark 9 tur attığında büyük çarkın kaç tur atacağını bulunuz.

ÇÖZÜM:
Dişli sayısı çok olan çark bir tur atarken dişli sayısı az olan çark daha
fazla tur atar. Bu sebeple iki çarkın attıkları tur sayıları arasında ters orantı vardır.
24 diş 9 tur
36 diş x tur
T.O
24 $ 9 = 36 $ x
24 $ 9
x = 36 = 6 tur atar.

ÖRNEK 20:
Aşağıda ters orantılı iki çokluğun grafiği verilmiştir.
y
18

x
3 9

Grafiğe göre a değerini bulunuz.

ÇÖZÜM:
Grafiğe bakıldığında verilenler arasında ters orantı olduğu görülüyor. Buna göre
3 $ 18
3 $ 18 = 9 $ a & a = 9 = 6 olur.

c) Bileşik Orantı

İki veya daha fazla oran bulunduran orantılara bileşik orantı denir.
a$c
a sayısı; b ile doğru, c ile ters orantılı ise bu durum b = k şeklinde gösterilir.

ÖRNEK 21:
x - 1 sayısı; y + 1 sayısı ile ters, z 3 sayısı ile doğru orantılıdır. x = 9, y = 3 iken z = 2 olduğuna göre
x = 5, z = 3 2 iken y değerinin kaç olduğunu bulunuz.

ÇÖZÜM:
]x - 1g^y + 1h ]9 - 1g]3 + 1g 8 $ 4
3 =k & = 8 = 4 = k bulunur.
z 23
]5 - 1g^y + 1h 4 $ ^y + 1h
=4 & = 4 & y + 1 = 2 & y = 1 olur.
^3 2 h 2
3

162 | Matematik 9
DENKLEMLER VE EŞİTSİZLİKLER

ÖRNEK 22:
a sayısı; b ile doğru, c ile ters orantılıdır. a sayısı 2 kat artırılıp b sayısı üçte iki azaltılırsa orantı sabitinin
değişmemesi için c sayısındaki değişimi bulunuz.

ÇÖZÜM:
a$c
b = k dir. 2b b
a sayısı iki kat artırılırsa 2a + a = 3a olur. b sayısı üçte iki azaltılırsa b - 3 = 3 olur.
3a $ x 9a $ x 9a $ x a$c c
c sayısının değişimine x denirse b = k & b = k & b = b & 9x = c & x = 9 bulunur. c
sayısı, dokuzda birine düşer. 3

ÖRNEK 23:
Eşit kapasitede çalışan 4 işçi, 8 günde 48 m 2 halı dokuyabildiğine göre aynı nitelikteki 12 işçinin 90 m 2
halıyı kaç günde dokuyacağını bulunuz.

ÇÖZÜM:
I. Yol _b
2
bb
4 işçi 8 gün 48 m bb
b
`b 12 $ x $ 48 = 4 $ 8 $ 90 & x = 5 günde dokur.
12 işçi x gün 90 m 2 bb
bb
T.O D.O b
a
II. Yol
Birinci iş Birinci iş ile ilgili verilenlerin çarpımı
= & 48 = 4 $ 8 & x = 5 günbulunur.
90 12 $ x
İkinci iş İkinci iş ile ilgili verilenlerin çarpımı

Altın Oran Eski Mısırlılar ve Yunanlılar tarafından keşfedilen, mimaride ve sanatta


kullanılmış olan altın oran; bir bütünün parçaları arasında gözlemlenen,
uyum açısından en yetkin boyutları veren sayısal bir oran bağıntısıdır.
İtalyan Matematikçi Leonardo Fibonacci [L. Fibonasi (1170-1250)]
tarafından oluşturulan Fibonacci sayıları arasındaki oran da altın orana
eşittir. Fibonacci sayıları, her sayının kendinden öncekiyle toplanması
sonucu oluşan bir sayı dizisidir. Bu sayı dizisinin bazı elemanları şunlardır:
1, 1, 2, 3, 5, 8, 13, 21, 34, 55, 89, 144, 233, 377, … Bu dizide bir sayı
kendisinden önceki sayıya bölündüğünde altın orana gittikçe yaklaşan
bir dizi elde edilir. Dizide 13. sırada yer alan sayıdan (233) itibaren bu
sayı sabitlenir ve 1,618 olarak hesaplanır.
Resim 9.3.4.1: Altın oran Evrendeki muhteşem düzenle bire bir örtüşen bu sayıları keşfettiği
için altın orana da Fibonacci’nin ilk iki harfi olan Fi ]Ugsayısı denilmiştir.
“ ^U = 1, 618033988749894...h ”
Altın oranla birçok alanda karşılaşılmaktadır.
Örneğin Edirne Selimiye Camisi'nin minarelerinde altın oran
kullanılmıştır. Dirseğinizin üstünde kalan bölümünün altında kalan
bölüme oranı, parmaklarınızın üst boğumunun alt boğumuna oranı,
Mısır piramitlerindeki her bir piramidin tabanının yüksekliğine oranı vb.
Resim 9.3.4.2: Edirne Selimiye Camisi altın orana örnek olarak gösterilebilir.

Matematik 9 | 163
DENKLEMLER VE EŞİTSİZLİKLER

ALIŞTIRMALAR

1
Dorukan 1996 yılında doğmuştur. 2016 yılında 7 a, b ve c sayıları sırasıyla 3, 4 ve ]- 2g sayıları
Dorukan’ın yaşının Alperen’in yaşına oranı 5 ile ters orantılıdır. Bu üç sayının toplamı 4
3 c
olacağına göre Alperen’in hangi yılda olduğuna göre a - b - 2 değerini
doğduğunu bulunuz. hesaplayınız.

2 a 1 ise a 2 + b 2 ifadesinin eşitini bulunuz.


b=2 2ab
8
Saatte 50 lt su akıtan eşit kapasiteli 4 tane
musluk, bir havuzu 12 saatte doldurmaktadır.
Her birinin kapasitesi saatte 80 lt ye çıkartılırsa
havuzu 5 saatte doldurması için kaç musluğun
ilave edilmesi gerekir?

x y z
3 4 = 5 = 7 ve x $ y $ z = 1680 olduğuna göre z
değerini bulunuz.

9
x sayısı y sayısı ile doğru, z sayısı ile ters
orantılıdır. x = 16, y = 4 iken z = 3 olduğuna
göre x = 4, y = 3 iken z değerinin kaç olduğunu
bulunuz.
4 3, 3 - 2 ve 3 + 2 sayıları ile dördüncü
orantılı olan sayıyı bulunuz.

10
Bir A şehrindeki belediye otobüsleri müşterinin
gittiği mesafeye göre ücret almaktadır. Şöyle
ki müşteri otobüse bindiğinde otobüs kartını
x + 1 sayısı, 3y ile doğru orantılıdır. x = 9 iken okutmaktadır. İneceği durağa geldiğinde
5
y = 2 ise x = 4 iken y değeri kaçtır? otobüsten inerken kartını tekrar okutarak
ücretini ödemektedir. Herhangi iki durak arası
en az 500 m, en fazla 1000 m dir.
A şehrinde yaşayan Mehmet, Öznur ve Arda
bir otobüse aynı duraktan biniyorlar. Bindikten
sonraki ilk durakta Arda inerek 0,80 lira ödeme
yapıyor. 3. durakta Öznur iniyor ve 3 lira ödeme
yapıyor. Mehmet ise 4. durakta inerek 4,40 lira
6
Bir üçgenin iç açıları sırasıyla 3, 4 ve 5 sayıları ödüyor.
ile doğru orantılı ise küçük dış açının büyük dış Buna göre
açıya oranını bulunuz.
a) Mehmet otobüs ile en az kaç km gitmiştir?
b) Bu otobüsün güzergâhı 10 km ise bu
güzergâhta en az kaç durak vardır?

164 | Matematik 9
DENKLEMLER VE EŞİTSİZLİKLER

2. Problemler
Bir problemi çözebilmek için sözel olarak belirtilen ifadeler matematiksel değişkenlere dönüştürülerek bir
denklem kurulmalıdır. Denklemin çözümü problemin çözümünü verir.
Problemler de kendi içerisinde sayı ve kesir, yaş, yüzde, karışım, hareket, işçi ve havuz gibi alt başlıklara
ayrılır.
Problemler çözülürken genellikle şu sıra takip edilir:
a) Problemde kullanılan veri veya veriler belirlenir.
b) Problemde istenen veri veya veriler belirlenir.
c) İstenen veriye uygun bir değişken atanır.
ç) Verilere göre denklem veya eşitsizlik yazılır.
d) Yazılan denklem veya eşitsizlik çözülür.

Sayı ve Kesir Problemleri


Aşağıdaki ifadeleri inceleyiniz.

a) Bir sayının 2 katının 5 fazlası 2x + 5

b) Bir sayının 3 eksiğinin 4 katı ]x - 3 g $ 4


3x
c) Bir sayının beşte üçü 5

ç) Bir sayının 2 katının küpünün 3 fazlasının yarısı ]2xg3 + 3


2
d) Bir sayının 4 katının 3 eksiğinin karekökü 4x - 3
x
e) Bir sayının küpünün 5 fazlası kendisinin yarısına eşitse x3 + 5 = 2

f) Ardışık üç çift sayı 2x, 2x + 2, 2x + 4

ÖRNEK 1:
Toplamları 60 olan iki sayıdan birisi, diğerinin 3 katının 4 eksiğine eşitse bu sayılardan küçük olanını bulunuz.

ÇÖZÜM:
Küçük sayıya x denirse büyük sayı 3x - 4 olur.
x + 3x - 4 = 60 & 4x = 64 & x = 16 olur.

ÖRNEK 2:
A, B ve C şehirleri arasındaki uzaklık ilişkisi aşağıda verilmiştir. Buna göre x in değer aralığını bulunuz.
A B C

4x - 36
3x + 20
ÇÖZÜM:
0 1 AB 1 AC olduğundan 0 1 4x - 36 1 3x + 20 olur.
00 1 4x - 36 4x - 36 1 3x + 20 _bb
bb
36 1 4x x 1 56f ^2h b` ^1 h ve ^2h den 9 1 x 1 56 bulunur.
bb
09 1 xf ^1h b
a

Matematik 9 | 165
DENKLEMLER VE EŞİTSİZLİKLER

ÖRNEK 3:
1
Bir demir çubuğun 9 u kesildiğinde orta noktası 4 cm kaymaktadır. Buna göre çubuğun kesilmeden önceki
boyunu hesaplayınız.

ÇÖZÜM:
A
Çubuk bir ucundan A cm kesilirse orta nokta 2 cm kayar.
x
Çubuğun uzunluğu x cm olsun. Bu durumda kesilen kısım 9 olur.
x 1 x
9 $ 2 = 4 & 18 = 4 & x = 18 $ 4 = 72 cm olur.

ÖRNEK 4:
Tiyatro bileti gişesindeki sırada Dila baştan 10, Kerem ise sondan 15.
sıradadır. Kerem ile Dila arasında 5 kişi olduğuna göre
a) Bu sırada en az kaç kişi,
b) Bu sırada en çok kaç kişi,
c) Kerem’in Dila’dan önde olması şartıyla sırada kaç kişi olduğunu
bulunuz.
ÇÖZÜM:
15
Kerem’in gişeye yakın olduğu durumda kişi sayısı
a) K D en az olacağından 10 + 15 - 5 = 20 olur.
Kerem ile Dila iki kez sayıldığından kişi sayısı 20 - 2 = 18 olarak
10 bulunur.
15

b) D K Dila’nın gişeye yakın olduğu durumda kişi sayısı


10
en çok olacağından 10 + 5 + 15 = 30 olur.
15
Bir kişi bir sırada baştan m, sondan n. ise sıradaki kişi sayısı
c) K D ]m + n - 1g dir. Kerem sondan 15. kişidir. Baştan ise yandaki
şekilde görüldüğü gibi 4. kişi olduğundan 4 + 15 - 1 = 18 kişi
vardır.
4 5 6 7 8 9 10

ÖRNEK 5:
Bir gösteri grubu 5 ileri, 2 geri hareket etmektedir. Bu gösteri grubunun 111 adım attığında kaç adım
ilerlemiş olduğunu bulunuz.

ÇÖZÜM:
5 + 2 = 7 adım atıldığında 3 adım ilerleme oluyor.

111 7 15 defa 7 adım atıldığında 15 $ 3 = 45 adım ilerleme gerçekleşmiş olur. Kalan 6 adımda 5 ileri,
105 15 1 geri adım atılacağı için 45 + 5 - 1 = 49 adım ilerlemiş olur.
6
ÖRNEK 6:
Bir poliklinikte bir doktora 60 hasta, bir hemşireye 30 hasta düşmektedir. Bu poliklinikteki doktor, hemşire
ve hasta sayılarının toplamı 378 olduğuna göre doktor sayısını bulunuz.
ÇÖZÜM:
Doktor sayısı : x _b x + 60x + 2x = 318
bb
Hasta sayısı : 60x b 63x = 378
`b
60x bb
Hemşire sayısı : 30 = 2x b x = 6 doktor bulunur.
a
166 | Matematik 9
DENKLEMLER VE EŞİTSİZLİKLER

ÖRNEK 7:
2
Belirli bir yükseklikten bırakılan bir top yere vuruşundan sonra bir önceki düşüş yüksekliğinin 3 ü kadar
yükselmektedir. Top yere üçüncü vuruşundan sonra 24 cm yükseldiğine göre topun başlangıçta kaç cm den
bırakıldığını bulunuz.
ÇÖZÜM:

_b
1. bb
bb
bb
bb
2x 2. bb
3 bb 8x 27 $ 24
x `b 27 = 24 & x = 8 = 81 cm
3. bb
4x bb
9 bb
8x 3 24 cm bbb
b
27 bb
a

ÖRNEK 8:
Bir öğrenci farklı sayfadaki iki kitaptan birisini 6 günde diğerini 4 günde okumaktadır. Bu kitaplar 3 gün
okunduktan sonra okunmayan sayfa sayıları birbirine eşit olduğuna göre kitapların sayfa sayılarının birbirine
oranını bulunuz.

ÇÖZÜM:
1. kitabın sayfa sayısı x ve ikinci kitabın sayfa sayısı y olsun.
3 1 x x
3 gün sonra 1. kitabın 6 = 2 si okunur. Geriye x - 2 = 2 si kalır.
3 3y y
3 gün sonra 2. kitabın 4 ü okunur. Geriye y - 4 = 4 ü kalır.
x y x 2 1
Kalan sayfalar birbirine eşit olduğuna 2 = 4 & y = 4 = 2 olur.

ÖRNEK 9:
1 1
Seçkin Bey; ürettiği meyve fidanlarının 8 ini 1. ayda, kalanının 3
1
ünü 2. ayda, daha sonra kalanının 2 sini de 3. ayda satmıştır. Seçkin
Bey’in elinde 980 adet meyve fidanı kaldığına göre ürettiği fidan sayısını
hesaplayınız.

ÇÖZÜM:
Bütün birden çok kesirli parçaya bölündüğünde bu bütünün tamamı
kesirlerin paydalarının EKOK u olarak seçilirse işlem kolaylaşır.
EKOK ^2, 3, 8 h = 24 tür. O hâlde fidan sayısının tamamına 24x denirse
24x
1. ayda sattığı miktar = 8 = 3x Kalan fidan sayısı = 24x - 3x = 21x tir.
21x
2. ayda sattığı miktar = 3 = 7x Kalan fidan sayısı = 21x - 7x = 14x tir.
14x
3. ayda sattığı miktar = 2 = 7x Kalan fidan sayısı = 14x - 7x = 7x tir.
7x = 980 & x = 140 olur.
Bu durumda ürettiği fidan sayısının tamamı = 24x = 24 $ 140 = 3360 olarak bulunur.

Matematik 9 | 167
DENKLEMLER VE EŞİTSİZLİKLER

Yaş Problemleri
Yaş problemlerinin çözümünde aşağıdaki bağıntılardan yararlanılabilir:

a yıl sonraki yaşı: x + a olur.


Bir kişinin bugünkü yaşı x olsun.
a yıl önceki yaşı: x - a olur.
Yaşları toplamı: x + y olur.
İki kişinin yaşları x ve y olsun. a yıl sonraki yaşları toplamı: x + a + y + a = x + y + 2a olur.
a yıl önceki yaşları toplamı: x + y - 2a olur.
İki kişi arasındaki yaş farkı daima sabittir. Yılların değişimi bu farkı etkilemez.
a yıl sonraki yaş ortalaması: k + a olur.
x kişinin bugünkü yaş ortalaması k olsun.
a yıl önceki yaş ortalaması: k - a olur.

ÖRNEK 1:
2 2
Emre ve Bahar’ın şimdiki yaşlarının birbirine oranı 5 tir. 8 yıl sonra bu oran 3 olduğuna göre Bahar ve
Emre’nin şimdiki yaşları toplamını bulunuz.

ÇÖZÜM:
Emre 2 & Emre 2x,
Bahar = 5 = Bahar = 5x şimdiki yaşları olsun.
2x + 8 2
8 yıl sonra 5x + 8 = 3 olacağından 10x + 16 = 6x + 24 & 4x = 8 & x = 2 bulunur.
Emre = 2 $ 2 = 4
3 4 + 10 = 14 olur.
Bahar = 5 $ 2 = 10

ÖRNEK 2:
Ecem’in yaşı kendisinden küçük olan Nehir’in yaşının 3 katından 30 eksiktir. Buna göre Nehir’in yaşının en
az kaç olabileceğini bulunuz.

ÇÖZÜM:
Nehir’in yaşı x olsun. Buna göre Ecem’in yaşı 3x - 30 olur.
Ecem’in yaşı Nehir’in yaşından büyük olduğu için
3x - 30 2 x
2x 2 30
x > 15 olur. x in en küçük değeri istendiği için x = 16 olur.

ÖRNEK 3:
Bir babanın yaşı, iki çocuğunun yaşları toplamından 25 büyüktür. 3 yıl sonra babanın yaşı, çocukların yaşları
toplamının 3 katı olacağına göre babanın bugünkü yaşını bulunuz.

ÇÖZÜM:

Baba 2 Çocuk
Bugün x + 25 x x + 28 = 3 ^x + 6 h = 3x + 18 & 2x = 10 & x = 5 olur.
3 yıl sonra x + 28 x+6 Baba x + 25 = 5 + 25 = 30 yaşındadır.

168 | Matematik 9
DENKLEMLER VE EŞİTSİZLİKLER

ÖRNEK 4:
Ender'in yaşı, Başak’ın yaşının 4 katıdır. Başak’ın doğmasına 10 yıl varken Ender şimdiki yaşının üçte biri
yaşta ise Başak’ın bugünkü yaşını bulunuz.

ÇÖZÜM:
Başak Ender
Bugün x 4x

Doğuma 10 yıl varken - 10 4x


3
Yaşlar farkı sabit olduğundan
4x - x = 3 - ]- 10g & 3x =
4x 4x + 30
3 & 9x = 4x + 30 & 5x = 30 & x = 6 olarak bulunur.

ÖRNEK 5:
Ali ile Mehmet’in yaşları toplamı, yaşları farkının 3 katıdır. Mehmet, Ali’nin yaşına geldiğinde yaşları toplamı
50 olacağına göre ikisinin yaşları arasındaki farkı bulunuz.

ÇÖZÜM:
Ali’nin yaşı x, Mehmet’in yaşı y olsun. Bu durumda
Ali Mehmet
x y
50 - x x

x + y = 3. ^x - yh & x + y = 3x - 3y & 4y = 2x & x = 2y olur. Yaş farkı değişmedinden


x - y = 50 - x - x olur. Buradan
3x - y = 50 olur. x = 2y yerine yazılırsa
3 $ 2y - y = 50 & 5y = 50 & y = 10 bulunur.
x = 2y = 2 $ 10 = 20 olur.
O hâlde yaşları farkı 20 - 10 = 10 olarak bulunur.

ÖRNEK 6:
Bir matematik öğretmeni, yaşını soran iki öğrencisine: “Benim yaşım sizin yaşlarınızın toplamının 15 fazlası,
farkının 18 katıdır. 8 yıl sonra benim yaşım sizin yaşlarınızın farkının 22 katı olacaktır.” demiştir. Buna göre
öğretmenin şimdiki yaşının öğrencilerin şimdiki yaşları toplamına oranını bulunuz.

ÇÖZÜM:

Öğretmen 1. Öğrenci 2. Öğrenci


Şimdiki yaş x y z

x = y + z + 15 & x - 15 = y + z
x = ^y - z h $ 18f ^1 h
4 (2) den (1) çıkarıldığında 8 = 4 $ ^y - z h & y - z = 2 bulunur.
x + 8 = ^y - z h $ 22f ^2h
x = 2 $ 18 = 36 & y + z = 36 - 15 = 21 olur.
x 36 12
y + z = 21 = 7 dir.

Matematik 9 | 169
DENKLEMLER VE EŞİTSİZLİKLER

ÖRNEK 7:
3 1
Begüm ile Susen’in 2005 yılındaki yaşları oranı 4 ve yaşları toplamı 42 dir. Hangi yılda yaşları oranı 3
olur?

ÇÖZÜM:
Begüm'ün yaşı 3
= 4 olduğundan Begüm = 3k ve Susen = 4k denilebilir.
Susen'in yaşı

3k + 4k = 42 & 7k = 42 & k = 6 olur.
Begüm’ün yaşı: 3 $ 6 = 18
Susen’in yaşı: 4 $ 6 = 24 olarak bulunur.
1
a yıl önceki yaşları oranına 3 denirse
18 - a 1
24 - a = 3 & 54 - 3a = 24 - a & 2a = 30 & a = 15 yıl olur.
1
2005 - 15 = 1990 yılında yaşları oranı 3 olur.

ÖRNEK 8:
Bir babanın yaşı; oğlunun yaşının 3 katı, kızının yaşının 4 katıdır. Kızı şimdiki yaşının 2 katına geldiğinde
3
babanın yaşı oğlunun yaşının 2 sinden 18 fazla oluyor. Bu durumda babanın bugünkü yaşını bulunuz.
ÇÖZÜM:
Baba Oğul Kız

3 3k yıl geçmiş olur.


Bugün 12k 4k 3k
15k 7k 6k

3 21k 9k
15k = 7k $ 2 + 18 & 15k - 2 = 18 & 2 = 18 & k = 4 olur.
Babanın bugünkü yaşı: 12 $ 4 = 48 olarak bulunur.

Yüzde Problemleri
Yüzde ifadesi, paydası 100 olan kesirler için kullanılır ve % sembolü ile gösterilir.
20
%20 yüzde yirmi diye okunur ve %20 = 100 şeklinde yazılır.
x$a
• Bir x sayısının yüzde a sı 100 ile hesaplanır.
100
• Yüzde a sı K olan sayının tamamı y ise y = K $ a ile hesaplanır.

ÖRNEK 1:
36 nın %25 ini bulunuz.

ÇÖZÜM:
Bir bütünün istenen kesir değeri, bütün ile kesrin çarpılması sonucu bulunur.
25 36
36 $ 100 = 4 = 9 olur.

170 | Matematik 9
DENKLEMLER VE EŞİTSİZLİKLER

ÖRNEK 2:
%15 i 45 olan sayıyı bulunuz.

ÇÖZÜM:
Verilen sayı x olsun.
15 4500
x $ 100 = 45 & x = 15 = 300 bulunur.

ÖRNEK 3:
%30 u, %40 ından 6 eksik olan sayıyı bulunuz.

ÇÖZÜM:
1. Yol:
Sayıya x denirse
30 40 30x - 40x - 10x 100
x $ 100 = x $ 100 - 6 & 100 =- 6 & 100 =- 6 & x =- 6 $ - 10 = 60 olur.

2. Yol: %40 - %30 = %10 olur. %10 u 6 olan sayının, %100 ü istenen değerdir.
%10 6

%100 x
D.O
6 $ 100
x= 10 = 60 olarak bulunur

ÖRNEK 4:
Bir karenin, bir kenar uzunluğunu %40 artırırsak alanın yüzde kaç artacağını bulunuz.

ÇÖZÜM:
Karenin bir kenarı 10 cm alınırsa karenin alanı 10 $ 10 = 100 cm 2 olur.
%40 artarsa bir kenarı 14 cm olur ve alanı 14 $ 14 = 196 cm 2 olur. %96 artmış olur.

ÖRNEK 5:
Bir yemek yapımında kullanılacak olan A, B, C ve D malzemelerinin
ağırlıkları ile C malzemesinin ağırlıkça yüzde oranı aşağıdaki tabloda
verilmiştir.
A B C D
Yüzde Oranı (%) 24
Ağırlık 540 480 360
Verilenlere göre A malzemesinin yemeğin yüzde kaçı olduğunu
bulunuz.
ÇÖZÜM:
Tüm malzemelerin toplam ağırlığı x olsun. A malzemesinin ağırlığı: 2000 - ]480 + 540 + 360g = 620 g
%24 480 g olur. Ağırlıkça yüzdesine a denirse
%100 xg %24 480 g
D.O %a 620 g
480 $ 100 D.O
x= 24 = 2000 g olur.
620 $ 24
a= 480 = 31 olur. Ağırlıkça yüzdesi %31 dir.

Matematik 9 | 171
DENKLEMLER VE EŞİTSİZLİKLER

ALIŞTIRMALAR

1
Bir sayının 2 katının 3 eksiği, aynı sayının 3 7
Bir babanın yaşı, iki çocuğunun yaşları farkının
katının 5 fazlasına eşitse bu sayıyı bulunuz. 11 katıdır. 2 yıl sonra babanın yaşı çocuklarının
yaşları farkının 9 katından 10 fazla olacağına
göre babanın şimdiki yaşını bulunuz.

2
Şükran Hanım, oğlunun kına gecesinde 8
Pınar ile Serap’ın bugünkü yaşları toplamı 30
dağıtmak için 40 g lık karışık çerez paketleri dur. Serap, Pınar’ın bugünkü yaşında iken Pınar
hazırlayacaktır. Kınaya 50 kişi katılacağına göre 3 yaşında olduğuna göre Serap’ın bugünkü
Şükran Hanım’ın kilosu 16 TL olan karışık çerez yaşını bulunuz.
için ödeyeceği parayı hesaplayınız.

9
10 300 sayısının % 15 i ile 5 sayısının
3 % 140 ının toplamını bulunuz.
3 Bir kesrin değeri 5 tir. Bu kesrin payına 3
eklenip paydasından 3 çıkarıldığında kesrin
3
değeri 4 oluyorsa bu kesrin başlangıç payını
bulunuz.

10
Aşağıdaki grafikte, çamaşır makinesi satan
bir firmanın 5 çeşit markaya göre bir yıllık
Gösterime yeni giren bir filmi izlemek için bilet satışı gösterilmektedir. Buna göre A marka
4 çamaşır makinesinin toplam satıştaki yüzdesini
kuyruğuna giren Mehmet; baştan m. sırada,
sondan ]3m + 5g . sıradadır. Bilet kuyruğunda bulunuz.
48 kişi olduğuna göre Mehmet'in sondan
kaçıncı sırada olduğunu bulunuz. Satış Adeti

350
300
250
200
150
5
Bir otomobilin yakıt deposu boş iken ağırlığı 100
2
a kg, 5 i dolu iken b kg dır. Deponun tamamı 50
Çamaşır
doluyken ağırlığının kaç kg olduğunu a ve b 0
Makinesi
A B C D E
cinsinden bulunuz.

8
Bir anne ile kızının yaşları oranı 3 tür. Kızı Bir dikdörtgenin kısa kenarı % 25 artırılır, uzun
6 11
doğduğunda anne 30 yaşında olduğuna göre kenarı % 20 azaltılırsa dikdörtgenin alanının
annenin bugünkü yaşını bulunuz. yüzde olarak ne kadar değişeceğini bulunuz.

172 | Matematik 9
DENKLEMLER VE EŞİTSİZLİKLER

Kâr Zarar Problemleri


Kâr zarar problemlerinde aşağıdaki bağıntılardan yararlanılır:
• Kâr = Satış fiyatı – Maliyet fiyatı Zarar = Maliyet fiyatı – Satış fiyatı
Kâr
• Kâr yüzdesi = $ 100
Maliyet fiyatı
Bir malın satışındaki kâr zarar durumu (soruda farklı durum yoksa) maliyet fiyatı üzerinden, zam indirim ise
satış fiyatı üzerinden hesaplanır.
• A liralık bir ürüne %x indirim yapıldığında ürünün yeni fiyatı:
A - A $ 100 = A $ b 100 l olur.
x 100 - x

• %x zam yapıldığında da

A + A $ 100 = A $ b 100 l olur.


x 100 + x

• İşlemlerde kolaylık sağlamak amacıyla genellikle bir ürünün alış fiyatına 100x denir.

ÖRNEK 1:
Aşağıdaki tabloda bir ürünün alış fiyatı, alış fiyatı üzerinden yapılan değişiklik ve satış fiyatı gösterilmiştir.
Alış Fiyatı (₺) 300 B 150 140 E 440 256 175 500 200
Yapılan %30 %10 %20 %10 %25 %20 %50 + %50 %10 + %10
C F indirim zam
Değişiklik kâr kâr zarar zarar indirim zam
Satış Fiyatı (₺) A 198 195 D 126 220 G H I K
Tabloya göre harflerle ifade edilen yerlerde olması gereken sayıları bulunuz.
ÇÖZÜM:
30
A değeri için 300 $ 100 = 90 TL kârla 300 + 90 = 390 TL sonucu bulunur.
198 198
B değeri 100x olsun, %10 kârla 110x olur. Buradan 110x = 198 & x = 110 & B = 100 $ 110 = 180 TL
bulunur.
45 $ 100
C değeri için 195 - 150 = 45 TL kâr yapılmıştır. Kâr yüzdesi 150 = 30 olduğundan %30 bulunur.
140 $ 20
D değeri için 100 = 28 TL zararla 140 - 28 = 112 TL sonucu bulunur.
126 126
E değeri 100x olsun. %10 zararla 90x olur. Buradan 90x = 126 & x = 90 & 100x = 100 $ 90 = 140 TL
bulunur.
220
F değeri için 440 - 220 = 220 TL zarar vardır. Zarar yüzdesi 440 $ 100 = 50 olur. Cevap %50 olarak
bulunur.
25
G değeri için 256 $ 100 = 64 TL indirimle G = 256 - 64 = 192 TL bulunur.
20
H değeri için 175 $ 100 = 35 TL zamla H = 175 + 35 = 210 TL bulunur.
50
I değeri için 500 $ 100 = 250 TL indirimle 500 - 250 = 250 olur. Bu fiyat üzerinden tekrar %50 indirim
yapılacağından sonuç:
50
250 $ 100 = 125 & 250 - 125 = 125 TL olarak bulunur.
10
K değeri için 200 $ 100 = 20 TL zamla 200 + 20 = 220 olur. Bu fiyat üzerinden tekrar %10 zam
yapılacağından sonuç:
10
220 $ 100 = 22 & 220 + 22 = 242 TL olarak bulunur.

Matematik 9 | 173
DENKLEMLER VE EŞİTSİZLİKLER

ÖRNEK 2:
Bir kitap dağıtım firması, kitapları etiket fiyatının %50 eksiğine alıp
etiket fiyatı üzerinden %20 indirim yaparak satmaktadır. Bu firmanın
kitap satışındaki kâr yüzdesini bulunuz.

ÇÖZÜM:
Etiket fiyatı 100x olsun. %50 eksiği 50x olur.
100x in 20x indirimli hâli 80x olur.
80x - 50x = 30x kâr edilir.
30 x
Kâr yüzdesi = 5 0 x $ 10 0 = 60 bulunur. Kâr yüzdesi %60 olur.

ÖRNEK 3:
Bir malın %40 ını %30 kârla satan bir tüccar, malın geri kalan kısmını % kaç zararla satarsa ne kâr ne de
zarar eder? Bulunuz.

ÇÖZÜM:
100x denirse %40 ı 40x eder.
30
40x %30 kârla 40x $ 100 = 12x kâr eder.
100x ten geriye kalan 60x %a zararla 12x e satılmalıdır.
a 12 x $ 100 100
60x $ 100 = 12x & a = 60 x = 5 = 20
Bu durumda malın geri kalan kısmını %20 zararla satılmalıdır ki kâr zarar eşitlensin.

ÖRNEK 4:
İki yıllık bir toplu sözleşmede memurlara, 1. yılın her altı aylık dilimi için %10 , 2. yılın her altı aylık dilimi
için 60 TL zam yapılması kararlaştırılmıştır. 2. yıl sonunda memurlar toplamda %25 zam almış olmaktadırlar.
Buna göre memurların başlangıçtaki maaşlarını bulunuz.

ÇÖZÜM:
Maaşları 100x olursa
10
1. altı ay sonunda 100x $ 100 = 10x zam alırlar ve maaşları 100x + 10x = 110x olur.
10
2. altı ayın sonunda 110x $ 100 = 11x zam alırlar.
1. yıl sonunda 11x + 10x = 21x zam almış olurlar.
2. yıl 60 + 60 = 120 TL zam alırlar.
21x + 120 = 25x & 120 = 4x & x = 30 olur.
Bu durumda başlangıçtaki maaşları 100x = 100 $ 30 = 3000 TL dir.

ÖRNEK 5:
Suat Bey, 10 eşit taksitle televizyon satın alıyor. İlk 2 taksitini ödemesi gereken miktardan %40 fazla ödüyor.
Ödediği fazla miktar, kalan 8 taksitten eşit olarak düşülüyor. Buna göre kalan taksitlerin her biri başlangıçtaki
durumdan yüzde kaç eksilir, bulunuz.
ÇÖZÜM:
Taksit miktarlarına 100 TL denirse ilk iki taksitte 40 + 40 = 80 TL fazla ödeme yapılmış olur. Bu fazla miktar
80
8 aya bölüneceğinden 8 = 10 TL eksik ödeyecektir. Bu durumda %10 azalma olur.

174 | Matematik 9
DENKLEMLER VE EŞİTSİZLİKLER

ÖRNEK 6:
Bir mağaza “3 Al 2 Öde” kampanyası düzenlemiştir. Kampanya koşullarına göre 3 ürün alan bir kişi en pahalı
olan 2 ürünün fiyatını ödeyecektir. 6 farklı ürün alan Mustafa Bey'in 140, 40, 120, 190, 80, 30 liralık 6 ürünü bu
kampanya sayesinde en çok yüzde kaç indirimle alacağını bulunuz.

ÇÖZÜM:
30 40 80 120 140 190 şeklindeki gruplamayla daha fazla indirim olacaktır.

30 + 40 + 80 + 120 + 140 + 190 = 600 TL de 30 + 120 = 150 TL indirim olacağından
600 TL de 150 TL indirim
100 TL de x TL indirim
D.O
150 $ 100
x= 600 = 25 olur. %25 indirimle almıştır.

ÖRNEK 7:
Alış fiyatı x TL olan bir ürün y = 3x - 900 TL ye satılırsa %20 kâr ediliyor. Buna göre bu ürünün satış fiyatını
bulunuz.

ÇÖZÜM:
%20 kârlı satış fiyatı:
20 120x
x + x $ 100 = 100 olur
120x
100 = 3x - 900 & 12x = 30x - 9000 & 9000 = 18x & x = 500
Satış fiyatı: y = 3x - 900 = 3 $ 500 - 900 = 1500 - 900 = 600 TL olur.

ÖRNEK 8:
Bir mağaza %30 indirim yaptığında satışları %70 artıyorsa bu mağazanın günlük satışındaki artış yüzdesini
bulunuz.

ÇÖZÜM:
Tanesi 10 TL den 10 ürün satarsa günlük satışı 10 $ 10 = 100 olur.
30
%30 indirim yaparsa tanesi 10 - 10 $ 100 = 7 TL olur.
70
Satışlar %70 artarsa 10 + 10 $ 100 = 17 tane ürün satılır.
Bu durumda günlük satış 7 $ 17 = 119 TL olur. Sonuç olarak satışlarda %19 artış olmuştur.

ÖRNEK 9:
x
Bir satıcı, 15 tanesini x TL ye aldığı limonların tanesini 6 TL ye satıyor. Buna göre satıcının kârının yüzde
kaç olacağını bulunuz.

ÇÖZÜM:
x
15 tanesi x ise 30 tanesi 2x olur. 1 tanesinin satışı 6 ise 30 tanesinin satışı 5x tir.
30 tanedeki kâr miktarı 5x - 2x = 3x olduğundan
2x de 3x kâr

100 $ 3x
100 de a kâr a= 2x = 150 olur. Satıcının kârı %150 dir.
D.O

Matematik 9 | 175
DENKLEMLER VE EŞİTSİZLİKLER

ÖRNEK 10:
Yaş üzüm kuruduğunda ağırlığının %20 sini kaybediyor. Kilosu 80 TL
den alınan yaş üzüm kuruduğunda satışından %25 kâr edebilmek için
kilosunun kaç TL den satıldığını bulunuz.

ÇÖZÜM:
Yaş Üzüm Kuru Üzüm
Miktar 100x 80x
Fiyat ₺80 ₺?
Yaş üzümün tamamının alış fiyatı: 100x $ 80 = 8000x olur. %25 kâr edilecekse
25
8000x $ 100 = 2000x kâr edilir. Satıştan kazanılan miktar 8000x + 2000x = 10000x olur.
10000x
80x = 125 TL olarak kuru üzümün fiyatı bulunur.

ÖRNEK 11:
Yıllık enflasyon oranının %30 olduğu bir ülkede maaşına %17 zam yapılan bir kişinin alım gücündeki
azalma yüzdesini bulunuz.

ÇÖZÜM:
Bir ürünün satış fiyatı 100 TL olursa %30 enflasyonlu fiyatı 130 TL olur
Maaş 100x olursa ve %17 zam yapılırsa maaş 117x olur.
100x
İlk durumdaki ürün alım gücü 100 = x tir.
117x 9x
Son durumdaki alım gücü 130 = 10 dur.
x
9x x 10 x 1
x - 10 = 10 luk azalma ile alım gücünde x $ 100 = 10 $ x $ 100 = 10 & %10 azalma olur.

Faiz Problemleri
Bankaya yatırılan anapara: A A TL nin yıllık %n faiz oranıyla
faiz oranı: n, A$n$t
• t yılda F = 100
zaman: t,
A$n$t
faiz getirisi: F olsun. • t ayda F = 100 $ 12
A$n$t
• t günde F = 100 $ 360 faiz getirisi olur.

ÖRNEK 1:
480 TL parası olan bir kişi parasını bankaya yatırıyor. Bu kişinin
a) %35 ten 1 yıllık,
b) %44 ten 10 aylık,
c) %15 ten 20 günlük faiz getirisini bulunuz.

ÇÖZÜM:
480 $ 35 $ 1
a) F = 100 = 168 TL dir.
480 $ 44 $ 10
b) F = 100 $ 12 = 44 $ 4 = 156 TL dir.
480 $ 15 $ 20
c) F = 100 $ 360 = 4 TL dir.

176 | Matematik 9
DENKLEMLER VE EŞİTSİZLİKLER

ÖRNEK 2:
3500 TL si 4 ayda 210 TL faiz getirdiğine göre bu paranın bankaya yıllık yüzde kaç faiz oranı ile yatırıldığını
bulunuz.

ÇÖZÜM:
35 00 $ n $ 4 210 $ 12
210 = 1 00 $ 12 & n = 35 $ 4 = 18 olur. Faiz oranı %18 dir.

ÖRNEK 3:
5000 TL si olan bir kişi, parasının bir kısmını yıllık %40, geri kalanını ise yıllık %30 faizle bankaya yatırıyor
ve 1 yıl sonunda toplam 1860 TL faiz geliri elde ediyor. Buna göre %30 faiz oranı ile bankaya yatırılan para
miktarını bulunuz.

ÇÖZÜM:
Paralar, x ve 5000 - x diye ayrıldığında
]5000 - xg $ 40 $ 1 x $ 30 $ 1
100 + 100 = 1860
200 000 - 40x + 30x = 186 000
14 000 = 10x & x = 1400 tir.

ÖRNEK 4:
12 000 TL önce yıllık %20 faiz oranıyla 1 yıllığına bankaya yatırılıyor. 1. yılın sonunda faiziyle birlikte tekrar
1 yıllığına %15 faiz oranıyla bankaya yatırılıyor. Buna göre 2. yıl sonundaki toplam faiz getirisini bulunuz.

ÇÖZÜM:

12 000 $ 20 $ 1
F1 = 100 = 2400 TL
12 000 + 2400 = 14 400 TL
14 400 $ 15 $ 1
F2 = 100 = 2160 TL
F = F1 + F2 = 2400 + 2160 = 4560 TL faiz getirisi olur.

ÖRNEK 5:
Umut, parasının %25 ini yıllık %12 , %35 ini yıllık %30 faiz üzerinden 8 aylığına farklı iki bankaya yatırmıştır.
Umut, kalan parasını aylık %5 faizle kaç aylığına başka bir bankaya yatırırsa toplamda parasının %29 u kadar
faiz geliri elde edeceğini bulunuz.

ÇÖZÜM:
Aylık %5 faiz, yıllık %60 faize eşittir.
Umut’un 100A TL si olsun.
25A $ 12 $ 8 35A $ 30 $ 8 40A $ 60 $ t
100 $ 12 + 100 $ 12 + 100 $ 12 = 29A
2A + 7A + 2At = 29A
2t = 20 & t = 10 ay olur.

Matematik 9 | 177
DENKLEMLER VE EŞİTSİZLİKLER

ÖRNEK 6:
Munise, bankaya yatırdığı x TL sinin aylık %2, 5 ten 8 aylık faiz getirisinin, y TL sinin yıllık %25 ten 6 aylık
faiz getirisine eşit olduğunu görmüştür. Buna göre x ile y arasındaki ilişkiyi bulunuz.

ÇÖZÜM:
Aylık %2, 5 faiz getirisi, yıllık 12 $ 2, 5 = 30 olduğundan %30 faiz getirisine eşittir.
x $ 30 $ 8 y $ 25 $ 6
100 $ 12 = 100 $ 12 & 240x = 150y & 8x = 5y olur.

Karışım Problemleri
Karışım problemlerinin çözümünde aşağıdaki bağıntılardan yararlanılır:
Saf madde miktarı
• Saf madde oranı =
Karışım miktarı
Saf madde miktarı
• Saf madde yüzdesi = $ 100
Karışım miktarı

Yeni karışımın tuz yüzdesi:


+ = a b
c 100 $ x + 100 $ y a$x+b$y
c = 100 = x+y &c = x+y dir.

%a %b %c
x g tuzlu su y g tuzlu su (x+y) g tuzlu su

Saf suyun tuz yüzdesi 0, tuzun tuz yüzdesi 100 dür.

ÖRNEK 1:
Tuz oranı %40 olan 240 g tuzlu sudaki tuz ve su miktarlarını bulunuz.

ÇÖZÜM:
40
Tuz miktarı = 240 $ 100 = 96 g ve su miktarı = 240 - 96 = 144 g

ÖRNEK 2:
Şeker oranı %30 olan 40 g şekerli su ile şeker oranı %60 olan 35 g şekerli su karıştırılıyor. Elde edilen
karışımın şeker yüzdesini bulunuz.

ÇÖZÜM:
30 $ 40 + 60 $ 35
Şeker yüzdesine x denirse x = 40 + 35 = %44

ÖRNEK 3:
Tuz oranı %60 olan 80 g tuzlu suya kaç g tuz konulursa tuz oranının %80 olacağını bulunuz.

ÇÖZÜM:
x g eklenirse
80 $ 60 + x $ 100 & 6400 + 80x = 4800 + 100x
80 = 80 + x
& & 6400 - 4800 = 100x - 80x
& & 1600 = 20x & x = 80 g

178 | Matematik 9
DENKLEMLER VE EŞİTSİZLİKLER

ÖRNEK 4:
Tuz oranı %18 olan 150 g çözeltiden kaç g su buharlaştığında kalan
çözeltinin tuz oranı %25 olur?

ÇÖZÜM:
18 $ 150 - x $ 0
25 = 150 - x & 3750 - 25x = 2700 olur.
Buradan x = 42 g bulunur.

ÖRNEK 5:
Aşağıdaki grafikte A ve B karışımlarında bulunan su ve şeker miktarları verilmiştir.
Su (g) A
B
220

Şeker (g)
180 280
A ve B şekerli su karışımlarının şeker yüzdelerini bulunuz.
ÇÖZÜM:
A karışımı 180 + 220 = 400 B karışımı 220 + 280 = 500
400 g 180 g şeker 500 g 280 g şeker
100 g x g şeker 100 g y g şeker
D.O D.O
180 $ 100 280 $ 100
x= 400 = 45 y= 500 = 56
A karışımında %45 şeker vardır. B karışımında %56 şeker vardır.

ÖRNEK 6:
1
%20 lik 500 g tuzlu suyun 5 i alınarak yerine alınan miktar kadar tuz ilave edildiğinde oluşan tuzlu suyun
yüzde kaçının tuz olduğunu bulunuz.

ÇÖZÜM:
1
500 $ 5 = 100 g alındığında 500 - 100 = 400 g kalır.
400 $ 20 + 100 $ 100 8000 + 10000
x= 400 + 100 = 500 = 36 Buna göre karışımda %36 oranında tuz vardır.

ÖRNEK 7:
Funda’nın kahvaltıda ikram etmek üzere hazırladığı tahin pekmez
karışımının tahin oranı %40 tır. Karışımın çok şekerli olduğunu fark eden
Funda pekmez oranını %40 a indirmek için karışıma 20 g tahin eklediğine
göre hazırladığı karışımın miktarını bulunuz.

ÇÖZÜM:

x g karışımın %40 ı tahin ise %60 ı pekmezdir.


x $ 60 + 20 $ 0
x + 20 = 40 & 60x = 40x + 800 & 20x = 800 & x = 40 g

Matematik 9 | 179
DENKLEMLER VE EŞİTSİZLİKLER

ÖRNEK 8:
Aşağıda A ve B kaplarındaki karışım miktarları ve karışımdaki tuz yüzdeleri verilmiştir. Buna göre
A B

a) A kabındaki karışımın yarısı alınıp B ye, ardından da B de biriken


karışımın yarısı alınıp A ya aktarılıyor. Oluşan karışımın yüzde
kaçının tuz olduğunu bulunuz.
3
b) A kabındaki karışımın 14 ü alınıp B ye, ardından B de oluşan
11
yeni karışımın 25 i alınıp A ya aktarılıyor. Oluşan karışımın
% 20 % 30 yüzde kaçının tuz olduğunu bulunuz.
140 g 70 g

ÇÖZÜM:
20 $ 70 + 30 $ 70 3500
a) 1. Durum: B kabında 70 + 70 = 140 = 25 olup yeni karışımda %25 oranında tuz bulunur.

20 $ 70 + 25 $ 70 3150
2. Durum: A kabında 140 = 140 = 22, 5 olup yeni karışımda %22, 5 oranında tuz
bulunur.

20 $ 30 + 30 $ 70 2700
b) 1. Durum: B kabında 30 + 70 = 100 = 27 olup yeni karışımda %27 oranında tuz bulunur.

20 $ 110 + 27 $ 44 3388
2. Durum: A kabında 110 + 44 = 154 = 22 olup yeni karışımda %22 oranında tuz bulunur.

ÖRNEK 9:
2
Alkol oranı %x olan bir kolonyaya içindeki miktarın 3 ü kadar %20
alkol oranına sahip başka bir kolonya ekleniyor. Yeni kolonyanın alkol
oranı %56 olmaktadır. Buna göre x değerini bulunuz.

ÇÖZÜM:
%x lik karışıma 3a denirse
3a $ 3 = 2a ]%20g miktarda karışım eklenirse
2

x $ 3a + 20 $ 2a
56 = 3a + 2a & 56 $ 5 = 3x + 40 & 240 = 3x & x = 80
bulunur.

ÖRNEK 10:
Bir kuyumcu 22 ayar 20 g bileziği, 16 ayar 100 g küpeleri ve saf altını
eritip karıştırarak 18 ayar bilezikler yapacaktır. Saf altın 24 ayar olduğuna
göre karışıma kaç g saf altın katılması gerektiğini bulunuz.

ÇÖZÜM:

20 $ 24 + 100 $ 24 + x $ 24 = ]20 + 100 + xg $ 24


22 16 24 18

440 + 1600 + 24x ]120 + xg 18
24 = $ 24
2040 + 24x = 2160 + 18x
6x = 120
x = 20 g saf altın katılmalıdır.

180 | Matematik 9
DENKLEMLER VE EŞİTSİZLİKLER

ÖRNEK 11:
Bir havuza %14 lük ve %25 lik tuzlu su akıtan iki musluktan birincisi 12, ikincisi 10 saatte bu havuzu tek
başına doldurabilmektedir. İki musluk aynı anda açılıp havuz dolduğunda havuzun içindeki suyun tuz yüzdesini
bulunuz.
ÇÖZÜM:
İki musluğun havuzu tek başlarına doldurması için geçen süreler ile musluklardan akan tuzlu su miktaları
birbiri ile ters orantılıdır.

10x $ 100 + 12x $ 100 = ]10x + 12xg $ 100


1. musluk 2. musluk 14 25 A
12 saat 10 saat 140x + 300x A
100 = 22x $ 100
10x tuzlu su 12x tuzlu su akar.
440x = 22x $ A
%20 = A

Hareket Problemleri
Hız ve hareket problemlerinde aşağıdaki bağıntılardan yararlanılır.
x V t
Yol Hız Zaman olmak üzere
Yol = Hız $ Zaman & x = V.t olur.
Yol x
Hız = Zaman & V = t olur.
Alınan toplam yol
Ortalama hız = dir.
Toplam süre
2.V .V
V1 ve V2 hızları ile alınan yollar eşit ise Vort = V1 +1 V22 (Aynı yolu gidip gelme durumunda kullanılır.)
Hız, yol ve zaman arasında daima bir orantı vardır.
• Hız, yol ile doğru; zaman ile ters orantılıdır.
• Yol ile zaman doğru orantılıdır.

ÖRNEK 1:
Aralarında 720 km uzaklık bulunan iki şehir arasında çalışan bir otobüs 60 km/sa ortalama hızla gittiğinde
otogara 3 saat geç kalıyor. Bu otobüsün zamanında seferini tamamlaması için hızını kaç km/sa artırması
gerektiğini bulunuz.

ÇÖZÜM:
t sürede varması gerekirse
60 $ ]t + 3g = 720 & t + 3 = 12 & t = 9
V $ t = 720 & 9V = 720 & V = 80 km/sa olduğundan 20 km/sa arttırılmalıdır.

• Bir araç bir noktadan geçerken kendi boyu, belli bir uzunluğu geçerken hem kendi boyu hem de
geçtiği uzunluğun toplamı kadar yol kat eder.
V $ 1000 V $ 1000
• V km/sa = 60 m/dk = 60 $ 60 m/sn

Matematik 9 | 181
DENKLEMLER VE EŞİTSİZLİKLER

ÖRNEK 2:
Ortalama hızı saatte 30 km olan bir aracın 5 dakikada kaç metre yol alacağını bulunuz.

ÇÖZÜM:
30 $ 1000
x = V$t V = 30 km/sa = 60 = 500 m/dk
x = 500 $ 5 = 2500 m dir.

ÖRNEK 3:
Ada, evden markete 25 m/dk hızla gidip eve 15 m/dk hızla dönmüştür. Gidiş dönüş toplam 16 dk sürdüğüne
göre ev ile market arasının kaç metre olduğunu bulunuz.

ÇÖZÜM:
Ev ile market arası uzaklık x metre olsun. Evden markete gidiş süresi de t olsun.
x = 25 $ t = 15 $ ]16 - tg & 25t = 15 $ 16 - 15t & 40t = 15 $ 16 & t = 40 = 6 dk
15 $ 16

X = 25 $ 6 = 150 metredir.

ÖRNEK 4:
2
Bir araç, gideceği yolun 3 ünü saatte 40 km lik hızla, kalan yolu da 60 km lik hızla gittiğine göre bu aracın
tüm yol boyunca yaptığı ortalama hızı bulunuz.

ÇÖZÜM:
Yolun tamamı 3x olsun.

2x x

40 km/sa t 1 60 km/sa t 2

2x x x
2x = 40 $ t 1 & t 1 = 40 = 20 saat x = 60 $ t 2 & t 2 = 60 saat

Alınan toplam yol 2x + x 3x 60.3x


Ortalama hız = & Vort = x x = 4x = 4x = 45 km/sa tir.
Toplam süre
20 + 60 60

ÖRNEK 5:
Arzu, evinden okula 3 km/sa hızla giderse 12 dakika geç kalıyor. 4 km/sa hızla giderse 6 dakika erken
varıyor. Arzu’nun evi ile okulu arasındaki mesafeyi metre cinsinden bulunuz.

ÇÖZÜM:
Cevap metre olarak istendiği için hızlar m/dk ya çevrilir. Alınan yollar eşit olduğundan
1000 1000
3 $ 60 $ ^t + 12h = 4 $ 60 $ ^t - 6h olur. 3t + 36 = 4t - 24 & t = 60 tir.
1000
Yol = 3 $ 60 $ 72 = 3600 metredir.

182 | Matematik 9
DENKLEMLER VE EŞİTSİZLİKLER

ÖRNEK 6:
Bir yarışta birinci olan yarışmacı; yarışı bitirdiğinde ikinci ile arasında 36 m, üçüncü ile de arasında 80 m
fark olmuştur. İkinci olan yarışmacı yarışı bitirdiğinde üçüncü ile arasında 52 m olduğuna göre yarışmanın kaç
metrelik mesafede yapıldığını bulunuz.

ÇÖZÜM:
2. yarışmacı 36 m mesafede 3.
ile arasını 8 m daha açmıştır. Bu
durumda
44 m 36 m
3 2 1 36 metrede 8 metre fark
x metrede 52 m fark
D.O
8x 36 $ 52 36 $ 52
8 = 8 & x = 8 = 234 m
52 m
3 2

A V1 km/sa C V2 km/sa B

Şekilde görüldüğü gibi A ve B noktalarından karşılıklı olarak aynı anda harekete başlayan iki araç t saat
sonra C noktasında karşılaşırsa
AB = ]V1 + V 2g .t

A V1 km/sa B V2 km/sa C

Şekilde görüldüğü gibi A ve B noktalarından aynı anda, aynı yöne doğru harekete başlayan iki araç t saat
sonra C noktasında yan yana gelirse ^V1 2 V2 h
AB = ]V1 - V 2g .t

ÖRNEK 7:
İki araç A ve B noktalarından aynı anda, aynı yöne doğru hareket ediyor. A dan hareket edenin hızı
90 km/sa ve B den hareket edenin hızı 70 km/sa tir. A dan hareket eden 5 saat sonra C noktasında diğerine
yetiştiğine göre AB değerini bulunuz.
ÇÖZÜM:

A 90 km/sa B 70 km/sa C

I. Yol II. Yol


AC = V $ t = 90 $ 5 = 450 AB = ]90 - 70g $ 5 = 100 km
BC = 70 $ 5 = 350
AB = 450 - 350 = 100

ÖRNEK 8:
A ve B kentlerinden birbirine doğru aynı anda harekete başlayan iki aracın saatteki ortalama hızları 70
ve 100 km dir. Bu iki araç, harekete başladıktan 3 saat sonra karşılaştığına göre A ile B arasındaki mesafenin
uzunluğunu bulunuz.

ÇÖZÜM:
AB = ]70 + 100g $ 3 = 510 km dir.

Matematik 9 | 183
DENKLEMLER VE EŞİTSİZLİKLER

İki trenin birbirini geçmesi, ön kısımlarının karşılaştığı andan arka kısımlarının birbirini geçtiği ana
kadar geçen süredeki hareketlerini belirtir. Aşağıda verilen A ve B trenlerinin boyları sırasıyla a ve b
metre olsun. Birbirlerini t dakikada geçsinler. Bu durumda iki tren, t dakikada aynı yöne doğru giderken
hızları farkı, zıt yöne doğru giderken hızları toplamı ile a + b metre yol alır.

A Treni

B Treni

A Treni

B Treni

Bir trenin bir tüneli geçmesi, trenin ön kısmının tünele girip arka kısmının tünelden çıktığı ana kadar
geçen süredeki hareketini belirtir. Trenin boyu x metre, tünelin boyu y metre olursa tren tüneli geçtiğinde
x + y metre yol almış olur.

Tünel

ÖRNEK 9:
150 metre uzunluğundaki bir tüneli saatte ortalama 60 km hızla 30 saniyede geçen trenin boyunu bulunuz.

ÇÖZÜM:
Tünel

x 150 m

Yol = 150 + x
Hız = 60km = 60 000 metredir.
30
Süre = 30 saniye = 60 $ 60 saattir.
30
150 + x = 60 000 $ 60 $ 60 & 150 + x = 500 & x = 350 metre olur.

ÖRNEK 10:
Saatteki hızları 40 km ve 80 km olan iki tren, karşılıklı hareket ettiğinde birbirlerini 6 saniyede geçtiklerine
göre trenlerin boyları toplamını bulunuz.

ÇÖZÜM:
Toplam uzunluk x metre olsun.
Zıt yönlü harekette hız = 40 + 80 = 120 km = 120 000 m
6
Süre = 60 $ 60 saat
6
x = 120 000 $ 60 $ 60 = 200 m dir.

184 | Matematik 9
DENKLEMLER VE EŞİTSİZLİKLER

ÖRNEK 11:
60 m uzunluğundaki bir tren; sabit hızla 1. tüneli 15 saniyede, 200 m uzunluğundaki 2. tüneli 26 saniyede
geçtiğine göre 1. tünelin uzunluğunu bulunuz.

ÇÖZÜM:
1. tünel için yol uzunluğu: x + 60 , 2. tünel için yol uzunluğu: 200 + 60 = 260 olur.
260 = 26 $ V & V = 10 m/sn
x + 60 = 15 $ 10 & x = 90 m dir.

V1
V1

V2

A A

V2

• Dairesel bir pistte aynı noktadan zıt yönde harekete başlayan iki araç, karşılaştıklarında dairesel pistin
çevresi kadar yol almış olur.
Pistin çevre uzunluğu ]V1 + V2g $ t dir.
• Dairesel bir pistte aynı noktadan, aynı yöne doğru harekete başlayan iki aracın tekrar yan yana gelmesi
için hızlı aracın yavaş olandan 1 tur fazla atması gerekir.
t saatte hızlı olan aracın hızı V1 , yavaş olan aracın hızı V2 olsun. Yan yana gelebilmeleri için
pistin çevre uzunluğu ]V1 - V2g $ t olur.

ÖRNEK 12:

Dairesel bir pistte zıt yönde, aynı anda 14 m/dk ve 16 m/dk hızlarla hareket eden iki bisikletli, 18 dk sonra
karşılaştığına göre pistin çevre uzunluğunu bulunuz.

ÇÖZÜM:
Çevre = ]V1 + V 2g $ t = ]14 + 16g $ 18 = 30 $ 18 = 540 metredir.

ÖRNEK 13:

Dairesel bir pistte aynı noktadan aynı yöne 20 m/dk ve 35 m/dk hızlarla aynı anda harekete başlayan iki
koşucu, 38 dk sonra tekrar yan yana geldiklerine göre pistin çevre uzunluğunu bulunuz.

ÇÖZÜM:
Hızlı koşucunun yavaş koşucuyu yakalayabilmesi için bir tur fazla atması gerekir. Bu durumda
Ç = ]35 - 20g $ 38 = 15 $ 38 = 570 metredir.

Matematik 9 | 185
DENKLEMLER VE EŞİTSİZLİKLER

ÖRNEK 14: A
C
Şekilde çevresi 200 m olan pistte A dan 15 m/sn ve B den 10 m/sn hızla
ok yönünde, aynı anda hareket eden iki hareketlinin kaç sn sonra dördüncü
kez karşılaşacağını bulunuz. B

ÇÖZÜM:
$ 1
AB değeri çevrenin 4 üne denk geldiğinden 50m dir.
C noktasında karşılaşılırsa alınan toplam yol:
50 = ]15 + 10g $ t 1 & 50 = 25 $ t 1 & t 1 = 2 sn bulunur.
C noktasında ilk karşılaşmadan sonra hareket aynı noktadan zıt yönlü harekete döner. 3 tur atmaları hâlinde
ise dördüncü kez karşılaşılır. Bu durumda alınan yol:
3 $ 200 = ]15 + 10g $ t 2 olduğundan & 600 = 25 $ t 2 & t 2 = 24 sn bulunur.
4. kez karşılaşma anına kadar geçen toplam süre: t 1 + t 2 = 2 sn + 24 sn = 26 sn olur.

ÖRNEK 15: V = 5 m/s


A
Şekildeki çevresi 120 m olan pistte A dan 5 m/sn ve B den 25 m/sn C
hızla aynı yönde, aynı anda hareket eden iki yayanın kaç sn sonra üçüncü
kez karşılaşacağını bulunuz. 120c
B
M
V = 25 m/s
ÇÖZÜM:
$ 1
AB değeri, pistin çevresinin 3 üne eşit olduğundan 40 m dir.
İlk kez yan yana geldiklerinde alınan yol:
x = ]25 - 5g $ t 1 olduğundan 40 m = 20 $ t 1 & t 1 = 2 sn sonra ilk kez yan yana gelirler. Geriye 2 kez
karşılaşma kalır. Bu durumda 3. kez yan yana gelme süresi:
2 $ 120 = ]25 - 5g $ t 2 & 240 = 20 $ t 2 & t 2 = 12 sn sonra gerçekleşir.
Bu durumda iki hareketlinin 3. kez yan yana gelinceye kadar geçen toplam süre:
t 1 + t 2 = 2 sn + 12 sn = 14 sn olur.

Nehirlerdeki hareket problemlerinde A noktasından B noktasına akıntı yönünde giderken akıntının


hızı VA , yüzücünün ya da nesnenin hızı VY olursa V = VA + VY olur.
B den A ya akıntıya ters yönde giderken V = VY - VA ]VY 2 V Ag olur.

ÖRNEK 16:
Bir tekne, akıntıya karşı 18 dakikada gidebildiği mesafeyi akıntı
yönünde 12 dakikada gittiğine göre teknenin hızının akıntının hızına
oranını bulunuz.

ÇÖZÜM:
VT Teknenin hızı, VA Akıntının hızı olsun.
Akıntı yönünde hız: VT + VA ve Akıntıya karşı hız: VT - VA olur.

x = ]VT + VAg $ 12 = ]VT - VAg $ 18 & 12VT + 12VA = 18VT - 18VA & 30VA = 6VT & VAT = 5
V
olur.

186 | Matematik 9
DENKLEMLER VE EŞİTSİZLİKLER

ÖRNEK 17:
Bir helikopterin hızı 240 km/sa tir. Helikopter, deposunda 12 saat
yetecek kadar yakıt varken arkasından 80 km/sa hızla esen rüzgârlı bir
havada helikopterin en fazla kaç km uzağa gidip gelebileceğini bulunuz.

ÇÖZÜM:
Gidilen yol geri dönüleceğinden
]240 + 80g $ t = ]240 - 80g $ ]12 - tg & 320t = 160 $ 12 - 160t & 480t = 160 $ 12 & t = 160 $ 12
480 = 4 tür.
Yol = 320 $ 4 = 1280 km olur.

İşçi Problemleri
İşçi problemlerinde işlemler, birim zamanda yapılan iş üzerinden gerçekleştirilir.
Bir işçi bir işin tamamını x günde yaparsa
1
• 1 günde x ini,
1
• a günde x $ a sını yapar.

1. işçinin a günde, 2. işçinin b günde bitirdiği bir işi ikisi birlikte x günde bitiriyorsa

b1
a + b l $ x = 1 veya a + b = x tir.
1 1 1 1

ÖRNEK 1:
Bir işi Umut 36 günde bitirebilmektedir. Aynı işi Umut ve Ufuk birlikte 12 günde bitirebildiklerine göre
Ufuk’un bu işi tek başına kaç günde bitirebileceğini bulunuz.

ÇÖZÜM:
Ufuk işi x günde bitirsin. Aynı işi Umut 36, Ufuk x ve ikisi birlikte 12 günde bitiriyorsa
1 1 1 1 1 1 1 2
36 + x = 12 eşitliği yazılır. x = 12 - 36 & x = 36 & x = 18 olur.
]3 g ]1 g

ÖRNEK 2:
Emel bir işi tek başına 24 günde, Yağmur ise aynı işi 36 günde yapabilmektedir. Emel 8 gün, Yağmur 9 gün
çalıştığında işin kaçta kaçının tamamlanacağını bulunuz.

ÇÖZÜM:
1 1
Emel tek başına işi 24 günde bitirdiğine göre 8 günde işin 24 $ 8 = 3 ünü yapar.
1 1
Yağmur tek başına işi 36 günde bitirdiğine göre 9 günde işin 36 $ 9 = 4 ünü yapar.
1 1 7
İkisi birlikte işin 3 + 4 = 12 sini tamamlamış olurlar.
]4 g ]3 g

Matematik 9 | 187
DENKLEMLER VE EŞİTSİZLİKLER

ÖRNEK 3:
Bir işin tamamını Sıla ile Gülce 18 günde, Mert ile Sıla 9 günde, Gülce ile Mert 12 günde bitirebilmektedir.
Üçü birlikte çalıştığında üçünün aynı işi kaç günde bitirebileceklerini bulunuz.

ÇÖZÜM:
İşi Sıla x günde, Gülce y günde, Mert z günde bitirsin. Bu durumda
1 1 1
Sıla ve Gülce: x + y = 18
1 1 1
Mert ve Sıla: x + z = 9
1 1 1
Gülce ve Mert: y + z = 12 denklemleri yazılır. Bu üç denklem taraf tarafa toplanırsa

2 $ c x + y + z m = 18 + 9 + 12 & 2 $ c x + y + z m =
1 1 1 1 1 1 1 1 1 2+4+3 1 1 1 1
36 & x + y + z = 8 olur.
]2 g ]4 g ]3 g
Üçü birlikte 8 günde bitirir.

ÖRNEK 4:
1 usta 3 günde 8 sehpa, 1 çırak 4 günde 4 sehpa yapabilmektedir. Buna göre ikisinin birlikte 220 sehpayı
kaç günde yapabileceklerini bulunuz.

ÇÖZÜM:
Günler eşitlenirse
1 usta 3 günde 8 sehpa " 12 günde 32 sehpa
3
1 çırak 4 günde 4 sehpa " 12 günde 12 sehpa Birlikte 12 günde 32 + 12 = 44 sehpa yaparlar.

12 günde 44 sehpa
x günde 220 sehpa
D.O

220 $ 12 = x $ 44 & x = 60 günde yapabilirler.

ÖRNEK 5:
Bir işi Senem 12 günde, Sinem 18 günde ve Öykü 36 günde bitirebiliyor. Üçü beraber işe başladıktan 2
gün sonra Senem, başlangıçtan 3 gün sonra da Sinem işten ayrılıyor. Öykü’nün kalan işi tek başına kaç günde
bitirebileceğini bulunuz.

ÇÖZÜM:
Öykü kalan işi t günde bitirsin.
1 1 1 1 1
e 12 + 18 + 36 o $ 2 + e 18 + 36 o $ 1 + 36
1
$t = 1
]3 g ]2 g ]1 g ]2 g ]1 g
6 3 1
36 $ 2 + 36 $ 1 + 36 $ t = 1
12 + 3 + t = 36
t = 36 - 15 & t = 21 günde bitirir.

188 | Matematik 9
DENKLEMLER VE EŞİTSİZLİKLER

ÖRNEK 6:
3 2
Filiz bir işin 4 ünü 6 günde, Sedef ise aynı işin 3 ünü 8 günde tamamlamaktadır. Filiz çalışma hızını 2 katına
çıkarıp Sedef çalışma hızını yarıya indirirse ikisi birlikte 3 günde aynı işin ne kadarını yapar?

ÇÖZÜM:
3
Filiz, işin 4 ünü 6 günde yaparsa tamamını 8 günde yapar. Hızını 2 katına çıkarırsa süre yarıya iner ve 4 gün
olur.
2
Sedef, işin 3 ünü 8 günde yaparsa tamamını 12 günde yapar. Hızını yarıya indirirse süre 2 katına çıkar ve 24
gün olur.
1
Filiz 1 günde işin 4 ünü
1
Sedef 1 günde işin 24 ünü yapar. İkisi birlikte 3 günde işin
1 1
e 4 + 24 o $ 3 = x & 24
7 7
$ 3 = x & x = 8 ini yaparlar.
]6 g ]1 g

Havuz Problemleri
Havuz problemlerinin çözümü de işçi problemlerine benzemektedir.
Bir havuzu bir musluk x saatte doldurursa
1
• 1 saatte x ini,
a
• a saatte x ini doldurur.
1. musluğun a saatte, 2. musluğun b saatte doldurduğu bir havuzu ikisi birlikte x saatte dolduruyorsa bu
durum,
b a + b l $ x = 1 veya a + b = x tir.
1 1 1 1 1

A B
Bir havuzu üsteki iki musluk, sırasıyla a ve b saatte doldururken dipte
bulunan 3. musluk dolu havuzu c saatte boşaltsın. Üçü birden aynı anda
açıldığında boş havuzu x saatte dolduruyorlarsa bu durum
1 1 1 1
a + b - c = x bağıntısıyla ifade edilir.

ÖRNEK 1:
A ve B muslukları, boş bir havuzu sırasıyla 12 ve 18 saatte doldururken havuzun dibindeki C musluğu dolu
havuzu 36 saatte boşaltmaktadır. Üç musluk aynı anda açıldığında boş havuzun kaç saatte dolacağını bulunuz.

ÇÖZÜM:
Üç musluk, aynı anda açıldığında boş havuzu x saatte doldursun.
1 1 1 1
12 + 18 - 36 = x
]6 g ]4 g ]2 g
6+4-2 1
72 = x
8 1
72 = x & x = 9 saatte dolar.

Matematik 9 | 189
DENKLEMLER VE EŞİTSİZLİKLER

ÖRNEK 2:
800 litrelik bir havuzu A musluğu 12 saatte, B musluğu 16 saatte dolduruyor. Havuzun dibindeki C musluğu,
dolu havuzu 24 saatte boşaltıyor. Havuz boşken 3 musluk birlikte açıldığında havuz dolana kadar C musluğu
nun kaç litre su boşaltacağını bulunuz.

ÇÖZÜM:
Üç musluk havuzu t saatte doldursun. C musluğu,
1 1 1 1 24 saatte 800 litre
12 + 16 - 24 = t 48
]4 g ]3 g ]2 g
5 x litre boşaltır.
4+3-2 1
48 = t D.O
5 1 48 48
24 $ x = 800 $ 5 & x = 320 litre su boşaltmış olur.
48 = t & t = 5 saatte dolar.

ÖRNEK 3:
Şekildeki gibi bir depoya üç musluk yerleştiriliyor. A musluğu, depoyu tek başına 6 saatte dolduruyor. C
musluğu, tek başına 12 saatte boşaltıyor. B musluğu da kendi seviyesine kadar olan kısmı tek başına 12 saatte
boşaltıyor.
A

_b Üç musluk aynı anda açıldığında boş deponun kaç saatte dolacağını


bb
bb bulunuz.
b
`b 3h
bb
bb B
b
a
3h
C

ÇÖZÜM:
A ve C muslukları açıkken geçen süre t 1 olsun.

b 1 - 1 l $ t1 = 1
6 12 4

1 1
12 $ t 1 = 4 & t 1 = 3 saat olur.
B musluğu için
12 saatte 3
4
x saatte 1 (tamamı) boşalır.
D.O
3
12 $ 1 = x $ 4 & x = 16 olur.
Muslukların üçü birden açıkken geçen süre t 2 olsun.

b 1 - 1 - 1 l $ t2 = 3
6 12 16 4
1 3
48 $ t 2 = 4 & t 2 = 36 saat olur.
t 1 + t 2 = 3 + 36 = 39 saatte dolar.

190 | Matematik 9
DENKLEMLER VE EŞİTSİZLİKLER

ALIŞTIRMALAR

1
%20 zararla 240 TL ye satılan bir malın %30 Güler, biriktirdiği parasını yıllık %40 faizle 3
5
kârla kaç TL ye satılacağını bulunuz. yıllığına bankaya yatırıyor. 3. yıl sonunda faizi
ile 8800 TL si olan Güler’in biriktirdiği para
miktarını bulunuz.

1
2 Bir tüccar malının 4 ünü %20 kârla, kalan malın
1
5 ini %40 zararla ve geri kalanını da %30 kârla
satmıştır. Bu tüccarın tüm satıştan yüzde kaç
kâr edeceğini bulunuz.

6
Şeker oranı %18 olan 40 g şekerli suya kaç g saf
su ilave edilirse şeker oranının %6 ya düşeceğini
bulunuz.

3
Bir miktar paranın %30 dan 1 yılda getirdiği
faizi, 6 ayda getirebilmesi için yıllık yüzde kaçtan
faize verilmesi gerektiğini bulunuz.

4
İrfan Bey; parasını yıllık %26 yerine aylık %4 7
%30 luk 500 g şekerli suya 1000 g saf su ve 300
faizle bankaya yatırsaydı 6 ayda 2200 TL daha g şeker eklendiğinde elde edilen yeni karışımın
fazla faiz geliri elde edecekti. Buna göre İrfan şeker yüzdesini bulunuz.
Bey’in bankaya yatırdığı para miktarını bulunuz.

Matematik 9 | 191
DENKLEMLER VE EŞİTSİZLİKLER

8
Ağırlıkça %30 u şeker olan 70 g lık un şeker 11
Aralarında 60 km uzaklık bulunan iki araç
karışımına 14 g şeker ve 21 g un eklenirse yeni aynı anda, aynı yöne doğru hareket ediyor.
karışımdaki un (g)/şeker(g) oranını bulunuz. Arkadakinin hızı 80 km/sa, öndekinin hızı 70
km/sa olduğuna göre arkadaki aracın öndekine
kaç saatte yetişeceğini bulunuz.

Bir araç 4 saatte gittiği yolu, 7 saatte geri 12


Dairesel pistin çevresi 360 m dir. A dan aynı
9
dönüyor. Gidiş hızı, geliş hızından 30 km fazla yöne doğru harekete başlayan iki hareketlinin
olduğuna göre yolun uzunluğunu bulunuz. kaç dakika sonra karşılaşacağını bulunuz.

A
15 m/dk
20 m/dk

Bir araç, A dan B ye (toprak yol) 30 km/sa, $


10 13 AB = 50 m olup şekilde gösterildiği gibi iki
B den C ye (asfalt yol) 90 km/sa hızla gitmiştir. hareketli aynı anda harekete başladıktan 46
AB = 3 $ BC ise ortalama hızı bulunuz. dakika sonra ikinci kez karşılaştığına göre pistin
çevresini bulunuz.
A
V 1 = 25 m/dk

V 1 = 20 m/dk

192 | Matematik 9
DENKLEMLER VE EŞİTSİZLİKLER

14
Bir yüzücünün durgun sudaki hızı 20 m/dk, akıntılı sudaki hızı 10 m/dk dır. Bu yüzücü, akıntıya karşı açılıp
belli bir süre yüzerek tekrar geri döndüğüne göre yüzücünün ortalama hızını bulunuz.

15
İzmir Mavi Treni, belli bir noktadan 5 sn de ve İzmir'in Buca ilçesi ile Şirinyer semti arasında bulunan
2000 m lik Şirinyer Tüneli’ni 30 sn de geçmektedir. Buna göre
a) Trenin boyunun kaç metre olduğunu bulunuz.
b) Trenin hızının kaç km/s olduğunu bulunuz.

16
A V 1 = 40 km/s C V 1 = 80 km/s B

Şekilde hızları V 1 = 40 km/s , V 2 = 80 km/s olan iki hareketli aynı anda, zıt yönde birbirlerine doğru
hareket ediyor. Aralarındaki mesafe AB = 600 km olduğuna göre bu iki hareketlinin kaç saat sonra
karşılaşacağını bulunuz.

17
Aralarında 180 km mesafe bulunan iki şehirden aynı anda aynı yöne doğru hareket eden iki araçtan hızlı
olan, yavaş olana 6 saat sonra yetişiyor. Araçlar zıt yönde hareket ettiğinde ise 2 saat sonra karşılaşıyor.
Her bir aracın hızını bulunuz.

Matematik 9 | 193
DENKLEMLER VE EŞİTSİZLİKLER

18
Tarımla geçimini sağlayan bir kişi, yaşadığı bölge tarıma uygun olmasına rağmen iklim şartlarından dolayı
istediği ürünü yetiştirememektedir. Bu sebeple bir sera kurmaya karar verir.
Bu seranın gelir gider ve maliyet bilgileri yaklaşık
aşağıdaki gibidir.
• Seranın kurulum maliyeti: 52 500 TL
• Yıllık yaklaşık gelir: 45 000 TL
• Kurulacak seranın 12 ay boyunca aylık gideri: 500 TL
• Verimlilik: Bir yılın yaklaşık %75 inde üretim
yapılabilmektedir.

Verilen bilgilere göre aşağıdaki soruları cevaplayınız.


1. Sera bir yılın kaç ayı tam kapasiteyle çalışır?
2. Seranın çalıştığı zamandaki üç aylık geliri en fazla kaç TL dir?
3. Bu seradan kâr edilebilmesi için en az kaç ay üretim yapılması gerekir?
4. Kurulacak seranın aylık geliri yüzde kaç arttırılırsa 7. aydan sonra kâr elde edilmeye başlanır?

19
Likya Yolu; Fethiye’den başlayıp Antalya’ya kadar devam eden, Antik Likya kalıntılarının olduğu,
etrafında dağ köyleri ve şelalelerin bulunduğu tarihi bir yürüyüş yoludur. Likya Yolu’nda her yıl yaklaşık
30 bin kişi yürümektedir.

Buna göre
1. Likya Yolu’nda günde ortalama kaç kişi yürüyüş yapmaktadır?
2. Furkan, gece karanlığına kalmadan saat 18.00 e kadar Mavikent-Gelidonya Feneri-Adrasan
etabını yürümeyi düşünüyor. Furkan’ın ortalama hızı saatte 5 km dir. ve 40 km lik bu etapta 2
saat yürüyüp 30 dk dinlenmeyi planlamaktadır. Furkan, en geç saat kaçta yürüyüşe başlamalıdır?
3. Furkan’ın 40 km lik yürüyüşünde kaç adım atmış olabileceğini bulunuz. (Dinlenme zamanındaki
adımları hesaplamaya katılmayacaktır.)

194 | Matematik 9
DENKLEMLER VE EŞİTSİZLİKLER

ÖLÇME VE DEĞERLENDİRME -1

1
x ve y negatif birer tam sayı olmak üzere 6 A = ^- 2, 4h ve B = 71, 3h olmak üzere A # B
4x + 9y =- 68 ise x + y değeri kaçtır? kümesinin tüm elemanlarını içeren en küçük
dairenin alanı kaç br 2 dir?
A) -15 B) -14 C) -13 D) -12 E) -10
A) 16r B) 15r C) 7r D) 4r E) 10r

3x - ky = 2k - 6
2 A = 3 - 2 + 28 7
17 15 11 kx - 12y = 12 denklem sisteminin çözümü

B= 13 14 6
15 - 17 + 11 sayısının A sayısı türünden boş küme ise k kaçtır?
eşiti nedir? A) 6 B) -6 C) 5 D) -7 E) -5

A) A-2 B) A-1 C) A+1


D) 2A E) A-3

3
5x - 13 ifadesini doğal sayı yapan x tam sayı 8 ^m - 2n - 5h2 + 4m + 5n - 7 + 2p 4 = 0 ise
x-4 m + n + p kaçtır?
değerlerinin toplamı kaçtır?
A) -5 B) -3 C) 1 D) 2 E) 3
A) 11 B) 12 C) 13 D) 14 E) 15

4 a,b,c ! N olmak üzere 9 a = b + 1 ifadesinde hangi a değeri için b


38 a 1 ise a b c değeri kaçtır? 3b - 5
7 = +b 1 + + hesaplanamaz?
+c 5
A) - 1 B) - 3 C) 1 D) 3 E) 3
2 2 3 2
A) 10 B) 11 C) 12 D) 13 E) 14

5
1 1 1 g 1 10 ^3a + bh x - 2 ^b + 1h = x + 6 + a denkleminin
1 $ 2 + 2 $ 3 + 3 $ 4 + + 99 $ 100 çözüm kümesi R ise a + b değeri kaçtır?
işleminin sonucu kaçtır?
A) 0,80 B) 0,85 C)0,87 A) -5 B) -3 C) 0 D) 1 E) 2

D) 0,90 E) 0,99

Matematik 9 | 195
DENKLEMLER VE EŞİTSİZLİKLER

11
x-y ^ - h
1 1 1 0, x 2 1 x, y x 3 $ z $ 0 16
2y $ x
olduğuna göre aşağıdakilerden hangisi 5x + 3y 1 15
kesinlikle doğrudur? eşitsizlik sisteminin çözüm kümesi aşağıdaki
bölgelerden hangilerinin birleşimidir?
A) y 1 0 B) x + y 2 1 C) - 1 1 x 1 1 y
D) z 2 0 E) x $ y $ z # 0 I

5 X x = 2y

12
x ve y reel sayıları için III II
x-1 IX
-4 1 2 1 1 0 VI
x
3 # 2-y 1 5 IV VII
3
VIII
V
ise x 2 - 2y 3 ifadesinin alabileceği en büyük ve 5x + 3y = 15
en küçük tam sayı değerlerinin toplamı kaçtır?
A) IX ve VIII
A) 112 B) 104 C) 72 D) 54 E) 26 B) V, VI ve VII
C) II, III ve IV
1 1 D) II ve IV
13
]x - 1g2
1 eşitsizliğinin çözüm kümesi
x2 + 5 E) I ve X
nedir?
A) ^- 2, - 3h - ! 1 + x+2 +5 = 6
17
B) ^1, 3h
C) ^- 3, - 2h
denklemini sağlayan x gerçek sayılarının
çarpımı kaçtır?
D) ^1, 4 h
E) ^- 3, 1h A) - 2 B) - 3 C) 0 D) 2 E) 3

14 -4 # a 1 0 x - 3 - 2x = 10 eşitliğini sağlayan x
18
11b#2 değerlerinin toplamı kaçtır?
-5 # c 1 3 -7 -5 2 4 7
A) 3 B) 3 C) 3 D) 3 E) 3
olduğuna göre ]a - 2bg $ c sayısı hangi aralıkta
değer alır?
A) ^- 16, 10h B) ^- 24, 40@ C) 6- 6, 20g
x - 3 + x + 4 = 8 denklemini sağlayan x
D) 6- 18, 10@ E) 6- 28, 36g 19
değerlerinin toplamı aşağıdakilerden
hangisidir?
7
z 1 0 1 y 1 x olmak üzere A) 4 B) 2 C) 3 D) - 1 E) - 2
15
y - z - x - y + y - 2x - - y ifadesinin
x-z
eşiti aşağıdakilerden hangisidir?
20 x - 3 - 12 - 4x 2 15 eşitsizliğinin çözüm
x-z y+z kümesi aşağıdakilerden hangisidir?
A) x + z B) x - z C) 2x
D) 1 E) - 1 A) ^- 3, 8 h B) ^- 3, 10h C) Q
D) ^3, 8 h E) ^3, 10h

196 | Matematik 9
DENKLEMLER VE EŞİTSİZLİKLER

ÖLÇME VE DEĞERLENDİRME -2

1 12 x = 3 x + 2 olduğuna göre 16 x ifadesinin 7 5 - 1 x olduğuna göre 11 - 1


değeri aşağıdakilerden hangisidir? =
11 + 1 5 +1
A) 36 B) 49 C) 64 D) 81 E) 111 ifadesinin x türünden eşiti, aşağıdakilerden
hangisidir?

A) 3x B) 2x C) 3x D) 5x E) x
5 3 2 2 3

2 5 -x + 2 - 2 $ 5 1 - x + 3 $ 5 -x = 90 denklemini
sağlayan x değeri aşağıdakilerden hangisidir?
A) - 2 B) - 1 C) 1 D) 2 E) 3 8 a=
3
64 $ 3 64 $ 3 64...
b= 343: 343: 343: ...
olduğuna göre a $ b değeri aşağıdakilerden
hangisidir?
^x + 1h1 - x = 1 denkleminin çözüm kümesi
2
3 A) 30 B) 42 C) 56 D) 63 E) 72
aşağıdakilerden hangisidir?

A) "- 1, 1,
B) "- 2, - 1, 0 ,
C) "- 2, 1 , 9
x, y, z sayıları sırasıyla 2, 4 ve (-3) sayılarıyla
D) "- 2, - 1, 1,
doğru orantılıdır. x + y + z = 12 olduğuna göre
x 2 + 4z - y değeri kaçtır?
E) "1, 0 ,
A) - 1 B) 0 C) 1 D) 2 E) 3

4 ]- xg-4 $ ]- xg-3
]- xg+2 $ ]- xg-1 $ ^- x -4h
işleminin sonucu kaçtır?

A) - x -5 10
420 tane badem 7, 9, 12 yaşlarındaki üç çocuğa
B) - x -4 yaşları ile doğru orantılı olarak paylaştırılıyor.
C) x 4 Bu paylaşım, 3 yıl sonra yaşları ile ters orantılı
olarak yapılsaydı büyük kardeşin alacağı badem
D) ^- xh-4
sayısı ilk duruma göre ne kadar azalırdı?
E) x2
A) 43 B) 52 C) 57 D) 68 E) 72

5 x - 5 = 2 ise x - 2 x değeri kaçtır?


x
A) 1 B) 2 C) 3 D) 4 E) 5 11 Bir izci grubundaki 80 izciye 37 gün yetecek
kadar yiyecek vardır. 7 gün sonra izcilerden 20
tanesi kamptan ayrılıyor. Kalan izcilerin her
birinin günlük yiyeceği 14 oranında artırılırsa
6 10 - 5 işleminin sonucu aşağıdakilerden kalan yiyecek izcilere kaç gün yeter?
3- 8 A) 25 B) 30 C) 32 D) 40 E) 54
hangisidir?

A) 5 B) 3 C) 2 D) 1 E) - 1

Matematik 9 | 197
DENKLEMLER VE EŞİTSİZLİKLER

Bir aracın duruş mesafesi, frene bastığı 17


Bir alışveriş merkezinde 3 döner menü alana
12
andaki hızının karesi ile doğru orantılıdır. 1 adet sinema bileti hediye edilmektedir. 1
Araç, saatte 80 km hızla giderken 40 m de adet sinema biletinin fiyatı 1 adet döner menü
durabiliyorsa saatte 120 km hızla giderken fiyatından 8 TL fazladır. Birlikte sinemaya
frene basıldığında kaç km de durabilir? giden 6 arkadaş, 6 adet döner menü alarak 2
hediye bilet kazanmıştır. Geriye kalan 4 bileti
A) 60 B) 80 C) 90 D) 100 E) 110
de sinema gişesinden satın almışlardır. Bu 6
arkadaş, toplam 132 TL harcadığına göre 1 adet
sinema biletinin fiyatı kaç TL dir?
A) ₺18 B) ₺20 C) ₺22 D) ₺24 E) ₺26
Bir merdivenin basamaklarını ikişer ikişer çıkıp
13
üçer üçer inen bir kişinin çıkarken attığı adım
sayısı, inerken attığı adım sayısından 8 fazladır.
Buna göre merdivenin basamak sayısı kaçtır?
A) 44 B) 45 C) 46 D) 47 E) 48
18
Bir annenin yaşı, üçer yıl arayla doğmuş 3
çocuğunun yaşları toplamından 6 eksiktir. En
küçük çocuk doğduğunda anne 39 yaşında
olduğuna göre en büyük çocuğun şimdiki yaşı
kaçtır?
14 Bir manavda satılan üç çeşit biberin kg satış
A) 18 B) 20 C) 21 D) 23 E) 24
fiyatının ortalaması 5 TL olup satışlardan elde
edilen kazanç sırasıyla 53 TL, 65 TL ve 70 TL dir.
Biberlerin kg satış fiyatları ve satış miktarları
birer doğal sayı olduğuna göre toplam satış
yapılan biber kaç kg dır?
A)115 B) 128 C) 90 D) 75 E) 46 Azime ve Elif’in yaşları toplamı 20 dir. Azime
19
Elif’in şimdiki yaşında iken Elif’in doğmasına 2
yıl vardı. Elif şimdi kaç yaşındadır?

Bir gezi grubundaki erkeklerin sayısı, A) 4 B) 5 C) 6 D) 7 E) 8


15
bayanların sayısının %60 ıdır. Bu grupta
bulunan erkeklerin sayısı 60 tan fazla olduğuna
göre gruptaki bayanların sayısı en az kaçtır?
A) 110 B) 101 C) 103 D) 105 E) 110
Ayşe kardeşine “Ben senin yaşındayken sene
20
2000 di. Sen babanın yaşına geldiğinde sene
2040 olacak.” diyor. Ayşe, kardeşinden 6
yaş büyük olduğuna göre Ayşe’nin babası
16
Bir benzin tankının içinde bir miktar benzin
kardeşinden kaç yaş büyüktür?
vardır. Tanka 400 litre benzin ilave edilirse
tankın 4 si doluyor. Oysa tanka benzin ilave A) 34 B) 30 C) 28 D) 25 E) 23
7
etmeyip 200 litre benzin boşaltılsaydı tankın 1
7
si dolu olarak kalacaktı. Buna göre tankın
tamamı kaç litre benzin alır?
A) 600 B) 980 C) 1240 D) 1300 E) 1400

198 | Matematik 9
DENKLEMLER VE EŞİTSİZLİKLER

ÖLÇME VE DEĞERLENDİRME -3

x sayısı, y sayısının % 80 i ise y sayısı, x sayısının 7


Bir kuru yemişçi %45 i kuru üzüm olan 20 kg
1
yüzde kaçıdır? kuru üzüm leblebi karışımı elde etmek istiyor.
Bunun için %30 ve %50 si kuru üzüm olan iki
A) 60 B) 85 C) 110 D) 125 E) 130 farklı kuru üzüm-leblebi karışımından sırasıyla
hangi miktarlarda almalıdır?
A) 8 ve 12
B) 10 ve 10
Ebru, 160 sorunun olduğu bir sınavda 85
2 C) 9 ve 11
sorudan 82 ini cevaplayabilmiştir. Tüm
soruların %70 ini cevaplayabilmesi için geriye D) 5 ve 15
kalan soruların yüzde kaçını cevaplaması E) 12 ve 8
gerekir?
A) 70 B) 60 C) 50 D) 40 E) 30 %20 si tuz olan 140 g ve %60 ı su olan 60
8
g tuz-su karışımları bir kapta karıştırılıyor.
Bir miktar para, yıllık %25 ten 1 yıllığına Oluşan karışımın %20 si buharlaştırılırsa son
3 durumdaki tuz oranının yüzdesi kaç olur?
bankaya yatırılmıştır. Aynı paranın %30 dan
aynı faizi getirmesi için kaç aylığına bankaya
A) 32,5 B) 33,5 C) 34,5 D) 35,5 E) 36,5
yatırılması gerekir?
A) 10 B) 11 C) 12 D) 13 E) 14
9
%25 tuz oranı ile su akıtan I. musluk boş bir
havuzu 8 saatte, %30 tuz oranı olan II. musluk
boş havuzu 12 saatte doldurmaktadır. Havuz
4
Bir ürün x TL ye satıldığında %50 kâr, y TL ye
boş iken iki musluk aynı anda açılırsa dolan
satıldığında %40 zarar ediliyor. Buna göre x havuzun tuz oranı % kaç olur?
y
oranı kaçtır?
A) 26 B) 27 C) 27,5 D) 28 E) 28,5
A) 3 B) 2 C) 5 D) 3 E) 7
2 2 2
Bir otomobil, A şehrinden B şehrine saatte
10
60 km hızla gitmiş ve saatte V km hızla
dönmüştür. Otomobilin gidiş dönüşteki
5
Kilogramı 22 liradan alınan bir miktar kabuklu ortalama hızı saatte 40 km olduğuna göre V
ceviz, kabuklarından temizlenirse %45 fire değeri kaçtır?
vermektedir. Ceviz içinin kilosu 50 liradan
satılırsa kâr % kaç olur? A) 20 B) 25 C) 27 D) 28 E) 30
A) 20 B) 25 C) 30 D) 35 E) 40
11

A B
6 Etiket fiyatının %25 eksiğine alınmış olan bir
mal, zam yapılarak etiket fiyatının %20 fazlasına
satıldığına göre bu satışta kâr yüzdesi kaçtır? Akıntı hızı 8 km/sa olan bir nehirde durgun
A) 45 B) 48 C) 50 D) 55 E) 60 sudaki hızı 24 km/sa olan bir kayık A
noktasından B noktasına gidiş ve dönüşünü
toplam 12 saatte tamamlıyor. Kayığın aldığı yol
kaç km dir?
A) 124 B) 188 C) 200 D) 250 E) 256

Matematik 9 | 199
DENKLEMLER VE EŞİTSİZLİKLER

12
200 m uzunluğundaki bir trenin hızı saniyede 16
Eş güçteki 2 usta ve 3 çırağın 12 saatte yaptığı
40 m dir. Bu trenin önüne geçmeyi planlayan bir işi, 1 usta ve 3 çırak birlikte 18 saatte
bir motosikletlinin trene uzaklığı 5 km bitirmektedir. Buna göre aynı işi 1 çırak kaç
olduğuna göre motorsiklet saatte kaç km hızla saatte bitirir?
giderse 12 dakika sonra trenin önüne geçer?
A) 100 B) 102 C) 105 D) 108 E) 110
A) 170 B) 180 C) 185 D) 190 E) 195

13 6AB@ çaplı dairesel pistin yarıçapı r m dir. A


40 A makinesi, B makinesinin 4 katı iş gücündedir.
17
dan 10 m/dk B den 8 m/dk hız ile iki cisim aynı İki makine birlikte bir işi 8 günde bitirebiliyor ise
anda harekete başlıyor. B makinesi aynı işi kaç günde bitirir?
A) 40 B) 42 C) 45 D) 48 E) 50

A B
18
Aynı güçteki 3 çırak bir işi birlikte 12 günde,
aynı güçteki 3 usta ise aynı işi 8 günde bitiriyor.
6 çırak ve 4 usta bu işi kaç günde bitirebilir?
Harekete başladıktan kaç dakika sonra hızlı A) 7 B) 6 C) 5 D) 4 E) 3
olan diğeriyle üçüncü kez karşılaşır?
A) 90 B) 100 C) 110 D) 120 E) 130

Şekildeki dikdörtgenin A noktasından, aynı A musluğu boş havuzu 8 saatte, B musluğu


14 19
anda belirtilen yönlerde harekete başlayan iki aynı boş havuzu 12 saatte doldurmaktadır. A
koşucunun dakikadaki hızları 12 m/dk ve 18 musluğundan birim zamanda akan su miktarı
m/dk dır. %60 arttırılıp B den akan su miktarı %20
D P C azaltılıyor. İki musluk aynı anda açılırsa boş
havuz kaç saatte dolar?
20
A) 25 B) 15 C) 3 D) 28 E) 24
9
4 4 5 5
12 m/dk

Şekilde verilen özdeş 6 musluk birlikte


A 21 B 20 açıldığında dolu havuzu 33 saatte
18 m/dk
Bu iki koşucu, P de karşılaştıklarına göre PC boşaltmaktadır.
uzunluğu kaçtır?
A) 4 B) 5 C) 6 D) 7 E) 8 E

F D

15
Bir araç; 320 km lik bir yolun yarısını 80 km/sa, A
1 lik kısmını 60 km/sa, kalan yolu 90 km/sa
8
hızla gitmiştir. Bu aracın ortalama hızı saatte Buna göre A musluğu tek başına havuzun
kaç km dir? tamamını kaç saatte boşaltır?
A) 60 B) 70 C) 80 D) 90 E) 100 A) 70 B) 72 C) 75 D) 80 E) 85

200 | Matematik 9
SAYILAR VE CEBİR
9.4. BÖLÜNEBİLME
Neler Öğreneceksiniz? Bölünebilmeyi
Öğrenmek Neden Önemlidir?

• Bölme işlemini ve özelliklerini, • Bölme işlemini ve özelliklerini öğrenmek,


• Tam sayılarda bölünebilme kurallarını ve bir işlemde eksik verilen veriler ile ilgili
bu kurallarla ilgili problemleri çözmeyi, yorumlar yapıp çıkarımlara varılmasını
sağlar.
• En az biri, sıfırdan farklı olan iki
tam sayının EBOB'unu ve EKOK’unu • Günlük hayatta bir bütünün parçalara
asal çarpanlarının kuvvetlerinden ayrılması, en çok karşılaşılan konulardan
faydalanarak bulmayı, biridir. Bütün artmayacak şekilde, bütünü
eşit parçalara ayırmak ve bunun için
• Öklid algoritmasını,
gerekli olan şartların neler olduğunu
• Elektronik tablolarda bulunan EBOB ve belirleyebilmek için bölünebilme kuralları
EKOK fonksiyonlarından yararlanmayı, bilinmelidir. Örneğin farklı türlerde
• Periyodik durum içeren problemleri malzemelerin eşit büyüklükte paketlere
çözmeyi öğreneceksiniz. ayrılmasında, bir duvarın eşit büyüklükte
kare aynalarla kaplanmasında en büyük
ortak bölene ihtiyaç duyulur.
• İki sayının ortak katlarının en
küçüğünü bulmak için çokluğun katları
alındığında bu iki sayının en az kaçta
eşitlenebileceklerinin bilinmesi gerekir.
Örneğin belli aralıklarla çalan iki zilin
aynı anda çalma zamanları bir çiçekçinin
satacağı çiçekleri üçerli, dörderli, beşerli
saydığında artan çiçek sayısına göre en
az kaç çiçeği olabileceği ortak katların en
küçüğü bulunarak hesaplanır.

Matematik 9 | 201
BÖLÜNEBİLME

9.4. BÖLÜNEBİLME KURALLARI

Etkinlik

1. Bir okuldaki öğrenciler, huzurevine yaşlıları ziyarete gideceklerdir. Öğrencilerin üç gruba


ayrılarak yaşlılarla görüşmesi planlanmaktadır. Bu gruplarla ilgili aşağıdaki sorulara uygun
cevapları veriniz.
2. İlk grup; aldığı çiçekleri yaşlılara ikişerli, üçerli veya beşerli şekilde dağıttığında sırayla 1, 2 ve 3
çiçek kaldığını görüyor. Çiçek sayısının alabileceği en küçük üç değeri bulunuz.
3. İkinci grup üyeleri, huzurevinde 100 ile 180 arasında yaşlı olduğunu ve yaşlı sayısının 7 ve 11 in
katı olduğunu söylüyor. Buna göre huzurevindeki yaşlı sayısını bulunuz.
4. Üçüncü grup üyeleri, yaşlıları gruplar hâlinde farklı günlerde ziyaret edecektir. Bu gruptaki
öğrenci sayısı; 4, 5, 6, 8 ve 9 a bölündüğünde 1 öğrenci artmaktadır. Buna göre bu grupta en az
kaç öğrenci vardır?

202 | Matematik 9
BÖLÜNEBİLME

1. Tam Sayılarda Bölünebilme


Bir tam sayının başka bir tam sayıya kalansız (tam) bölünebilmesi, o sayının bölen sayı kadar eşit parçalara
ayrıldığını göstermektedir. Bunun yanında kalanlı bölme işlemlerinde bölme işlemi yapmadan kalanın
bulunmasını sağlayan bazı kurallar da vardır. Bu kurallardan bölünen, bölüm, bölen ve kalan arasındaki ilişkiler
aşağıda gösterilmiştir.

A, B, C, K tam sayılar ve B Y
= 0 olmak üzere
A B A " bölünen,
B " bölen,
C C " bölüm,
K " kalan şeklinde gösterilir.
K
Bir bölme işleminde
1. A = B $ C + K
2. 0 # K 1 B
3. K 1 C ise bölen ile bölüm yer değiştirebilir.
4. K = 0 ise A sayısı B ye tam bölünür ve bu durum B Aşeklinde gösterilir.

ÖRNEK 1: 2a-5b b-3


Yandaki bölme işlemine göre a nın b türünden ifadesini bulunuz
4
ÇÖZÜM:
2
Bölünen = Bölen x Bölüm + Kalan eşitliğinden
2a - 5b = 4 ]b - 3g + 2 & 2a - 5b = 4b - 12 + 2
2a - 5b = 4b - 10 & 2a = 9b - 10
9b - 10
a= 2 olur.

ÖRNEK 2:
k bir doğal sayı olmak üzere
A 20
işleminde A nın alabileceği en büyük değeri bulunuz.
3k

k2
ÇÖZÜM:
k2 1 20 olması gerektiğinden k nin alabileceği en büyük doğal sayı değeri 4 olur.
A = 20 $ 3k + k 2

k = 4 için A = 20 $ 12 + 16
= 240 + 16
= 256

Matematik 9 | 203
BÖLÜNEBİLME

ÖRNEK 3:
Yandaki bölme işlemine göre X in Z türünden ifadesini yazınız X Y+1 Y Z

2 Y-3

3 1

ÇÖZÜM:
Bölme işleminin sağlamasından (2) ifadesi (1) de yerine konulduğunda
X = 2 ]Y + 1g + 3 & X = 2Y + 5 ... (1) X = 2b 1 - Z l + 5
- 3Z + 1
Y = Z ]Y - 3g + 1 & Y = ZY - 3Z + 1
- 6Z + 2 5
& Y ]1 - Zg =- 3Z + 1 X = 1-Z + 1
1-Z] g
- 3Z + 1 - 6Z + 2 + 5 - 5Z
&Y = 1-Z ... (2) X= 1-Z
- 11Z + 7
X = 1 - Z bulunur.

ÖRNEK 4:
Yanda verilen bölme işlemine göre x bir tam sayı olmak üzere T nin alabileceği farklı T x+3
değerler toplamını bulunuz.
2

ÇÖZÜM: 3x-4

3 # x 1 2 & x ! "2, 3,
_b 4 7
4 bb
Kalan $ 0 & 3x - 4 $ 0 & x $ 3
b T = ^x + 3h $ 2 + 3x - 4 = 5 $ x + 2 olur.
`
7 bbb
Kalan 1 Bölen & 3x - 4 1 x + 3 & 2x 1 7 & x 1 2 b x = 2 için: x = 3 için:
a T = 5 $ 2+ 2 T2 = 5 $ 3 + 2
1

T1 = 12 T2 = 17
T1 + T2 = 12 + 17 = 29 bulunur.

ÖRNEK 5:
2m-1 doğal sayısı n+2 doğal sayısına bölündüğünde bölüm 9, kalan 2 dir. m+n=16 olduğuna göre m.n
ifadesinin değerini bulunuz.

ÇÖZÜM:
Bölme işleminin sağlamasından
2m - 1 = ]n + 2g $ 9 + 2 2m - 9n = 21 iki bilinmeyenli denklem çözülürse

2m - 1 = 9n + 18 + 2 - 2/ m + n = 16
2m = 9n + 21 - 11n =- 11
n = 1 ve m = 15 olur. Buradan m $ n = 15 $ 1 = 15 bulunur.

204 | Matematik 9
BÖLÜNEBİLME

ÖRNEK 6:
ab0ab3 ab Verilen bölme işleminde a ve b birer rakamdır. Buna göre x ve y değerlerinin toplamını
bulunuz.
x

ÇÖZÜM:
ab0ab3 ab
ab Bölme işlemine göre x = 10 010
10 010 2 olduğundan
y=3

000ab x + y = 10 010 + 3
ab = 10 013 bulunur.
003

ÖRNEK 7:
AB A+B
AB iki basamaklı bir sayıdır. Yandaki bölme işlemine göre kaç farklı AB sayısı
yazılabileceğini bulunuz.
4
3
ÇÖZÜM:
AB = ^A + Bh $ 4 + 3 Bölme işleminin sağlamasından
10A + B = 4A + 4B + 3 3 1 A + B olması gerekir. A = 1 ve B = 1
6A = 3B + 3 için 3 1 A + B sağlamaz.
6A = 3 (B + 1)
23 _b
2A = B + 1
35 b 4 tane AB iki basamaklı sayısı
1 1 AB " `
47b yazılabilir.
59 b
2 3

3 5
a
4 7

5 9

ÖRNEK 8: 24
abc4
abc4 dört basamaklı, xy iki basamaklı doğal sayı olmak üzere xy nin
alabileceği farklı değerleri bulunuz. k

xy
ÇÖZÜM:
Bölme işleminin sağlamasından

: : 5
abc4 = 24 $ k + xy
. . e
çift sayı çift sayı xy sayısının da çift sayı olması gerekir.

5
xy 1 24
24 ten küçük çift sayılar e 22, 20, 18, 16, 14, 12, 10 olur.

Matematik 9 | 205
BÖLÜNEBİLME

Sıra Sizde

SORU: a 41
a) Yandaki bölme işleminde a ! N ve b ! R olmak üzere
a nın alabileceği en büyük değeri bulunuz. b2
b) Yukarıdaki bölme işleminde b ! N ise a nın alabileceği en büyük değeri
bulunuz. b2

ÇÖZÜM:

Bir A sayısının x ile bölümünden kalan m, bir B sayısının x ile bölümünden kalan n olmak üzere
a) A ! B nin x ile bölümüden kalan m ! n
b) A $ B nin x ile bölümünden kalan m $ n
c) t $ B nin x ile bölümünden kalan t $ n
ç) B t nin x ile bölümünden kalan n t dir.
Eğer elde edilen kalanlar x ten büyük ise kalan sayı tekrar x e bölünerek kalan bulunur.

ÖRNEK 9:
K sayısının 6 ile bölümünden kalan 4, M sayısının 6 ile bölümünden kalan 2 ise aşağıdaki ifadelerin 6 ile
bölümünden kalanları bulunuz.
a) K + M
b) K - M
c) K $ M
ç) M 3
d) 2K 2 + 3M 3

ÇÖZÜM:
K=6x+4 ve M=6y+2 olsun.
a) K + M = 6x + 4 + 6y + 2 = 6 ^x + y + 1h olduğundan K+M nin 6 ile bölümünden kalan 0 dır.

b) K nin 6 ile bölümünden kalan 4 ve M nin 6 ile bölümünden kalan 2 olduğundan K-M nin 6 ile bölümünden
kalan 4 - 2 = 2 dir.

c) K nin 6 ile bölümünden kalan 4 ve M nin 6 ile bölümünden kalan 2 olduğundan K.M nin 6 ile bölümünden
kalan 4 $ 2 = 8 bulunur. 8 sayısı, 6 dan büyük olduğundan sayı 6 ya bölünür ve kalan 2 bulunur.

ç) M nin 6 ile bölümünden kalan 2 olduğundan M 3 ün 6 ile bölümünden kalan 2 3 = 8 dir. 8 sayısı, 6 dan
büyük olduğundan 6 ya bölünür ve kalan 2 olarak bulunur.

d) K nin 6 ile bölümünden kalan 4 ve M nin 6 ile bölümünden kalan 2 olduğundan 2K 2 + 3M 3 ün kalanı
bulmak için K = 4 ve M = 2 yazılırsa 2K2 + 3M3 = 2 $ 42 + 3 $ 23 = 56 dır. 56 nın 6 ile bölümünden kalan
2 olur.

206 | Matematik 9
BÖLÜNEBİLME

ALIŞTIRMALAR

1
Yandaki bölme işlemine 2a + b b 5
x doğal sayısının 12 ile bölümünden kalan 6, y
göre b nin a türünden eşitini doğal sayısının 20 ile bölümünden kalan 7 dir.
bulunuz. 3 Buna göre x $ y nin 4 ile bölümünden kalanı
bulunuz.
2b - 1

2 2x + 3y 3y x-y+1 y 6
Bir P sayısının 13 ile bölümünden kalan, 3 ise
3P - P3 ün 13 ile bölümünden kalanı bulunuz.
7 z

4 3
Yukarıda verilen bölme işlemlerine göre z
değerini bulunuz.

3 a, b, c d N+ olmak üzere 7
Dört basamaklı bir sayının yüzler basamağı
4, onlar basamağı 2 arttırıldığında kalan
a b-3 bölme işlemine göre değişmiyor; bölüm 15 artıyor. Bölen sayının
a + b + c toplamının rakamları toplamı kaçtır?
c+4 alabileceği en küçük
değerini bulunuz
7

4
x 8 y 8
8 Yandaki bölme işlemine göre .... ..
bölünen sayının rakamları
toplamını bulunuz.
... 34
5 6 ...
x, y birer doğal sayıdır. x $ y nin 8 ile bölümünden 220
kalanı bulunuz.
7

Matematik 9 | 207
BÖLÜNEBİLME

2 ile Bölünebilme
Birler basamağında " 0, 2, 4, 6, 8 , olan sayılar 2 ile tam bölünür.
Örneğin 854, 48, 1000 sayıları 2 ile tam bölünür.

ÖRNEK 10:
Aşağıdaki sayıların 2 ile bölümünden kalanları bulunuz.
a) 123 123
b) 15! - 1
c) 1! + 2! + 3! + ... + 20!

ÇÖZÜM:

a) 123 sayısı tek sayı olduğundan bütün kuvvetleri tektir. Bu durumda kalan 1 olur.
b) 15! çift sayıdır. 15! - 1 tek sayı olduğundan kalan 1 dir.
c) 1! dışındaki diğer sayılar 2 nin katı olduğundan kalanları sıfırdır. Bu durumda kalan 1 dir.

3 ile Bölünebilme

Rakamları toplamı 3 ün katı olan sayılar 3 ile tam bölünebilir. Bir doğal sayının 3 ile bölümünden
kalan, sayının rakamları toplamının 3 ile bölümünden kalana eşittir.
Örneğin 57, 102 sayıları 3 ile tam bölünebilir.

ÖRNEK 11:
abc üç basamaklı sayısının 3 ile bölümünden kalanın bu sayının rakamları toplamına eşit olduğunu
gösteriniz.
ÇÖZÜM:
abc = 100a + 10b + c
= 99a + a + 9b + b + c
= 3 ]33a + 3bg + a + b + c abc üç basamaklı sayısının 3 ile bölümünden kalan a + b + c olur.
144444424444443 a + b + c sayısı 3 ten büyük ise yeniden 3 ile bölünmelidir.
.
3 ün tam katı

ÖRNEK 12:
5a2b sayısı, 3 ile tam bölünebildiğine göre a $ b nin alabileceği en büyük değeri bulunuz.
ÇÖZÜM:
5a2b sayısının 3 ile tam bölünebilmesi için rakamlarının toplamı 3 ün katı olmalıdır.
5 + a + 2 + b = 3k ]k ! Z g

:
7 + a + b = 3k
.
2, 5, 8, 11, 14, 17
a $ b nin en büyük değerini bulabilmek için a + b = 17 alınır. a $ b nin en büyük değeri 8 $ 9 = 72 olur.

208 | Matematik 9
BÖLÜNEBİLME

4 ile Bölünebilme
Bir sayının 4 ile tam bölünebilmesi için son iki basamağındaki sayının 4 ün katı olması gerekir.
Bir doğal sayının 4 ile bölümünden kalan, sayının son iki basamağının 4 ile bölümünden kalana eşittir.
Örneğin 20, 248, 152 sayıları 4 ile tam bölünür.

ÖRNEK 13:
abcd dört basamaklı sayısının 4 ile bölümünden kalanın cd nin 4 ile bölümünden kalana eşit olduğunu
gösteriniz.

ÇÖZÜM:
abcd dört basamaklı sayısı çözümlendiğinde
abcd = 1000a + 100b + 10c + d
= 4 ]250a + 25bg + cd
14444444244444443
.
4 ile tam bölündüğünden abcd dört basamaklı sayısının 4 ile bölümünden kalan, cd iki basamaklı sayısının
4 ile bölümünden kalana eşittir.

ÖRNEK 14:
3x5y dört basamaklı sayısı, 4 ile tam bölünebildiğine göre x + y toplamının alabileceği en küçük ve en
büyük değeri bulunuz.

ÇÖZÜM:
3x5y sayısının 4 ile tam bölünebilmesi için 5y iki basamaklı sayısının 4 ün katı olması gerekir.
52 ve 56 sayıları 4 ile tam bölündüğünden y = 2 ve y = 6 olabilir.
x in alabileceği değerler "0, 1, 2, 3, 4, 5, 6, 7, 8, 9, olur.
x + y toplamının en büyük değeri: x = 9 ve y = 6 için x + y = 9 + 6 = 15 olur.
x + y toplamının en küçük değeri: x = 0 ve y = 2 için x + y = 0 + 2 = 2 olur.

ÖRNEK 15:
Üç basamaklı x0y sayısı, 3 ile tam bölünebilmektedir. Bu sayının 4 ile bölümünden kalan 3 olduğuna göre x
in alabileceği değerler toplamını bulunuz.

ÇÖZÜM:
x0y sayısının 4 ile bölümünden kalan 3 olduğuna göre y = 3 veya y = 7 olur.
x0y sayısının 3 ile tam bölünebilmesi için rakamları toplamı 3 ün katı olmalıdır. Buna göre
y = 3 için y = 7 için
x + 0 + 3 = 3k ]k ! Zg x + 0 + 7 = 3k ]k ! Zg
x + 3 = 3k x + 7 = 3k
x ! "3, 6, 9 , x ! "2, 5, 8 ,
x in alabileceği değerler "2, 3, 5, 6, 8, 9 , olur. x in değerleri toplamı 2 + 3 + 5 + 6 + 8 + 9 = 33 bulunur.

Matematik 9 | 209
BÖLÜNEBİLME

5 ile Bölünebilme
Birler basamağındaki rakam 0 veya 5 olan sayılar 5 ile tam bölünür. Bir doğal sayının 5 ile bölümünden
kalan, birler basamağındaki rakamın 5 ile bölümünden kalana eşittir.

ÖRNEK 16:
Üç basamaklı 7ab sayısı, 4 ve 5 ile tam bölünebilmektedir. Buna göre a nın alabileceği değerler toplamını
bulunuz.
ÇÖZÜM:
7ab sayısı, 5 ile tam bölünebildiğinden birler basamağı 0 veya 5 olmalıdır. Buna göre b ! " 0, 5 , tir. 7ab
sayısı, 4 ile tam bölünebildiğinden son iki basamağındaki sayı 4 ün katı olmalıdır. Buna göre b = 0 olur.
b = 0 için a nın alabileceği değerler a ! " 0, 2, 4, 6, 8 , dir.
a nın alabileceği değerler toplamı 0 + 2 + 4 + 6 + 8 = 20 olur.

ÖRNEK 17:
2xyx dört basamaklı sayısı 3 ile tam bölünebilmektedir. Bu sayının 5 ile bölümünden kalan 2 ise x + y nin
alabileceği en büyük değeri bulunuz.
ÇÖZÜM:
2xyx sayının 5 ile bölümünden kalan 2 ise x ! " 2, 7 , olmalıdır. 2xyx sayısının 3 ile tam bölünebilmesi için
rakamları toplamı 3 ün katı olmalıdır.
2 + x + y + x = 3k
x = 2 için 2 + 4 + y = 3k x = 7 için 2 + 14 + y = 3k

6 + y = 3k 16 + y = 3k
y ! " 0, 3, 6, 9 , y ! " 2, 5, 8 ,
x + y en büyük değerini x = 7 ve y = 8 için alır. Buna göre x + y = 7 + 8 = 15 tir.

8 ile Bölünebilme
Verilen sayının son 3 basamağındaki rakamların oluşturduğu sayı, 8 in katı ise sayı 8 ile tam bölünür.
Sayının 8 ile bölümünden kalan, son 3 basamağındaki rakamların oluşturduğu sayının 8 ile bölümünden
kalana eşittir.
Örneğin 1000, 224, 25008 sayıları 8 ile tam bölünür.

ÖRNEK 18:
İki basamaklı, 8 ile bölündüğünde 2 kalanını veren kaç sayı olduğunu bulunuz.

ÇÖZÜM:
İki basamaklı, 8 ile bölündüğünde 2 kalanını veren sayılar "10, 18, 26, ..., 98, dir.
Terim Sayısı = 98 - 10 + 1
8
88 1
= 8 +
= 12
İki basamaklı, 8 ile bölündüğünde 2 kalanını veren 12 tane sayı vardır.

210 | Matematik 9
BÖLÜNEBİLME

ÖRNEK 19:
Tabloda verilen sayıların 2, 4 ve 8 ile bölümünden kalanları bulunuz. Tabloya yazınız.
Sayı 2 ile Bölümünden Kalan 4 ile Bölümünden Kalan 8 ile Bölümünden Kalan
5 000
13 007
17 083
185 132

ÇÖZÜM:
Bir sayının 2 ile bölümünden kalanı bulmak için birler basamağındaki sayıya, 4 ile bölümünden kalanı
bulmak için son iki basamağındaki sayıya ve 8 ile bölümünden kalanı bulmak için son üç basamağındaki sayıya
bakılır.
Sayı 2 ile Bölümünden Kalan 4 ile Bölümünden Kalan 8 ile Bölümünden Kalan
5 000 0 0 0
13 007 1 3 7
17 083 1 3 3
185 132 0 0 4

9 ile Bölünebilme
Bir sayının 9 ile tam bölünebilmesi için sayının rakamları toplamı 9 un katı olmalıdır.
Bir doğal sayının 9 ile bölümünden kalan, sayının rakamları toplamının 9 ile bölümünden kalana eşittir.
Örneğin 54, 27, 1206 sayıları 9 ile tam bölünür.

ÖRNEK 20:
x3x1 sayısı, 9 ile tam bölünebildiğine göre x in değerini bulunuz.

ÇÖZÜM:
x3x1 sayısının 9 ile tam bölünebilmesi için sayının rakamları toplamı 9 un katı olmalıdır.
x + 3 + x + 1 = 9k ]k ! Zg
2x + 4 = 9k olur. k = 2 için 2x + 4 = 18 & x = 7 bulunur.

ÖRNEK 21:
33 basamaklı 444...4 sayısının 9 ile bölümünden kalanı bulunuz.
ÇÖZÜM:
Sayının 9 ile bölümünden kalanı bulmak için rakamları toplamı hesaplanır.
4 + 4 + 4 + ... + 4 = 4 $ 33 = 132 dir.
1444444442444444443
33 tane

132 nin 9 ile bölümünden kalan: 1 + 3 + 2 = 6 dır.

Matematik 9 | 211
BÖLÜNEBİLME

10 ile Bölünebilme
Birler basamağındaki rakamı 0 olan sayılar 10 ile tam bölünür.
Bir sayının birler basamağındaki rakam, o sayının 10 ile bölümünden kalan sayıdır.

ÖRNEK 22:
4ab sayısı 10 ile tam bölünebilmektedir. Bu sayının 9 ile bölümünden kalan, 1 olduğuna göre a sayısını
bulunuz.
ÇÖZÜM:
4ab sayısı 10 ile tam bölündüğünden b=0 dır.
4a0 sayısının 9 ile bölümünden kalan 1 ise sayının rakamları toplamının 9 ile bölümünden kalan 1 dir.
4 + a + 0 = 9k + 1 ]k ! Zg

3 + a = 9k olur. k = 1 için 3 + a = 9 & a = 6 bulunur.

11 ile Bölünebilme
abcdef sayısının 11 ile bölümünden kalanı bulmak için sayının birler basamağından başlayarak
sırasıyla + - + - + - ... işaretleri yazılır ve aşağıdaki işlemler yapılır.
a b c d e f $ ]b + d + fg - ]a + c + eg = 11k ]k ! Zg oluyorsa sayı 11 ile tam bölünür.
- + - + - +

Sayının 11 ile bölümünden kalan, bulunan toplamın 11 ile bölümünden kalana eşittir.

ÖRNEK 23:
Aşağıdaki sayıların 11 ile bölümünden kalanları bulunuz.
a) 14 873
b) 867 501
c) aabbccdd
ÇÖZÜM:
11 ile bölünebilme kuralı uygulanırsa
a) 14873 " ]1 + 8 + 3g - ]4 + 7g = 12 - 11 = 1 dir. Kalan 1 olur.
+-+-+

b) 867501 ]6 + 5 + 1g - ]8 + 7 + 0g = 12 - 15
-+-+- + "
=- 3 tir. Kalan negatif olduğundan 11 eklenerek -3+11=8
bulunur.
c) aabbccdd ]a + b + c + dg - ]a + b + c + dg = 0 olduğundan 11 ile tam bölünür. Kalan 0 dır.
-+ - +-+- + "

212 | Matematik 9
BÖLÜNEBİLME

ÖRNEK 24:
8x042 sayısının 11 ile bölümünden kalan 2 ise x rakamını bulunuz.
ÇÖZÜM:
8x042 " ]8 + 0 + 2g - ]x + 4g = 11k + 2 ]k ! Zg
+-+ - +
6 - x = 11k + 2
4 - x = 11k
k=0 için x=4 bulunur. k nin diğer değerleri için x rakamı yoktur.

ÖRNEK 25:
Dört basamaklı a86b sayısının 11 ile bölümünden kalan 6 ise kaç farklı (a,b) ikilisi olduğunu bulunuz.
ÇÖZÜM:
a86b ]b + 8g - ]a + 6g = 11k + 6 ]k ! Zg
-+- + "
b - a + 2 = 11k + 6
b - a - 4 = 11k bulunur.
k = 0 için b - a = 4 ve k =- 1 için b - a =- 7 b_b
bb
. . . . bb
4 0"a!0 0 7 bb
bb
1 8 bb
5 1 bb
`b^ a, b h sıralı ikililerinin sayısı
2 9 b
6 2 bb
bb 5 + 3 = 8 tanedir.
7 3 bb
bb
8 4 bb
bb
9 5 bb
a

Aralarında asal iki sayıdan her birine tam bölünebilen bir sayı, bu sayıların çarpımına da tam bölünür.
6 ya tam bölünebilen bir sayı, 2 ve 3 ile tam bölünür.
12 ye tam bölünebilen bir sayı, 3 ve 4 ile tam bölünür.
15 e tam bölünebilen bir sayı, 3 ve 5 ile tam bölünür.
24 e tam bölünebilen bir sayı, 3 ve 8 ile tam bölünür.
36 ya tam bölünebilen bir sayı, 4 ve 9 ile tam bölünür.
44 e tam bölünebilen bir sayı, 4 ve 11 ile tam bölünür.

ÖRNEK 26:
Rakamları farklı 23ab sayısı, 15 ile tam bölünebildiğine göre a nın alabileceği değerleri bulunuz.
ÇÖZÜM:
Bir sayının 15 ile tam bölünebilmesi için 3 ve 5 ile tam bölünebilmesi gerekir. 23ab sayısı, 5 ile tam
bölünüyorsa b = 0 veya b = 5 olmalıdır.
b rakamı yerine yazıldığında oluşan sayıların 3 ile tam bölünebilmesi için sayının toplamı 3 ün katı olmalıdır.
23a0 için 23a5 için
2 + 3 + a + 0 = 3k 2 + 3 + a + 5 = 3k ]k ! Zg Rakamları farklı denildiğinden a nın alabileceği
a ! " 1, 4, 7 , a ! " 2, 5, 8 ,
değerler 1, 4, 7 ve 8 dir.

Matematik 9 | 213
BÖLÜNEBİLME

ÖRNEK 27:
1x0yy sayısının 12 ile bölümünden kalan 2 ise x in alabileceği değerler toplamını bulunuz.
ÇÖZÜM:
Sayının 12 ile bölümünden kalan 2 olduğundan 3 ve 4 ile bölümünden kalanlar da 2 olur. 4 ile bölümünden
kalanın 2 olması için son iki basamağındaki sayı 22 veya 66 olmalıdır.
1x022 ve 1x066 sayılarının 3 ile bölümünden kalanın 2 olması için rakamları toplamına bakılırsa
1x022 için 1x066 için
1 + x + 2 + 2 = 3k + 2 1 + x + 6 + 6 = 3k + 2 ]k ! Zg
3 + x = 3k x + 11 = 3k
x ! " 0, 3, 6, 9 , x ! " 1, 4, 7 ,
x in alabileceği değerler toplamı 0 + 1 + 3 + 4 + 6 + 7 + 9 = 30 olur.

ÖRNEK 28:
Yandaki bölme işlemine göre (a,b) sıralı ikilisinin alabileceği farklı değerleri ab0ba 55
bulunuz.

24
ÇÖZÜM:
55 ile bölünebilmeyi incelemek için 11 ve 5 ile bölünebilme incelenmelidir. Sayının 55 ile bölümünden
kalan 24 olduğundan 24 ün 5 ile bölümünden kalan 4 tür. Buna göre a = 4 veya a = 9 olur.
24 ün 11 ile bölümünden kalan 2 olacağından
ab0ba " 2a - 2b = 11k + 2 ]k ! Zg
+ -+ -+

k = 0 için a - b = 1
. .
4 3

9 8

^a, bh sıralı ikilileri ^4, 3 h ve ^9, 8 h dir.

ÖRNEK 29:
3434...343 sayısının 36 ile bölümünden kalanı bulunuz.
1444442444443
21 basamaklı

ÇÖZÜM:
Verilen sayının 4 ile bölümünden kalanı bulmak için 43 sayısı 4 e bölünür ve kalan 3 bulunur.
Verilen sayının 9 ile bölümünden kalanı bulmak için sayının rakamları toplamı 11 $ 3 + 10 $ 4 = 73
olduğundan 73 ün 9 ile bölümünden kalan 1 dir.
3434...343 sayının 36 ile bölümünden kalan 4 ile bölündüğünde 3 kalanını, 9 ile bölündüğünde 1 kalanını
veren en küçük ortak sayıdır.
4 ile bölündüğünde 3 kalanını veren sayılar " 3, 7, 11, 15, 19, 23, 27, ... , dır.
9 ile bölündüğünde 1 kalanını veren sayılar " 1, 10, 19, 28, 37, ... , olduğundan sayı 36 ile bölündüğünde
kalan 19 olur.

214 | Matematik 9
BÖLÜNEBİLME

ALIŞTIRMALAR

1
Beş basamaklı x201y sayısı, 10 ile tam 5
5abc4 sayısının 13 ile bölümünden kalan 7
bölünebilmektedir. Bu sayının 3 ile bölümünden ise 4abc5 sayısının 13 ile bölümünden kalanı
1 kalması için x in alabileceği değerleri bulunuz. bulunuz.

2
Rakamları farklı 45ab sayısı, 15 ile tam 6 2ab3 36 Bölme işlemine göre ab iki
bölündüğüne göre a + b nin en büyük değerini basamaklı en büyük sayıyı
bulunuz. bulunuz.

3
Dört basamaklı bir doğal sayının 4 ile 7
4 ile bölündüğünde 3, 3 ile bölündüğünde 1
bölümünden kalan 3, 11 ile bölümünden kalan kalanını veren en büyük 3 basamaklı doğal sayı
5 ise 44 ile bölümünden kalanı bulunuz. ile en küçük 4 basamaklı doğal sayının farkının
pozitif değerini bulunuz.

4
201ab sayısının 72 ile bölümünden kalan 28 ise 8 2x2 = y + z şartını sağlayan ve 4 ün katı olan
ab sayısını bulunuz. xyz üç basamaklı sayılarını bulunuz.

Matematik 9 | 215
BÖLÜNEBİLME

Bi̇r Tam Sayının Asal Çarpanları ve Tam Sayı Bölenleri

x, y birer tam sayı sayı olmak üzere x = k $ y koşulunu sağlayan bir k doğal sayısı bulunabiliyorsa y
sayısı x in bir çarpanıdır. “y böler x” i denir ve y ; x biçiminde gösterilir.
Örneğin 12 sayısını tam bölen doğal sayılar; 1, 2, 3, 4, 6, 12 dir. 12 sayısını tam bölen asal sayılar 2 ve
3 olur.
180 i tam bölen sayıları bulmak zaman alır. Bu nedenle bir yöntem kullanılması gerekir. Bir tam
sayının asal sayıların çarpımı biçiminde yazılmasına bu sayının asal çarpanlara ayrılmış hâli denir.
A bir tam sayı; x, y, z asal sayı ve a, b, c doğal sayı olmak üzere A tam sayısının A = x a $ y b $ z c şeklinde
ifade edilmesine asal çarpanların çarpımı ile gösterim denir. x, y, z sayıları A nın asal çarpanlarıdır.

ÖRNEK 30:
280 ve 720 sayılarını asal çarpanlarına ayırınız.
ÇÖZÜM:
280 2 720 2
140 2 360 2
70 2 280 = 23 $ 51 $ 71 180 2
35 5
90 2
7 7 720 = 2 4 $ 3 2 $ 5
45 3
1
15 3
5 5
1

ÖRNEK 31:
x ve y pozitif tam sayılardır. 120 $ x = y2 koşulunu sağlayan en küçük x ve y değerlerini bulunuz.
ÇÖZÜM:
120 sayısı asal çarpanlarına ayrıldığında
120 = 4 $ 3 $ 2 $ 5
= 23 $ 31 $ 51 dir.
120 yerine asal çarpanları yazılırsa 2 3 $ 3 1 $ 5 1 $ x = y 2 olur. y nin tam sayı olabilmesi için asal çarpanların
üslerinin 2 nin katı olması gerekir.
x in alabileceği en küçük değer x = 21 $ 31 $ 51 olacağından 2 4 $ 32 $ 52 = y2 olur. Buna göre 22 $ 31 $ 51 = y
bulunur.
x + y = 30 + 60 = 90 olur.

A bir tam sayı; x, y, z asal sayı; a, b, c doğal sayı ve A = x a $ y b $ z c olmak üzere


• A sayısının pozitif tam sayı bölenleri sayısı: ]a + 1g $ ]b + 1g $ ]c + 1g
• A sayısının negatif tam sayı bölenleri sayısı: ]a + 1g $ ]b + 1g $ ]c + 1g
• A sayısının tam sayı bölenleri sayısı: 2 $ ]a + 1g $ ]b + 1g $ ]c + 1g tanedir.
• A sayısının asal sayı bölenleri x, y, z olmak üzere 3 tanedir.
• A sayısının tam sayı bölenleri toplamı 0 dır.

216 | Matematik 9
BÖLÜNEBİLME

ÖRNEK 32:
126 sayısının pozitif tam sayı bölenlerinin sayısını ve tam sayı bölenlerinin sayısını bulunuz.
ÇÖZÜM:
126 sayısı asal çarpanlarına ayrıldığında 126 = 21 $ 32 $ 71 olur.
126 nın pozitif tam sayı bölenleri sayısı: ]1 + 1g $ ]2 + 1g $ ]1 + 1g = 2 $ 3 $ 2 = 12 ,
126 nın tam sayı bölenleri sayısı: 2 $ ]1 + 1g $ ]2 + 1g $ ]1 + 1g = 2 $ 2 $ 3 $ 2 = 24 olur.

ÖRNEK 33:
7! sayısının
a) Pozitif tam sayı bölenlerinin sayısını,
b) Tam sayı bölenlerinin toplamını,
c) Asal olmayan pozitif tam sayı bölenlerinin sayısını,
ç) Asal olmayan tam sayı bölenlerinin toplamını,
d) Çift tam sayı bölenlerinin sayısını bulunuz.
ÇÖZÜM:
7! = 7 $ 6 $ 5 $ 4 $ 3 $ 2 $ 1 = 2 4 $ 32 $ 51 $ 71 asal çarpanlarına ayrılmış hâlidir.
a) Pozitif tam sayı bölenlerinin sayısı ]4 + 1g $ ]2 + 1g $ ]1 + 1g $ ]1 + 1g = 5 $ 3 $ 2 $ 2 = 60 bulunur.
b) Pozitif tam sayı bölenlerinin toplamaya göre tersleri, negatif tam sayı bölenleri olacağından tam sayı
bölenlerinin toplamı 0 dır.
c) 7! sayısının asal olmayan pozitif tam sayı bölenlerinin sayısını bulmak için pozitif tam sayı bölenlerinin
sayısından asal sayı bölenlerinin sayısı çıkarılır. Asal bölenleri 2, 3, 5 ve 7 olmak üzere 4 tanedir. Asal
olmayan pozitif tam sayı bölenlerinin sayısı: 60 - 4 = 56 dır.
ç) 7! sayısının asal olmayan tam sayı bölenlerinin toplamını bulmak için tüm bölenler yazılır ve asal sayılar
çıkartılır.

4 Pozitif ve negatif bölenlerin toplamı sıfır olduğundan


- 1, - 2, - 3, - 4, - 5, - 6, - 7, - 8, - 10f

1, 2 3 4, 5, 6, 7 8, 10f
sayının geriye kalan bölenleri toplandığında - 2 - 3 - 5 - 7 =- 17 bulunur.
d) Çift tam sayı bölenleri 2 nin katı olacağından asal çarpanlardan bir tane 2 ayrılır. Geriye kalan asal
çarpanların bölenlerinin sayısı hesaplanır.
7! = 2 $ 23 $ 32 $ 51 $ 71
Pozitif tam sayı bölenleri sayısı: ^3 + 1h $ ^2 + 1h $ ^1 + 1h $ ^1 + 1h = 4 $ 3 $ 2 $ 2 = 48 bulunur.
Tam sayı bölenleri sayısı: 48 $ 2 = 96 olur.

Sıra Sizde

SORU:
x 1, x 2, x 3,... x n asal sayılar olmak üzere A = x1a1 $ x2a2 $ x3a3 $ f $ xnan sayısının pozitif tam sayı bölenlerinin
sayısı ]a1 + 1g $ ]a2 + 1g $ f $ ]an + 1g olduğunu gösteriniz.
ÇÖZÜM:

Matematik 9 | 217
BÖLÜNEBİLME

ÖRNEK 34:
8 4 $ 67 $ 353 sayısının pozitif tam sayı bölenlerinin sayısını bulunuz.
ÇÖZÜM:
Verilen sayı asal çarpanlara ayrılırsa 8 4 $ 6 7 $ 35 3 = ]2 3g4 $ ]2 $ 3g7 $ ]5 $ 7g3
= 2 19 $ 3 7 $ 5 3 $ 7 3 bulunur.
Pozitif tam sayı bölenleri sayısı ^19 + 1h $ ]7 + 1g $ ^3 + 1h $ ^3 + 1h = 2 560 olur.

ÖRNEK 35:
15 $ 42 x sayısının asal olmayan tam sayı bölenlerinin sayısı 140 olduğuna göre x in tam sayı değerini
bulunuz.
ÇÖZÜM:
Asal sayı bölenleri 2, 3, 5 ve 7 olmak üzere 4 tanedir. Asal olmayan tam sayı bölenlerinin sayısı 140 ve asal
sayı bölenlerinin sayısı 4 olduğundan 140 + 4 = 144 tam sayı bölenlerinin sayısıdır.
Pozitif tam sayı bölenlerinin sayısı tam sayı bölenleri sayısının yarısı olacağından 72 dir.
Sayı asal çarpanlarına ayrıldığında 15 $ 42 x = 3 $ 5 $ 2 X $ 3 X $ 7 X = 2 x $ 3 x + 1 $ 5 1 $ 7 x olur.
Pozitif tam sayı bölen sayısı: ]x + 1g $ ]x + 2g $ ]1 + 1g $ ]x + 1g = 72
]x + 1g2 $ ]x + 2g = 36
x = 2 dir.
ÖRNEK 36:
n bir tam sayı olmak üzere 3n - 240 ifadesi bir tam sayı ise n nin alabileceği kaç farklı değer olduğunu
n
bulunuz.
ÇÖZÜM:
Verilen ifade aşadaki gibi düzenlendiğinde 3n - 240 = 3n - 240
n n n
240
= 3- n

3 - 240 ifadesinin tam sayı olması için n sayısı 240 ın tam böleni olmalıdır.
n
240 asal çarpanlarına ayrıldığında: 240 = 2 4 $ 3 1 $ 5 1 olur.
240 sayısının tam sayı bölenlerinin sayısı: 2 $ ^4 + 1h $ ^1 + 1h $ ^1 + 1h = 40 bulunur.

Sıra Sizde

SORU:
605 sayısının
a) 6 nın katı olan pozitif tam sayı bölenlerinin sayısını,
b) Tam kare olan pozitif tam sayı bölenlerinin sayısını bulunuz.
ÇÖZÜM:

218 | Matematik 9
BÖLÜNEBİLME

2. Tam Sayılarda EBOB ve EKOK


En Büyük Ortak Bölen (EBOB)

İki veya daha fazla sayıyı aynı anda tam bölebilen sayılara ortak bölenler denir. 1 sayısı bütün
sayıları tam böler.
Örneğin 18 ve 24 sayılarının bölenleri incelendiğinde
18 in pozitif bölenleri: 18 " 1 , 2 , 3 , 6 , 9,18
24 ün pozitif bölenleri: 24 " 1 , 2 , 3 , 4, 6 , 8,12, 24
ve 24 ün ortak pozitif tam sayı bölenleri: 1, 2, 3, 6
18 ve 24 ü ortak bölen en büyük tam sayı 6 dır.
Birden fazla sayının ortak bölenlerinin en büyüğüne, bu sayıların en büyük ortak böleni denir. Bu
durum, EBOB ile gösterilir.
İki sayının EBOB’u EBOB ^x, y h veya ^x, yhEbob şeklinde gösterilir.
18 ve 24 ün EBOB’u aşağıdaki gibi de bulunabilir..
18 24 2*
9 12 2
9 6 2
9 3 3 * 18 ve 24 ü ortak bölen sayıların çarpımı EBOB’u
3 1 3 oluşturur. EBOB (18, 24) = 2 $ 3 = 6 olur.
1

18 = 2 $ 3 $ 3
4 18 ve 24 ün ortak asal çarpanlarının çarpımı EBOB’u verir. EBOB (18, 24) = 2 $ 3 = 6
24 = 2 $ 2 $ 2 $ 3

Sayıların EBOB değeri bulunurken sayılar asal çarpanlarına ayrılır. Ortak olan asal çarpanlardan üssü küçük
olanların çarpımı bu sayıların EBOB değerini verir.

ÖRNEK 1:

A = 23 $ 32 $ 51 sayılarının en büyük ortak bölenini bulunuz.


B = 2 4 $ 3 $ 52 $ 7
ÇÖZÜM:
İki sayının ortak asal çarpanlarının en küçük dereceli olanlarının çarpımı, bu sayıların EBOB değerini verir.
A = 23 $ 32 $ 5
4 EBOB (A, B) = 23 $ 3 $ 5 = 120 dir.
B = 2 4 $ 3 $ 52 $ 7

x, y, z, k ! Z ve EBOB ^x, yh = z olmak üzere


a) EBOB ^x, xh = x c) EBOB ^kx, kyh = kz
b) EBOB ^ky, yh = y ç) EBOB b xz , yz l = 1

Matematik 9 | 219
BÖLÜNEBİLME

ÖRNEK 2:
Toplamları 140 olan iki sayının en büyük ortak böleni 5 olduğuna göre bu sayılar arasındaki farkın en az kaç
olabileceğini bulunuz.
ÇÖZÜM:
Sayılar a ve b olmak üzere toplamları 140 ve en büyük ortak bölenleri 5 olduğundan
a + b = 140
4 a = 5x ve b = 5y ise
EBOB (a, b) = 5
5x + 5y = 140
x + y = 28

x = 15 ve y = 13 için a = 75 ve b = 65
a - b = 75 - 65 = 10
Farkın en az olabilmesi için sayılar birbirine yakın seçilmelidir.

ÖRNEK 3:
Yatılı bir okula kırtasiye yardımı yapılacaktır. Her öğrenciye aynı
sayıda kırtasiye malzemesi içeren paketler hazırlanacaktır. Her pakette
tüm malzeme çeşitlerinden olacak ve aynı malzemeden her pakette eşit
sayıda bulunacaktır. Yapılacak kırtasiye yardımı için toplam 126 silgi,
180 defter ve 324 kalem olduğuna göre en fazla kaç öğrenciye kırtasiye
yardımı yapılabilir ve bir pakette her malzemeden kaçar tane bulunur?
ÇÖZÜM:
En fazla kaç öğrenciye kırtasiye yardımı yapılacağını bulmak için 126, 180 ve 324 sayılarının EBOB’u bulunur.
126 180 324 2 )
63 90 162 2
63 45 81 3 )
21 15 27 3 )
7 5 9 3 EBOB (126, 180, 324) = 2 $ 3 $ 3 = 18 olduğundan en fazla 18 öğrenciye
kırtasiye yardımı yapılabilir.
7 5 3 3
7 5 1 5
7 1 7
1
Bir pakette her malzemeden kaç tane olduğunu bulmak için malzeme sayıları öğrenci sayısına bölünür.
126
18 = 7 silgi
180
18 = 10 defter
324
18 = 18 kalem
Bir pakette 7 silgi, 10 defter ve 18 kalem bulunur.

220 | Matematik 9
BÖLÜNEBİLME

ÖRNEK 4:
Dörtgen şeklinde bir atletizm stadyumunda koşu dalında pist yarışları
yapılmaktadır. Sporcular eşit aralıklarla yerleştirilmiş engelleri aşarak
koşuyu tamamlamaya çalışmaktadır. Pistin kenar uzunlukları 20 m, 30 m,
40 m ve 50 m dir. Köşelere de engel konulacağına göre en az sayıda engel
yerleştirilebilmesi için iki engel arasındaki mesafeyi ve engel sayısını
bulunuz.
ÇÖZÜM:
En az sayıda engel olabilmesi için engeller arası mesafenin en büyük değeri alması gerekir. Bu nedenle 20,
30, 40 ve 50 nin EBOB değeri hesaplanır.
20 30 40 50 2 )
10 15 20 25 2
5 15 10 25 2
5 15 5 25 3 EBOB(20, 30, 40, 50) = 10 olduğundan engeller arası mesafe en fazla 10 m olur.
5 5 5 25 5 )
1 1 1 5 5
1

Engel sayısını bulmak için çevre uzunluğunun iki engel arasındaki mesafeye bölünmesi gerekir.
Çevre 140
tane engel vardır.
10 = 10 = 14
ÖRNEK 5:
Uzunlukları 48, 60 ve 84 m olan türdeş tahta bloklar kesilerek birbirine eşit en büyük parçalara ayrılmak
isteniyor. Her kesim için 2 dk harcanmaktadır. Buna göre tüm blokların kesim işleminin kaç dakikada
tamamlanacağını bulunuz.
ÇÖZÜM:
48, 60 ve 84 sayılarının EBOB’unun bulunması gerekir.
48 60 84 2 )
24 30 42 2 )
12 15 21 2
6 15 21 2 EBOB ^48, 60, 84 h = 2 $ 2 $ 3 = 12
3 15 21 3 )
1 5 7 5
1 7 7
1
Kesilen parçaların uzunlukları 12 m olacağından parça adetleri ve kesim işlemi sayıları aşağıdaki gibi
bulunur.
48 _b
= 4 parça " 4 - 1 = 3 kesim bb
12 bb
60 bb
12 = 5 parça " 5 - 1 = 4 kesim `b 3 + 4 + 6 = 13
bb
84 bb
= 7 parça " 7 - 1 = 6 kesim bb
12
a
Tahta blokların parçalanması için toplam 13 kesim işlemi yapılır. Bir kesim işlemi 2 dakika sürdüğünden
2 $ 13 = 26 dakikada işlem tamamlanır.

Matematik 9 | 221
BÖLÜNEBİLME

ÖRNEK 6:
Bir bale salonunun bir duvarı hiç boşluk kalmayacak şekilde eş kare
aynalar ile kaplanacaktır. Duvarın boyutlarının 3,3 m ve 15,4 m olduğu
bilinmektedir. En az sayıda ayna kullanılabilmesi için aynaların boyutunu
ve adedini bulunuz.
ÇÖZÜM:
3, 3 m = 330 cm, 15, 4 m = 1540 cm dir.
330 ve 1540 ın EBOB’unun bulunması gerekir.

3 330 cm
14444444444444444244444444444444443
1540 cm
330 1540 2 ) _bb
bb
165 770 2 bb
bb
165 385 3 bbb
5 ) b` EBOB ^330, 1540h = 110 olduğundan kare aynanın bir kenarı 110 cm olur.
b
55 385
bb
11 77 7 bbb
b
11 11 11 ) bbb
bb
1 1
a
Duvarın alanı 330 $ 1540 Kullanılabilecek en az ayna sayısı 42 tanedir.
Bir aynanın alanı = 110 $ 110

a) a ve b aralarında asal sayılar olmak üzere EBOB ^a, b h = 1 dir.


b) Ardışık tam sayılar aralarında asaldır.
c) Ardışık tek sayılar aralarında asaldır.

ÖRNEK 7:
x ve y ardışık doğal sayılar olmak üzere x $ y = x + y + EBOB ^x, yh + 40 koşulunu sağlayan x ve y değerlerini
bulunuz.
ÇÖZÜM:
x ve y ardışık sayılar olduğundan y = x + 1 olsun. x ve y ardışık sayılar olduğundan EBOB ^x, yh = 1 dir.
x $ y = x + y + EBOB ^x, yh + 40
x $ y = x + y + 1 + 40 olur.
y yerine x + 1 yazıldığında x $ ]x + 1g = x + ]x + 1g + 41
x 2 + x = 2x + 42
x 2 - x = 42
x ]x - 1g = 42
denklemini sağlayan x doğal sayısı 7 dir. x = 7 için y = 8 bulunur.

222 | Matematik 9
BÖLÜNEBİLME

Ökli̇d Algori̇tması
Öklid algoritması iki sayının EBOB’u bulunurken kullanılan bir yöntemdir. Ardışık bölme işlemleri ile
sonuca gidilir. Özellikle büyük sayılar verildiğinde asal çarpanlara ayırarak EBOB’u bulmak çok uzayabilir.
Bunun yerine Öklid algoritması kullanılır. Öklid algoritmasında büyük sayı, küçük sayıya bölünür. Kalan
sıfır değil ise bölen kalana tekrar bölünür. Sıfır kalanı elde edilene kadar bölme işlemi tekrarlanır. Sıfır
kalanını veren bölme işleminden önceki kalan, verilen sayıların EBOB’u olur. Bu işlem aşağıdaki gibi
gösterilebilir.
a, b ! Z ve a 2 b $ 1 olsun. a sayısı, b sayısına bölünüp ardışık bölme işlemleri kalan sıfır oluncaya
kadar aşağıdaki gibi devam ettirilir.
a = t1 $ b + k1, 0 # k1 1 b
b = t2 $ k1 + k2, 0 # k2 1 k1
k1 = t3 $ k2 + k3, 0 # k3 1 k2
h
kn - 1 = t n + 1 $ kn + 0
Kalanın sıfır olduğu bölme işlemindeki bölen, a ve b nin EBOB değeridir. EBOB ^a, bh = kn olur.
Öklid algoritması yardımıyla iki sayının EBOB’unu bulmak yanında ax + by = EBOB ^a, b h = k n denklemini
sağlayan x ve y tam sayılarının genel ifadeleri de bulunabilir.

ÖRNEK 8:
196 ve 108 sayılarının en büyük ortak bölenini Öklid algoritması ile bulunuz.
ÇÖZÜM:
196 ve 108 sayılarının EBOB’unu bulmak için Öklid algoritması uygulandığında

196 108 108 88 88 20 20 8 8 4

108 1 88 1 80 4 16 2 8 2

88 20 8 4 0

EBOB ^196, 108h = 4 bulunur.

İki sayının EBOB’unu bulmak için büyük sayıdan küçük sayı çıkarılır. Elde edilen sayı ile küçük sayının
EBOB’u hesaplanır. Bu işlem, eşit sayıların EBOB’u alınana kadar devam eder. Çıkarma yöntemi aşağıdaki
gibi uygulanabilir.
x 2 y için EBOB ^x, yh = EBOB ^x - y, yh olur.

ÖRNEK 9:
27 ve 36 sayılarının en büyük ortak bölenini çıkarma yöntemi ile bulunuz.
ÇÖZÜM:
27 ve 36 sayılarının EBOB’u çıkarma yöntemi ile bulunursa
EBOB ^27, 36h = EBOB ^36 - 27, 27 h
= EBOB ^9, 27h
= EBOB ^27 - 9, 9h
= EBOB ^18, 9 h
= EBOB ^18 - 9, 9h
= EBOB ^9, 9h
= 9 olur.

Matematik 9 | 223
BÖLÜNEBİLME

a ve b sıfırdan farklı tam sayılar ise EBOB ^a, b h = ax + by olacak şekilde sonsuz sayıda ^x, y h ikilisi
vardır.

ÖRNEK 10:
21x + 27y = EBOB ^21, 27h ifadesini sağlayan ^x, yh sıralı ikililerinin genel ifadesi ile en küçük iki pozitif x
değerinin toplamını bulunuz.
ÇÖZÜM:
Öklid algoritmasına göre (2) de 6 yerine (1) ifadesi yazılırsa
27 = 21 $ 1 + 6 3 = 21 - ]27 - 21g $ 3
21 = 6 $ 3 + 3 3 = 21 - 3 $ 27 + 3 $ 21
6 = 3 $2+0 & EBOB ^21, 27h = 3 3 = 4 $ 21 + ]- 3g $ 27 bulunur.
27 = 21 $ 1 + 6 & 6 = 27 - 21 f (1) 3 = 21m + 27n ifadesindeki m değeri 4 ve n değeri - 3
21 = 6 $ 3 + 3 & 3 = 21 - 6 $ 3f (2) olur.

^x, yh sıralı ikililerinin genel ifadesini bulmak için 3 = 21m + 27n denklemi sadeleştirilirse 1 = 7m + 9n
elde edilir. Bu denkleme 7 $ 9 $ k ifadesi eklenip çıkartılırsa
1 = 7m + 9n & 1 = 7m + 9n + 7 $ 9 $ k - 7 $ 9 $ k (k ! Z olmak üzere)
1 = 7 ]m + 9kg + 9 ]n - 7kg

Buradan x = m + 9k ve y = n - 7k dir.
Buna göre x = 4 + 9k ve y =- 3 - 7k bulunur.
^x, y h = ^4 + 9k, - 3 - 7k h genel ifadesidir.
k nin alabileceği farklı değerler için farklı ^x, yh sıralı ikilileri bulunur.
k = 0 için x = 4 + 9 $ 0 & x = 4

k = 1 için x = 4 + 9 $ 1 & x = 13 olmak üzere
x in alabileceği en küçük iki pozitif tam sayı değerinin toplamı 4 + 13 = 21 bulunur.

Sıra Sizde

SORU:
1. 1035 ve 1071 sayılarının en büyük ortak bölenini Öklid algoritması ile bulunuz.
2. EBOB ^3x + 5, x + 2h ifadesinin değerini bulunuz.
3. Ardışık iki çift sayının en büyük ortak bölenini Öklid algoritması ile bulunuz.
ÇÖZÜM:

224 | Matematik 9
BÖLÜNEBİLME

En Küçük Ortak Kat (EKOK)


Ortak katların en küçüğü, sayıların katlarını alıp belli bir noktada buluşturmak istendiğinde veya
parçaları birleştirerek bir bütün elde etmek amaçlandığında kullanılır.
12 ve 18 sayılarının EKOK’unu bulmak için 12 ve 18 in katları alınır.
12 nin pozitif tam katları 12 " " 12, 24, 36, 48, 60, 72, 84, 96, 108, f ,
18 in pozitif tam katları 18 " " 18, 36, 54, 72, 90, 108,126, f ,
12 ve 18 sayılarının pozitif ortak katları 36, 72, 108, 144 ... olduğundan sayının en küçük ortak katı
36 olur.
Birden fazla sayının pozitif ortak katlarının en küçüğüne, bu sayıların en küçük ortak katı denir. EKOK
ile gösterilir.
İki sayının EKOK’u “ EKOK ^x, y h veya ^x, yhEKOK ” şeklinde gösterilir.
12 18 2
6 9 2
3 9 3
1 3 3 12 ve 18 in ortak katlarının en küçüğünü bulmak için bölenlerin hepsini çarpmak gerekir.
EKOK ^ 12, 18 h 2 2
= 2 $ 3 = 36 bulunur.
1

Sayıların EKOK değeri bulunurken sayılar asal çarpanlarına ayrılır. Ortak olan asal çarpanlardan üssü büyük
olanlarla ortak olmayan asal çarpanların çarpımı bu sayıların EKOK değerini verir.

ÖRNEK 11:
K = 20 4 $ 12 2, M = 18 3 $ 35 K ve M sayılarının EKOK ve EBOB’unu bulunuz.

ÇÖZÜM:
K ve M sayılarının EKOK’unu bulmak için sayılar asal çarpanlarına ayrılır.
K = 20 4 $ 12 2 = ]2 $ 5g $ ]2 $ 3g M = 18 3 $ 35 = ]2 $ 3 g $ 5 $ 7
2 4 2 2 2 3

= 28 $ 54 $ 24 $ 32 = 23 $ 36 $ 5 $ 7
= 2 12 $ 3 2 $ 5 4
EKOK ^K, M h = 2 12 $ 3 6 $ 5 4 $ 7 EBOB ^K, M h = 2 3 $ 3 2 $ 5 1 bulunur.
ÖRNEK 12:
İki öğrencinin bir günde çözdüğü soru sayılarının en küçük ortak katı 120 dir. Buna göre
a) Toplam çözülen soru sayısı en az kaçtır?
b) Toplam çözülen soru sayısı en çok kaçtır?
c) Birbirlerinden farklı sayıda soru çözdükleri biliniyorsa çözülen soru sayısı toplamının en büyük değerini
bulunuz.
ÇÖZÜM:
a) Çözülen soru sayısının en az olması için x ve y nin aralarında asal ve birbirine yakın sayılar olması gerekir
EKOK ^x, yh = 120 olmak üzere
x = 8 ve y = 15 için x + y = 8 + 15 = 23 bulunur.
b) Çözülen soru sayısının en çok olması için sayıların EKOK değerine eşit olması gerekir.
EKOK ^x, yh = 120 olmak üzere
x = 120 ve y = 120 için x + y = 120 + 120 = 240 bulunur.
c) Çözdükleri soru sayısı birbirinden farklı ise x ve y toplamının en büyük olması için en küçük tam sayı
katı 120 olan sayıların seçilmesi gerekir.
EKOK ^x, yh = 120 olmak üzere
x = 60 ve y = 120 için x + y = 60 + 120 = 180 bulunur.

Matematik 9 | 225
BÖLÜNEBİLME

ÖRNEK 13:
Bir akvaryumda bulunan balıklar üçerli satıldığında 1 balık, dörderli satıldığında 2 balık ve beşerli satıldığında
3 balık kalmaktadır. Buna göre
a) Akvaryumdaki balık sayısının en az kaç tane olduğunu bulunuz.
b) Balıkların sayısı 200 den fazla olsaydı akvaryumda en az kaç balık olurdu?
ÇÖZÜM:

a) Balıkların sayısı A olsun.


A = 3x + 1 = 4y + 2 = 5z + 3
A + 2 = 3 ]x + 1g = 4 ^y + 1h = 5 ]z + 1g
_
3 4 5 2 bbb
b
3 2 5 2 bbb Bölen ile kalanlar arası farklar 2 olduğundan
b
3 1 5 3 `b EKOK ^3, 4, 5h = 60 A + 2 sayısı 3, 4 ve 5 in katı olduğundan EKOK değeri
bb hesaplanır.
1 1 5 5 bb
bb
1 b
a ^ h
A + 2 = k $ EKOK 3, 4, 5
A + 2 = k $ 60
k = 1 için A + 2 = 60 & A = 58
b) Balık sayısı 200 den fazla olacağından EKOK değerinin katları alınır.
A + 2 = k $ EKOK ^3, 4, 5 h
A + 2 = 60 $ k
k = 4 için A + 2 = 240
A = 238 bulunur.

ÖRNEK 14:
Bir fabrikada üretilen gofret çeşitlerinden birinin boyutları 1 cm, 3 cm ve 7 cm dir. Bu gofretler, boşluk
bırakılmadan küp şeklindeki en küçük kutuya konmak isteniyor. Buna göre
a) Bu kutunun hacmi kaç cm 3 olabilir?
b) Bu kutuda kaç gofret vardır?

ÇÖZÜM:
a) En küçük kutunun bir kenarı gofret boyutlarının EKOK’u olur. EKOK ^1, 3, 7h = 21 dir. Kutunun hacminin
en küçük değeri 21 $ 21 $ 21 = 9 261 cm 3 olur.
b) Kutudaki gofret sayısını bulmak için kutunun hacmi, bir gofretin hacmine bölünür.
Küpün hacmi 21 $ 21 $ 21
Dik. prizma hacmi = 1 $ 3 $ 7 = 441
Kutuda 441 adet gofret bulunur.

226 | Matematik 9
BÖLÜNEBİLME

1. x, y, z, k ! Z ve EKOK ^x, yh = z olmak üzere


a) EKOK ^x, xh = x b) EKOK ^ky, yh = ky c) EKOK ^kx, kyh = kz

2. x ile y asal sayıları,


Her x, y ! Z + için EBOB ^x, yh $ EKOK ^x, yh = x $ y şeklinde ifade edilir.
3. x ve y aralarında asal tam sayılar ise EBOB değeri 1 olduğundan EKOK değeri, sayıların çarpımına
eşittir.
EKOK ^x, yh = x $ y
4. İki tam sayının EBOB değeri, bu sayılardan küçüktür ya da küçük sayıya eşittir. Bu sayıların EKOK
değeri ise bu sayılardan büyüktür ya da büyük sayıya eşittir.
Her x, y ! Z + için, x 1 y ise EBOB ^x, yh # x 1 y # EKOK ^x, yh
5. x, y, z ve t pozitif tam sayılar olmak üzere
EKOK ^x, z h
EKOK c y , t m =
x z
EBOB ^y, t h
tir.

ÖRNEK 15:
x ve y aralarında asal sayılar olmak üzere
EKOK ^x, yh = 120

40
y = x + 10 şartını sağlayan x değerini bulunuz.

ÇÖZÜM:
Sayılar aralarında asal ise EBOB ^x, yh = 1 tir.
EBOB ^x, yh $ EKOK ^x, yh = x $ y
1 $ 120 = x $ y
Sayıların çarpımı EBOB ve EKOK’ları çarpımına eşittir.
Verilen eşitlik düzenlendiğinde
40
y = x + 10
40 + 10x
y= x
xy = 40 + 10x ve x $ y = 120 için 120 = 40 + 10 $ x
80 = 10 $ x
x = 8 olur.

ÖRNEK 16:
a ve b pozitif birer tam sayıdır. EBOB ^a, bh = 14 ve EKOK ^a, bh = 420 olduğuna göre a + b nin alabileceği
en küçük değeri bulunuz.

ÇÖZÜM:
EBOB ^a, bh = 14 & a = 14x ve b = 14y olacak şekilde aralarında asal x ve y sayıları vardır.
EKOK ^14x, 14yh = 420 ve EKOK ^14x, 14yh = 14xy olduğundan 14xy = 420
xy = 30
a + b toplamının küçük olabilmesi için x ve y sayıları birbirine yakın seçilmelidir.
x = 5 ve y = 6 için a = 70 ve b = 84 olur. Buna göre a + b = 154 tir.

Matematik 9 | 227
BÖLÜNEBİLME

ÖRNEK 17:
Farklı büyüklükte olan iki çark, kırık dişleri yan yana olacak şekilde birlikte dönmeye başlıyor. Birincisi bir
turu 5/12 saatte, diğeri ise 3/7 saatte tamamlıyor. Buna göre
a) Kaç saat sonra kırık dişlerin tekrar yan yana geleceğini bulunuz.
b) Çarkların kırık dişleri tekrar yan yana gelene kadar attıkları tur sayısını bulunuz.

ÇÖZÜM:
EKOK ^3, 5h
a) 5/12 ve 3/7 sayılarının EKOK’unun bulunması gerekir. EKOK b 12 7 l = EBOB ^7, 12h = 15 saat sonra kırık
5 ,3
dişler yan yana gelir.
15 12
b) 1. dişlinin attığı tur sayısı: 5 = 15 $ 5 = 36 tir.
12
15 7
2. dişlinin attığı tur sayısı: 3 = 15 $ 3 = 35 tir.
7

ÖRNEK 18:
a ve b birbirinden farklı pozitif tam sayılar olmak üzere EBOB ^a, bh = a ve EKOK ^a, bh = b - a + 7 olduğuna
göre a + b nin alabileceği en küçük değeri bulunuz.

ÇÖZÜM:
Sayıların çarpımı EBOB ve EKOK’larının çarpımına eşittir. EBOB ^a, bh $ EKOK ^a, bh = a $ b
a $ EKOK ^a, bh = a $ b
EKOK ^a, bh = b olur.
EKOK ^a, bh = b - a + 7 ifadesinde EKOK değeri yazılırsa b = b - a + 7 & a = 7 olur.
a ve b sayıları birbirinden farklı ve EBOB ^a, bh = 7 olduğundan b nin alabileceği en küçük değer 14 tür.
Buna göre a + b = 7 + 14 = 21 olur.

Sıra Sizde

SORU:
Bir lisenin mezuniyet töreni için masa düzenlemesi yapılacaktır. Masaları süslemek için aşağıda
belirtilen adetlerde çiçek siparişi verilmiştir.
Gül Karanfil Papatya
200 240 280
Her çiçek türünden bütün masalarda eşit sayıda olması istenmektedir. Masalar 6 kişilik olduğuna göre
bu çiçeklerle en çok kaç kişilik masa ayarlanabilir?
ÇÖZÜM:

228 | Matematik 9
BÖLÜNEBİLME

ALIŞTIRMALAR

1 2 3 4 5 6 9 10 15 4
Boyutları 15 cm, 20 cm, 25 cm olan
dikdörtgen prizması şeklinde 7000 adet kutu
A bulunmaktadır. Bu kutular birleştirilerek farklı
B iki küp yapılmak isteniyor. Bunun için en az kaç
3A kutuya daha ihtiyaç olduğunu bulunuz.
A2
A+B
A$B
Yukarıdaki tabloda satırlarda bulunan sayıların
5 3030 ve 4220 sayılarının ortak tam sayı
sütundaki sayılarla tam bölündüğünü
bölenlerinin sayısını bulunuz.
göstermek için “ ” işareti kullanılmıştır. Siz de
tabloyu benzer şekilde doldurunuz.

2 Zeytin Türleri Zeytin Miktarı Yağ Oranı %


6
a ve b doğal sayı olmak üzere
A 300 kg 30
ab = 1616 ifadesine göre kaç farklı ^a, bh sıralı
B 200 kg 25 ikilisi yazılabilir?
C 350 kg 20
D 150 kg 40
Yukarıdaki tabloda bir işletmenin farklı
türdeki zeytin miktarları ve zeytinyağı oranları
verilmiştir. Zeytinyağları türlerine göre 7
180 kişinin katıldığı bir panele en az kaç kişi
birbirine karışmayacak şekilde şişelenecektir. daha katılmalıdır ki katılımcılar sekizerli ve
Buna göre onarlı gruplara ayrılabilir hâle gelsin?
a) İşletmenin toplam zeytinyağı üretim
miktarını bulunuz.
b) Zeytinyağı şişeleri en fazla kaç litrelik
olabilir?
c) Zeytinyağları için en az kaç şişe gerektiğini
bulunuz.

8
Bir fabrikada bir saatte, dört farklı makinede
ve farklı kalitede üretilen kağıt havlu adetleri
aşağıdaki tabloda verilmiştir.

Makine A B C D
Boyutları 25 cm ve 160 cm olan dikdörtgen Üretim
3 48 36 54 60
şeklindeki minderler kullanılarak kare Miktarı
biçiminde bir oyun alanı oluşturulacaktır. Oyun
Farklı kalitedeki havlular, ayrı paketlerde ve
alanı için en az kaç minder kullanılacağını
eşit sayıda olacak şekilde paketlenecektir.
bulunuz.
Buna göre
a) Bir pakette en çok kaç tane kağıt havlu
olabileceğini bulunuz.
b) Paket sayısı en az kaç tane olabilir?

Matematik 9 | 229
BÖLÜNEBİLME

ÖLÇME VE DEĞERLENDİRME

1
Ardışık 3 çift doğal sayının çarpımı, Bir bölme işleminde bölen, bölümün 7 katı ve
4
aşağıdakilerden hangisine daima bölünmez? kalan 22 ise bölünen doğal sayı en az kaçtır?
A) 4 B) 6 C) 12 D) 16 E) 18 A) 29 B) 50 C) 85 D) 134 E) 147

2
Bölme işlemine göre bölüm ab0ab7 ab 2x+y x-3 y z+2
5
ile kalanın toplamı kaçtır?
y-8 2
A) 117 B) 1107 C) 10017
4 2
D) 10017 E) 10117 Verilen bölme işlemlerine göre z nin x
türünden ifadesi, aşağıdakilerden hangisidir?
4x - 4 2x - 2 - 2x + 2
A) 8 - 2x B) x - 4 C) - x - 6
x-6 -x - 1
D) 2x - 8 E) 2x - 6

x = 4322 xyz7 dört basamaklı sayısının 18 ile


3 4 ise x $ y + xy - x + y 6
bölümünden oluşabilecek iki basamaklı
y = 7328
kalanların toplamı kaçtır?
ifadesinin 11 ile bölümünden kalan kaçtır?
A) 25 B) 52 C) 56 D) 81 E) 108
A) 8 B) 7 C) 5 D) 3 E) 2

230 | Matematik 9
BÖLÜNEBİLME

7
Verilen bölme işleminde 67.... 3x 362 + 452 - 272 - 4 $ 3 4 sayısının asal olmayan
10
3x iki basamaklı bir pozitif tam sayı bölen sayısı kaçtır?
sayıdır. Buna göre x 2
aşağıdakilerden hangisi A) 15 B) 18 C) 24
olamaz? D) 27 E) 30
A) 0 B) 1 C) 2 D) 3 E) 4

8
Dört basamaklı abcd sayısının 13 ile Rakamları çarpımı 32 olan en büyük 4
11
bölümünden kalan 5 tir. Bu sayının birler ve basamaklı doğal sayının 11 ile bölümünden
yüzler basamağındaki rakamlar 1 azaltılıp kalan kaçtır?
onlar ve binler basamağındaki rakamlar
2 arttırılıyor. Oluşan yeni sayının 13 ile A) 8 B) 7 C) 6 D) 5 E) 4
bölümünden kalan kaçtır?
A) 0 B) 2 C) 3 D) 7 E) 8

a bir doğal sayı olmak a 41 12


Üç basamaklı, rakamları farklı bir doğal
9
üzere a nın alabileceği en sayının 5 ile bölümünden kalan 3 tür. Bu
büyük değer kaçtır? b2 + 4 sayı 36 ile tam bölünebildiğine göre yüzler
b2 basamağındaki rakam en çok kaç olabilir?

A) 1453 B) 1516 C) 1681 A) 5 B) 6 C) 7 D) 8 E) 9


D) 1724 E) 1844

Matematik 9 | 231
BÖLÜNEBİLME

13
7 ile bölünebilen üç basamaklı kaç tane çift
16 3 4 5 11
sayı vardır?
3AB4
A) 64 B) 65 C) 66 D) 67 E) 68 B10A

Tabloda satırda bulunan dört basamaklı


sayıların 3, 4, 5 ve 11 sayılarından hangileri ile
tam bölündükleri işaretlenmiştir. Buna göre B
sayısı kaçtır?
A) 0 B) 1 C) 4 D) 6 E) 8

14 x = 2y olmak üzere beş basamaklı x613y 17


168 ile en küçük hangi doğal sayı çarpılırsa
sayısı 2 ile tam bölünebilmektedir. Bu sayının 3 sonuç, bir doğal sayının tam karesi olur?
ile bölümünden kalan 1 ise x değerlerinin
A) 42 B) 56 C) 72 D) 84 E) 168
toplamı kaçtır?
A) 8 B) 10 C) 12 D) 14 E) 16

15
Bir mimar tanesini 45 TL den aldığı lambalar 18
1500...0 sayısının asal olmayan tam sayı
için ödeme yapıyor. Faturada toplam tutarın bölenlerinin sayısı 285 olduğuna göre bu
1 5 TL şeklinde silik çıktığını görüyor. Tek sayının sondan kaç basamağı sıfırdır?
sayıda lamba alındığı bilindiğine göre silik
çıkan rakamların çarpımı kaçtır? A) 5 B) 6 C) 7 D) 8 E) 9

A) 0 B) 15 C) 25 D) 35 E) 45

232 | Matematik 9
BÖLÜNEBİLME

19
x ve y tam sayı olmak üzere 22
Bir lise, öğrencilerinin sosyal faaliyetlerdeki
y = 2xx++152
1 eşitliğini sağlayan kaç farklı x fotoğraflarından oluşacak kare bir pano
değeri vardır? yaptıracaktır. Bir fotoğrafın boyutu 10 cm ve 15
cm olduğuna göre panonun alanı en az kaç
A) 8 B) 12 C) 18 D) 24 E) 30 cm2 olabilir.
A) 500 B) 600 C) 700 D) 800 E) 900

3 3 5 4x + 3y sayısı 5 ile tam bölünebiliyorsa


20 8 $ 9 $ 125 sayısının kaç tane tam kare olan 23
pozitif tam sayı böleni vardır? aşağıdakilerden hangisi 5 ile daima tam
bölünebilir?
A) 64 B) 160 C) 320 D) 480 E) 810
A) 2x + 3y B) x + 2y C) x - 2y
D) x - y E) x + y

21 A = "0, 1, 2, ..., 20 , kümesinin 2 elemanlı alt 24


Rakamları farklı 31x5y sayısı 12 ve 18 ile tam
kümelerinin kaç tanesinde elemanlar toplamı 3 bölünebilmektedir. Buna göre x in alabileceği
ün katıdır? değer kaçtır?

A) 49 B) 61 C) 70 D) 82 E) 94 A) 7 B) 6 C) 5 D) 4 E) 3

Matematik 9 | 233
BÖLÜNEBİLME

Bir pasta atölyesinde boyutları 24 cm, 42 cm 27 EBOB^a,28,36h = 4


25
ve 54 cm olan dikdörtgen prizması şeklinde EKOK ^a,28,36h = 1260
kek üretilmektedir. Bu kek küp biçiminde koşulunu sağlayan a nın alabileceği kaç doğal
eşit parçalara ayrılarak şeker hamuru ile sayı değeri vardır?
kaplanacaktır. Kaplama için az miktarda şeker
hamuru kullanılmak isteniyor. Buna göre A) 6 B) 8 C) 12 D) 15 E) 16
oluşacak küp şeklinde pasta sayısı kaçtır?
A) 240 B) 252 C) 264 D) 275 E) 286

26
Bir sempozyumda 3 farklı oturum şekli 28 A = a3 $ b3 $ c verilenlere göre EKOK ^A,Bh = ?
planlanmıştır.
B = a2 $ b 4 $ c3 EBOB^A,Bh
Oturum Çalışma Dinlenme
A) a $ b 2 $ c B) a $ b $ c 2 C) a 2 $ b $ c
A 20 dk 10 dk
D) a 2 $ b 2 $ c E) a $ b $ c
B 50 dk 10 dk
C 80 dk 10 dk
09.00 da başlayan sempozyumda 12.00 den
sonra öğle yemeği verilecektir. Tüm grupların
aynı anda katılabileceği öğle yemeği en erken
saat kaçta başlayabilir?
A) 12.00 B) 12.10 C) 13.20
D) 13.30 E)13.40

29
n bir tam sayı olmak üzere
EBOB^4n + 7,n - 3h en çok kaç olabilir?
A) 1 B) 4 C) 13 D) 16 E) 19

234 | Matematik 9
GEOMETRİ
9.5. ÜÇGENLER
Neler Öğreneceksiniz? Üçgenleri
Öğrenmek Neden Önemlidir?

• Üçgende açı özelliklerini kullanarak • Açılar ve üçgenler, geometrinin en temel


uygulamalar yapabilmeyi, konularıdır. Karşılaşılan şekillerin çevre,
• Üçgenin kenar uzunlukları ile bu kenarları yükseklik, açıortay, kenarortay ve alanını
gören açılar arasındaki ilişkilendirmeyi, bulmak gerekebilir. Mesela inşaatçılık,
haritacılık, mobilyacılık, arazi ölçümü ve
• Üçgen eşitsizliğini, süslemecilik gibi alanlarda açıların ve
• Üçgenin iç ve dış açıortayları ile üçgenlerin bu özellikleri ve hesaplamaları
bunların özelliklerini, iç ve dış açıortay kullanılmaktadır.
uzunluklarını hesaplayabilmeyi, • Üçgenler konusu ne kadar iyi öğrenilip
• Üçgende kenarortay ve özellikleri ile uygulanabilirse dörtgenler, beşgenler
kenarortay uzunluğunu hesaplayabilmeyi, vb. konuların öğrenilmesi de o ölçüde
• Üçgende kenarortayı, kolaylaşır. Bu da günlük hayatta
• Farklı üçgen çeşitlerine göre kenarlara ait karşılaşılan karmaşık şekillerin üçgenlere
yükseklikleri çizebilmeyi, bölünerek daha iyi anlaşılmasını sağlar.
• İki üçgenin eş olması için gerekli asgari • Ayrıca günlük hayatta karşılaşılan
koşullar ile üçgende eşlik kurallarını, hesaplaması zor bazı uzunlukların
(örneğin bir ağacın boyu, bir binanın ya
• İki üçgenin benzer olması için gerekli
da dağın yüksekliği, bir deniz aracının
asgari koşullar ile üçgendeki temel
kıyıya uzaklığı, çok uzak mesafedeki
benzerlik kurallarını (Menelaus, Seva,
uzunluklar vb.) bulunmasında geometri
Stewart, Carnot ve Thales teoremleri),
teoremlerinden yararlanılır.
• Dik üçgendeki Pisagor ve Öklid
• Geometri, bazı matematik problemlerine
teoremleri, bu teoremlerin uygulamaları
farklı açılardan bakılmasını sağlayarak
ile dar açıların trigonometrik oranlarını,
yeni çözüm yöntemlerinin geliştirilmesine
• Yükseklikleri aynı olan üçgenlerin alanları olanak sağlar.
ile tabanları arasındaki ilişkiyi,
• Benzer üçgenlerin alanları ile benzerlik
oranı arasındaki ilişkiyi öğreneceksiniz.

Matematik 9 | 235
ÜÇGENLER

(Mustafa Kemal Atatürk 1881-1938)

İ nsanoğlu, çok eski çağlardan bugüne kadar geometriye ve geometrik


hesaplamalara ihtiyaç duymuştur.
Günlük yaşantı ihtiyaçları geometriye gereksinim duyulmasının ana
sebebidir. Örneğin Eski Mısır’da Nil Nehri’nin taşması sonucu kaybolan
arazilerin ölçümlerinde ve piramitlerin yapımında geometri bilimi
kullanılmıştır.
Sümerler, Babiller ve Akadlara ait kil tabletlerde de geometrik
hesaplamalara rastlanmıştır. Yunan Matematikçi Thales (Tales)
piramitlerin ve dağların yüksekliklerini, nehirlerin genişliklerini ve
denizdeki bir geminin kıyıya olan uzaklığını benzerlik teoremlerini
kullanarak hesaplamıştır. Ayrıca Yunan matematikçilerden Pisagor,
Euclides (Öklid), Apollonius (Apolyonus) ve Archimedes (Arşimet) de
geometrinin gelişiminde katkıları olan önemli bilim insanlarındandır.
Yunan matematikçilerin geometri bilimi için ortaya attığı bu teoremler;
Arap ve Türk matematikçileri Harezmî, Ömer Hayyam, Sabit Bin Kurra,
Beyrûni ve Nasîrûddin Tûsî’nin yazdıkları şerhler ve ortaya koydukları
görüşler neticesinde gelişerek yeni boyutlar kazanmıştır.
Tûsî’nin Öklid’in elementleri için yazdığı “Tahrîru Öklidis ( Tahrirü’l
usûl)” adlı eser, asırlarca kendi alanında en önemli çalışmalardan biri
olarak ün yapmıştır. Tûsî, Öklid dışındaki geometri çalışmalarının ortaya
çıkmasında öncülük etmiştir.
Sabit Bin Kurre, iki geometrik problemin cebir yöntemiyle çözülmesini
sağlamıştır. Bu açıdan matematik tarihinde cebrin geometrik problemlere
uygulanışını açık şekilde gösteren ilk bilim insanı Sabit Bin Kurre’dir.
Ebu’l Vefa, pergelin bir tek açıklığıyla daire içine kare çizimini ve
verilen bir kare içine eşkenar üçgen çizimini ilk kez yapan bilim insanıdır.
Cahit Arf ise cebir konusundaki çalışmaları ile ün kazanmıştır. Sentetik
geometri problemleri ile uğraşmıştır. Arf değişmezi, Arf halkaları, Arf
kapanışı gibi çalışmaları yanında “Hasse-Arf Teoremi”ni de ispatlamıştır.
Matematiğe yapmış olduğu köklü katkılardan dolayı 1974’te de TÜBİTAK
Bilim Ödülü’ne layık görülmüştür.
Atatürk’ün iyi bir asker, politikacı ve vatansever olmasının yanında
matematik bilimine de katkıları olmuştur. Atatürk'ün hazırladığı geometri
kitabı 1937 yılında Kültür Bakanlığınca kılavuz kitap olarak yayımlanmıştır.
Bu geometri kitabında yer alan pek çok terim ilk kez kullanılmıştır. Sivas
Lisesinde bizzat kendisi geometri dersi anlatmıştır. Türkçeye kazandırdığı
terimlerden bazıları aşağıda örnek olarak verilmiştir.

Hendese Geometri Murabba Kare


Zaviye Açı Muhit-i daire Çember
Müselles Üçgen Kutur Çap
Mustatil Dikdörtgen Mesaha-ı sathiyye Alan

236 | Matematik 9
ÜÇGENLER

9.5.1 ÜÇGENLERDE TEMEL KAVRAMLAR


1. Üçgende Açı Özellikleri
Düzlemde Açılar
Düzlemde başlangıç noktaları ortak olan iki ışının birleşiminin oluşturduğu açıklığa açı denir. Buradaki
ışınlara açının kolları, ortak noktaya da açının köşesi adı verilir. Açılar adlandırılırken açının köşe ve
kolları üzerindeki noktalar kullanılır.
% %
Örneğin şekildeki açı AOB, BOA veya O W şeklinde adlandırılır.
5OA ve 5OB ışınlarına açının kolları, O noktasına açının köşesi denir.

Açının Ölçüsü
Tam çember yayının (çevresinin) 360 eş parçasından birini gören
merkez açının ölçüsüne 1 derecelik açı denir ve 1c ile gösterilir.

AB yayı, çemberin çevresinin 360 ta biri ise m^AOBh = 1c olur.


%

Açı Çeşitleri

• Dar Açı
Ölçüsü 0c den büyük, 90c den küçük olan açılardır.

• Dik Açı
Ölçüsü 90c olan açılardır.

• Geniş Açı
Ölçüsü 90c den büyük, 180c den küçük olan açılardır.
90c 1 m^AOBh 1 180c
%

• Doğru Açı
Ölçüsü 180c olan açılardır.

Matematik 9 | 237
ÜÇGENLER

• Tam Açı
O
Ölçüsü 360c olan açılardır.
m^AOBh = 360c
%

• Tümler Açılar
Ölçüleri toplamı 90c olan açılardır.
5OA = 5OC ve m^AOB %
h = x, m^BOC h = y ise x + y = 90c tir. Buna
%
göre x ile y birbirinin tümleridir.

• Bütünler Açılar
Ölçüleri toplamı 180c olan açılardır. A, O ve C noktaları doğrusal
olmak üzere
m^AOBh = x, m^BOC h = y ise x + y = 180c olur.
% %
x ile y birbirinin bütünleridir.

ÖRNEK 1:
Tümler iki açının farkı 20c olduğuna göre büyük açının ölçüsünü bulunuz.

ÇÖZÜM:
Tümler açılar a ve b olsun. O zaman
a + b = 90c
3

+ a - b = 20c
2a = 110c & a = 55c olur. 55c - b = 20c & b = 35c bulunur.
Büyük açının ölçüsü 55c olur.

ÖRNEK 2:
Bir açının tümleri ile bütünlerinin toplamı 130c olduğuna göre bu açının ölçüsünü bulunuz.

ÇÖZÜM:

Açı a ise tümleri 90c - a , bütünleri 180c - a olur.


Buradan
]90c - ag + ]180c - ag = 130c yazılır.
270c - 2a = 130c
270c - 130c = 2a
140c = 2a ise a = 70c bulunur.

238 | Matematik 9
ÜÇGENLER

ÖRNEK 3:
Bir açının tümlerinin bütünleri, bu açının bütünlerinden 20c fazla olduğuna göre bu açının ölçüsünü
bulunuz.

ÇÖZÜM:
Açı a ise tümleri 90c - a , bütünleri 180c - a , tümlerinin bütünleri 180c - ]90c - ag olur.
Buradan 180c - ]90c - ag = 180c - a + 20c yazılır.
a - 90c = 20c - a ise 2a = 110c & a = 55c bulunur.

• Komşu Açılar
İç bölgeleri ayrık ve birer ışını ortak olan açılara komşu açılar denir.
AOB açısının kolları: 5OA ve 5OB
BOC açısının kolları: 5OB ve 6OC olup 5OB ışını ortaktır.
% %
Bu durumda AOB ve BOC komşu açılardır.

ÖRNEK 4:
Komşu tümler iki açıdan birinin ölçüsü, diğerinin ölçüsünün iki katı olduğuna göre büyük açının ölçüsünü
bulunuz.

ÇÖZÜM:
Hem komşu hem de tümler olan açılara komşu tümler açılar denir. Şekilde,
% %
AOB ile BOC komşu açılar ve
m^AOBh + m^BOC h = 90c olduğundan komşu tümler açılardır.
% %

m^AOBh = 2x alınırsa m^BOC h = x olur.


% %

m^AOBh + m^BOC h = 2x + x = 90c


% %

3x = 90c & x = 30c


m^AOBh = 2 $ 30c = 60c bulunur.
%

ÖRNEK 5:

Şekilde A, O ve D noktaları doğrusaldır. x, y ve z açıları sırasıyla 2, 3 ve 5


sayıları ile doğru orantılı olduğuna göre x açısının tümlerinin ölçüsünü
bulunuz.

ÇÖZÜM:
x, y ve z açıları sırasıyla 2, 3 ve 5 sayıları ile doğru orantılı ise
x = y = z = k dir. x = 2k, y = 3k, z = 5k ve x + y + z = 180c olduğundan
2 3 5
2k + 3k + 5k = 180c
10k = 180c & k = 18c bulunur.
x = 2k olduğundan x = 2 $ 18c = 36c olur. Tümleri ise 90c - 36c = 54c bulunur.

Matematik 9 | 239
ÜÇGENLER

Açıortay

Bir açıyı iki eş açıya bölen ışına açıortay denir.


Şekilde 5OB ışını, AOC açısını iki eş açıya böldüğünden açıortaydır.
m^AOBh = m^BOC h
% %
Bir açının açıortayı üzerinde alınan herhangi bir noktadan açının
kollarına çizilen dik uzunluklar birbirine eşittir.
Şekilde AB = BC ve OA = OC tir.

ÖRNEK 6:

Şekilde A, O ve E noktaları doğrusal ve 5OB ve 5OD açıortay olduğuna


göre BOD açısının ölçüsünü bulunuz.

ÇÖZÜM:
AOC ve COE komşu bütünler açılardır.
5OB açıortay ise m^AOB
%
h = m^BOC h = x ve 5OD açıortay ise m^COD h = m^EOD h = y alınırsa
% % %
x + x + y + y = 180c yazılır. Buradan
%
x + y = 90c = m_BODi olur.

Komşu bütünler iki açının açıortayları arasındaki açı 90c dir.

ÖRNEK 7:

Şekilde m^AOBh = 80c dir. Buna göre BOC ve AOC açılarının açıortayları
%
arasındaki açıyı bulunuz.

ÇÖZÜM:

5OE, BOC
%
nın ve 5OD, AOC nın açıortayı olsun. O zaman m^BOC h = 2a
% %
seçilirse
m^AOC h = 80c + 2a, m^AOD h = 40c + a ve m^BOD h = 40c - a olur.
% % %

Buradan BOC ve AOC açılarının açıortayı arasındaki DOE açısının ölçüsü


m^DOE h = 40c - a + a = 40c bulunur.
%

240 | Matematik 9
ÜÇGENLER

Paralel İki Doğrunun Bir Kesenle Yaptığı Açılar

d1 ' d2 ve d3 doğrusu bu doğruların kesenidir.

Kesişen iki doğrunun oluşturduğu a ile c


Ters Açılar açılardan komşu olmayanlarına 3 ters açılar olup ölçüleri eşittir.
ters açılar denir. b ile d

Birer ışınları paralel zıt yönlü, diğer a ile p


İç Ters Açılar ışınları ortak olan zıt yönlü açılara 3 iç ters açılar olup ölçüleri eşittir.
iç ters açılar denir. d ile n

Birer ışınları aynı doğru üzerinde c ile m


Dış Ters Açılar zıt yönlü, diğer ışınları farklı doğru 3 dış ters açılar olup ölçüleri eşittir.
üzerinde paralel olan zıt yönlü b ile r
açılara dış ters açılar denir.
n ile b _b
Birer ışınları aynı doğru üzerinde
aynı yönlü, diğer ışınları farklı m ile ab
Yöndeş Açılar doğru üzerinde paralel olan aynı ` yöndeş açılar olup ölçüleri eşittir.
p ile c b
yönlü açılara yöndeş açılar denir.
r ile d b
a
n ile a karşı durumlu açılardır. Ölçüleri
toplamı
Birer ışınları aynı doğru üzerinde
180c dir. n + a = 180c
Karşı Durumlu Açılar zıt yönlü, diğer ışınları farklı doğru
üzerinde paralel olan aynı yönlü d ile p karşı durumlu açılardır. Ölçüleri
açılara karşı durumlu açılar denir. toplamı
180c dir. d + p = 180c

ÖRNEK 8:

d1 + d2 + d3 = !O +
m^BOC h = 5x
%

m^DOE h = 2x
%

m^AOF h = 3x
%

Şekilde verilenlere göre x açısının değerini bulunuz.

ÇÖZÜM:
% % % % % %
Şekle göre BOC ile EOF , DOE ile AOB ve AOF ile COD ters açılardır. Ters açıların ölçüleri eşit
olduğundan
m^BOC h = m^EOF h = 5x dir. AOD
% % % doğru açı olup ölçüsü 180c olduğundan

m^AOD h = m^AOF h + m^EOF h + m^DOE h = 180c tir.


% % % %

3x + 5x + 2x = 180c

10x = 180c & x = 18c bulunur.

Matematik 9 | 241
ÜÇGENLER

ÖRNEK 9:

Şekilde
d1 ' d2
m^RCNh = 3x + y
%

m^PBNh = 60c
%

m^MAL h = 2x + y
%

m^MDK h = 50c
%
Yukarıdaki değerlere göre x + y toplamını bulunuz.

ÇÖZÜM:
% % % %
RCN ile PBN ve MDK ile MAL açıları yöndeş olduğundan bu açıların ölçüleri eşittir.
m^RCNh = m^PBNh & 3x + y = 60c ... ]1g
% %

m^MDK h = m^MAL h & 2x + y = 50c ... ]2g


% %
3x + y = 60c
4 sistemi çözülürse
2x + y = 50c
x = 10c, y = 30c bulunur.
Bu durumda x + y = 10c + 30c = 40c olur.

ÖRNEK 10:

Şekilde
d 1 ' d 2 ve d 3 ' d 4
m^ABC h = 2x + 30c
%

m^ADE h = 5x + 10c tir.


%

Yukarıda verilenlere göre x değerini bulunuz.

ÇÖZÜM:
Şekilde verilenlere göre

m^ADE h = m^BCE h = 5x + 10c (Yöndeş açılar)
% %

m^ABC h + m^BCE h = 180c (Karşı durumlu açılar)


% %

2x + 30c + 5x + 10c = 180c


7x + 40c = 180c
7x = 140c
x = 20c bulunur.

242 | Matematik 9
ÜÇGENLER

Sonuçlar

d1 ' d2 & y = x + z

d1 ' d2 & Sağa bakan açılar: x, y, z


Sola bakan açılar: a, b
Sağa bakan açıların toplamı, sola bakan açıların toplamına eşittir. Bu
durumda
x + y + z = a + b olur.

x+y = a+b+c+d

d1 ' d2 & x + y + z = 360c

d 1 ' d 2 ise a1 + a2 + a3 + ... + an = ]n - 1g $ 180c

a + b + c + d + e = 180c

Matematik 9 | 243
ÜÇGENLER

ÖRNEK 11:

Şekilde 6KT@, 6MS@ açıortay ve d 1 ' d 2 dir.


m^KTM h = 75c, m^KSM h = 60c olduğuna göre
% %

m^KRM h nın değerini bulunuz.


%

ÇÖZÜM:

5KT ve 5MS açıortay olduğundan m^%


SKL h = 2a ve m^NMT h = 2b yazılır.
%
a
a
Ayrıca 75c = a + 2b ve
60c = b + 2a tir.
Taraf tarafa toplama yapılırsa 135c = 3 $ ]a + bg
a + b = 45c bulunur.
m^KRM h = 2a + 2b = 2 $ ]a + bg olduğundan m^KRM h = 2 $ 45c = 90c
b % %
b
olur.

ÖRNEK 12:

AB ' DC
5KL? ve 5ML? açıortay
m^KLM h = 35c
%

m^MND h = 50c
%

m^ABK h = x
%

m^NDE h = y olduğuna göre x - y farkının kaç derece olduğunu


%
bulunuz.

ÇÖZÜM:

AB ' DC olduğundan sağa bakan açılar toplamı, sola bakan açılar


toplamına eşittir. m^BKM h = 2a, m^KMNh = 2b alınırsa
a % %
x + 35c + 50c = a + b + yf ]1g yazılır.
a

b & de + + c =
KLM a b 35 180c eşitliğinden
a + b = 180c - 35c = 145cf ]2g bulunur.
b

(2), (1) de yerine yazıldığında x + 35c + 50c = 145c + y


x - y = 145c - 85c = 60c olur.

244 | Matematik 9
ÜÇGENLER

ÖRNEK 13:

Şekilde 5AK ' 5DL ' 5EM ve 5CD? açıortaydır.


%h = 130c, m ^%h = 100c, m ^%h = 110c olduğuna göre
m ^KAB ABC CDL
^ % h
m CEM = x açısının kaç derece olduğunu bulunuz.

ÇÖZÜM:

C köşesinden 5DL ' CN olacak şekilde CN doğrusu çizilir ve m^ECNh = a


%
5CD? açıortay olduğundan m ^% ECB h = 2y alındığında
130c + 100c + y + y + a = 360c ]) g yazılır.
m ^%
LDC h + m ^NCD h = 180c (Karşı durumlu açılar)
%
a 110c + y + a = 180c & a + y = 70c bulunur.

Bu değer ]) g eşitliğinde yerine yazıldığında


130c + 100c + y + 70c = 360c & y = 60c bulunur. Bu durumda a = 10c olur.
x + a = 180c (karşı durumlu açılar)
x + 10c = 180c & x = 170c olarak bulunur.

ÖRNEK 14:

Şekilde 5DC? ' 5FE?


m^BAHh = 70c, m^ABC h = 10c
% %

m^EFG h = 30c, m^GHA h = 40c


% %

olduğuna göre m^BCD h = x açısının değerini bulunuz.


%

ÇÖZÜM:

DEF h = m ^EDC h = a olur.


5DC? ' 5FE? & m ^% %

x + a + x = 70c + 40c + 30c + a + 10c olduğundan


2x = 150c & x = 75c olur.

Matematik 9 | 245
ÜÇGENLER

ÖRNEK 15:

Şekilde d 1 ' d 2 olduğuna göre a + b + c + d toplamını bulunuz.

ÇÖZÜM:

B ve C köşelerinden d 1 ' 5BK ' 5CL ' d 2 olacak şekilde 5BK ve 5CL
çizildiğinde
a + x = 180c_bb Z]a + ^x + yh + ]z + tg + d = 3 $ 180c
bb ]] 144424443 \
y + z = 180cb` & ][ b c
bb ]]
t + d = 180cb \ a + b + c + d = 540c tir.
a

Üçgende Açı

Düzlemde doğrusal olmayan üç noktanın ikişer ikişer birleştirilmesi


ile elde edilen geometrik şekle üçgen adı verilir.
• 5AB? , 5BC? , 5AC? = ABC
&
• A, B, C noktaları üçgenin köşeleridir.
• 5AB?, 5BC? ve 5AC? üçgenin kenarlarıdır.
• a, b ve c üçgenin iç açıları; x, y ve z üçgenin dış açılarıdır.
• Bir üçgende açıortay, kenarortay, yükseklik ve kenar orta dikme
üçgenin yardımcı elemanlarıdır.

Üçgende Açı Özellikleri


1. Bir üçgenin iç açılarının ölçüleri toplamı 180c dir. Bu ifadenin doğruluğu aşağıdaki gibi gösterilir.
B köşesinden 5AC? kenarına paralel d doğrusu çizildiğinde a = x (İç ters
açılar)
c = y (İç ters açılar)
x + b + y = 180c (Doğru açı)
a + b + c = 180c elde edilir.

2. Bir üçgenin dış açılarının ölçüleri toplamı 360c dir. Bu ifadenin doğruluğu aşağıdaki gibi gösterilir.
a + x = 180c (Doğru açı)
b + y = 180c (Doğru açı)
c + z = 180c (Doğru açı)
a + b + c + x + y + z = 540c bulunur. Buradan
14444244443
180c
x + y + z = 540c - 180c = 360c elde edilir.

246 | Matematik 9
ÜÇGENLER

3. Bir üçgende bir dış açının ölçüsü, kendisine komşu olmayan iki iç açının ölçüleri toplamına eşittir.
a + b + c = 180c olduğundan
a = 180c - ]b + cg f ^1h
a + a = 180c (Doğru açı) f ^2h
^1 h eşitliği ^2 h eşitliğinde yerine yazılırsa
180c - ]b + cg + a = 180c
a = b + c elde edilir.
Benzer düşünceyle b = a + c ve i = a + b olduğu görülür.

4. Bir içbükey (konkav) dörtgende içbükey köşenin dıştaki açısının ölçüsü, içteki açıların ölçüleri toplamına
eşittir. a = a + b + c

5BD? uzatılıp 5AC? kestiği noktaya E denirse


ABE de m ^% BEC h = a + b olur. (Dış açı özelliği)
&
&
DEC de a = a + b + c elde edilir. (Dış açı özelliği)

ÖRNEK 16:
Bir üçgenin dış açıları 2, 3 ve 7 sayıları ile doğru orantılı ise en büyük dış açının ölçüsünü bulunuz.

ÇÖZÜM:
ABC üçgeninin dış açıları x, y ve z olsun. x, y ve z sırasıyla 2, 3 ve 7 ile doğru orantılı olduğundan
x y z
2 = 3 = 7 = k eşitliğinden x = 2k, y = 3k, z = 7k yazılır.
Dış açıların ölçüleri toplamı x + y + z = 360c olduğundan
2k + 3k + 7k = 360c & 12k = 360c & k = 30c bulunur.
En büyük dış açının ölçüsü z = 7k, z = 7 $ 30c = 210c olarak bulunur.

ÖRNEK 17:

ABC bir üçgen. B, C ve D doğrusal.


m ^ACD
% h = 5 $ m ^%h, m ^%h = 80c olduğuna göre
ABC CAB
m ^ACB h = a değerini bulunuz.
%

ÇÖZÜM:
m ^%
ABC h = x alınırsa m ^ACD h = 5x olur.
% m ^BCD
%h = m ^%h + m ^ % h = 180c (Doğru açı)
ACB ACD
m ^ACD
% h = m ^%h + m ^%h (Dış açı özelliği)
ABC CAB a + 5x = 180c & a + 5 $ 20c = 180c
5x = x + 80c
& a = 80c tir.
4x = 80c & x = 20c olur.

Matematik 9 | 247
ÜÇGENLER

ÖRNEK 18:

ABC bir üçgen,


m ^%
ACB h = 65c
m ^%
ABE h = 3x + 25c
m ^CAD
% h = 4x + 10c
olduğuna göre x in değerini bulunuz.

ÇÖZÜM:

ABC üçgeninde
m ^%
ACBh + m ^ BCF h = 180c
%
65c + m ^% BCF h = 180c
m ^ BCF h = 115c tir.
%

m ^ABE h + m ^%
%
BCF h + m ^CAD h = 360c
%
3x + 25c + 115c + 4x + 10c = 360c
7x + 150c = 360c & 7x = 210c & x = 30c olur.
F

ÖRNEK 19:

Yandaki şekilde
m ^CAB
%h = 40c
m ^BDC
%h = 100c
m ^ACD
% h = 3 $ m ^%h
DCE
%h olduğuna göre m ^%
m ^ABD h = 3 $ m ^DBE
% BEC h = a açısını bulunuz.

ÇÖZÜM:
ABDC içbükey dörtgeninde DBEC içbükey dörtgeninde
100c = 3x + 3y + 40c m ^%
BEC h = a = 100c + x + y
60c = 3x + 3y & 3 $ ^x + yh = 60c a = 100c + 20c
& x + y = 20c a = 120c bulunur.

248 | Matematik 9
ÜÇGENLER
ÖRNEK 20:

Şekilde verilen değerlere göre x + y + z değerini bulunuz.

ÇÖZÜM:
ABE üçgeninde 65c = x + 30c eşitliğinden x = 35c
ACF üçgeninde 140c = 35c + y eşitliğinden y = 105c
DEF üçgeninde 105c = z + 30c eşitliğinden z = 75c bulunur.
Buradan x + y + z = 35c + 105c + 75c = 215c olur.

Üçgende Açıortay Özellikleri


1. Bir üçgende iç açıortaylar bir noktada kesişir. Bu nokta, üçgenin iç teğet çemberinin merkezidir.

ÖRNEK 21:

ABC bir üçgen,


5AD?, 5CD? iç açıortay,
m ^BAC
%h = 86c olduğuna göre
m ^BDC
%h = a açısını bulunuz.

ÇÖZÜM:
5AD? ve 5CD? iç açıortaylarının kesişimi 5AD? + 5CD? = !D + ise bu nokta,
ABC üçgeninin iç açıortaylarının kesişme noktası olur. Bu durumda 5BD?
de iç açıortay olur.
% h = m ^%h = x ve m ^ % h = m ^%h = y alındığında
m ^ABD CBD ACD BCD
43c + 43c + 2x + 2y = 180c eşitliğinden
2 $ ^x + yh = 180c - 86c = 94c
x + y = 47c bulunur.
x + y + a = 180c
47c + a = 180c
a = 133c bulunur.

Matematik 9 | 249
ÜÇGENLER

2. Bir üçgende iki iç açıortay arasındaki açının ölçüsü, açıortayı çizilmeyen açı ölçüsünün yarısından 90c
fazladır.
m ^W
Ah
a = 90c + 2

ÖRNEK 22:
ABC bir üçgen,
5BD?, 5CD? iç açıortay,
m ^BAC
%h = 120c olduğuna göre

m ^BDC
%h = a açısının ölçüsünü bulunuz.

ÇÖZÜM:
1. Yol:
m ^W
Ah
a = 90c + 2
120c
a = 90c + 2
a = 90c + 60c = 150c tir.

2. Yol:
&
ABC de
m ^W Uh + m ^ C
A h + m ^B Vh = 180c olduğundan 2x + 2y + 120c = 180c
2 $ ^x + yh = 60c
x + y = 30c
&
BDC de a + x + y = 180c dir. a + 30c = 180c & a = 150c tir.

3. Bir üçgende iki dış açıortay arasındaki açının ölçüsü ile açıortayı çizilmeyen iç açı ölçüsünün yarısı,
birbirinin tümleridir.
m ^W
Ah
a+ 2 = 90c veya
m ^W
Ah
a = 90c - 2 tir.

250 | Matematik 9
ÜÇGENLER

ÖRNEK 23:

ABC bir üçgen, 5BD? ve 5CD? dış açıortay, m ^BAC


%h = 110c olduğuna göre
m ^BDC
%h = a açısının ölçüsünü bulunuz.

ÇÖZÜM:

1. Yol: Formül kullanılırsa


m ^W
Ah
a = 90c - 2
110c
a = 90c - 2 = 90c - 55c = 35c olarak bulunur.

E F

2. Yol:
EBC açısının bütünleri " 180c - 2y
FCB açısının bütünleri " 180c - 2x olduğundan
& & de
ABC de BCD
m ^W B h + m ^V
A h + m ^V C h = 180c B h + m ^V
m ^V C h + m ^W
D h = 180c
110c + 180c - 2y + 180c - 2x = 180c y + x + a = 180c
470c - 2 $ ^x + yh = 180c 145c + a = 180c
470c - 180c = 2 $ ^x + y h a = 180c - 145c = 35c bulunur.
145c = x + y bulunur.

4. Bir üçgende bir köşenin iç açıortayı ile diğer bir köşenin dış açıortayı arasındaki açının ölçüsü, açıortayı
çizilmeyen köşenin iç açı ölçüsünün yarısıdır.

m^BAC h m ^W
Ah
%
a= 2 = 2

Matematik 9 | 251
ÜÇGENLER

ÖRNEK 24:

ABC bir üçgen,


5BD? iç açıortay, 5CD? dış açıortay,
m ^W
A h = 2x + 22c
m ^BDC h = 3x + 5c olduğuna göre x değerini bulunuz.
%

ÇÖZÜM:

Vh = m ^W
Ah
m ^D
2x + 22c
2 olduğundan 3x + 5c = 2 & 6x + 10c = 2x + 2 & 4x = 12 & x = 3c bulunur.

5. Bir üçgende farklı köşelerdeki iki dış açıortay ile bir iç açıortay bir noktada kesişir. Bu nokta üçgenin dış
teğet çemberinin merkezidir.

ÖRNEK 25:

Yanda ABCD bir dörtgendir. Şekilde verilenlere göre m ^%


AEB h = a
açısının ölçüsünü bulunuz.

ÇÖZÜM:

Şekilde 5DA? ve 5DC? kenarları uzatıldığında


FAB h = 55c ve m ^BCG h = 65c bulunur. Bu durumda
m ^% %
& de 5AB? ve 5CB? dış açıortay olduğundan 6DB@ iç açıortay olur.
ADC
m ^W
D h = 180c - ^50c + 70ch = 60c olduğundan
m^CDE h = m^EDA h = 30c olur. Buradan
% %

m^AEBh = m^ADBh + m^EAD h


% % %

= 70c + 30c
= 100c bulunur.

252 | Matematik 9
ÜÇGENLER

ÖRNEK 26:

Şekilde verilenlere göre m ^%


ACB h = a değerini bulunuz.

ÇÖZÜM:

5DA? ve 5DC? şekildeki gibi uzatıldığında


m ^%BAE h = 180c - ]62c + 59cg = 59c olduğundan 5AB?, CDA nin dış
&
& de 5AB? 5DB? = !B + olduğundan 5CB? dış
açıortayı olur. Ayrıca ADC +
açıortaydır.
2 $ m ^%
ACB h = m ^ACF h = 62c + 68c = 130c & m ^ACB h = 65c olur.
% %

İkizkenar ve Eşkenar Üçgende Açı Özellikleri

İki kenarının uzunluğu, eşit olan üçgenlere ikizkenar üçgen denir.


ABC de 5BC? taban, 5AB? ve 5AC? yan kenarlardır.
&
W
A tepe açısı, BU ve C
V taban açılarıdır.
AB = AC
Uh = m ^ C
m ^B Vh

İkizkenar üçgende tabana ait yükseklik, hem açıortay hem de


kenarortaydır.
5AH? = 5BC? ise
% h = m^ % h
m ^BAH CAH
BH = CH

Matematik 9 | 253
ÜÇGENLER

Üç kenarının uzunluğu, eşit olan üçgenlere eşkenar üçgen denir.


AB = AC = BC
m ^W Uh = m ^ C
A h = m ^B Vh = 60c

ÖRNEK 27:
ABC ve ADC birer ikizkenar üçgen,
AB = AC ve AD = DC
m ^CAD
% h = 27c olduğuna göre

m ^DAB
%h = a açısını bulunuz.

ÇÖZÜM:
Uh = m ^ C
ABC ve ADC ikizkenar üçgen olduğundan m ^B Vh ve m ^CAD Vh tir.
% h = m ^C
Vh = m ^CAD h = 27c olduğundan m ^B
m ^C % Uh = 27c olur.
m ^W Uh + m ^ C
A h + m ^B Vh = 180c & a + 27c + 27c + 27c = 180c & a + 81c = 180c & a = 99c bulunur.

ÖRNEK 28:

ABC eşkenar üçgen,


A, D ve E doğrusaldır. AC = AE olduğuna göre m ^%
CBE h + m ^BCE h
%
değerini bulunuz.

ÇÖZÜM:
ABC eşkenar üçgen olduğundan AB = AC = BC tir.
AC = AE ise ACE ve ABE ikizkenar üçgen olur.

O hâlde m ^%ACE h = m ^AEC h ve m ^ABE h = m ^AEB h tir.


% % %
m ^%
BCE h = y alınırsa m ^AEC h = y + 60c
%
m ^%
CBE h = x alınırsa m ^AEB h = x + 60c olur.
%
&
BCE de iç açılar toplamından
y + x + ]60c + xg + ^60c + yh = 180c
120c + 2 $ ^x + yh = 180c
2 $ ^x + yh = 180c - 120c
x + y = 30c olur.
m ^CBE h + m ^BCE h = x + y = 30c bulunur.
% %

254 | Matematik 9
ÜÇGENLER

ÖRNEK 29:

ABC eşkenar üçgen,


BD = CD
m ^ACD
% h = 34c olduğuna göre

m ^ADB
% h = b değerini bulunuz.

ÇÖZÜM:
ABC eşkenar üçgen olduğundan her bir açısı 60c dir.
BD = CD olduğundan ikizkenar üçgen olur.
m ^DCB
%h = m ^%h = x alınırsa
DBC
Vh = x + 34c = 60c & x = 26c bulunur.
m ^C
& de _%i
BDC m BDC = 180c - 52c
= 128c olur.
m ^ADB h = m ^ADC h = b tir.
% %
128c + b + b = 360c
2b = 232c
b = 116c bulunur.

2. Üçgende Açı Kenar Bağıntıları


Teknoloji Uygulaması
Dinamik Geometri Yazılımı GeoGebra ile aşağıdaki yönergelerden yararlanarak bir üçgen çiziniz. Çizdiğiniz
üçgenin kenar uzunluklarını ve iç açı ölçülerini hesaplatınız

Çokgen aracını etkinleştiriniz.


ABC üçgeni oluşturmak için A, B ve C noktalarını art arda seçiniz.
(A noktasından başlayıp yine en son A noktasına tıklayınız.)

Açı aracını etkinleştiriniz.


Üçgenin iç açı ölçülerini belirlemek için sırasıyla köşeleri seçiniz.

Uzunluk aracını etkinleştiriniz.


Üçgenin kenar uzunluklarını ölçmek için kenarları seçiniz.

Taşı aracını etkinleştiriniz.


Üçgenin köşelerini hareket ettirerek farklı üçgenler oluşturunuz.

• Üçgenin kenar uzunluklarını ve iç açı ölçülerini kendi aralarında küçükten büyüğe sıralayınız.
• Elde ettiğiniz verilere göre üçgenin kenar uzunlukları ile açılarının ölçüleri arasında küçüklük büyüklük
bakımından bir ilişki var mı? İnceleyiniz.
• Ulaştığınız sonucu köşeleri hareket ettirerek elde edeceğiniz yeni birkaç üçgen için test ediniz.

Matematik 9 | 255
ÜÇGENLER
Üçgenin açı ve kenarları arasındaki ilişkiyi inceleyebileceğiniz GeoGebra programında çizilmiş benzer bir
örnek aşağıda verilmiştir.

Bir üçgende büyük kenar karşısında büyük açı, küçük kenar karşında
küçük açı bulunur. Başka bir ifadeyle büyük açı karşısında büyük
kenar, küçük açı karşısında küçük kenar bulunur.
ABC de, a 2 b 2 c & m ^W B h 2 m ^V
A h 2 m ^V C h veya
&

m ^W B h 2 m ^V
A h 2 m ^V C h & a 2 b 2 c tür.

Bu ifadenin doğruluğu aşağıda gösterilmiştir:


Şekilde AC 1 BC olsun. Bu durumda m ^% ABC h 1 m ^CAB h olduğu
%
gösterilmelidir.
AC = DC olacak şekilde D ! 5BC? olsun.
Buna göre m ^CAD
% h = m ^% h = x olur.
CDA
O zaman m ^ABC h 1 m ^CDA h, z 1 x tür.
% %
Aynı zamanda x 1 x + y olduğundan z 1 x 1 x + y yazılır.
Bu durumda z 1 y + x olduğundan m ^% ABC h 1 m ^CAB h elde edilir.
%

ÖRNEK 1:

Yandaki şekilde kenar uzunlukları verilen ABC üçgeninin açıları


arasındaki sıralamayı bulunuz.

ÇÖZÜM:
Verilenlere göre AC = 12 cm, AB = 8 cm, BC = 15 cm olduğundan AB 1 AC 1 BC yazılır.
Küçük kenar karşısında küçük açı bulunduğundan
AB 1 AC 1 BC & m ^C Vh 1 m ^BUh 1 m ^ W
A h elde edilir.

256 | Matematik 9
ÜÇGENLER

ÖRNEK 2:

Şekilde verilenlere göre en uzun ve en kısa kenarı bulunuz.

ÇÖZÜM:
ABC üçgeninde
m ^W Uh = 55c ise m ^W
A h = 65c, m ^B Uh + m ^ C
A h + m ^B Vh = 180c

65c + 55c + m ^V
C h = 180c& m ^V
C h = 180c - 120c
& m ^V

C h = 60c bulunur.
Bu durumda m ^V C h 1 m ^W
B h 1 m ^V A h & e 1 a 1 b f ]1 g
Vh = 61c, m ^D
ACD üçgeninde m ^C Vh = 67c ise m ^W Vh + m ^D
A h + m ^C Vh = 180c
m ^W
A h + 61c + 67c = 180c& m ^W A h = 180c - 128c
W

& m ^ A h = 52c bulunur.
Bu durumda m ^W A h 1 m ^V
C h 1 m ^W D h & c 1 d 1 e f ]2 g
(1) ve (2) sıralamaları karşılaştırılırsa c 1 d 1 e 1 a 1 b kenar sıralaması elde edilir. Bu durumda en uzun
kenar b, en kısa kenar c olur.

ÖRNEK 3:

ABC bir üçgendir. m ^BUh = 66c, BC 1 AB olduğuna göre


m ^ACB h = x in alabileceği en küçük tam sayı değerini bulunuz.
%

ÇÖZÜM:

BC 1 AB & m ^W A h 1 m ^V
Ch
& m ^W

A h 1 x f ]1 g
m ^W Uh + m ^ C
A h + m ^B Vh = 180c olduğundan

m ^W
A h + 66c + x = 180c & m ^W

A h = 114c - x ... ^2 h
(2) ifadesi (1) de yerine yazıldığında
114c - x 1 x & 114c 1 2x
& 57c 1 x olur.
Bu durumda x in değer aralığı: 57c 1 x 1 114c bulunur.

En küçük değeri: m ^%
ACB h = 58c olur.

Matematik 9 | 257
ÜÇGENLER

A ABC üçgeninde
a 1 90c ise b 2 1 a 2 + c 2
a = 90c ise b 2 = a 2 + c 2
c b a 2 90c ise b 2 2 a 2 + c 2 olur.

B a C

ÖRNEK 4:

Uh 1 90c ve AB = 6, BC = 5 olduğuna göre


ABC bir üçgendir. m ^B
AC = x in alabileceği en büyük tam sayı değerini bulunuz.

ÇÖZÜM:
m ^V
B h 1 90c & x 2 1 6 2 + 5 2
& x 2 1 61
& x 1 61, ^ 61 . 7, 81h

& x = 7 olur.

3. Üçgen Eşitsizliği
Teknoloji Uygulaması
Aşağıda GeoGebra programı kullanılarak uzunluğu verilen üç doğru parçasının hangi durumlarda üçgen
oluşturduğu incelenmiştir.

Sürgü aracını etkinleştiriniz.


En küçük değeri 1 en büyük değeri 10 olacak şekilde a, b, c sürgülerini oluşturunuz.

Verilen uzunlukta doğru parçası aracını etkinleştiriniz.


A noktasını seçiniz ve doğru parçasının uzunluğunu a olarak belirleyiniz. Bu şekilde AB doğru parçasını
oluşturunuz.

Merkez ve yarıçapla çember aracını etkinleştiriniz.


A noktası merkez olarak işaretleyiniz ve yarıçapı b olan çemberi çiziniz.
B noktasını merkez olarak işaretleyiniz ve yarıçapı c olan çemberi çiziniz.

Nokta aracını etkinleştiriniz.


Oluşturduğunuz iki çemberin kesiştikleri noktayı C noktası olarak belirleyiniz.

Doğru parçası aracını etkinleştiriniz.


A ve C noktalarını seçerek AC doğru parçasını oluşturunuz.
B ve C noktalarını seçerek BC doğru parçasını oluşturunuz.

• Sürgüleri hareket ettirerek oluşan şekilleri inceleyiniz.


• a, b ve c uzunluklarından herhangi ikisi sabit tutulduğunda, üçüncü uzunluk hangi aralıkta değer alırsa
bir üçgen oluşur? Açıklayınız.

258 | Matematik 9
ÜÇGENLER
GeoGebra programında çizilmiş benzer bir örnek aşağıda verilmiştir.

Bir üçgende iki kenar uzunluğunun toplamı, üçüncü kenarın


uzunluğundan büyüktür.
ABC üçgeninde
a 1 b+c
b 1 a+c
c 1 a + b tür.
a 1 b + c ...(1)
b 1 a+c&b-c 1 a
3 b - c 1 a ... ]2 g
c 1 a+b&c-b 1 a
(1) ve (2) den b - c 1 a 1 b + c elde edilir.

ABC üçgeninde herhangi bir kenar uzunluğu, diğer iki kenar


uzunluğu toplamından küçük; farklarının mutlak değerinden
büyüktür. Bu bağıntıya üçgen eşitsizliği denir.
b-c 1 a 1 b+c
a-c 1 b 1 a+c
a - b 1 c 1 a + b tür.

Matematik 9 | 259
ÜÇGENLER

ÖRNEK 1:
Temel, A şehrinden hava yolu ile C şehrine gitmek istiyor. A şehri ile C şehri arasındaki hava yolculuğu
ancak B şehrine uğrayarak mümkün olmaktadır. A, B ve C şehirlerinin birbirine göre konumları bir üçgen
oluşturmaktadır. A şehri ile C şehri arası hava yolu ile 900 km, A şehri ile B şehri arası hava yolu ile 1120 km
olduğuna göre B ile C şehri arası hava yolu tam sayı değeri olarak en az kaç kilometredir?
ÇÖZÜM:
A Üçgen eşitsizliği kullanılırsa
km 1120 - 900 1 z 1 1120 + 900
y=1120
220 1 z 1 2020
B B ile C şehri arası hava yolu en az 221 km dir.
km
00
x=9
z=?

ÖRNEK 2:
Aşağıda uzunlukları verilen doğru parçaları ile bir üçgen oluşturulup oluşturulamayacağını inceleyiniz.
a) AB = 5 cm, CD = 10 cm, EF = 17 cm ve b) AB = 12 cm, CD = 8 cm, EF = 15 cm

ÇÖZÜM:
a) AB = 5 cm
CD = 10 cm & 10 - 5 1 17 1 10 + 5
EF = 17 cm
Üçgen eşitsizliği sağlanmadığından verilen ölçülerde bir üçgen çizilemez.
b) AB = 12 cm, CD = 8 cm, EF = 15 cm
12 - 8 1 15 1 12 + 8 & 4 1 15 1 20

15 - 8 1 12 1 15 + 8 & 7 1 15 1 23
15 - 12 1 8 1 15 + 12 & 3 1 8 1 27
Üçgen eşitsizliği sağlandığında verilen ölçülerde bir üçgen çizilebilir.

Üçgen eşitsizliği, uzunlukları verilen üç doğru parçasından birisi için sağlanıyorsa üçgen çizilebilir.

ÖRNEK 3:

Şekilde verilenlere göre BC kenar uzunluğunun alabileceği en küçük ve


en büyük tam sayı değerlerini bulunuz.

ÇÖZÜM:

ABC de " 15 - 10 1 BC 1 15 + 10 & 5 1 BC 1 25 ... ]1 g


&

BCD de " 18 - 6 1 BC 1 18 + 6 & 12 1 BC 1 24 ... ]2 g
&
(1) ve (2) den 12 1 BC 1 24 yazılır. Bu durumda BC kenarının alabileceği tam sayı değerleri,
BC = 13 (en küçük)
BC = 23 (enbüyük) olarak bulunur.

260 | Matematik 9
ÜÇGENLER

ÖRNEK 4:

ABC bir üçgen,


Vh 2 m ^ W
m ^C Ah
AC = 7
BC = 4 olduğuna göre
AB = x in kaç farklı tam sayı değeri olabileceğini bulunuz.

ÇÖZÜM:
&
ABC de üçgen eşitsizliğinden
7 - 4 1 x 1 7 + 4 & 3 1 x 1 11 bulunur.
C h 2 m ^W
m ^V A h olduğundan 4 1 x 1 11 olur.
Bu durumda x in alabileceği tam sayı değerleri 5, 6, 7, 8, 9, 10 olup 6 tanedir.

ÖRNEK 5:

Yandaki ABCD dörtgeninde BD = 9 cm olduğuna göre


Ç ]ABCDg alabileceği en küçük tam sayı değerini bulunuz.

ÇÖZÜM:
Üçgen eşitsizliği kullanılırsa
&
ABD de " a + d 2 9
& de " b + c 2 9 eşitsizlikleri taraf tarafa toplanırsa
BCD
Ç ]ABCDg = a + b + c + d 2 18 bulunur. Buradan Ç ]ABCDg = 19 olur.

Dar açılı, dik açılı ve geniş açılı üçgenlerde açı ve kenarlar arasındaki ilişkiler şunlardır:

a) ABC dar açılı üçgen ise b) ABC dik açılı üçgen ise c) ABC geniş açılı üçgen ise
m ^VB h 1 90c olduğundan m ^VB h = 90c olduğunda m ^VB h 2 90c olduğundan
b 1 a 2 + c 2 tir.
2 b 2 = a 2 + c 2 tir. b 2 a 2 + c 2 tir.
2

b 1 a2 + c2 (Pisagor bağıntısı) b 2 a2 + c2
& a - c 1 b 1 a 2 + c 2 tir. & a + c 2 b 2 a2 + c2

Matematik 9 | 261
ÜÇGENLER

ÖRNEK 6:

m ^W
A h 2 90c olduğuna göre
DB = x in alabileceği tam sayı değerlerinin toplamını bulunuz.

ÇÖZÜM:
& de m ^W
A h 2 90c & x 2 2 5 2 + ^2 6 h
2 & de üçgen eşitsizliği kullanılırsa
BCD
ABD
& x 2 2 49 9 - 6 1 x 1 9 + 6 & 3 1 x 1 15 ... ]2g
& x 2 7 olur.... ]1g

(1) ve (2) den x = 8, 9, 10, 11, 12, 13, 14 bulunur.


Bu durumda x değerleri toplamı 8 + 9 + 10 + 11 + 12 + 13 + 14 = 77 olur.

ÖRNEK 7:

ABC ikizkenar üçgen DC = 8 olduğuna göre


AD = x in alabileceği tam sayı değerlerini bulunuz.

ÇÖZÜM:
& %
ABC ikizkenar üçgen olduğundan ACD geniş açı olur.
m ^ACD
% h 2 90c & x 2 2 8 2 + 6 2
& de üçgen eşitsizliği kullanılırsa
ACD
8 - 6 1 x 1 8 + 6 & 2 1 x 1 14 ... ]2g
2
& x 2 100
& x 2 10 olur. ... ]1g
(1) ve (2) den AD = x in alabileceği tam sayı değerleri 11, 12, 13 olarak bulunur.

Sıra Sizde

A
SORU:
ABC üçgeninin çevresi 24 cm AD = x, BD = y, DC = z x + y + z ! N
olmak üzere x + y + z nin alabileceği en küçük ve en büyük değerlerin
toplamı kaçtır?
D
ÇÖZÜM: B C

262 | Matematik 9
ÜÇGENLER

ALIŞTIRMALAR

1 5

ABC üçgeninde, 5CD? ve 5BD? açıortaylar,


m ^BDC
%h = 42c olduğuna göre
d1 ' d2 ' d3 ise a açısının ölçüsünü
m ^DAE
%h = x açısının ölçüsünü bulunuz.
bulunuz.

ABC bir üçgen, AB = 2 DC ise m ^DAC


%h
açısını bulunuz.
ABC bir üçgen, 5BP? ve 5CP? iç açıortaylar,
BP = 3 br, CP = 8 br olduğuna göre
BC uzunluğunun alabileceği tam sayı
değerlerini bulunuz.
3

ABC üçgeninde
5AF?, 5BE? ve 5CD? iç açıortaylar,
m ^BDC
%h = x, m ^%h = y, m ^%h = z
AFC BEA
olmak üzere x + y + z = 270c olduğunu
gösteriniz.

Şekilde verilenlere göre m ^%


ABC h = x
değerini bulunuz.

Yukarıdaki şekilde OD = DC = CE , Kenar uzunlukları tam sayı olan bir çeşit kenar
OE = EB = BF , m ^% CEB h = 32c
8
üçgenin çevresinin alabileceği en küçük tam
olduğuna göre FBA açısının ölçüsünü sayı değerini bulunuz.
bulunuz.

Matematik 9 | 263
ÜÇGENLER

9.5.2. ÜÇGENİN YARDIMCI ELEMANLARI

Etkinlik

Üçgen şeklindeki bir salonun tavanına duvarlara eşit uzaklıkta olacak şekilde bir çubuk
sabitlenecek ve bu çubuğa 3 tane projeksiyon cihazı takılacaktır. Bu cihazların her biri
farklı bir duvara yansıtılacağına göre çubuğun takıldığı noktayı üçgen geometrisi yönünden
inceleyiniz. Bu nokta ile üçgenin köşe açıları ve kenarları arasında nasıl bir bağıntı vardır?
a) Eğer bu salon, bir kenarı 24 m olan bir eşkenar üçgen şeklinde ise çubuğun yerini
hesaplayınız.
b) Boyutları 20-20-24 m olan ikizkenar üçgen bir salonda çubuk nereye takılmalıdır?
Hesaplayınız.
c) Boyutları 30-40-50 m olan dik üçgen bir salonda çubuğun yerini hesaplayınız

264 | Matematik 9
ÜÇGENLER

1. Üçgende Açıortay
Açıortay

Bir açıyı iki eş açıya ayıran ışına açıortay denir.


m ^AOB
% h = m ^ % h ise 5OB AOC %
BOC nın açıortayıdır.

Verilen bir açı cetvel ve pergel kullanılarak şu şekilde iki eş açıya ayrılabilir.

I. Adım: II. Adım:


Cetvel yardımıyla AOB açısı çiziniz. Pergel yardımıyla O merkezli bir çember yayı
çiziniz. Bu çember yayının açının kollarını
kestiği noktalar K ve N olsun.

III. Adım: IV. Adım:


Pergel yardımıyla K ve N merkezli iki eş çember yayını Son olarak O ve P noktaları birleştirildiğinde
kesişecek şekilde çiziniz. Bu çemberlerin kesiştiği elde edilen [OP AOB açısının açıortayıdır.
noktalardan birisi P olsun.

Açıortay doğrusu üzerindeki herhangi bir noktanın açının


kollarına olan uzaklıkları eşittir.
5OP ışını, AOB açısının açıortayı ve herhangi bir N ! 5OP olsun.
N noktasından 6OA ve 5OB ye indirilen dikmeler sırasıyla
5NC? ve 5ND? olsun.
Elde edilen üçgenlerin ortak doğru parçası, 5ON? olduğundan
A.K.A. eşlik teoreminden
& &
NCO b NDO olur.
O hâlde NC = ND ve OC = OD dir.
Bu ifadenin karşıtı da doğrudur. “Bir açının iç bölgesinde alınan bir noktanın açının kollarına olan
uzaklıkları eşit ise bu nokta, açının açıortay doğrusu üzerindedir.”

Matematik 9 | 265
ÜÇGENLER

ÖRNEK 1:

BTN h = m ^\
m ^\ ATN h
5KL? = 5TA, 5KM? = 5TB, 5PR? = 5TA, 5PS? = 5TB
KL = 3x - 16, KM = 2x - 9, PS = x - 4
TR
olduğuna göre RL oranını bulunuz.

ÇÖZÜM:
KL = KM & 3x - 16 = 2x - 9 & x = 7 br olur.
PR = PS = x - 4 = 3 br ve KL = 3x - 16 = 5 olarak bulunur.
& &
TRP + TLK olduğundan
PR TR 3
KL = TL = 5 tir.
TR 3
O hâlde RL = 2 tir.

ÖRNEK 2

Yandaki şekilde
FD = FE , m ^% ABC h = 40c, m ^ AFE h = 52c
%
olduğuna göre m ^%
FKC h nin kaç derece olduğunu bulunuz.

ÇÖZÜM:

FD = FE olduğundan 5AF?, BAC açısının açıortayıdır.


Bu durumda m ^% AFE h = m ^AFD h = 52c tir.
%

m ^%
DAF h = m ^ FAE h = 90c - 52c = 38c tir.
%

m^AKC h = 40c + 38c = 78c & x = 180c - 78c = 102c tir.


%

266 | Matematik 9
ÜÇGENLER

Üçgende İç Açıortay

Bir üçgenin bir iç açısını iki eş açıya ayıran ışına o üçgenin iç açıortayı denir. Bir üçgende iç açıortaylar
tek noktada kesişir.
5AS?, A açısına ait açıortay olmak üzere AS = n A
5BR?, B açısına ait açıortay olmak üzere BR = n B
5CP?, C açısına ait açıortay olmak üzere CP = n C ile gösterilir.
I noktası, iç açıortayların kesişim noktasıdır.

Bir üçgende iç açıortayların kesişim noktası üçgenin iç teğet çemberinin merkezidir.


I noktası iç açıortayların kesişim noktası ve üçgenin iç teğet
çemberinin merkezdir.
D, E, F noktaları çemberin üçgene teğet noktaları olmak üzere
IE = ID = IF = r iç teğet çemberinin yarıçapıdır.
AD = AE , BD = BF , CF = CE dir.

ÖRNEK 3:

ABC üçgeninde
5CE? iç açıortay, DC = 17 br
BC = 20 br, ED = 4 br olduğuna göre BE = x değerini bulunuz.

ÇÖZÜM:

E noktasından 5BC? ye indirilen dikmenin kestiği nokta H olsun.


5CE? iç açıortay olduğundan ED = EH = 4 br dir.
Bu durumda DC = HC = 17 br ve BH = 20 - 17 = 3 br olur.
EBH üçgeninde Pisagor teoremi uygulandığında x = 5 br olarak
bulunur.

Matematik 9 | 267
ÜÇGENLER

ÖRNEK 4:

ABC üçgeninde
I noktası üçgenin iç açıortaylarının kesişim noktasıdır.
ID = 3 br, BD = 5 br, DC = 12 br
olduğuna göre ABC üçgeninin çevresini bulunuz.

ÇÖZÜM:

I noktası üçgenin iç teğet çemberinin merkezidir.


Çember çizilirse AEIF dörtgeninin kare olduğu görülür.
Dolayısıyla AE = AF = 3 br dir.
Bu durumda BE = BD = 5 br, CF = CD = 12 br olur.
Ç^ABC h = 8 + 15 + 17 = 40 br dir.
&

ÖRNEK 5:

ABC üçgeninde
I noktası üçgenin iç teğet çemberinin merkezidir.
5ID? ' 5AB?, 5IE? ' 5AC?,
BD = 3 br, DE = 6 br, EC = 5 br
olduğuna göre IDE üçgeninin çevresini bulunuz.

ÇÖZÜM:

I noktası üçgenin iç açıortaylarının kesişim noktasıdır.


O hâlde 5IB? ve 5IC? iç açıortaylardır.
ID = BD = 3 br, IE = EC = 5 br dir.
Bu durumda Ç ^IDE h = 14 br olarak bulunur.
&

268 | Matematik 9
ÜÇGENLER

Üçgende İç Açıortay Teoremi

ABC üçgeninde
A açısına ait açıortay doğrusunun 5BC? yi kestiği nokta N olsun.
AB = c, AC = b, BN = m, NC = n olmak üzere
c m
b = n dir.

İspat:
B noktasından AC kenarına paralel çizilen doğru AN yi P noktasında
kesiyor.
İç ters açılardan m^CANh = m^BPA h olduğundan
% %
ABP üçgeni ikizkenar olur.
Dolayısıyla AB = BP = c dir.
& &
BPN + CAN kelebek benzerliğinden
BP BN c m
AC = NC & b = n olarak bulunur.

ABC üçgeninde 5AN? iç açıortay ve AN = x olsun.


AB = c, AC = b, BN = m, NC = n olmak üzere
x 2 = bc - mn dir.

ÖRNEK 6:

ABC üçgeninde
AB = 8 br, BN = 2 br, NC = 3 br
olduğuna göre AC = x ve AN = y değerlerini bulunuz.

ÇÖZÜM:
8 2
İç açıortay teoreminden x = 3 & x = 12 br dir.
y 2 = 8 $ 12 - 2 $ 3 = 96 - 6 = 90 & y = 90 = 3 10 br olarak bulunur.

Matematik 9 | 269
ÜÇGENLER

ÖRNEK 7:

ABC dik üçgeninde 5AD? iç açıortay,


BD = 3 br, DC = 4 br
ise AD değerini bulunuz.

ÇÖZÜM:
AB 3
İç açıortay teoreminden AC = 4 olduğundan
AB = 3k ise AC = 4k dir.
Pisagor teoreminden
]4kg2 = ]3kg2 + 7 2 & 7k 2 = 49 & k = 7 br olur.
AB = 3 7 br ve AC = 4 7 br dir.
Bu durumda
x 2 = 12k 2 - 12 = 12 $ 7 - 12 = 72
x = 72 = 6 2 br olarak bulunur.

ÖRNEK 8:
ABC üçgeninde
m^BAC h = 2 $ m^CBA h,
% %
BC = 9 br, AC = 6 br
olduğuna göre AB değerini bulunuz.

ÇÖZÜM:

5AN?, BAC açısının açıortayı olmak üzere


m^ABC h = a denirse m^BAC h = 2a olur.
% %
BAC açısının açıortayı olmak üzere
BN = x, AB = y & AN = x ve NC = 9 - x olur.
Bu durumda üçgende iç açıortay bağıntılarından
x 2 = 6y - x ]9 - xg & x 2 = 6y - 9x + x 2 & 6y = 9x & x = 2 ... (1)
y 3

y x y 6
6 = 9 - x & x = 9 - x ... (2) dir.
(1) ve (2) den
6 3 15
9 - x = 2 & 27 - 3x = 12 & 3x = 15 & x = 5 & y = AB = 2 br olarak bulunur.

270 | Matematik 9
ÜÇGENLER

ÖRNEK 9:

KLP üçgeninde
LH = HM , 5KH? = 5LM?,
KL = 9 br, LN = 6 br, NP = 12 br
olduğuna göre HN değerini bulunuz.

ÇÖZÜM:
5KH? hem yükseklik hem de kenarortay olduğuna göre KLM
üçgeni ikizkenar üçgendir.
Dolayısıyla 5KH? açıortay ve KL = KM = 9 br olur.
MP = x olsun.
KLP üçgeninde 5KN? iç açıortaydır.
O hâlde iç açıortay teoreminden
6 9
12 = 9 + x & x = 9 dur.

5MR? ' 5KN? olacak şekilde R ! 5LP?


Bu durumda
HN = y ise MR = 2y ve KN = 4y olur.
KN 2 = KL $ KP - LN $ NP & KN 2 = 9 $ 18 - 6 $ 12 = 90 & KN = 3 10 br dir.
3 10
O hâlde 4y = 3 10 & y = 4 br olarak bulunur.

ÖRNEK 10:

ABC üçgeninde
AB = 10 br, AC = 15 br olduğuna göre x in alabileceği
kaç tane tam sayı değeri vardır?

ÇÖZÜM:

İç açıortay teoreminden
BD 10 2k
DC = 15 = 3k dir.
D noktasından AB kenarına paralel çizilirse AED ikizkenar üçgeni
elde edilir.
Bu durumda AE = ED = 2 t ve EC = 3 t dir.
5t = 15 & t = 3 & AE = ED = 6 br ve EC = 9 br
olarak bulunur.
AED üçgeninde üçgen eşitsizliği kullanılırsa 0 < x < 12 olduğundan x in
alabileceği tam sayı değerleri 11 tanedir.

Matematik 9 | 271
ÜÇGENLER

Üçgende Dış Açıortay


1. Bir üçgenin bir dış açısını iki eş açıya ayıran ışına o üçgenin dış açıortayı denir.

ABC üçgeninde ACP dış açısının açıortayı olan 5CK , C açısına ait dış
açıortaydır.

2. Bir üçgende iki dış açıortay ile üçüncü açının iç açıortayı tek noktada kesişir. Bu nokta, üçgenin dış
teğet çemberinin merkezidir. Bir üçgenin üç tane dış teğet çemberi vardır.

A, B ve C açılarına ait dış


teğet çemberlerin merkezleri
sırasıyla
I A, I B, I C
ile gösterilir.
D, E ve F noktaları çemberin
değme noktaları olmak üzere
I A D = I A E = I A F = rA
olup A açısına ait dış teğet
çemberinin yarıçapı rA dır.

Benzer şekilde: B açısına ait dış teğet çemberin yarıçapı rB ,


C açısına ait dış teğet çemberinin yarıçapı rC dir.

ÖRNEK 11:

P noktası ABC üçgeninin dış teğet çemberinin merkezidir.


5PD? = 5BC, BC = 8 br, CD = 4 br, AC = 5 br
olduğuna göre ABC üçgeninin çevresini bulunuz.

ÇÖZÜM:
P noktası, üçgenin dış teğet çemberinin merkezi olduğuna göre
5PA?, 5PC? ve 5PB? açı ortaylardır.
5PF? = 5BA, 5PE? = 5AC?,
PD = PE = PF dir.
x Bu durumda
CD = CE = 4 br, AE = AF = 1 br ve BD = BF
olduğundan AB = x iken
x + 1 = 8 + 4 & x = 11 olarak bulunur.
Ç^ABC h = 11 + 5 + 8 = 24 br bulunur.
&

272 | Matematik 9
ÜÇGENLER

Üçgende Dış Açıortay Teoremi

ABC üçgeninde
5AK? dış açıortay, K ! 6BC,
AB = c, AC = b olmak üzere
KC b
KB = c dir.

İspat:

C den 5AB? na paralel olacak şekilde çizilen doğru 5AK? nı L


noktasında kesiyor.
m^DAKh = m^ALC h olduğundan ACL üçgeni ikizkenar olur.
% %
Bu durumda AC = CL = b dir.
& & b KC
Dolayısıyla KLC + KAB olduğundan c = KB olarak bulunur.

ABC üçgeninde
5AK? dış açıortay ve AK = y olsun.
AB = c, AC = b olmak üzere
y 2 = KC $ KB - bc dir.

ÖRNEK 12:

ABC üçgeninde
5AD? üçgenin dış açıortayı, D ! 6BC,
AB = 12 cm, BC = 5 cm, CD = 10 cm
olduğuna göre AC ve AD uzunluklarını bulunuz.

ÇÖZÜM:

ABC üçgeninde dış açıortay teoremi kullanılırsa


10 AC
15 = 12 & AC = 8 cm dir.
AD 2 = 10 $ 15 - 12 $ 8 = 150 - 96 = 54 & AD = 3 6 cm olarak bulunur.

Matematik 9 | 273
ÜÇGENLER

ÖRNEK 13:

ABC üçgeninde
5BD? iç açıortay, 5CD? dış açıortay, m^BDC
%
h = 45c,
BC = 10 br, EC = 5 br olduğuna göre
BE = x değerini bulunuz.

ÇÖZÜM:

m^CBE h = a ise m^DCP h = m^DCE h = 45 + a olur.


% % %

m ^W
A h + 2a = 90c + 2a & m ^W
A h = 90c tir.
ABC üçgeninde 5BE? iç açıortay olduğundan
iç açıortay teoremi kullanılırsa
AE 5 AE = k ise AB = 2k tir.
AB = 10 olduğundan

Pisagor teoreminden
]2kg2 + ]k + 5g2 = 10 2 & k 2 + 2k - 15 = 0 & ]k + 5g]k - 3g = 0 & k = 3 olur.
Bu durumda x 2 = 20k - 5k = 15k = 45 & x = 45 = 3 5 br olarak bulunur.

ÖRNEK 14:

ABC üçgeninde
5AK? iç açıortay, 5AN? dış açıortay,
AB = 8 br, AC = 6 br, BC = 7 br
olduğuna göre KN değerini bulunuz.

ÇÖZÜM:
İç açıortay teoreminden
AB 4 BK
AC = 3 = KC
olduğundan BK = 4k ise KC = 3k tir.
BC = 4k + 3k = 7k = 7 olduğundan k = 1 tir.
O hâlde BK = 4 br ve KC = 3 br olur.

CN = x olsun. Dış açıortay teoreminden


x 6 3
x + 7 = 8 = 4 & 4x = 3x + 21 & x = 21 tir. Bu durumda
KN = 3k + x = 3 + 21 = 24 br olarak bulunur.

274 | Matematik 9
ÜÇGENLER

ÖRNEK 15:

ABC üçgeninde
I, üçgenin iç teğet çemberinin merkezi
P, üçgenin dış teğet çemberinin merkezi
AB = 6 br, AC = 5 br, BC = 9 br
olduğuna göre I ile P arasındaki uzaklığı bulunuz.

ÇÖZÜM:

Verilenlerden I noktası iç açıortayların kesişim noktası, P noktası


ise dış açıortayların kesişim noktasıdır.
Dolayısıyla B, I ve P noktaları doğrusal olup
5BI? ile 5CI? iç açıortaylar, 5CP? ise dış açıortaydır.

CI = x, CP = y, 5BP? + 5AC? = ! D + olsun.

ABC üçgeninde iç açıortay teoremi uygulandığında


AD 6 2k
DC = 9 = 3k & 2k + 3k = 5k = 5 & k = 1 dir.
O hâlde AD = 2 br ve DC = 3 br olur.

BCD üçgeninde iç açıortay teoremi uygulandığında


ID 3 n
BI = 9 = 3n dir.
5CP? ,CDB üçgeninin dış açıortayı olduğundan dış açıortay teoreminden
PD 3 PD 1
PD = 2n dir.
PB = 9 & PD + 4n = 3 &
ABC üçgeninde iç açıortay uzunluk formülü uygulandığında
]4ng2 = 6 $ 9 - 2 $ 3 = 54 - 6 = 48 & 16n 2 = 48 & n = 3 & IP = 3n = 3 3 br

Sıra Sizde
A
SORU:
ABC bir ikizkenar üçgen ve E noktası üçgenin iç teğet çemberinin
merkezidir.
m]EDCg = 14c
E %
F
AB = AC , DB = EC
m]BACg değerini bulunuz.
%
D B C
ÇÖZÜM:

Matematik 9 | 275
ÜÇGENLER

2. Üçgende Kenarortay

Üçgenin bir köşesinden karşı kenarın ortasına çizilen ve bu kenarı iki eşit uzunluğa bölen doğru parçasına
kenarortay denir.

ABC üçgeninde BD = DC olduğundan 5AD? , BC kenarının


kenarortayıdır. Bu kenarortayın uzunluğu AD = Va şeklinde
gösterilir.
Benzer şekilde AC kenarının kenarortayı Vb,
AB kenarının kenarortayı Vc ile gösterilir.

Teknoloji Uygulaması
Aşağıda GeoGebra programı kullanılarak bir üçgen ve bu üçgenin kenarlarına ait kenarortayları çizilmiştir.
Farklı üçgen çeşitlerine göre kenarortayların kesiştiği nokta incelenmiştir.
Çokgen aracını etkinleştiriniz. A, B ve C noktalarını sırasıyla işaretleyerek bir ABC üçgeni oluşturunuz.
(A noktasından başlayıp yine en son A noktasını seçiniz.)

Orta nokta veya merkez aracını etkinleştiriniz. Bir kenara ait orta noktayı bulmak için sırasıyla o kenarı
oluşturan köşeleri seçiniz. Bunu diğer kenarlara da uygulayarak kenarların orta noktalarını bulunuz.

Doğru parçası aracını etkinleştiriniz. Kenarortayları çizmek için üçgenin köşesi ile karşısındaki kenarın orta
noktasını birleştiriniz.

Kesiştir aracını etkinleştiriniz. Üçgenin kenarortaylarının kesişim noktasını bulmak için kenarortayları seçiniz.

Taşı aracını etkinleştiriniz. Üçgeni köşelerinden tutarak hareket ettiriniz. Farklı üçgen çeşitleri için
kenarortayların kesiştiği noktanın yerini inceleyiniz.

276 | Matematik 9
ÜÇGENLER

Ağırlık Merkezi

Kenarortaylar üçgenin içinde bir noktada kesişir. Bu noktaya üçgenin ağırlık merkezi denir.

Ağırlık merkezi, kenarortayı köşeye 2 birim, kenara 1 birim


oranında böler.
AE = EC , AF = FB , BD = DC olduğundan
5AD?, 5BE? ve 5CF? kenarortaydır.
AD = Va b_b
bb
BE = Vb `b şeklinde gösterilir.
b
CF = Vc bb
a

5AD? + 5BE? + 5CF? = ! G + noktası üçgenin ağırlık merkezidir. Bu durumda


AG = 2 GD , BG = 2 GE , CG = 2 GF tir.

ÖRNEK 1:

G noktası ABC üçgeninin, L noktası ise BEF üçgeninin ağırlık merkezidir.


BL = 8 cm olduğuna göre BD nu bulunuz.

ÇÖZÜM:

BEF üçgeninde L ağırlık merkezi olduğundan BL = 2 LG dir.


8 = 2 LG & LG = 4 cm olur.
ABC üçgeninde G ağırlık merkezi olduğundan BG = 2 GD dir.
12 = 2 GD & GD = 6 cm olur.
O hâlde BD = 12 + 6 = 18 cm olarak bulunur.

Matematik 9 | 277
ÜÇGENLER

BD = DC ve AE = EC ise 5AD? ve 5BE?


kenarortaydır. Bu durumda K noktası, ABC BD = DC ve AP = 2 $ PD
üçgeninin ağırlık merkezidir. ise P noktası, ABC üçgeninin ağırlık merkezi olur.

AR = 2 $ RD ve BR = 2 $ RE ise R noktası 5AD? kenarortay ve BS = 2 $ SE ise 5BE?


ABC üçgeninin ağırlık merkezi olur. kenarortaydır. Bu durumda S noktası ABC
Bu durumda 5AD? ve 5BE? kenarortaydır. üçgeninin ağırlık merkezi olur.

ÖRNEK 2:

ABC üçgeninde 5AE? = 5BC? ve 5BD? kenarortaydır.


AK = 8 cm, KE = 4 cm, AB = 13 cm
olduğuna göre ABC üçgeninin çevre uzunluğunu bulunuz.

ÇÖZÜM:

AK = 2 $ KE olduğundan AE kenarortay olur. Bu durumda K noktası,


ABC üçgeninin ağırlık merkezidir.
5AE? = 5BC? ve 5AE? kenarortay ise ABC ikizkenar üçgendir.
AB = AC = 13 cmbulunur.
2
ABE dik üçgeninde Pisagor teoreminden BE + 12 2 = 13 2 olur.
O hâlde
BE = 5 cmbulunur. ^5, 12 , 13 üçgeni h
Ç]ABCg = 13 + 13 + 10 = 36 cm olarak bulunur.
&

278 | Matematik 9
ÜÇGENLER

ÖRNEK 3:

ABC bir üçgen;


B,G,D ve C,G,E doğrusaldır.
m]BGFg = m]CGFg, AD = DC , AE = EB , BG = 6 br,
% %
EG = 2 br, FC = 8 br
olduğuna göre BF = x uzunluğunu bulunuz.

ÇÖZÜM:

ABC üçgeninde 5EC? ve 5BD? kenarortay olduğundan G, ağırlık


merkezidir.
GC = 2 $ GE olduğundan GC = 4 br olur.
BGC üçgeninde 5GF? iç açıortaydır.
İç açıortay teoreminden
GC FC
GB = FB olduğundan
4 8
6 = x & 4 $ x = 6 $ 8 & x = 12 br bulunur.

ÖRNEK 4:

ABC üçgeninde
m]BACg 2 90c
%
BD = DC , AB = 12 br, AC = 16 br
olduğuna göre AD = x değerinin alabileceği kaç tane tam sayı değeri
olduğunu bulunuz.

ÇÖZÜM:

D noktasından AB kenarına paralel olacak şekilde çizilen doğru AC


kenarını E noktasında kessin.
Bu durumda
m]BACg = m]DECg = b açısı geniş açı olup a açısı dar açıdır.
% %

Üçgen eşitsizliğinden 2 1 x 1 14 ...(1)


a dar açı olduğunda, x 2 1 6 2 + 8 2 & x 2 1 100 olup x 1 10 ...(2)
(1) ve (2) den 2 1 x 1 10 olduğundan x değerinin alabileceği 7 tane tam sayı değeri vardır.

Matematik 9 | 279
ÜÇGENLER

Bir dik üçgende hipotenüse ait kenarortayın uzunluğu, hipotenüsün uzunluğunun yarısına eşittir

ABC dik üçgeninde 5AD? , hipotenüse ait kenarortay ise


BD = DC = AD olup BC = 2 $ AD tir.

ÖRNEK 5:

ABC bir dik üçgen;


G, ABC üçgeninin ağırlık merkezi.
5AB? = 5AC? ve 5AH? = 5BC?, BH = 6 cm, AG = 8 cm
olduğuna göre AH = x değerini bulunuz

ÇÖZÜM:

5AG nın hipotenüsü kestiği nokta D olsun.


AG = 2 $ GD olduğundan 8 = 2 $ GD & GD = 4 br olur.
Buradan AD = 8 + 4 = 12 br elde edilir.

AD hipotenüse ait kenarortay olduğundan AD = BD = 12 br dir. Buradan HD = 6 br olur.


AHD dik üçgeninde Pisagor teoremi uygulandığında
144 = 36 + x 2 & 144 - 36 = x 2 & 108 = x 2 & x = 6 3 br bulunur.

ÖRNEK 6:

ABC dik üçgeninde G noktası, üçgenin ağırlık merkezidir.


AH = 9 br, HC = 15 br
olduğuna göre GH nun kaç birim olduğunu bulunuz.

ÇÖZÜM:

5BD? kenarortayı çizildiğinde


AD = DC = BD = 12 br ve HD = 3 br olur.
G noktası üçgenin ağırlık merkezi olduğundan
BG = 2 GD , BG = 8 br, & GD = 4 br olur.
Pisagor teoreminden
GH 2 + 3 2 = 4 2 & GH = 7 br olarak bulunur.

280 | Matematik 9
ÜÇGENLER

Orta Taban

Bir üçgenin iki kenarının orta noktalarını birleştiren doğru parçasına


orta taban denir.
ABC üçgeninde 5DE? orta taban ise 5DE? ' 5BC? ve BC = 2 $ DE tir.

L
ABC üçgeninde D ve E kenarların orta ABC üçgeninde
noktaları ise AK = KL olur. 5AF? kenarortay ve 5KL? ' 5BC? ise KE = EL olur.

ÖRNEK 7:

ABC üçgeninde
AD = DB ve AE = EC dir.
DE = 3x, BC = 4x + 10
olduğuna göre x değerini bulunuz.

ÇÖZÜM:
D ve E orta noktalar ise BC = 2 $ DE tir. Bu durumda
4x + 10 = 2 $ 3x & 2x = 10 & x = 5 br olarak bulunur.

ÖRNEK 8:

ABC üçgeninde
5AN? kenarortay ve 5KM? ' 5BC? dir.
KL = 5x + 3, LM = 7x - 9
olduğuna göre x değerini bulunuz.

ÇÖZÜM:
ABC üçgeninde 5KM? ' 5BC? ve 5AN? kenarortay olduğuna göre KL = LM dir. Buna göre
5x + 3 = 7x - 9 & 2x = 12 & x = 6 olarak bulunur.

Matematik 9 | 281
ÜÇGENLER

Bir ABC üçgeninde orta tabanların oluşturduğu üçgenin ağırlık


merkezi ile ABC üçgeninin ağırlık merkezi aynı noktadır.
5ED? orta tabanının 5AF? kenarortayını kestiği nokta L ise
AL - LG - GF
. . .
3k k 2k orantısı vardır.
2 AL = 6 LG = 3 GF tir.

ÖRNEK 9:

ABC üçgeninde
K, L, N noktaları kenarların orta noktalarıdır.
GN = 8 cm
olduğuna göre AP = x değerini bulunuz.

ÇÖZÜM:
K noktası 5AC? nın, L noktası 5AB? nın orta noktası olduğundan
5KL? orta tabandır.
Ayrıca 5AN? ve 5CL? kenarortay olduğundan G noktası üçgenin ağırlık
merkezidir.
Sırıasıyla 3k, k, 2k oranından AP = 3k, PG = k, GN = 2k tir.
O hâlde 2k = 8 ise k = 4 olarak bulunur.
Bu durumda x = 3k = 12 cm olur.

ÖRNEK 10:

ABC üçgeninde
AL = LB , BM = MC , 5BK? = 5AC?
Ç ^&
ABC h = 36 cm
olduğuna göre KLM üçgeninin çevre uzunluğunu bulunuz.

ÇÖZÜM:
BKC dik üçgeninde BM = MC = KM = y tir.
AKB dik üçgeninde AL = LB = KL = x tir.
Diğer taraftan ABC üçgeninde M ve L, kenar orta noktalar olduğundan
5ML? orta tabandır. AC = 2z yazılırsa ML = z olur.
Ç^KLM h = x + y + z olduğundan
&

Ç^ABC h = 2x + 2y + 2z = 2 ^x + y + z h = 36 & x + y + z = 18 cm tir.


&

Ç^KLM h = x + y + z = 18 cm olarak bulunur.


&

282 | Matematik 9
ÜÇGENLER

Kenarortay Uzunluğu

a) ABC üçgeninde a, b ve c kenarlarına ait kenarortaylar sırasıyla


Va, Vb, Vc olmak üzere üçgenin kenarları ile kenarortayları
arasında
a2
2V 2a = b 2 + c 2 - 2
b2
2V 2b = a 2 + c 2 - 2
c2
2V 2c = a 2 + b 2 - 2 bağıntıları vardır.

Bu bağıntılar taraf tarafa toplanıp düzenlenirse


4 ^V 2a + V 2b + V 2c h = 3 ^a 2 + b 2 + c 2 h bağıntısı bulunur.

b)

ABC dik üçgeninde m ^W A h = 90c ve


AS = Va, BP = Vb, CR = Vc olmak üzere
5V 2a = V 2b + V 2c tir.

ÖRNEK 11:

ABC üçgeninde
BD = DC = 4 br, AB = 9 br, AC = 7 br
olduğuna göre AD = x uzunluğunu bulunuz.

ÇÖZÜM:
ABC üçgeninde 5AD?, BC kenarına ait kenarortay olduğundan AD = x = Va tir.
a2
2V 2a = b 2 + c 2 - 2 eşitliği kullanılırsa
82
2x 2 = 7 2 + 9 2 - 2 & 2x 2 = 130 - 32 = 98
& x 2 = 49
x = x = 7 br olarak bulunur.

Matematik 9 | 283
ÜÇGENLER

ÖRNEK 12:

ABC dik üçgeninde G noktası ağırlık merkezidir.


BD = 5 2 cm ve CE = 5 3 cm
olduğuna göre BC = x değerini bulunuz.

ÇÖZÜM:

ABC dik üçgeninde D, E ve F kenar orta noktalar,


AF = Va, BD = Vb, CE = Vc
olduğundan
5V 2a = V 2b + V 2c ve BC = 2V a tir.
F
5V 2a = ^5 3 h + ^5 2 h = 75 + 50 = 125 olduğundan
2 2

V 2a = 25 & Va = 5 cm olur.
O hâlde BC = 2 $ Va = 2 $ 5 = 10 cm tir.

ÖRNEK 13:

ABC üçgeninde G noktası ağırlık merkezidir


5AN? = 5BM?, AN = 9 cm, BM = 12 cm
olduğuna göre AB kenarına ait kenarortayın uzunluğunu bulunuz.

ÇÖZÜM:

AB kenarına ait kenarortay 5CP? olsun.


G noktası ABC üçgeninin ağırlık merkezi olduğundan
PG = n ise GC = 2n tir.
GN = 3 cm, AG = 6 cm,
GM = 4 cm, BG = 8 cm tir.

AGB dik üçgen olduğundan AB = 10 cm tir.


O hâlde AP = PB = PG = 5 cm olup GC = 10 cm tir.
Bu durumda CP = 15 cm olur.

284 | Matematik 9
ÜÇGENLER

ÖRNEK 14:

ABC üçgeninde
5AF? açıortay, 5BD? kenarortay,
AB = 8 br, BF = 4 br, FC = 3 br
olduğuna göre DF = x değerini bulunuz.

ÇÖZÜM:

İç açıortay teoreminden
AB BF 8 4 AC = 6 br olur.
AC = FC & AC = 3 &

5AF? iç açıortay olduğundan


AF 2 = 8 $ 6 - 4 $ 3 = 48 - 12 = 36 olduğundan AF = 6 br olarak bulunur.
5FD?, AFC üçgeninin kenarortayı olduğundan kenarortay uzunluk bağıntısından:
62 27 3 3 3 6
2x 2 = 6 2 + 3 2 - 2 & 2x 2 = 27 & x 2 = 2 & x = = 2 br olarak bulunur.
2

Sıra Sizde

SORU:
ABC üçgeninde G noktası üçgenin ağırlık merkezi, P noktası
üçgenin iç açıortaylarının kesişim noktasıdır.
EC = 5 br, AC = 10 br, GP = 1 br, olduğuna göre AB
kenarının uzunluğunu bulunuz.

ÇÖZÜM:

Matematik 9 | 285
ÜÇGENLER

3. Üçgenin Kenar Orta Dikmeleri


Orta Dikme

Bir doğru parçasının orta noktasından geçen ve doğru


parçasına dik olan doğruya orta dikme doğrusu denir.
Yandaki şekilde AB doğru parçasının CH orta
dikmesidir.

Teknoloji Uygulaması
Aşağıda GeoGebra programı kullanılarak verilen bir doğru parçasının orta dikme doğrusunun nasıl
çizileceği açıklanmıştır.
Doğru parçası aracını etkinleştiriniz.
A ve B iki nokta seçerek doğru parçasını oluşturunuz.

Merkez ve bir noktadan geçen çember aracını etkinleştiriniz.


A merkezli B den geçen ve B merkezli A dan geçen iki çember çiziniz.

Kesiştir aracını etkinleştiriniz.


Bu iki çemberi seçerek kesiştikleri iki noktayı belirleyiniz.

Doğru aracını etkinleştiriniz.


Çemberlerin kesiştikleri iki noktayı seçerek doğruyu oluşturunuz.

Nokta aracını etkinleştiriniz.


Doğru üzerinde herhangi bir nokta seçiniz.

Doğru parçası aracını etkinleştiriniz.


Doğru üzerinde aldığınız herhangi bir noktadan (E noktası) A ve B noktalarına doğru parçaları çiziniz.

Uzunluk aracını etkinleştiriniz.


Çizilen doğru parçalarını seçerek bunların uzunluklarını bulunuz.

Taşı aracını etkinleştiriniz.


E noktasını, doğru üzerinde hareket ettirerek doğru parçalarının uzunluklarındaki değişimi inceleyin.

286 | Matematik 9
ÜÇGENLER

Üçgende Kenar Orta Dikme


Bir ABC üçgeninde A, B ve C noktalarından geçen çember üçgenin çevrel çemberi olarak isimlendirilir.
Bir üçgenin kenar orta dikmeleri tek noktada kesişir. Bu nokta, üçgenin çevrel çemberinin merkezidir.

Teknoloji Uygulaması
Aşağıda GeoGebra programı kullanılarak verilen bir üçgenin kenarlarına ait orta dikmelerinin ve üçgenin
çevrel çemberinin nasıl çizileceği açıklanmıştır.
Çokgen aracını etkinleştiriniz.
ABC üçgenini oluşturmak için A, B ve C noktalarını art arda seçiniz.
(A noktasından başlayıp yine en son A noktasını seçiniz.)

Orta dikme aracını etkinleştiriniz.


Kenarlara ait orta dikmeleri oluşturmak için kenarları seçiniz.

Kesiştir aracını etkinleştiriniz.


Orta dikmelerin kesiştikleri noktayı bulmak için orta dikmeleri seçiniz.
Kesişim noktasını D harfi ile isimlendiriniz.

Merkez ve bir noktasından geçen çember aracını etkinleştiriniz.


Çevrel çemberi oluşturmak için D ve A noktalarını seçiniz.

Doğru parçası aracını etkinleştiriniz.


Kenar orta dikmelerin kesiştiği nokta ile üçgenin köşelerinin oluşturduğu doğru parçalarını çiziniz.

Uzunluk aracını etkinleştiriniz.


Çizilen doğru parçalarını seçerek bunların uzunluklarını bulunuz.

Üçgeni köşelerinden tutarak hareket ettiriniz. Üçgenin farklı durumları için çevrel çemberi inceleyiniz.

Matematik 9 | 287
ÜÇGENLER

Yapılan GeoGebra çiziminde de görüldüğü gibi bir üçgenin kenar orta dikmelerinin kesiştiği noktanın
üçgenin köşelerine olan uzaklıkları eşittir ve bu uzaklık üçgenin çevrel çemberinin merkezidir.
ABC üçgeninde çevrel çemberin merkezi O, yarıçapı R ise AO = BO = CO = R birimdir.
Üçgenin çeşidine göre çevrel çemberinin merkezi için 3 farklı durum vardır.

Dar açılı üçgenin çevrel Dik üçgenin çevrel çemberinin Geniş açılı üçgenin çevrel
çemberinin merkezi, üçgenin iç merkezi, hipotenüsün orta çemberinin merkezi, üçgenin
bölgesindedir. noktasıdır. dış bölgesindedir.

ÖRNEK 1:

ABC üçgeninde D noktası, çevrel çemberinin merkezidir.


m ^BDC
% h = 114c olduğuna göre m ^%h = x değerini bulunuz.
BAC

ÇÖZÜM:

ABC üçgeninde D noktası çevrel çemberinin merkezi olduğundan


üçgenin köşe noktalarına eşit uzaklıktadır.
m^DAC h = m^DCA h = a
% %

m^DABh = m^DBA h = b
% %

m^DBC h = m^DCB h = i
% %

BDC üçgeninin iç açılarının ölçüleri toplamı: 2i + 114 = 180 & i = 33c dir.
ABC üçgeninin iç açılarının ölçüleri toplamı: 2a + 2b + 2i = 180 & a + b = 57c olarak bulunur.
O hâlde x = a + b = 57c tir.

288 | Matematik 9
ÜÇGENLER

ÖRNEK 2:

ABC üçgeninde P noktası, kenar orta dikmelerinin kesiştiği noktadır.


PH = 8 cm, AP + BP + PC = 51 cm
olduğuna göre 5AB? kenarının uzunluğunu bulunuz.

ÇÖZÜM:

P noktası, kenar orta dikmelerinin kesiştiği nokta olduğundan


AP = BP = PC = 17 cm olur.
PAH dik üçgeninde Pisagor teoremi uygulandığında
AH 2 + 8 2 = 17 2 & AH = 15 cm olduğundan AB = 30 cm tir.

ÖRNEK 3:

ABC üçgeninde D noktası, çevrel çemberinin merkezidir.


5DE? = 5AC?, 5DF? = 5BC?, EC = 7 cm, DE = 15 cm,
DF = 2 7 cm olduğuna göre BC kenarının uzunluğunu bulunuz.

ÇÖZÜM:

D noktası, ABC üçgeninde çevrel çemberin merkezi olduğundan


5DF?, 5BC? kenarına ait orta dikmedir.
DEC dik üçgeninde
DC 2 = DE 2 + EC 2 & DC 2 = 15 + 49 = 64 & DC = 8 cm tir.

DFC dik üçgeninde


DC 2 = DF 2 + FC 2 & 64 = 28 + FC 2 & FC = 6 cm tir.
FC = 6 cm olduğundan BC = 12 cm olarak bulunur.

Matematik 9 | 289
ÜÇGENLER

ÖRNEK 4:

ABC üçgeninde K noktası, kenar orta dikmelerinin kesişim


noktasıdır.
K noktasının 5AC? kenarına göre simetriği D,
5AB? kenarına göre simetriği E noktasıdır.
Buna göre BK = 6 br ise D ve E noktaları arasındaki
uzaklığı bulunuz.

ÇÖZÜM:

K noktası, üçgenin kenar orta dikmelerinin kesişim noktası olduğundan


üçgenin çevrel çemberinin merkezidir.
Bu durumda üçgenin çevrel çemberinin yarıçapı
BK = AK = 6 br
olduğundan AK = AE = AD = 6 br olur.
a + b = 30c & 2a + 2b = 60c olup AED
üçgeni eşkenardır.
O hâlde ED = 6 br olarak bulunur.

Sıra Sizde

SORU:
Üçgen biçiminde bir bahçenin tamamını sulayabilmek için bir fıskiye kullanılmak isteniyor. Buna göre
a) Bahçe dar açılı bir üçgen şeklinde olursa fıskiye nereye konulmalıdır?
b) Bahçe geniş açılı üçgen şeklinde olursa fıskiye nereye konulmalıdır?
a ve b seçeneklerinde bulduğunuz sonuçlardan yararlanarak bu durumun uygulanabilirliği hakkında ne
düşünüyorsunuz? Açıklayınız.

ÇÖZÜM:

290 | Matematik 9
ÜÇGENLER

4. Üçgende Yükseklik
Üçgenin bir köşesinden karşısındaki kenara çizilen dikmeye üçgenin o kenarına ait yüksekliği denir.
Bir ABC üçgeninin a, b, c kenarlarına ait h a, h b, h c olmak üzere üç yüksekliği vardır. Bir üçgende yükseklikler
tek noktada kesişir. Bu noktaya üçgenin diklik merkezi denir.

Teknoloji Uygulaması:
Aşağıda GeoGebra programı kullanılarak bir üçgen ve bu üçgenin yükseklikleri çizilmiştir. Farklı üçgen
çeşitlerine göre yüksekliklerin kesiştiği nokta incelenmiştir.

Çokgen aracını etkinleştiriniz. ABC üçgenini oluşturmak için A, B ve C noktalarını art arda seçiniz.

Dik doğru aracını etkinleştiriniz. Kenarlara ait yükseklikleri oluşturmak için üçgenin bir köşesini ve
karşısındaki kenarı seçiniz.
Kesiştir aracını etkinleştiriniz. Üçgenin yüksekliklerinin kesiştiği noktayı bulmak için yükseklikleri
seçiniz.
Taşı aracını etkinleştiriniz. Üçgeni köşelerinden hareket ettirerek farklı üçgen çeşitlerinde
yüksekliklerin kesiştiği noktanın yerini inceleyiniz.

Matematik 9 | 291
ÜÇGENLER

Üçgenin çeşidine göre diklik merkezi için üç farklı durum vardır.

1. ABC üçgeni dar açılı üçgen ise diklik merkezi üçgenin iç


bölgesindedir.
Diklik merkezi D noktasıdır.

2. ABC üçgeni dik üçgen ise diklik merkezi üçgenin dik köşesidir.
A noktası aynı zamanda diklik merkezidir.

3. ABC üçgeni geniş açılı üçgen ise diklik merkezi üçgenin dış
bölgesindedir.
Diklik merkezi D noktasıdır.

ÖRNEK 1:

ABC üçgeninde P noktası üçgenin diklik merkezi olmak üzere


m]BACg = 3x - 72c olduğuna göre x in alabileceği kaç tam sayı değeri
%
olduğunu bulunuz.

ÇÖZÜM:
ABC üçgeninde diklik merkezi üçgenin iç bölgesinde olduğundan üçgen dar açılı bir üçgendir. Buna göre
0c 1 m]BACg 1 90c & 0c 1 3x - 72c 1 90c
%

72c 1 3x 1 162c
24c 1 x 1 54c
olup x in alabileceği 29 farklı tam sayı değeri vardır.

292 | Matematik 9
ÜÇGENLER

İkizkenar Üçgenin Yardımcı Elemanları

ABC üçgeninde H d 5BC? olmak üzere

1. AB = AC Bu dört özellikten herhangi ikisi sağlanıyorsa


2. BH = HC üçgen ikizkenar olup diğer özellikler de
3. m]BAHg = m]HACg
% % sağlanır.

4. 5AH? = 6BC@

İkizkenar üçgende eşit kenarlara ait kenarortay


uzunlukları birbirine eşittir.

]]Z BD = CE
]]
]] AE = EB = AD = DC
AB = AC & []
]] BG = GC
]]
] EG = GD
\

İkizkenar üçgende eşit açılara ait açıortay


uzunlukları birbirine eşittir.

]]Z BP = CR
]]
AB = AC & [] AR = AP , RB = PC
]]
] SR = SP , SB = SC
\

İkizkenar üçgende eşit kenarlara ait yükseklik


uzunlukları birbirine eşittir.

]]Z BK = CL
]]
AB = AC & [] AL = AK , LB = KC
]]
] LM = MK , BM = MC
\

Matematik 9 | 293
ÜÇGENLER

ÖRNEK 2:

ABC ikizkenar üçgeninde


5BE? = 5AC?, AC = BC , m]ADB%g
= 45c, AD = 8 br
olduğuna göre BE kenarının uzunluğunu bulunuz.

ÇÖZÜM:
5BC? kenarına ait yükseklik 5AF? çizildiğinde
İkizkenar üçgende eşit kenarlara ait yükseklikler eşit olduğundan
AF = BE dir.
AFD ikizkenar dik üçgeninde
AF = FD = 4 2 br olduğundan
AF = BE = 4 2 br olarak bulunur.

ÖRNEK 3:

ABC ücgeninde
BD = 5 DC
AB = AC = 13 br
AD = 89 br olduğuna göre
BD değerini bulunuz.

ÇÖZÜM:
Tabana ait yükseklik 5AH? çizildiğinde
ABC üçgeni, ikizkenar olduğundan 5AH? aynı zamanda tabana ait
kenarortaydır.
DC = k denirse BD = 5k olduğundan BH = HC = 3k olur.
Pisagor teoreminden
AH 2 + 4k 2 = 89 ve AH 2 + 9k 2 = 169
89 - 4k 2 = 169 - 9k 2
5k 2 = 80 & k = 4 tür.
O hâlde BD = 5k = 20 br olarak bulunur.

294 | Matematik 9
ÜÇGENLER

ÖRNEK 4:

ABC ikizkenar üçgen ve AB = AC , 5CD? kenarortay ve


CD = BF , AE = EC , m]DCE
%g
= 50c
olduğuna göre m]BFEg = a açısının ölçüsünü bulunuz.
%

ÇÖZÜM:

AE = EC olduğundan B ile E noktaları birleştirilirse 5BE? kenarortayı


elde edilir.
AB = AC olduğundan bu kenarlara ait kenarortay uzunlukları eşittir.
O hâlde CD = BE = BF ve m]FBEg = m]ECDg = 50c tir.
% %

Bu durumda FBE üçgeni ikizkenar üçgen olup m]BFEg = m]BEFg = a tir.


% %

Dolayısıyla 2a + 50c = 180c & a = 65c olarak bulunur.

ABC ikizkenar üçgen,

İkizkenar üçgende taban üzerindeki AB = AC


herhangi bir noktadan eşit kenarlara P d 5BC?
çizilen paralellerin uzunlukları toplamı, eşit 5PE? ' 5AC?
5PD? ' 5AB? ise
kenarlardan birinin uzunluğuna eşittir.

PE + PD = AB = AC tir.

ABC ikizkenar üçgen,

AB = AC
İkizkenar üçgende taban üzerindeki P d 5BC?
5PE? = 5AB?
herhangi bir noktadan eşit kenarlara
çizilen dikmelerin uzunlukları toplamı, eşit
kenarlara ait yüksekliğe eşittir. 5PD? = 5AC? ise
PE + PD = BH tir.

Matematik 9 | 295
ÜÇGENLER

ÖRNEK 5:

ABC ikizkenar üçgeninde


AB = AC = 15 br,
5DE? ' 5AC?, 5DF? ' 5AB?,
DE - FD = 3
olduğuna göre DE değerini bulunuz.

ÇÖZÜM:
İkizkenar üçgende taban üzerindeki herhangi bir noktadan eşit kenarlara çizilen paralellerin toplamı, eşit
kenarlardan birinin uzunluğuna eşit olduğundan
DE + FD = 15

DE - FD = 3
+
2 DE = 18 & DE = 9 br olarak bulunur.

ÖRNEK 6:

ABC ikizkenar üçgeninde


AB = AC = 18 cm,
5DF? = 5AB?, 5DE? = 5AC?,
m]ABCg = 75c
%
DF = 5 cm
olduğuna göre DE değerini bulunuz.

ÇÖZÜM:

ABC ikizkenar üçgen olduğundan


m]ABCg = m]ACBg = 75c olduğundan m]BACg = 30c tir.
% % %
O hâlde B noktasından AC kenarına indirilen dikme
5BH? ise BH = 9 cm tir.
İkizkenar üçgende taban üzerindeki herhangi bir noktadan kenarlara
indirilen dikmelerin uzunlukları toplamı eşit kenarlara ait yüksekliğe eşit
olacağından
DE + DF = BH = 9 & DE = 9 - 5 = 4 cm olarak bulunur.

296 | Matematik 9
ÜÇGENLER

Eşkenar Üçgenin Yardımcı Elemanları

Eşkenar üçgende üç kenara ait yükseklik ve kenarortay uzunlukları ile üç açıya ait açıortay uzunlukları
eşittir.

AB = AC = BC = a ise
BD = DC = FC = AF = AE = EB = 2a
a 3
Va = Vb = Vc = n A = n B = n C = h a = h b = h c = h = 2 dir.

ÖRNEK 7:

ABC eşkenar üçgeninde


2 AB = 3 BD , AD = 4 7 br
olduğuna göre AB değerini bulunuz.

ÇÖZÜM:

AB = 6k ise BD = 4k ve DC = 2k dir.
A noktasından BC kenarına ait yükseklik çizilirse
6k 3
BH = 3k, HD = k ve AH = 2 = 3k 3 tür.
Pisagor teoreminden
^3k 3 h + k 2 = ^4 7 h & 27k 2 + k 2 = 16 $ 7
2 2

28k 2 = 16 $ 7
k2 = 4
k = 2 tir.
O hâlde AB = 6k = 12 br olarak bulunur.

Matematik 9 | 297
ÜÇGENLER

ÖRNEK 8:

ABCD dörtgeninde
Uh = m ^ C
m ^B Vh = 60c, 5DA? = 5AB?, AB = 6 br, AD = 2 3 br,
olduğuna göre DC değerini bulunuz.

ÇÖZÜM:

BA ve CD kenarları uzatıldığında oluşan BEC üçgeni eşkenardır.


EAD üçgeni 30c - 60c - 90c üçgeni olduğundan
AD = 2 3 br & AE = 2 br ve ED = 4 br olur.
BEC eşkenar üçgen olduğundan
BE = EC & 8 = x + 4 & x = 4 br olarak bulunur.

ÖRNEK 9:

ABC eşkenar üçgeninde


m]BADg = m]DBAg = 15c, AD = 7 2 br
% %
olduğuna göre AC değerini bulunuz.

ÇÖZÜM:

5DH? = 5AC? ve 5CE? = 5AB? olacak şekilde 6DH@ ve 6CE@ çizildiğinde


DHA üçgeni 45c - 45c - 90c ,
DHC üçgeni 30c - 60c - 90c olduğundan
AD = 7 2 br & AH = DH = 7 br olur.
DH = 7 br & HC = 7 3 br olup AC = 7 + 7 3 br olarak bulunur.

298 | Matematik 9
ÜÇGENLER

ÖRNEK 10:

ABC eşkenar üçgeninde


5AH? = 5BC?,
BF = FA ,
AH = CD olduğuna göre

m]EDCg = x açısının ölçüsünü bulunuz.


%

ÇÖZÜM:

C ile F noktaları birleştirilirse 5CF? kenarortay olacağından


CF = AH = CD dir.
Dolayısıyla FCD ikizkenar üçgen olduğundan m]CFDg = x olur.
%
ABC eşkenar üçgen olduğundan
m]FCBg = 30c & 2x = 30c & x = 15c olarak bulunur.
%

ABC eşkenar üçgen,


Eşkenar üçgenin iç bölgesinde P üçgenin içinde
veya kenarlarının üzerinde alınan herhangi bir nokta,
herhangi bir noktadan kenarlara 5PE? ' 5AC?
çizilen paralellerin uzunlukları 5PD? ' 5BC?
5PF? ' 5AB? ise
toplamı, eşkenar üçgenin bir kenar
uzunluğuna eşittir.

PD + PE + PF = AB = AC = BC tir.

Eşkenar üçgenin iç bölgesinde


veya kenarlarının üzerinde alınan ABC eşkenar üçgen, P üçgenin
herhangi bir noktadan kenarlara içinde herhangi bir nokta,
indirilen dikmelerin uzunlukları 5PE? = 5AC?
5PD? = 5BC?
toplamı eşkenar üçgenin bir
yüksekliğinin uzunluğuna eşittir.
5PF? = 5AB?
5AH? = 5BC? ise
PD + PE + PF = AH tir.

Matematik 9 | 299
ÜÇGENLER

ÖRNEK 11:

ABC eşkenar üçgeninde


5DH? = 5AB?, 5HF? = 5AC?, 5HE? = 5BC?,
AC = 16 br, HF = 4 3 br, HE = 3 3 br
olduğuna göre DH = x değerini bulunuz.

ÇÖZÜM:

ABC eşkenar üçgeninin bir kenarının uzunluğu 16 br olduğundan üçgenin


bir yüksekliği
16 3
CK = 2 = 8 3 br olur.
Eşkenar üçgenin içinde alınan bir noktanın kenarlara olan uzaklıklarının
toplamı, eşkenar üçgenin yüksekliğine eşit olduğundan
HD + HE + HF = CK & x + 3 3 + 4 3 = 8 3
& x = 3 br olarak bulunur.

ÖRNEK 12:

ABC eşkenar üçgen,


5EP? ' 5AC?, 5PD? ' 5BC?, 5PF? ' 5AB?,
2 PE = PD , PF = 5 cm, Ç]ABCg = 33 cm
&
olduğuna göre PD değerini bulunuz.

ÇÖZÜM:

PE = x ise PD = 2x tir.
5LP? ' 5BC? ve 5KP? ' 5AB?
çizildiğinde PEL ve PKD üçgenleri eşkenar olduğundan
PL = PE = EL = x ve PD = KP = KD = 2x olur.
PF = LB = 5 cm ve AE = KP = 2x & 3x + 5 = 11 & x = 2 cm tir.
Bu durumda PD = 2x = 4 cm olarak bulunur.

300 | Matematik 9
ÜÇGENLER

ALIŞTIRMALAR

1 5

ABC bir üçgen, 5AD? açıortay,


5AD? = 5AE?, DC = 2 br, BD = 3 br
ABC bir üçgen, K noktası üçgenin iç teğet
olduğuna göre EC değerini bulunuz.
çemberinin merkezi,|AB|=c, |AC|=b, |BC|=a
olmak üzere AB = c, AC = b, BC = a
AK b+c
2 olmak üzere KN = a olduğunu
gösteriniz.

6 ABC bir üçgen, G ağırlık


merkezi ve GL = 6 cm
olduğuna göre BD
ABC bir üçgen, 5BE? ve 5AD? iç açıortaylar, uzunluğunu bulunuz.
BD = 6 br, DC = 3 br, AB = 8 br
olduğuna göre PD değerini bulunuz.

3 7

ABC bir üçgen, 5AE? ve 5ED? açıortaylar,


ABC bir dik üçgen, G noktası üçgenin ağırlık
merkezidir.
DB = 2 br, AD = 3 br, ED = 3 2 br
GD = 2 br, AC = 4 3 br olmak üzere
olduğuna göre AC değerini bulunuz.
AB uzunluğunu bulunuz.

4 8

ABC bir üçgen, 5BD? iç açıortay, ABC üçgeninin ağırlık merkezi G,


AD = 5 br, EH = 3 br olduğuna göre ABG üçgeninin ağırlık merkezi T,
AB = x değerini bulunuz. ST = 1 br AP = 15 br, BR = 12 br
olduğuna göre AB uzunluğunu bulunuz.

Matematik 9 | 301
ÜÇGENLER

9 13

ABC ikizkenar üçgeninde,


m]BACg = 140c, AB = AC , 5BD? açıortay,
%
AD = AE , 5FH? = 5BC?, FE = 8 br
olduğuna göre FH değerini bulunuz.
ABC dik üçgen, AB = 10 br, EC = 2 br,
AD = DC , m]BAE
%g
= m]EADg olduğuna göre
%
BD uzunluğunu bulunuz.

10 14

ABC ikizkenar üçgeninde, ABC üçgeninde, AB = 10 cm, BC = 16 cm


AC = BC , m]ACB %g
= 30c ve G ağırlık merkezi, O iç teğet çemberin
5FE? ' 5AB?, 5FD? ' 5AC?, A noktasının 5BC? na merkezidir. OG = x değerinin bulunuz.
uzaklığı 7 br olduğuna göre EF = x değerini
bulunuz.

15
11

m]BACg = m]CADg, BE = 2 ED
% %
ABC eşkenar üçgen, 5DE? = 5AC?, 5FD? = 5AB?, AD = 4 cm, BC = 2 7 cm olduğuna
5BH? = 5AC?, DE = 4 3 br, EH = 2 br FD göre AC uzunluğunu bulunuz.
kenarının uzunluğunu bulunuz.

12 16 A
x

B E
18
6
ABC üçgeninde, 5AD? = 5BC? olmak üzere C D
AB = 17 br, CD = 2 br, m]CAD
%g
= a, m]DACg = 2m]CABg, AC = AD ,
% %
]% g
m ABC = 2a olduğuna göre AC kenarının AE = EC , BC = 6 cm, ED = 18 cm
uzunluğunu bulunuz. ise AB = x değerini bulunuz.

302 | Matematik 9
ÜÇGENLER

9.5.3. ÜÇGENLERDE EŞLİK VE BENZERLİK


1. Üçgenlerin Eşliği
Eşlik Kavramı
Sanayi, hizmet sektörü, sağlık vb. alanlarda yapılan üretimler belli bir standart ölçüsünde yapılmaktadır.
Belirlenen standarttan olumsuz bir sapma olduğunda yapılan bu üretimler, defolu ürünler kategorisine girer.
Bu doğrultuda, kullanılan bir ürün ya da parçanın bozulması ve yedeğinin ürüne uymaması hâlinde bu ürünlerin
eşliği hakkında düşüncelerinizi matematik diliyle nasıl ifade edersiniz? Bir ürünün yedeğinden bekledikleriniz
özellikler nelerdir? Belirtiniz.
Günlük hayatta eş parçalar olduğu gibi geometride de birbirine eş olan üçgen ve kare gibi şekiller bulunur.
Eş şekillerde karşılıklı noktalar, kenarlar ve açılar birbirine eştir. Eşlik " , " sembolü ile gösterilir.
• İki doğru parçasının uzunluğu eşit ise bu doğru parçaları eştir.
AB = CD ise 5AB? , 5CD? yazılır.
• Ölçüleri eşit olan iki açı eş açıdır.

m ^AOB
% h = m ^%h ise AOB
% %
KOL , KOL yazılır.

• Kenar uzunlukları eşit olan iki kare ya da yarıçapları eşit olan iki çember ise aynı şekillerdir.

AB = KL ise ABCD , KLNM M ve N merkezli çemberlerin yarıçapları eşit


olduğundan bu çemberler aynı çemberlerdir.
Üçgenlerde Eşlik
Karşılıklı kenarlarının uzunlukları ve karşılıklı açılarının ölçüleri eşit olan üçgenlere eş üçgenler denir.
&
ABC ve &
DEF için
Z] ^Vh
]]m C = m ^FTh
Z] AB = DE
]]
[] BC = EF ve []m ^W Vh
A h = m ^D
] ]
Uh = m ^EUh
]] ]]
] AC = DF ]m ^B
\ \
eşitlikleri sağlanıyorsa & ABC , &DEF yazılır.

Z] ^Vh
]Z] AB = DE
] ]]m C = m ^FTh
DEF & [] BC = EF ve []m ^W Vh eşitlikleri yazılır.
A h = m ^D
] ]
Tersine &
ABC , &
Uh = m ^EUh
]] ]]
] AC = DF ]m ^B
\ \
&
ABC , &DEF nin gösteriminde, karşılıklı açı ve kenarlar eşleştirildiğinden harflerin yazılış sırası önemlidir.

Matematik 9 | 303
ÜÇGENLER

Üçgenlerde Eşlik Kuralları


İki üçgenin eş olması için karşılıklı üçer kenarının ve karşılıklı üçer açısının eş olması gerekmez.
Verilen üçgenlerde en az biri kenar olmak üzere üçer elemanın eş olması üçgenlerin eşliği için yeterlidir.

a) Kenar-Açı-Kenar (K.A.K.) Eşlik Kuralı

İki üçgen arasında yapılan bire bir eşlemede karşılıklı ikişer kenar uzunlukları ve bu kenarlar arasında
kalan açı ölçüleri eşit ise bu üçgenler eş üçgenler olur. Örneğin
A D

55c 55c

9 12 9 12

B C E F
Şekildeki ABC ve DEF üçgenlerinde
AB = DE = 9 br, AC = DF = 12 br ve m ^W
A h = m ^W
D h = 55c olduğundan &
ABC , &
DEF dir.

ÖRNEK 1:
K 16
N
Yandaki şekilde 5LN? açıortay olduğuna göre x uzunluğunu bulunuz.
3x - 5

M
L

ÇÖZÜM:
KL = LM , LN = LN ve m ^%
KLN h = m ^MLN h olduğundan K.A.K. eşlik kuralına göre KLN , MLN dir.
% & &
Bu durumda KN = NM olur. Buradan 16 = 3x - 5 & 21 = 3x & x = 7 br bulunur.

ÖRNEK 2:

Yandaki şekilde verilenlere göre DE = x değerini bulunuz.

ÇÖZÜM:

ML = MP = 7 br, MN = MK = 12 br ve m ^LMK
% h = m ^ % h olduğundan K.A.K. eşlik kuralına göre
PMN
& &
KML , NMP dir. Bu durumda KL = NP olur. Buradan 15 = 2x - 3 & 18 = 2x & x = 9 br bulunur.

304 | Matematik 9
ÜÇGENLER

b) Açı-Kenar-Açı (A.K.A.) Eşlik Kuralı


İki üçgen arasında yapılan bire bir eşlemede karşılıklı ikişer açının ölçüleri ve bu açılar arasında kalan
kenar uzunlukları eşit ise bu üçgenler eş üçgenler olur. Örneğin

Şekildeki ABC ve DEF üçgenlerinde


m ^W D h = 50c, m ^V
A h = m ^W B h = m ^V
E h = 65c ve
AB = DE = 6 br olduğundan & ABC , & DEF dir.

ÖRNEK 3:

Yandaki şekilde
AE = 15 br, CD = x + 4, BD = 12 br, AB = y
olduğuna göre x + y değerini bulunuz

ÇÖZÜM:
m ^BCD
%h = m ^%h, m ^%h = m ^%h ve BC = BE olduğundan
BEA CBD EBA
& &
A.K.A. eşlik kuralına göre ABE , DBC dir.
Bu durumda AB = DB olduğundan y = 12 br ve AE = DC olduğundan 15 = x + 4 & x = 11 br
bulunur.
Buradan x + y = 11 + 12 = 23 br bulunur.

ÖRNEK 4:

Yandaki şekilde
5AB? = 5AC?, 5DE? = 5EC?, 5DC? = 5CB?, DC = CB ve
CE = 6 br, AC = 3 br olduğuna göre AB + ED değerini
bulunuz.

ÇÖZÜM:

m ^%
ACBh = x ve m ^DCE h = y olsun. m ^ACBh + m ^DCE h = 90c
% % %
% %
olduğundan ACB ve DCE tümler açılardır.
m ^%
ACB h = m ^CDE h = x ve m ^DCE h = m ^ABC h = y, DC = BC
% % %
olduğundan (A.K.A.) eşlik kuralına göre & & olur.
ABC , ECD
Bu durumda EC = AB = 6 ve AC = ED = 3 bulunur.
Buradan AB + ED = 6 + 3 = 9 br olur.

Matematik 9 | 305
ÜÇGENLER

c) Kenar-Kenar-Kenar (K.K.K.) Eşlik Kuralı

İki üçgen arasında yapılan bire bir eşlemede karşılıklı bütün kenar uzunlukları eşit ise bu üçgenler eş
üçgenlerdir. Bu durumda üçgenlerin karşılıklı açıları da eştir. Örneğin
A E
4

B 6
8
4

8
F 6 D
C
Şekildeki ABC ve DEF üçgenlerinde kenar uzunlukları eşit olduğundan bu üçgenler eş üçgenlerdir.
AB = DE = 4, BC = EF = 8, AC = DF = 6 olduğundan bu eşlik & ABC , &
DEF şeklinde gösterilir.
W W V V V U
Bu durumda m ^ A h = m ^ D h, m ^ B h = m ^ E h, m ^ C h = m ^ F h tir.

ÖRNEK 5:

K Yandaki şekilde
m ^% Xh
KLN h = 25c, m ^MNL h = 43c olduğuna göre m ^M
%
değerini bulunuz.
25 ° N
L 43 °

ÇÖZÜM:
Şekilde KL = ML , KN = MN , NL = NL olduğundan
& &
K.K.K. eşlik kuralına KLN , MLN tir.
m ^%
KLN h = m ^MLN h = 25c, m ^MNL h = m ^ KNL h = 43c olur.
% % %

LMN de m ^UL h + m ^YM h + m ^W


N h = 180c
&

25c + m ^Y
M h + 43c = 180c
m ^YM h = 180c - 68c = 112c olur.

ÖRNEK 6:
A D Yandaki şekilde
&
ABC , &DEF olduğuna göre x $ y $ z
2x + 5
14 15 3y + 2
çarpımını bulunuz.

B 17 C E F
2z + 3

ÇÖZÜM:
& &
ABC , DEF & AB = DE , AC = DF , BC = EF tir. Buna göre
AB = DE & 2x + 5 = 15 AC = DF & 3y + 2 = 14 BC = EF & 2z + 3 = 17
&x = 5 &y = 4 & z = 7 bulunur.
O hâlde x $ y $ z = 5 $ 4 $ 7 = 140 bulunur.

306 | Matematik 9
ÜÇGENLER

2. Üçgenlerin Benzerliği
Benzerlik Kavramı
Belli oranda büyültülmüş veya küçültülmüş şekillere benzer şekiller adı verilir.
Belli oranda büyütülmüş veya küçültülmüş şekillere benzer şekiller adı verilir.

Herhangi iki eşkenar üçgen birbirine Herhangi iki çember birbirine benzerdir.
benzerdir.

Üçgenlerde Benzerlik
İki üçgen arasında kurulan bire bir eşlemede, karşılıklı açıları eş veya karşılıklı kenarlarının uzunlukları
orantılı olan üçgenlere benzer üçgenler denir. Benzerlik durumu " + " sembolü ile gösterilir.

]Z] m ^W h ^Wh
]] A = m D
& & [ ^Vh
ABC + DEF ise ] m B = m ^V Eh
] m ^V
C h = m ^U
]]
Fh
\

&
ABC + &
DEF şeklinde gösterilir. Bu durumda üçgenlerin karşılıklı kenarları orantılıdır.
AB BC AC
DE = EF = DF = k ise k ye benzerlik oranı denir.
• k = 1 ise üçgenler eştir.

ÖRNEK 1:

& &
ABC + DEF
BC = 15 br, AC = 9 br
DF = 6 br ise x, a ve b
değerlerini bulunuz.

ÇÖZÜM:

DEF olduğundan m ^W D h = 70c, a = m ^V


A h = m ^W E h = 50c ve b = m ^V
B h = m ^V C h = m ^U
F h tir.
&
ABC + &
Bu durumda 50c + 70c + b = 180c olduğundan b = 60c tir.
AB BC AC 9 15
DE = EF = DF olduğundan 6 = x & x = 10 br olur.

Matematik 9 | 307
ÜÇGENLER

Üçgenlerde Benzerlik Teoremleri


a) Kenar-Açı-Kenar (K.A.K.) Benzerlik Teoremi
İki üçgenin karşılıklı ikişer kenarlarının uzunlukları orantılı ve bu orantılı kenarlar arasındaki açıları eş
ise bu iki üçgen benzerdir. Bu benzerliğe Kenar-Açı-Kenar (K.A.K.) benzerliği denir.
m ^W Vhb_b
A h = m ^D b & &
AB AC `bb ise ABC + DEF
DE = DF b
a
veya
AB AC & &
= ise ABC + DFE
DF DE

ÖRNEK 2:

Yandaki şekilde 5AB? ' 5DC?, AB = 16 br, AC = 12 br, BC = 18 br,


DC = 9 br ise AD değerini bulunuz.

ÇÖZÜM:
5AB? ' 5DC? olduğundan m ^DCA
% h = m ^%h dır.
CAB
Bu açıları oluşturan kenarların karşılıklı oranları
9 3 12 3
12 = 4 ve 16 = 4 olarak bulunur.
O hâlde K.A.K. benzerlik teoreminden & & dir.
ABC + CAD
Dolayısıyla orantılı kenarların karşısındaki açılar da eşittir.
3
Benzerlik oranı 4 tür. AD = x denildiğinde
x 3 54 27
18 = 4 & x = 4 = 2 br olur.

b) Kenar-Kenar-Kenar (K.K.K.) Benzerlik Teoremi

İki üçgenin karşılıklı kenar uzunlukları orantılı ise bu iki üçgen benzerdir. Bu benzerliğe Kenar-Kenar-
Kenar (K.K.K.) benzerliği denir.

AB AC BC Bu durumda
DE = DF = EF ise m ^W Vh, m ^B
A h = m ^D Uh = m ^EUh
& &
ABC + DEF dir. ve m ^CVh = m ^FTh olur.

308 | Matematik 9
ÜÇGENLER

ÖRNEK 3:

AB = 16 br, BC = 30 br, DE = 8 br, DC = 13 br, EC = 15 br,


AE = 11 br, m ^DEC
%h = 60c olduğuna göre m ^%h = a açısının
ABC
ölçüsünü bulunuz.

ÇÖZÜM:
DE 8 1 DC 13 1 EC 15 1
AB = 16 = 2 , AC = 26 = 2 , BC = 30 = 2 olup üçgenlerin kenarları orantılıdır. O hâlde K.K.K.

benzerlik teoreminden & & dir. Bu durumda orantılı olan kenarların karşısındaki açılar eşittir.
ABC + DEC
m ^W Vh, m ^BCA
A h = m ^D Uh = m ^EUh & a = 60c olur.
%h = m ^%h, m ^B
ECD

c) Açı-Açı (A.A.) Benzerlik Teoremi

İki üçgen arasında yapılan bire bir eşlemede karşılıklı ikişer açısı eş ise üçgenler benzerdir. Bu
benzerliğe Açı-Açı (A.A.) benzerliği denir.

m ^BUh = m ^EUh ve m ^W Vh
A h = m ^D
&+&
ise ABC DEF dir.
Dolayısıyla, k orantı sabiti ise
AB AC BC
DE = DF = EF = k tir.

ÖRNEK 4:

5AB? = 5BD?, 5ED? = 5BD?, 5AC? = 5EC?,


BC = 8 br, CD = 9 br, ED = 12 br,
AB = x, AC = y ise x + y değerini bulunuz.

ÇÖZÜM:
a + b = 90c olmak üzere
m ^BAC
%h = m ^%h = a ise
ECD
m ^ACB h = m ^CED
% %h = b olur.
& + CDE
O hâlde A.A. benzerlik teoreminden ABC & dir.
x 8 8
9 = 12 & x = 9 $ 12 = 6 br olur.
Pisagor teoreminden
x 2 + 8 2 = y 2 & 6 2 + 8 2 = y 2 & y 2 = 100 & y = 10 & x + y = 16 tir.

Matematik 9 | 309
ÜÇGENLER

ÖRNEK 5:

5AB? ' 5DE?, 3 BC = 2 CD , AE = 20 cm ise AC nu bulunuz.

ÇÖZÜM:
5AB? ' 5DE? olduğundan A.A. benzerliğinden & & olur. Bu
ABC + EDC
benzerliğe kelebek benzerliği denir.
AC = x denirse
BC 2 x 2
CD = 3 & 20 - x = 3 & 40 - 2x = 3x & 5x = 40 olup x = 8 tir.

ÖRNEK 6:

AFC bir üçgen


5AB? ' 5DE?, 5BD? ' 5FA?, m ^DBA
%h = m ^%h, DE = 4 br, EC = 6 br,
DBE
AF = 10 br, DB = x uzunluğunu bulunuz.

ÇÖZÜM:
Verilenlerden BED üçgeni ikizkenar üçgendir.
BE = 4 br dir. & &
ABC + DEC olduğundan
EC CD CD 3
EB = DA olup DA = 2 tir.
&+&
A.A. benzerliğinden CDB CAF olduğundan
DB CD CB x 3
AF = CA = CF & 10 = 5 & x = 6 br olur.

ÖRNEK 7:

ABC bir üçgen


m ^%
ABC h = m ^ACD h, AD = 4 br, BC = 8 br, AC = 6 br olduğuna
%
göre BD = x ve CD = y değerlerini bulunuz.

ÇÖZÜM: ABC ve ACD üçgenlerinin birer açıları eşittir. A açısı da ortak açı
olduğundan A.A. benzerlik teoreminden yola çıkıldığında bu iki üçgen
benzerdir. Bu durumda & & olur.
ABC + ACD
Dolayısıyla üçüncü açılar da birbirine eşit olup m ^ADC
% h = m ^%h olur.
ACB
O hâlde benzerlik oranı:
AD AC CD 4 6 y
AC = AB = BC & 6 = x + 4 = 8
& 4x + 16 = 36 & x = 5 tir.
32 16
& y = 6 = 3 br olarak bulunur.

310 | Matematik 9
ÜÇGENLER

ÖRNEK 8:

Aybige, güneşli bir günde bir binanın boyunu hesaplamak istiyor.


Aybige’nin boyu 160 cm, gölgesi ise 40 cm dir.
Binanın gölgesinin uzunluğu 12 m olduğuna göre binanın boyunu
hesaplayınız.

160 cm
9

1444442444443 5
12 m 40 cm

ÇÖZÜM:
İlk olarak problemde verilenlere karşılık gelen temsili şekiller çizilirse

12 m = 1200 cm
&
ABC + &
DEF olduğundan
x 1200
160 = 40 & x = 1200 $ 4 = 4800 cm = 48 m olarak bulunur.

ÖRNEK 9:

Sena, bir otoyolun genişliğini hesaplamak istiyor. Bunun için yolun karşı
tarafında bir tabela seçiyor. Tabelanın hizasından başlayarak yolun
kenarında 12 m yürüyor ve geldiği noktayı bir çubuk ile sabitliyor. Yolun
12 m 3m kenarında 3 m daha ilerledikten sonra yola dik olarak dönüp çubuk,
5m

tabela ve kendisi aynı hizaya gelinceye kadar 5 m daha yürüyor. Buna


göre otoyolun genişliğinin kaç m olduğunu bulunuz.

ÇÖZÜM:
Problemde verilenleri gösteren temsili bir şekil çizilirse

Otoyolun genişliği x olmak üzere A.A. benzerliğinden yola çıkıldığında


&+& 3 5
DEC ABC olduğundan 12 = x & x = 20 m olarak bulunur.

Matematik 9 | 311
ÜÇGENLER

ÖRNEK 10:

Bilge ile Belinay duvardaki aynaya baktıklarında birbirlerini

2m
görebilmektedirler. Bilge’nin duvara uzaklığı 2 m, aynaya uzaklığı 3 m
3m
dir. Belinay’ın duvara uzaklığı 4 m olduğuna göre Bilge ile Belinay’ın
4m

aralarındaki uzaklığı bulunuz. (Işığın ayna yüzeyine geliş açısı, yansıma


Bilge açısına eşittir.)

Belinay

ÇÖZÜM:
Buna göre verilen probleme karşılık gelen temsili şekil çizildiğinde ve
& + BDC
A.A. benzerlik teoreminden yola çıkıldığında ADE & dir.
Benzerlik oranı:
BC 2 1
AE = 4 = 2 tir.
Pisagor teoreminden
DC = 5 br olup ED = 2 5 br olur.
Bu durumda
EC = FB = 3 5 br & x 2 = 2 2 + ^3 5 h
2

= 4 + 45 = 49 & x = 7 m olarak bulunur.

ÖRNEK 11:
6m
Boyutları yandaki şekilde görüldüğü gibi olan bir duvarın alt tabanı üst
tabanına paraleldir. Tabanlara paralel olacak şekilde eşit aralıklarla 4
sıra hâlinde renkli şerit yapıştırılacaktır. Kaç metre şeride gereksinim
15 m 20 m
olduğunu bulunuz.

16 m
ÇÖZÜM:

5AB? ve 5DC? ye paralel çizilen 4 doğru parçası yapıştırılacak olan


şeritlerin yerlerini belirtsin. 5CB? ye paralel olacak şekilde 5DE?
çizildiğinde ADE üçgeninde karşılıklı kenarlar aynı sayıda eşit parçalara
ayrılırsa elde edilen üçgenler benzerdir.
En üstteki küçük üçgenin diğer üçgenler ile olan benzerlik oranları
1 1 1 1
sırasıyla 2 , 3 , 4 , 5 olur. Bu durumda üçgeni bölen doğru parçalarının
uzunlukları sırasıyla x, 2x, 3x, 4x, 5x olur.
5x = 10 & x = 2 tir.
Toplam şerit uzunluğu:
]x + 6g + ]2x + 6g + ]3x + 6g + ]4x + 6g = 10x + 24 = 44 m olur.

312 | Matematik 9
ÜÇGENLER

Benzerlik Özellikleri
&ABC + & DEF ve benzerlik oranı k olmak üzere
a) Benzer üçgenlerin karşılıklı kenarlara ait yükseklikleri, kenarortayları ve açıortayları orantılıdır. Bu oran
benzerlik oranına eşittir.

Yükseklikler orantılıdır.

ha hb hc
h d = h e = h f = k tir.

Açıortaylar orantılıdır.

nA nB nC
n D = n E = n F = k tir.

Kenarortaylar
orantılıdır.

Va Vb Vc
Vd = Ve = Vf = k tir.

Ç ^&
ABC h
b) Benzer üçgenlerin çevreleri oranı, benzerlik oranına eşittir. = k olur.
Ç ^ DEF h
&

Matematik 9 | 313
ÜÇGENLER

ÖRNEK 12:

& &, 3 BE = 4 LP , LN = 2 br, KN = 9 br


ABC + KLM
olduğuna göre AD ve BC değerlerini bulunuz.

ÇÖZÜM:
Üçgenler benzer ve AC ile KM kenarları orantılı olduğundan bu kenarlara ait yüksekliklerin oranı da
LP 3
benzerlik oranına eşittir. O hâlde BE = 4 benzerlik oranıdır.
LM 4 3 16
Bu durumda BC = BC = 4 & BC = 3 br olur.

Benzer şekilde, BC ile LM kenarları orantılı olduğundan bu kenarlara ait kenarortayların oranı da
benzerlik oranına eşittir.
KN 9 3
AD = 36
3 = 12 br olarak bulunur.
AD = AD = 4 &

ÖRNEK 13:

ABC üçgeninde KLMN karedir. BC = 12 cm, A]ABCg = 36 cm2 olduğuna


&
göre KLMN karesinin alanını bulunuz.

ÇÖZÜM:

Karenin bir kenarının uzunluğuna x denirse


KN = KL = MN = LM = x dir. AH = h olmak üzere

A]ABCg = 2 = 36 & 6h = 36 & h = 6 cm olur.


& h $ 12

5KN? ' 5BC? olduğundan AKN&+& ABC dir.


Benzer üçgenlerin yükseklikleri oranı da benzerlik oranına eşit
olduğundan
AP KN 6-x x x
AH = BC & 6 = 12 & 6 - x = 2 & 3x = 12 & x = 4 br dir.
O hâlde A ]KLMNg = x2 = 42 = 16 br2 olarak bulunur.

314 | Matematik 9
ÜÇGENLER

ÖRNEK 14:

5AB? ' 5EF? ' 5DC?, AB = y, EF = x, DC = z ise 1x = 1y + 1z


olduğunu gösteriniz.

ÇÖZÜM:
& & olduğundan x = CF CE
CEF + CAB y CB = CA tir.
& & olduğundan x = BE BF
EBF + DBC z BD = BC tir.
= 1 & x $ c y + z m = 1 & y + z = x tir.
x x CF BF BC 1 1 1 1 1
y + z = CB + BC = BC

ÖRNEK 15:
Aşağıda verilen şekilde A noktasında bulunan Serhat müzeye, B noktasında bulunan Serkan ise kütüphaneye
en kısa yoldan gitmek istemektedir. Kütüphanenin yola uzaklığı 600 m, müzenin yola uzaklığı 400 m dir.

Kütüphane
Müze

600 m C 400 m

A B

Serkan ve Serhat C noktasında karşılaştıklarına göre bu noktanın yola uzaklığını bulunuz.

ÇÖZÜM:

Problemde verilenleri gösteren temsili bir şekil çizildiğinde istenen uzaklık:


1 1 1 1 5
x = 600 + 400 & x = 1200 & x = 240 m olur.
^2h ^3 h

Matematik 9 | 315
ÜÇGENLER

ÖRNEK 16:

ABC dik üçgeninde, G noktası üçgenin ağırlık merkezidir.


BH = 2 br, GH = 4 br olduğuna göre AB + HC toplamını bulunuz.

ÇÖZÜM:

G noktası ağırlık merkezi olduğundan


GB = 2 5 br olup GD = 5 br olur.
AD = DC = BD = 3 5 br ve GH = KG = 4 br olarak bulunur.
& + DAB & olup 4 = 1 & x = 12 br olur.
DKG x 3
Pisagor teoreminden
AB 2 + BC 2 = AC 2 & 122 + BC 2 = ^6 5 h & BC = 6 br & y = 4 br
2

bulunur.
O hâlde AB + HC = x + y = 12 + 4 = 16 tir.

ÖRNEK 17:

5AC? ' 5DE?, AB = 8 br, BC = 4 br, DE = 6 br, PR = 2 br olduğuna


göre, AP uzunluğunu bulunuz.

ÇÖZÜM:

Kelebek benzerliğinden & & ve &


ABP + EDP & dir. Dolayısıyla
ACR + EDR
DR ER 6 1 DP EP 6 3
RC = RA = 12 = 2 ve PB = PA = 8 = 4 olur.
RE = x denirse AR = 2x olup AP = 2x - 2 dir. Benzerlik oranından
x+2 3
2x - 2 = 4 & 4x + 8 = 6x - 6 & 2x = 14 & x = 7 tir.
AP = 2x - 2 = 14 - 2 = 12 br olarak bulunur.

316 | Matematik 9
ÜÇGENLER

ÖRNEK18:

ABCD kare, AK = BL = LM , KB = 7 br, DM = 13 2 br olmak


üzere BN uzunluğunu bulunuz.

ÇÖZÜM:

AK = BL = LM = x, BN = y olsun.
AD = AB = KL = x + 7 olur. m]DAK %g
= m]KLMg = 90c
%
olduğundan K.A.K. eşlik teoreminden dolayı DAK ve KLM
& , KLM
üçgenleri eştir. DAK & dir. Dolayısıyla DK = KM
dir.
Bulunan eşitlikler şekil üzerinde aşağıdaki gibi gösterilirse
a + b = 90c olduğundan m]DKMg = 90c dir. O hâlde
%
DKM ikizkenar dik üçgendir. Pisagor teoreminden
DK = KM = 13 br olur. Ayrıca

x2 + ]x + 7g2 = 132 & 2x2 + 14x + 49 = 169 & 2x2 + 14x - 120 = 0
& x2 + 7x - 60 = 0 & ]x + 12g]x - 5g = 0 & x = 5 tir.

5BN? ' 5LM? olduğundan KBN


& + KLM
& dir. O hâlde

KB BN 7 y 7 y 35
KL = LM & x + 7 = x & 12 = 5 & y = 12 br olarak bulunur.

Temel Orantı Teoremi


Bir üçgenin bir kenarına paralel olan ve diğer iki kenarını kesen bir
doğru, kestiği kenarları orantılı olarak böler.

d ' 5BC? ise DB = EC olur.


AD AE

Bu teoremin karşıtı da doğrudur. ABC üçgeninde, D d 5AB? ve E d 5AC? olmak üzere


5DE? ' 5BC? dir.
AD AE
DB = EC ise

Matematik 9 | 317
ÜÇGENLER

ÖRNEK 19:

5DE? ' 5KL? ' 5BC?, AD = 6 br, DK = 3 br, KB = EL = 2 br,


LC = x - 3, AE = y olduğuna göre x ve y değerlerini bulunuz.

ÇÖZÜM:
Temel orantı teoreminden
6 y 9 y+2 9 6 12 4 13
3 = 2 & y = 4 olup 2 = x - 3 & 2 = x - 3 & x - 3 = 9 & x = 3 + 3 = 3 olarak bulunur.

ÖRNEK 20:

5PR? ' 5BC?, 5RS? ' 5AD?, 5ST? ' 5CE?


DT = 15 br, TE = 10 br, AP = x + 3 br, PB = x br olduğuna
göre x i bulunuz.

ÇÖZÜM:
Temel orantı teoreminden
x+3 AR DS 15 3
x = RC = SC = 10 = 2 & 2x + 6 = 3x & x = 6 tir.

ÖRNEK 21:

Yandaki şekilde ABC üçgenini kesen farklı doğrular ile bu doğruların


üçgenin kenarlarında ayırdığı uzunluk birimleri gösterilmiştir.
Buna göre verilen şekilde birbirine paralel olan doğrular olup
olmadığını inceleyiniz.

ÇÖZÜM:
d 1 ve d 2 için d 1 ve d 3 için d 1 ve d 4 için
10 8 olduğundan 10 Y 8 olduğundan 10 Y 8 olduğundan
5 = 4 6 = 4 22 = 16
d 1 ' d 2 dir. d 1 ve d 3 paralel değildir. d 1 ile d 4 paralel değildir.
d 2 ve d 4 için d2 ile d3 d 3 ve d 4 için
15 Y 12 olduğundan doğrularının ortak bir 16 12 olduğundan
17 = 12 16 = 12
noktaları olduğundan d2 ile d3
d 2 ile d 4 paralel değildir. d 3 ' d 4 dir.
paralel değildir.

318 | Matematik 9
ÜÇGENLER

3. Thales Teoremi
I. Thales Teoremi

İki farklı doğru, en az üç paralel doğru ile kesildiğinde orantılı parçalara


ayrılır.

AC BD
d 1 ' d 2 ' d 3 ise CE = DF tir.

ÖRNEK 1:

5KL? ' 5MN? ' 5OP? ' 5RS?


KM = 9 br, MO = 6 br, LN = 2x - 1 br,
NP = x + 2 br, PS = 2 br, OR = y br olduğuna göre x ve y
değerlerini bulunuz.

ÇÖZÜM:
9 2x - 1
I. Thales teoremine göre 6 = x + 2 & 3x + 6 = 4x - 2 & x = 8 br olur.
6 x+2 6 10 6
y = 2 & y = 2 & y = 5 br olur.

II. Thales Teoremi


Kesişen iki doğrunun, paralel iki doğru tarafından kesilmesiyle oluşan üçgenlerin kenarları orantılıdır.

d1 ' d2

AD AE DE AB AC BC
AB = AC = BC AE = AD = DE

Matematik 9 | 319
ÜÇGENLER

ÖRNEK 2:

5AP? ' 5BR? ' 5CS? ' 5DT?, 3 PR = 2 RS , AP = 3 br, BR = 5 br,


RS
CS = x br, DT = 10 br ise
ST oranını bulunuz.

ÇÖZÜM:

5AE? ' 5PT? doğru parçası çizilsin.


PR 2 2n
RS = 3 = 3n dir.
AETP bir paralelkenar olacağından AE = PT olur.
Thales teoremlerinden
2 2n 2 2n
x - 3 = 5n & x - 3 = 5 & x = 8 ve 7 = AE & AE = 7n tir.
Buradan PT = 7n ve ST = 2n olur.
RS 3
O hâlde ST = 2 olarak bulunur.

4. Benzerlik Uygulamaları
Menelaus Teoremi
ABC üçgeni verilsin. D d BC olmak üzere D den çizilen bir doğru
üçgenin diğer kenarlarını E ve F de kessin.
DB CF AE
DC $ FA $ EB = 1 tir.

İspat:
ABC üçgeninde D d BC olmak üzere D den çizilen bir doğru,
üçgenin diğer kenarlarını E ve F de kessin. ABC üçgeninde B den 5AC?
ye çizilen paralel bir doğru 5DF? yi K noktasında kessin.
Kelebek benzerliğinden & KBE + & FAE olduğundan
BK BE KE
AF = AE = EF tir. ... (1)
& + DCF
A.A. benzerliğinden DBK & olduğundan
BK DB DK
CF = DC = DF tir. ... (2)
BE
(1) bağıntısında BK = AF $ AE eşitliği (2) de yerine yazıldığında
BE
AF $
AE DB AF BE DB DB CF AE
CF = DC & CF $ AE $ DC & DC $ FA $ EB = 1 olarak bulunur.

320 | Matematik 9
ÜÇGENLER

ÖRNEK 1:

Şekilde verilen ABC üçgeninde


AF = FC , BC = 2 BD , AE = x + 2, EB = x olduğuna göre
x değerini bulunuz.

ÇÖZÜM:

1. Yol: DB = nise BC = 2n tir.


n x+2
Menelaus teoreminden: 3n $ 1 $ x = 1 & x + 2 = 3x & 2x = 2 & x = 1 olarak bulunur.

B den 5AC? ye çizilen paralel doğru 5DF? yi P de kessin.


2. Yol:

DB = n, PB = m ise BC = 2n ve FC = 3m olur.
&
EPB + &
EFA olduğundan
PB EB 1 x
AF = AE & 3 = x + 2 & 3x = x + 2 & x = 1 tir.

Seva Teoremi
ABC üçgeni verilsin. 5AF?, 5BE? ve 5CD? doğru parçaları üçgenin içindeki
P noktasında kesişsin. Bu durumda
AD BF CE
DB $ FC $ EA = 1 tir.

İspat:
BF CE AP
AFC üçgenine Menelaus teoremi uygulandığında $ $ = 1 tir. ... ^1 h
BC EA PF
CF BD AP
ABF üçgeninde Menelaus teoremi uygulandığında $ $ = 1 tir. ... ^2 h
BC DA PF
AP BC EA
^1 h den = $ ifadesi yazılır. Bu ifade (2) de yerine yazıldığında
PF BF CE
CF BD AP CF BD BC EA
$ $ = $ $ $
BC DA PF BC DA BF CE
CF BD AE
= $ $ = 1 olarak bulunur.
FB DA CE

Matematik 9 | 321
ÜÇGENLER

ÖRNEK 2:

Şekildeki ABC üçgeninde


AE = 2 EC , 3 BF = 2 FC , BD = 2 br olduğuna göre AB değerini
bulunuz.

ÇÖZÜM:

EC = n, AE = 2n, BF = 2k, FC = 3k,


ve AB = x olsun. AD = x - 2 tir.

Seva teoreminden
2 1 x-2
3 $ 2 $ 2 = 1 & x - 2 = 6 & x = 8 tir. O hâlde, AB = 8 br olur.

Steawart Teoremi
ABC üçgeninde D d 5BC?, BD = m, DC = n, AB = c, AC = b
olmak üzere

İspat:
A noktasından 5BC? ye bir dikme indirilsin.
AH = h, DH = y, HC = n - y olmak üzere Pisagor teoreminden
h 2 + y 2 = x 2 ... ]1g
h 2 + ^n - y h = b 2 ... ]2g
2

h 2 + ^m + yh = c 2 ... ]3g
2

(1) ve (2) den 2ny = n2 + x2 - b2


(1) ve (3) den 2my = c2 - x2 - m2 eşitlikleri bulunur.
Bulunan eşitlikler oranlanırsa
n n2 + x2 - b2 2 2 2 2 2 2
m c2 - x2 - m2 & nc - nx - nm = mn + mx - mb
=

& ]m + ng x2 = nc2 + mb2 - mn ]m + ng


nc2 + mb2 - mn ]m + ng c2 n + b2 m
& x2 = m+n = m + n - mn olarak bulunur.

322 | Matematik 9
ÜÇGENLER

ÖRNEK 3:

ABC üçgeninde AB = 6 br, AC = 4 br, BD = 3 br, AD = x br


olduğuna göre x değerini bulunuz.

ÇÖZÜM:
Steawart teoremi kullanılırsa

36 $ 2 + 16 $ 3
x2 = 3+2 -2$3
72 + 48
x2 = 5 -6
x 2 = 24 - 6
x 2 = 18 ise x = 3 2 br olarak bulunur.

Steawart
Carnot Teoremi
Teoremi

ABC üçgeninde P, üçgenin içindeki herhangi bir noktadır.


AD = m, AE = y, EC = n, DB = x, BF = p, FC = z
olmak üzere
x 2 + y 2 + z 2 = m 2 + n 2 + p 2dir.

İspat:

5AP?, 5BP? ve 5CP? doğru parçaları çizilsin. Pisagor teoreminden


m2 + DP 2 = AP 2 = y2 + PE 2
p2 + PF 2 = BP 2 = x2 + DP 2 eşitlikleri yazılır.
n2 + PE 2 = CP 2 = z2 + PF 2

Taraf tarafa toplama işlemi yapılırsa


m2 + DP 2 + p2 + PF 2 + n2 + PE 2 = y2 + PE 2 + x2 + DP 2 + z2 + PF 2 olup, x2 + y2 + z2 = m2 + n2 + p2
eşitliği bulunur.

Matematik 9 | 323
ÜÇGENLER

ALIŞTIRMALAR

1 4

ABC bir üçgen, m]BAEg = m]CBFg = m]ACDg


% % %
Yandaki şekilde AB = BC , 5AB? = 5BC?,
5AE? = 5BD?, 5BD? = 5DC?, DC = 4 br,
DE = 4 br, DF = 6 br, EF = 5 br,
AC = 18 br olduğuna göre AB + BC
DE = 8 br olduğuna göre AF - FC
toplamını bulunuz.
değerini bulunuz.

2 5

ABC bir üçgen, AC = AB , CF = EB ,


CD = FB , m]CAB
%g
= 42c olduğuna göre
m]DEFg = x açısının ölçüsünü bulunuz. ABCD bir dörtgen, m]ACDg = m]BCAg
% % %
AB = 12 br, BC = 16 br, AC = 24 br,
DC = 36 br olduğuna göre AD = x
uzunluğunu bulunuz.

3 6

ABCD karesinde ABCD bir paralelkenar olmak üzere,


AE = 3 br, EB = 6 br, 5AE? = 5EB? ise AE 2 = EF $ ET olduğunu gösteriniz.
AED açısının ölçüsünü bulunuz.

324 | Matematik 9
ÜÇGENLER

Mustafa, deniz fenerine çıkarak denizdeki bir


geminin kıyıya uzaklığını gönye kullanarak ölçmek
istiyor. Gönyenin dik kenarları 20 cm-60 cm dir.
Gönyeyi şekilde görüldüğü gibi uzun dik kenarı
zemine, paralel ve ucu ile gemi göz hizasında
olacak şekilde tutuyor. Gönyenin zemine olan
uzaklığı 70 m olduğuna göre geminin fenere olan
uzaklığının kaç m olduğunu bulunuz.

8
Aylin, iki dağın zirvesi arasındaki yaklaşık mesafeyi ölçmek için sırasıyla şu adımları izlemektedir.
I. Adım:
Bulunduğu A noktasından kulelere baktığında iki çubuğu uç noktaları dağların zirvesi ile aynı hizaya
gelecek şekilde göz hizasında tutuyor.
II. Adım:
Bu şekilde elde ettiği üçgenin resmini, tabanı dağların zirvelerinin bulunduğu hatta paralel olacak
şekilde karton üzerine çiziyor. Çizdiği üçgenin taban uzunluğunu 30 cm, yüksekliğini ise 4 cm olarak
ölçüyor.

30 cm
C
4 cm B
A
Aylin

III. Adım:
Bulunduğu noktadan 100 m geriye giderek (dağların zirvelerinin bulunduğu hatta dik olacak şekilde)
D noktasına geldiğinde de aynı şekilde bir üçgen daha oluşturuyor. Yeni elde ettiği üçgenin taban
uzunluğunu 25 cm, yüksekliğini ise 5 cm olarak ölçüyor.

Verilenlere göre aşağıdaki soruları cevaplayınız.


a) Dağların zirveleri arasındaki mesafe yaklaşık kaç
metredir?
b) Aylin’in bulunduğu A noktasından 150 m geriye
giderek geldiği K noktasında oluşturacağı temsili
bir KLM üçgeninin taban ve yükseklik uzunluğunu
bulunuz.
c) Bu şekildeki benzer soruların daha kolay
hesaplanabilmesi için bir genelleme yapılabilir mi?
Açıklayınız.

Matematik 9 | 325
ÜÇGENLER

9.5.4 DİK ÜÇGENDE TRİGONOMETRİ


1. Dik Üçgende Pisagor Teoremi
Pisagor, Yunan filozof ve matematikçidir. Matematik bilimine ispat
özelliğini getiren bilim insanıdır. Doğadaki her şeyi matematik bilimi ile
açıklayıp yorumlamaya çalışmıştır.
Pisagor’a göre sayılar nesnelerin özünü oluşturmaktadır. Pisagor,
matematiğin doğanın temel yasalarını çözen bir anahtar olduğunu
kabul eder. Bu konudaki kayıtlı ilk sistematik düşünceler, MÖ 600-400
yıllarında Pisagor yanlısı bir gruba aittir. Bu gruba Pisagorcular denir.
Pisagor’un hayatı hakkında çok fazla bilgi olmamasına rağmen
dik üçgen ile ilgili teoremi, geçmişten günümüze dek gelen en ünlü
teoremlerinden biridir. Pythagoras (Pisagor)

Bir açısı 90 derece olan üçgene dik üçgen denir. Dik üçgende 90
derecenin karşısındaki kenara hipotenüs, diğer kenarlara dik kenar
a
adı verilir.
c
Pisagor Teoremi
Bir dik üçgende dik kenarların kareleri toplamı hipotenüsün
karesine eşittir.
b
a2 = b2 + c2

Pisagor teoreminin birçok farklı şekilde ispatı yapılmıştır. Benzerlik yöntemi ile ispatı aşağıdaki gibi
yapılabilir.

ABC dik üçgeninde


m ^AVh = 90c, AB = c, BC = a, AC = b
6AD@ = 6BC@ lacak biçimde h yüksekliği çizilir.
& + CBA
Açı açı benzerlik kuralından CAD & olacağından
CA CD
= & CA 2 = CD $ CB ... (1)
CB CA
& + CBA
Açı açı benzerlik kuralından ABD & olacağından
AB BD
= & AB 2 = CB $ BD ... (2)
CB BA
(1) ve (2) eşitliklerin toplamı alınırsa
CA 2 + AB 2 = CD $ CB + CB $ BD
b 2 + c 2 = CB $ _ CD + BD i
b2 + c2 = a2

Şekildeki dik kenarlar


üzerindeki karelerin alanları 2
toplamı hipotenüsün üzerinde c
kurulan karenin alanına eşittir. a
2
a c
Bu ispat, 1917 yılında H.E. b
Dudeney tarafından yandaki
şekildeki gibi gösterilmiştir. b
2

a2 + b2 = c2

326 | Matematik 9
ÜÇGENLER

ÖRNEK 1:

a
bb
Sulama yapmak için Nil Nehri’nin suyunu yükseltmeye yarayan

bb
bb
bb
13 br

bb
Arşimet vidası yanda verilmiştir. Şekilde verilen bilgilere göre vidanın uç

bb
_
b

bb
b

`b
b
kısmı duvardan kaç m uzağa yerleştirilmelidir? b

bb
b

bb
b
b

bb
b

bb
5 br

bb
`
b
ÇÖZÜM:

bb
b

bb
b
b

_b
b
b
b
b
a
13 br

a
bbb
bb
bb
bb
bb
bb
bb
`
bbb
bb
bb
bb
bb
bb
bb_
5 br Pisagor teoreminden x 2 + 5 2 = 13 2 ?

x = 12 br
x

ÖRNEK 2:
Yandaki şekilde
5AB? = 5CD?
AC = 6 cm
AD = 4 cm
DB = 3 cm olduğuna göre
CB = x değerini bulunuz.
ÇÖZÜM:
ACD üçgeninde Pisagor teoremi yapılarak CD CDB üçgeninin üçgeninde Pisagor teoremi
kenarının uzunluğu bulunur. yapılarak CB kenarının uzunluğu bulunur.
2
CD = 62 - 42 CB 2 = CD 2 + DB 2

x 2 = ^2 5 h + 3 2
2 2
CD = 36 - 16
2
CD = 20 x 2 = 29
CD = 2 5 cm x = 29 cm

ÖRNEK 3:
Bir bilardo oyuncusu yaptığı gösteride şekildeki gibi bir vuruş
125 cm

gerçekleştirerek topu deliğe sokmak istiyor. Topun alacağı en kısa yolun


uzunluğunu bulunuz.
25 cm
200 cm

ÇÖZÜM:

Topun alacağı en kısa yolu bulmak için A noktasının simetriği alınır.


BCD dik üçgeni oluşturulup Pisagor teoremi uygulanırsa
x 2 = 200 2 + 150 2
x = 250 cm olur.

Matematik 9 | 327
ÜÇGENLER

ÖRNEK 4:
Yandaki şekilde BC = BD + 1
5CB? = 5BA?
AD = 4 cm 13
CD = 13 cm olduğuna göre
AC = x değerini bulunuz.
ÇÖZÜM:
BCD üçgeninde Pisagor teoremi uygulandığında CBA üçgeninde Pisagor
2
BC + BD = 13 2 teoremi uygulandığında
BC = BD + 1 için ^ BD + 1h2 + BD 2 = 13 x2 = 32 + 62
BD 2 + 2 BD + 1 + BD 2 = 13 x 2 = 9 + 36
BD 2 + BD - 6 = 0 x = 3 5 cm bulunur.
^ BD - 2 h $ ^ BD + 3 h = 0
& BD = 2 cm bulunur.
2 2 2
BC = CD - BD = 13 - 4 = 9 & BC = 3 cm olur.

ÖRNEK 5:
Yandaki şekilde
5BC? = 5AC?
5BD? = 5AD?
BC = 4 cm
AC = 5 cm
BD = 3 cm olduğuna göre
AD = x değerini bulunuz.

ÇÖZÜM:
BCA ve ADB üçgenlerinde Pisagor teoremi uygulanıp birbirine eşitlendiğinde
AB 2 = 4 2 + 5 2 AB 2 = x 2 + 3 2
4 2 + 5 2 = x 2 + 3 2
16 + 25 = x 2 + 9
x 2 = 32
x = 4 2 cm bulunur.

Kenarları Tam Sayı Olan Dik Üçgenler

3k 4k 5k 5k 12k 13k 8k 15k 17k 7k 24k 25k


3 4 5 5 12 13 8 15 17 7 24 25
6 8 10 10 24 26 16 30 34 14 48 50
... ... ... ... ... ... ... ... ... ... ... ...

328 | Matematik 9
ÜÇGENLER

ÖRNEK 6:
Yandaki şekilde ABC, CBD ve EFC dik üçgenler
AC = 3 cm
AB = 4 cm
BD = 12 cm
DE = 2 cm
EF = 8 cm
olduğuna göre CF kenarının uzunluğunu bulunuz.

ÇÖZÜM:
ABC dik üçgeninde dik kenar uzunlukları 3 cm ve 4 cm olduğundan bu
üçgen 3-4-5 dik üçgeni olup CB kenarının uzunluğu 5 cm dir.

CBD dik üçgeninde dik kenar uzunlukları 5 cm ve 12 cm


olduğundan bu üçgen 5-12-13 dik üçgeni olup CD kenarının
uzunluğu 13 cm dir.
CEF dik üçgeninde dik kenar uzunlukları 8 cm ve 15
cm olduğundan bu üçgen 8-15-17 dik üçgeni olup CF kenarının
uzunluğu 17 cm olur.

2. Öklid Teoremi
5AB? = 5AC? ve 5AD? = 5BC?
ABC üçgeninde hipotenüse ait yükseklik çizildiğinde oluşan ABD
ve ACD üçgenlerinin açıları eş olduğundan Açı Açı benzerlik
& &
kuralına göre ABD + CAD olur.
Benzer üçgenlerde eş açılar karşısındaki kenarlar orantılı
olduğundan
AD BD AD 2 = CD $ BD
CD = AD ise
Bir dik üçgende hipotenüse ait yüksekliğin uzunluğunun
karesi, bu yüksekliğin hipotenüs üzerinde ayırdığı parçaların
uzunluklarının çarpımına eşittir.
h2 = p $ k
Bir dik üçgende, bir dik kenarın uzunluğunun karesi,
hipotenüse ait yüksekliğin hipotenüste ayırdığı parçalardan kenara
yakın olanın uzunluğu ile hipotenüsün uzunluğunun çarpımına
eşittir.
&
ABC + DBA& olduğundan c 2 = p $ a
& &
ABC + DAC olduğundan b 2 = a $ k

Matematik 9 | 329
ÜÇGENLER

ÖRNEK 1:
ABC üçgeninde
5AB? = 5AC?, 5AD? = 5BC?
AB = 6 cm, DC = 5 cm olduğuna göre
AD = x değerini bulunuz.

ÇÖZÜM:
ABC üçgeninde Öklid teoremi AB dik kenarı için uygulandığında
6 2 = BD $ ^ BD + 5h
36 = BD 2 + 5 BD BD 2 + 5 BD - 36 = 0
BD = 4 cmbulunur.
Benzer şekilde Öklid teoremi yükseklik için uygulandığında
x2 = 4 $ 5
x 2 = 20

x = 2 5 cm bulunur.
ÖRNEK 2:

ABC üçgeninde
5AB? = 5BC?
5BH? = 5AD?
DH = 2cm
AH = 8cm
DC = 2 5 cm olduğuna göre AC kenarının uzunluğunu bulunuz.

ÇÖZÜM:
DBA dik üçgeninde BD ve AB dik kenarları için Öklid teoremi uygulandığında
BD 2 = 2 $ 10 AB 2 = 8 $ 10
BD = 2 5 cm AB = 4 5 cm
ABC üçgeninde AC kenarının uzunluğunu bulmak için Pisagor teoremi uygulandığında
AC 2 = _4 5 i + _4 5 i & AC 2 = 160 & AC = 4 10 cm elde edilir.
2 2

ÖRNEK 3:
5AD? = 5BD?, 5BC? = 5AC?
5CE? = 5BA?, 5DF? = 5BA?
BE = 2cm
AF = 3cm
BC = x
AD = y olduğuna göre x oranını bulunuz.
y

ÇÖZÜM:
ABC dik üçgeninde BC dik kenarının, ADB dik üçgeninde de AD dik kenarının uzunluğunu bulmak için Öklid
teoremi uygulanıp oranlama yapılırsa
x 2 = 2 $ BA
4 x2 = 2
2
& x = 2 bulunur.
2
y = 3 $ AB y 3 y 3

330 | Matematik 9
ÜÇGENLER

ÖRNEK 4:
5AD? = 5CB?
5AC? = 5CE?
5BA? = 5CA?
AD = 4cm
DE = 1cm
AB = x olduğuna göre değerini bulunuz.

ÇÖZÜM:
ACE dik üçgeninde CD dik kenarının uzunluğunu bulmak için Öklid teoremi uygulandığında

AD 2 = CD $ DB
4 2 = 2 $ DB
DB = 8 cm bulunur.
2 8 CD 2 = AD $ DE
CD 2 = 4 $ 1
CD = 2 cm bulunur.
ADB dik üçgeninde AB hipotenüs uzunluğunu bulmak için
Pisagor teoremi uygulandığında
x2 = 42 + 82
x 2 = 80
x = 4 5 cmbulunur.

ÖRNEK 5:

5AC? = 5BD?

AB = 2 3 cm
BC = 5cm
CD = 7cm
AD = x olduğuna göre değerini bulunuz.

ÇÖZÜM:
OB = a, AO = b, OD = d, OC = c
olmak üzere ABO, AOD,COB ve DOC dik üçgenlerinde Pisagor
teoremi uygulandığında aşağıdaki eşitlikler elde edilir.
a 2 + b 2 = 12 a 2 + c 2 = 25
+ c + d 2 = 49 + b2 + d = x2
2 2

a 2 + b 2 + c 2 + d 2 = 61 ... (1) a 2 + b 2 + c 2 + d 2 = x 2 + 25...(2)


(1) ve (2) denklemleri eşitlenirse x 2 + 25 = 61
x 2 = 36
x = 6 bulunur.

Matematik 9 | 331
ÜÇGENLER

6AC@ = 6BD@ olmak üzere 6AH@ = 6BC@ olmak üzere


a +d = b +c
2 2 2 2
a2 + d2 = b2 + c2

ÖRNEK 6:
ABC dik üçgeninde G ağırlık merkezi olmak üzere

6AB@ = 6AC@
6GE@ ' 6BC@
GD = 3 cm
GE = x olduğuna göre değerini bulunuz.

ÇÖZÜM:

ABC dik üçgeninde G ağırlık merkezi olduğundan AG kenarının uzunluğu GD kenarının uzunluğunun iki
katıdır. Dik üçgende hipotenüse ait kenarortayın uzunluğu hipotenüsün uzunluğunun yarısı olduğundan BC
kenarının uzunluğu 18 cm bulunur. GE ile DC paralel olduğundan benzerlik kuralı uygulandığından
AG GE 6 GE
= & 9 = 9 & GE = 6 cm bulunur.
AD DC

332 | Matematik 9
ÜÇGENLER

ÖRNEK 7:
A
Yandaki şekilde
6AB@ = 6BC@
D AD = DC
E
AB = 16cm
FB = 6cm
F
BC = 18cm olduğuna göre FD kenarının uzunluğunu bulunuz.
6 B 18 C

ÇÖZÜM:

A
ABC dik üçgeninde D noktasından BC kenarına inilen yüksekliğin
uzunluğu AB kenarının uzunluğunun yarısı kadardır. Ayrıca BC
16
kenarını iki eş parçaya böler.
D Yeni oluşan DEF dik üçgeninin dik kenarları 8cm ve 15 cm
olduğundan bu üçgen 8-15-17 dik üçgeni olup FD kenarının
17
8 uzunluğu 17 cm olur.

F C
6 B 9 E 9

ÖRNEK 8:
Yandaki ABC dik üçgeninde E noktası AB kenarının orta noktası
A olmak üzere
6AB@ = 6AC@
E
BE = EA
3 BD = DC
B C
D DC + ED = 16 olduğuna göre ED kenarının uzunluğunu bulunuz.

ÇÖZÜM:
BD = x ise DC = 3x
A Hipotenüse ait kenarortayın uzunluğu, hipotenüsün uzunluğunun yarısı
olacağından 2x olur.
E ED kenarı, kenarortaya paralel olacağından ED = x olur.
2x DC + ED = 16
x

B x x C 3x + x = 16
D 2x
4x = 16
x=4

Matematik 9 | 333
ÜÇGENLER

ALIŞTIRMALAR

ABC ikizkenar dik üçgen olmak üzere Aşağıdaki şekilde


1 5

6AB@ = 6AC@ 6AB@ = 6AC@


AB = AC AE = EC
BC = 12 cm ED = 8 cm
CD = 2 cm DC = 6 cm

AD kenarının uzunluğunu bulunuz. olduğuna göre AB kenarının uzunluğunu


bulunuz.

2
Aşağıdaki şekilde 6 Aşağıdaki şekilde
6AB@ = 6AC@
6AB@ = 6AD@
AD = DC
6AC@ = 6BC@
BD = 37 cm
AB = AD
BC = 8 cm
AC = 3 br
olduğuna göre DC = x uzunluğunu bulunuz. BC = 6 br

olduğuna göre DC = x uzunluğunu bulunuz.

Aşağıda verilen ABC dik üçgeninde 7


Aşağıdaki şekilde
3
6AB@ = 6AC@
6AD@ = 6BC@
6AD@ = 6BC@
AB = 3 cm
AD = 2 6 cm
BE = 2 cm
AB = 33 cm
EC = 5 cm
AC = x olduğuna göre
olduğuna göre x değeri kaç cm dir? AC kenarının uzunluğunu bulunuz.

4
Aşağıdaki şekilde 8
Aşağıdaki şekilde C noktası AD doğru parçası
üzerindedir.
6BH@ = 6AC@
6AB@ = 6AD@
AB = AC
6DE@ = 6AD@
DC = 1 cm
AB = 14cm
BC = 17 cm
BE = 17cm
DE = 6cm
olduğuna göre AB = x uzunluğunu bulunuz. BC =x
CE =y
olduğuna göre x+y toplamının alabileceği en
küçük değeri bulunuz.

334 | Matematik 9
ÜÇGENLER

3. Dik Üçgende Dar Açıların Trigonometrik Oranları

Trigonometri, bilim ve teknolojide en çok kullanılan matematik tekniklerinden biridir. El Battani, Güneş’in ve
Ay’ın hareketlerinin ölçümüne bağlı olarak gece ve gündüzün başlangıç saatlerinin tam olarak belirlenebilmesi
için trigonometriyi kullanmıştır. Trigonometrinin tam bir disiplin hâline getirilmesi Battani’nin zamanından
itibaren Müslüman matematikçilerin eseridir.
Matematikçi Nasırüddin Tusi “Dörtgenler Üzerine İncelemeler” adlı çalışmasında trigonometriye yer
vermiştir. Nasuriddin Tusi, konuya gökbilimin bir parçası olarak değil matematiksel olarak yaklaşır.
Trigonometri; arazi ölçümlerinde, haritacılıkta, GPS uydu sistemlerinde veya bir kıtanın haritasının
çıkarılmasında hayati önem taşır. Özellikle engebeli arazilerde mesafe ölçümünde topograflar, nirengi
(üçgenlere ayırma) sürecinde hesaplama yaparken trigonometriyi kullanırlar.

Trigonometri, açıları aynı olan benzer dik üçgenlerin belirlenen kenarlarının uzunlukları arasındaki
oranların değişmediğini gösterir. Bu oranlara trigonometrik oranlar denir.

Yandaki şekilde ABG, ACF ve ADE üçgenleri Bu benzerlik aşağıdaki şekildeki gibi ifade
A.A. benzerlik kuralına göre benzerdir. edilebilir.
AB AC AD
AG = AF = AE
AB AC AD
BG = CF = DE
BG CF DE
AG = AF = AE
BG CF DE
AB = AC = AD
Yukarıdaki oranların sabitliği trigonometride sinüs, kosinüs, tanjant, kotanjant olarak isimlendirilir.

• Bir açının sinüs değeri, dik üçgende açının karşısında bulunan dik kenarın uzunluğunun hipotenüs
uzunluğuna oranıdır. Kısaca sin ile gösterilir.
• Bir açının kosinüs değeri, dik üçgende açıya komşu olan dik kenar uzunluğunun hipotenüs
uzunluğuna oranıdır. Kısaca cos ile gösterilir.
• Bir açının tanjant değeri, dik üçgende açının karşısında bulunan dik kenar uzunluğunun açıya
komşu olan dik kenar uzunluğuna oranıdır. Kısaca tan ile gösterilir.
• Bir açının kotanjant değeri, dik üçgende açıya komşu olan dik kenar uzunluğunun açının karşısında
bulunan dik kenar uzunluğuna oranıdır. Kısaca cot ile gösterilir.

Matematik 9 | 335
ÜÇGENLER

Karşı Dik Kenar Uzunluğu


sin a =
Hipotenüsün Uzunluğu
Komşu Dik Kenar Uzunluğu
cos a =
Hipotenüsün Uzunluğu
Karşı Dik Kenar Uzunluğu
tan a =
Komşu Dik Kenar Uzunluğu
Komşu Dik Kenar Uzunluğu
cot a =
Karşı Dik Kenar Uzunluğu

ÖRNEK 1:
Bir ABC dik üçgeninde
m ^W
A h = 90c
AB = 4 cm
AC = 3 cm
olmak üzere sin V
C, cos V
C, tan V
C, cot V
C değerlerini bulunuz.
ÇÖZÜM: B

Pisagor teoremini kullanarak BC kenarının uzunluğu bulunur.


BC 2 = 3 2 + 4 2
BC = 5 cm 4
5

C açısının karşısındaki dik kenarın uzunluğu 4 cm, C açısının


komşusundaki dik kenarın uzunluğu 3 cm ve hipotenüs uzunluğu 5 cm dir.
Uygun dik üçgen yandaki gibi çizilirse aşağıdaki oranlar bulunur.

sin V cos V
AB 4 AC 3 A 3
C
C = BC = 5 C = BC = 5

tan V cot V
AB 4 AC 3
C = AC = 3 C = AB = 4

ÖRNEK 2:
Sandal üzerinde hareket eden bir kişi 5 m ilerledikten sonra
12 m
suyun derinliğini 2 m olarak belirliyor. Su üzerinde 12 m ilerlediğinde 5m
derinliğin kaç metre olduğunu bulunuz. 32m
_b
bb
bb
b
`b
bb
bb
b
a
ÇÖZÜM:
A açısının tanjant değeri ADE ve ABC üçgenlerini kullanılarak hesaplandığında eşit olacağından

_
V=
tan A
DE 2
= 5 bb
AD 2 x 24
` 5 = 12 & 5x = 24 & x = 5 m dir.
V=
tan A
BC x
= 12 bb
AB
a

336 | Matematik 9
ÜÇGENLER

ÖRNEK 3:
Bir lisede öğrencilerden boylarını
farklı bir yöntem kullanarak
hesaplamaları istenmiştir.
Öğrenci, kendi boyunu
hesaplamak için iletki kullanacaktır.
Öğrencinin iletkinin orta noktasına
olan uzaklığı 200 cm ve şekildeki gibi
açısı 40c olarak hesaplanmıştır.
Öğrencinin boyunu bulunuz.
^tan 40c = 0, 84 h
200 cm

ÇÖZÜM:
A
Öğrencinin boyu x olarak alındığında aşağıdaki dik üçgen oluşturulur. C açısına ait
tanjant değeri yazıldığında
AB = x için
AB x
tan 40c = BC & tan 40c = 200

x = 200 $ 0, 84
40c x = 168 cm tir.
B 200 cm C

Tümler Açıların Trigonometrik Oranları

Dik üçgende bulunan dar açıların trigonometrik oranları alındığında


a a
sin x = c cos y = c
b b
cos x = c sin y = c
a a
tan x = b cot y = b
b b
cot x = a tan y = a

Toplamları 90 dereceye eşit olan açılardan birinin sinüs değeri diğerinin kosinüs değerine, birinin
tanjant değeri diğerinin kotanjant değerine eşittir.
x + y = 90c & sin x = cos y ve tan x = cot y
Örneğin
sin 30c = cos 60c tan 25c = cot 65c
sin 70c = cos 20c cot 3c = tan 87c
sin 1c = cos 89c tan a = cot ^90 - ah

Matematik 9 | 337
ÜÇGENLER

ÖRNEK 4:

sin 24c $ tan 56c işleminin sonucunu bulunuz.


cot 34c $ cos 66c
ÇÖZÜM:
Tümler açılardan birinin sinüsü diğerinin kosinüsüne, birinin tanjantı diğerinin kotanjantına eşittir. Buna
göre
sin 24c = cos 66c sin 24c $ tan 56c
olacağından = 1 elde edilir.
cot 34c = tan 56c cot 34c $ cos 66c

ÖRNEK 5:

7a = 90c ise cos 5a $ tan 3a işleminin sonucunu bulunuz.


sin 2a $ cot 4a
ÇÖZÜM:
2a + 5a = 7a = 90c & cos 5a = sin 2a cos 5a $ tan 3a
olacağından = 1 elde edilir.
3a + 4a = 7a = 90c & tan 3a = cot 4a sin 2a $ cot 4a

ÖRNEK 6:
sin ^a - 2bh $ cot ^a + 4bh
tan ^2a - 3bh $ cos ^2a + 3bh
3a + b = 90c ise işleminin sonucunu bulunuz.

ÇÖZÜM:

^a - 2bh + ^2a + 3bh = 3a + b = 90c & sin ^a - 2bh = cos ^2a + 3bh
^2a - 3bh + ^a + 4bh = 3a + b = 90c & cot ^a + 4bh = tan ^2a - 3bh
sin ^a - 2bh $ cot ^a + 4bh
tan ^2a - 3bh $ cos ^2a + 3bh
= 1 elde edilir.

45°- 45°- 90° Dik Üçgeninde Trigonometrik Oranlar

m ^\
ABC h = 90c
Yandaki ABC ikizkenar dik üçgeninde
AB = BC = k
Pisagor teoreminden AC = k 2 olarak bulunur.
Buna göre ABC üçgenindeki A açısının trigonometrik oranları
aşağıdaki gibidir.

BC k 2
sin 45c = AC = = 2
k 2
AB k 2
cos 45c = AC = = 2
k 2
BC k
tan 45c = AB = k = 1
AB k
cot 45c = BC = k = 1

338 | Matematik 9
ÜÇGENLER

30°- 60°- 90° Di̇k Üçgeni̇nde Tri̇gonometri̇k Oranlar

Yandaki ABC eşkenar üçgeninin bir köşesinden karşı kenara dik


inildiğinde bu yükseklik aynı zamanda açıortay ve kenarortaydır.
Buradan 30c - 60c - 90c üçgeni elde edilmiş olur.
30c lik açının karşısındaki dik kenarın uzunluğu, hipotenüs
uzunluğunun yarısına eşittir.
60c lik açının karşısındaki dik kenarın uzunluğu, 30c lik açının
karşısındaki dik kenarın uzunluğunun 3 katına eşittir. Buna göre 30c
ve 60c lik açıların trigonometrik oranları aşağıdaki gibidir.

BH k 1 AH k 3 3
sin 30c = AB = 2k = 2 sin 60c = AB = 2k = 2
AH k 3 3 BH k 1
cos 30c = AB = 2k = 2 cos 60c = AB = 2k = 2
BH k 3 AH k 3
tan 30c = AH = = 3 tan 60c = BH = 3 = 3
k 3
AH k 3 BH k 3
cot 30c = BH = 3 = 3 cot 60c = AH = = 3
k 3

ÖRNEK 7:

sin 30c $ cos 60c + tan 45c $ sin 2 60c işleminin sonucunu bulunuz.
cot 60c $ cos 30c - cot 45c
ÇÖZÜM:
c 3m
2
1 1 1 3
$ + 1 $
sin 30c $ cos 60c + tan 45c $ sin 2 60c 2 2 2 4 + 4 = 1 =- 2
= = 1 1
cot 60c $ cos 30c - cot 45c 1 3
$ 2 -1 2 - 1 -2
3

ÖRNEK 8:

6AB@ = 6BC@
\h = 15c
m ^CAD
\h = 30c
m ^ADC
8
2
AC = 8 2 cm olduğuna göre CD uzunluğunu bulunuz.

ÇÖZÜM:
İki iç açının toplamı kendilerine komşu olmayan bir dış açıya eşit
olduğundan m ^ACB\h = 45c dir.
Buradan AB = BC = 8 cm olur. ABD dik üçgeninin açıları
30c - 60c - 90c olduğundan BD kenarının uzunluğu AB kenarının
uzunluğunun 3 katıdır.
8
2

BD = 8 3 cm
CD = BD - BC
CD = _8 3 - 8i cm dir.

Matematik 9 | 339
ÜÇGENLER

Sıra Sizde

SORU:
A noktasında bulunan Mert, eli hizasında bulunan sepeti 12 m
uzunluğundaki ip ile makara yardımıyla yukarı çıkarmak istiyor. Buna
göre
• Mert ipi çekerek B noktasına geldiğinde kova ne kadar
yükselmiş olur?
• Mert’in sepeti tam yukarıya çıkarabilmesi için ip ile zemin
60c ?
arasındaki açı ölçüsü kaç derece olmalıdır? A B C
ÇÖZÜM:

15°- 75°- 90° Di̇k Üçgeni̇nde Tri̇gonometri̇k Oranlar

15c - 75c - 90c dik üçgeninde hipotenüsün uzunluğu,


hipotenüse ait yüksekliğin uzunluğunun 4 katıdır.
BC = 4 AD

İspat:
ABC dik üçgeninde A açısından 15 derecelik kısım
ayrıldığında ikizkenar üçgen elde edilmiş olur. Oluşan
30c - 60c - 90c üçgeninden dolayı hipotenüse ait
kenarortay uzunluğu 2h olur. Hipotenüsün uzunluğu,
hipotenüse ait kenarortay uzunluğunun 2 katı
olacağından Hipotenüs uzunluğu 4h olur.

ÖRNEK 9:
A
6AB@ = 6AC@
AB = 3 cm olduğuna göre AC kenarının uzunluğunu bulunuz.
3

15c
C
B

ÇÖZÜM:
BD = DC olacak şekilde bir BD doğru parçası
çizildiğinde
3 3 \h = 15c, m ^BDA
m ^DBC \h = 30c, m ^ABD
\h = 60c
D
3 15c DC = BD = 6 cm
6 6
60c AD = 3 3 cm
15c 15c
AC = 6 + 3 3 cm bulunur.

340 | Matematik 9
ÜÇGENLER

ÖRNEK 10:

Yandaki ABC dik üçgeninde


x 6AB@ = 6AC@
AB = 4 cm
olduğuna göre AC kenarının uzunluğunu bulunuz.

ÇÖZÜM:
BD = DC olacak şekilde bir BD doğru parçası çizildiğinde
\h = 22, 5c, m ^BDA
m ^DBC \h = m ^ABD
\h = 45c olacağından

AD = 4 cm
DC = BD = 4 2 cm
AC = 4 + 4 2 cm bulunur.

Tri̇gonometri̇k Oranlardan Bi̇ri̇ Belli̇ İken Di̇ğeri̇ni Bulma


Trigonometrik oranlardan biri belli iken diğerlerinin bulunması istendiğinde
• Verilen trigonometrik orana uygun dik üçgen çizilir.
• Çizilen dik üçgen yardımıyla diğer trigonometrik oranlar hesaplanır.

ÖRNEK 11:
0c 1 a 1 90c ve cos a = 5 ise sin a, tan a ve cot a değerlerini bulunuz.
13
C
ÇÖZÜM:
5
cos a = 13 & AB = 5, AC = 13

BC 2 = 13 2 - 5 2
13 12
BC = 12 cm
12 12 5
sin a = 13 , tan a = 5 , cot a = 12
olacak şekilde bir ABC dik üçgeni çizilir. A 5 B

ÖRNEK 12:
0c 1 2a 1 90c ve sin 2a = 4 olduğuna göre cot a değerini bulunuz.
5
ÇÖZÜM:
sin 2a = 4 oranına göre aşağıdaki üçgen oluşturulur.
5
ABC dik üçgenine göre

BD 8
cot a = = 4 = 2 olarak bulunur.
AB

Matematik 9 | 341
ÜÇGENLER

ALIŞTIRMALAR

1
A Yandaki ABC dik üçgeninde 4 Aşağıda verilen şekilde 6AB@ = 6BC@
x 6AB@ = 6BC@ AB = 2 br
AB = 2 10 br A AC = 3 br
CE = 4 + 20 br
2 10 BC = 3 br olduğuna göre
2
3 CD = 6 br olduğuna göre

2y 4+ 20
B C E
3 B C
Aşağıdaki trigonometrik oranları bulunuz.
sin x = tan x = 6 x
DE kenarının uzunluğunu
sin 2y = tan y =
bulunuz. D
cos x = cot x =
cos 2y = cot y =

5 26x = 180 ise sin 10x $ cos 4x $ tan x


sin 9x $ cos 3x $ cot 12x
işleminin sonucunu bulunuz.
2 Aşağıda birim karelerle oluşturulmuş şekilde
tan a + cot b
işleminin sonucunu bulunuz.
sin a + cos 2 b

6 sin 2 45c + cos 30c $ tan 30c


cot 2 30c - cos 60c $ sin 2 60c
işleminin sonucunu bulunuz.

Yandaki şekilde 7 2 sin x - 3 cos x 2


3
3 cos x - 7 sin x = 3
6CF@ = 6AB@
olmak üzere sinx değerini bulunuz.
AD = DC
\h = 30c
m ^FCA
AF = 6 cm
olduğuna göre

BF uzunluğunu bulunuz.

342 | Matematik 9
ÜÇGENLER

8 10

60c
53c
16 m
37c 53c
3.15 m x
A B

Bir elektrik teknisyeni, kopan bir kabloyu Bir güvenlik kamerası, 16 m yükseklikteki
şekildeki gibi A noktasından başlayarak bir binanın üzerine yerleştiriliyor. Kamera
toplamaya çalışıyor. Gergin durumdaki yatayla 53c lik açı yapacak şekilde ayarlanırsa
kablonun zemin ile yaptığı açı 37cdir. karşıdaki binanın en alt noktasını, yatayla 60c
Teknisyen 3,15 m yürüdüğünde B noktasına lik açı yapacak şekilde konumlandırılırsa aynı
geliyor ve kablo ile zemin arasındaki açının binanın en üst noktasını görüntüleyebiliyor.
53c olduğunu görüyor. Buna göre Buna göre
a) Elektrik direğinin uzunluğunu a) Kameranın yerleştirildiği bina ile diğer
hesaplayınız. bina arasındaki uzaklığı bulunuz.
b) Teknisyenin A noktasından B noktasına
b) İkinci binanın yüksekliğini hesaplayınız.
gelinceye kadar kaç m kablo sardığını
bulunuz. ^sin 53c = 0, 8 h
^sin 37c = 0, 6 h

9 Aşağıda verilen ABD üçgeninde 11 6BC@ = 6AC@, 6DE@ = 6AC@, AE = EC


A \h = 105c ve AD = 8 br
m ^EDB
4 2 olduğuna göre BC kenarının uzunluğunu
4 bulunuz. C
22, 5c
B
C 4 D
E
\h = 22, 5c, AC = CD = 4cm ve
m ^ADB x
AB = 4 2 cm
105c
olduğuna göre BC kenarının uzunluğunu
bulunuz. A 8 D B

Matematik 9 | 343
ÜÇGENLER

9.5.5 ÜÇGENİN ALANI

Etkinlik
ALAN HESAPLAMA
Birim karelerden oluşan bir düzlemde karelerin köşelerini kullanarak oluşturulan şekillerin
alanlarını farklı bir yöntem geliştirerek bulmaya çalışınız.
1. Aşağıdaki şekilde verilen çokgenlerin alanlarını bulunuz. Tabloya yazınız.

A C A B C
Şeklin
B Alanı

2. Çokgenlerle ilgili aşağıdaki tabloyu doldurunuz.

Çokgenlerin Kenarlarının Üzerinden Çokgenlerin İçinde Kalan Nokta


Şekil Adı
Geçtiği Nokta Sayısı Sayısı
A
B
C

3. Çokgenlerin kenarlarının geçtiği nokta sayısı ve içindeki nokta sayısını kullanarak çokgenin alanını
veren bir formül bulunuz.

4. Bulduğunuz formülü kullanarak aşağıdaki şekillerin alanlarını bulunuz. Elde ettiğiniz bu formüle
Pick teoremi denir.

5. Sizce bu yöntemi kullanarak günlük hayatta karşılaşılan hangi şekillerin alanlarını bulabilirsiniz?

344 | Matematik 9
ÜÇGENLER

1. Üçgenlerde Alan Uygulamaları


Üçgenlerde Temel Alan Bağıntısı
Alan hesaplama işlemi ile hayatın birçok yerinde karşılaşılmaktadır.
Mesela bir ülkenin yüzölçümü, bir ilin yüz ölçümü, bir bina veya oyun
parkının kapladığı alan, yol yapımı için gerekli alan, takım elbise dikmek
için gereken kumaşı hesaplama, inşaat sektörü gibi birçok konuda alan
hesaplamalarına ihtiyaç duyulur.
Örneğin çocuklarına pasta paylaştırmak isteyen Gönül Hanım,
pastanın bir kenarının orta noktasına bıçakla iz bırakmıştır. Dilimin bu
kenarının karşısındaki köşeden işaretli yere kadar pastayı iki parçaya
ayırıyor. Pastanın eşit iki eşit parçaya ayrıldığını çocuklarına nasıl
anlatabilir?

Bir üçgensel bölgenin alanı, bir kenar uzunluğu ile o kenar uzunluğuna ait yüksekliğin uzunluğunun
çarpımının yarısına eşittir. A, B, C köşelerinden oluşan üçgensel bölgenin alanı A _& ABCi biçiminde
gösterilir.

Dar Açılı Üçgende Alan

& A(&
a.h b. h c. h
ABC) = 2 a = 2 b = 2 c dir.

Dik Üçgende Alan

Dik üçgensel bölgenin alanı, dik kenar uzunluklarının


çarpımının yarısının alınması ile bulunur. Eğer hipotenüse ait
yükseklik biliniyorsa taban ile yükseklik çarpımının yarısı da
alınabilir.

A _&
ABCi = 2 = 2 buna göre
b $ c a $ ha

b $ c = a $ ha olur.

Geniş Açılı Üçgende Alan

Geniş açılı üçgenlerde 6AB@ ve 6BC@ kenarlarına ait


yükseklikler üçgenin dış bölgesindedir.
A(&
a$h b$h c$h
ABC) = 2 a = 2 b = 2 c şeklinde bulunur.

Matematik 9 | 345
ÜÇGENLER

ÖRNEK 1:

6AD@ = 6BC@, b
_

AD = 8cm, bb
` ise BE kaç cm olur bulunuz.
BC = 12cm, b
AC = 16cm b
a
ÇÖZÜM:
Üçgensel bölgelerin alan bağıntısını iki kenar için de uygulanırsa
AD $ BC AC $ BE
A _&
ABCi =
=
2 2
8 $ 12 16 $ BE
= 2 = 2 & 8 $ 12 = 16 $ BE & BE = 6 cm olur.

ÖRNEK 2:
ABC üçgeninde
6AB@ = 6BC@
AB = 5 cm
6DC@ = 12 cm ise
A _ADC
& i kaç cm 2 olduğunu bulunuz.

ÇÖZÜM:
AB kenarı aynı zamanda DC kenarının yüksekliği olduğu için
& i = DC $ AB = 12 $ 5 = 30 cm 2 olur.
A _ADC 2 2
ÖRNEK 3:

ABC dik üçgen 6AD@ açıortay olmak üzere


6AB@ = 6BC@, AC = 22cm
& nin alanını bulunuz.
BD = 6 cm ise ADB

ÇÖZÜM:
Açıortay üzerinde alınan bir noktadan açıortayın kollarına inilen
dikmelerin uzunlukları eşit olduğundan DH = 6 cm olur. Bu durumda
& 22 $ 6
A^ADCh = 2 = 66 cm 2dir.

ÖRNEK 4:
ABC ikizkenar üçgeninde AB = AC = 10 cm ve BC = 16 cm ise ABC üçgeninin alanını bulunuz.

ÇÖZÜM:

ABH 6-8-10 özel üçgeni olduğundan AH = 6 cm olur.


Buradan
& 6 $ 16
A^ABCh = 2 = 48 cm 2 olur.

346 | Matematik 9
ÜÇGENLER

ÖRNEK 5:

6AB@ = 6AC@ ve 6AH@ = 6BC@ dir.


Şekilde verilenlere göre ABD üçgeninin alanını bulunuz.

ÇÖZÜM:
ABD üçgeninin alanını ve BD kenarına ait yüksekliği bulmak için Öklid teoremi kullanılırsa
h 2 = BH $ HC
h 2 = 12 $ 3
& 6 $ 10
h 2 = 36 & h = 6 cm dir. A(ABD) = 2 = 30 cm 2

ÖRNEK 6:

%
ABC üçgeninde m^ABCh = 30c, AB = 8 cm, BC = 3 cm ise
ABC üçgeninin alanını bulunuz.

ÇÖZÜM:
6BC@ kenarına ait yükseklik üçgenin dışından çizilir. ABH dik
üçgeninde 30c nin karşısındaki kenar hipotenüsün yarısına eşit
8
olduğundan AH = 2 = 4 cm olur.
& 3$4
A^ABCh = 2 = 6 cm 2 bulunur.

ÖRNEK 7:

ABCD dörtgeninde
6AB@ = 6BC@, 6EF@ ' 6BC@,
AB = 11 cm
EF = 6 cm ise DEC üçgeninin alanını bulunuz.

ÇÖZÜM:

A _DEC
&i = A _&DEF i + A _&
ECF i
6 $ h 6 $ ]11 - hg
= 2 + 2
6 $ h + 66 - 6h
= 2
= 33 cm 2

Matematik 9 | 347
ÜÇGENLER

ÖRNEK 8:
Dik üçgen şeklindeki bir masa yüzeyinin en uzun kenarının uzunluğu 7 m dir. Masanın yüzeyinin çevresinin
toplam uzunluğu 18 m olduğuna göre masanın üst yüzünün alanını bulunuz.
ÇÖZÜM:
Masanın diğer kenarlarının uzunlukları x ve y olsun.
x + y + 7 = 18
x + y = 11
^x + yh2 = 11 2
x 2 + 2xy + y 2 = 121
49 + 2xy = 121
Pisagor bağıntısı uygulanırsa
x 2 + y 2 = 7 2 ...(1) 2xy = 72 her iki tarafın karesi alınırsa
olur. xy = 36 (1) denklemde yerine yazılırsa
xy 36
Alan = 2 = 2 = 18 m 2 olur.

Teknoloji Uygulaması
Aşağıda GeoGebra programı kullanılarak taban ve yüksekliği değiştirilen bir üçgenin alanının nasıl değiştiği
incelenmiştir.

Doğru aracını etkinleştiriniz.


Nokta seçerek iki farklı doğru oluşturunuz.

Nokta aracını etkinleştiriniz.


Oluşturduğunuz doğrulardan birinin üzerinde yeni bir nokta oluşturunuz. Bu noktayı E olarak adlandırınız.

Çokgen aracını etkinleştiriniz.


EAB üçgeni oluşturmak için E, A ve B noktalarını art arda seçiniz.
(E noktasından başlayıp yine en son E noktasına tıklayınız.)

Dik doğru aracını etkinleştiriniz.


Üçgenin bir köşesini ve bu köşenin karşısındaki kenarı işaretleyerek yüksekliği oluşturunuz.

Uzunluk aracını etkinleştiriniz.


Üçgene ait taban ve yüksekliği işaretleyerek uzunluklarını bulunuz.

Alan aracını etkinleştiriniz.


Üçgeni işaretleyerek alanını hesaplatınız.

Taşı aracını etkinleştiriniz.


Üçgenin köşelerini hareket ettirerek farklı üçgenler oluşturunuz.

Üçgenin taban ve yüksekliği değiştiğinde alanının nasıl değiştiğini açıklayınız.


Sadece E noktasını hareket ettirdiğinizde üçgenin taban, yükseklik ve alanında bir değişim olup olmadığını
gerekçesiyle açıklayınız.

348 | Matematik 9
ÜÇGENLER
Üçgenin taban ve yüksekliği değiştiğinde alanının nasıl değiştiğiyle ilgili GeoGebra çizimi aşağıda verilmiştir.

Yandaki şekilde d1 ' d2 olmak üzere ABC üçgeninin


A noktası hareket ettirildiğinde oluşan ABC üçgeninin
alanı değişmez. Bu üçgenlerin tabanları ve yükseklikleri
aynı uzunluktadır.
& & BC $ h
A^ABCh = A^Al BCh = 2

ÖRNEK 9:
ABC dik üçgeninde
6AC@ = 6AB@,
6DE@ ' 6BC@,
AD = 5 br,
EB = 10 br ise CDE üçgeninin alanını bulunuz.

ÇÖZÜM:

CDE ve DEB üçgenlerinde DE kenarı ortak


ve bu kenara ait yükseklik (paralellikten dolayı)
değişmediğinden A^CDEh = A]DEBg dir.
& &

A^CDEh = A]DEBg = 2 = 25 br 2 olur.


& & 10 $ 5

Matematik 9 | 349
ÜÇGENLER

Sıra Sizde

SORU:
Yandaki verilen şekilde 6AC@ ' 6ED@ ve 6AB@ = 6BD@ dir.
AB = 4 cm
BC = 3 cm
CD = 1 cm olduğuna göre taralı alanın kaç cm 2 olduğunu
bulunuz.

ÇÖZÜM:

Yükseklikleri eşit olan üçgenlerin alanları oranı bu yüksekliklere


ait taban uzunlukları oranına eşittir.
Yukarıdaki kurala benzer olarak tabanları eşit olan üçgenlerin
B H C D alanları oranı yükseklikleri oranına eşittir.
& BC $ AH
A_ABCi 2 BC
& = CD $ AH = DC
A^ACDh
2

ÖRNEK 10:

A Yandaki ABD üçgeninde B, C, D doğrusaldır.


4 BC = 5 CD dir. ABD üçgeninin alanı 54 cm 2 ise
A^ABC h yi bulunuz.
&

B C D

ÇÖZÜM:

4 BC = 5 CD ise BC = 5k
A CD = 4k olur.
&
A^ABCh 5 & &
& = 4 ise A^ABCh = 5s, A^ACDh = 4s olur.
A^ACDh

A]ABDg = 54 cm 2
& &
B C D A^ABCh = 5s
9s = 54 = 5$6
s = 6 cm 2 = 30 cm 2

350 | Matematik 9
ÜÇGENLER

ÖRNEK 11:

ABC üçgeninde
BD = 3 cm, DC = 5 cm
CE = 6 cm, EA = 3 cm

A]&
BDEg
olduğuna göre oranını bulunuz.
A]ABCg
&

ÇÖZÜM:

A]BDEg
&
BDEg = 3S ve A]&
3 & ]& DCEg = 5S olur.
BD
& =
A]DCEg DC = 5 A

A]ABEg 3 & A]ABEg 3 & ]&


& &
8S = 6 A ABEg = 4S olur.
AE
& = =
A]BCEg EC 6

A]BDEg
&
3S 1
& = 12S = 4
A]ABCg

ÖRNEK 12:

ABC üçgeninde
AB = AD = 10 cm
BD = 16 cm, DC = 6 cm
BC kenarı 5 eşit parçaya ayrılmıştır.
Buna göre DGK üçgeninin alanını bulunuz.

ÇÖZÜM:
5AH? = 5BD? olacak şekilde 5AH? çizildiğinde BH = HD olur.
AHD üçgeninde Pisagor teoreminden AH = 6 cm bulunur.
A]ABDg =
& BD $ AH 16 $ 6
2 = 2 = 48 cm 2
&h
A ^ABD
= 6 & & = 3 & A ^BCD & h = 144 cm 2
10 48 5
A ^BCD h
& A ^BCD h 5
H
144
A]DBCg & ]DGK
&
A]DGKg =
& &g 5 144 2
5 A = 5 = 25 cm

ABC üçgeninde
BD = DC
A]ABHg = A]AHCg
& &

Kenarortay, üçgeni alanları eşit Ağırlık merkezi üçgenin alanını


olan iki parçaya ayırır. 6 eşit parçaya ayırır.

Matematik 9 | 351
ÜÇGENLER

ÖRNEK 13:

ABC üçgeninde G ağırlık merkezi


5BG? = 5GC?, BG = 5 cm, CG = 8 cm
olduğuna göre A]ABCg nin değerini bulunuz.
&

ÇÖZÜM:
Üçgenin kenarortayları çizildiğinde alanın 6 eşit parçaya ayrıldığı görülür.
A]ABCg = 6S
&
A]BCGg = 2
& 5$8
A]ABCg = 60 cm 2 bulunur.
&
2S = 20
S = 10 cm 2

ÖRNEK 14:
ABC üçgeninde
5AB? ' 5ED?, AE = EC , BD = DC , A]&
ABCg = 48cm 2 olduğuna
göre A]KDEg yi bulunuz.
&

ÇÖZÜM:
5BE? ve 5AD? kenarortay olduğundan kesiştikleri nokta K ağırlık
merkezidir. Buna göre AK = 2 KD olacağından
A]KDEg = S & A]AKEg = 2S
& &

A]ADEg = 3S ve A]EDCg = 3Sdir.


& &

BD = DC olduğundan A]ABD = A]ADCg = 6S olur.


&g &

A]ABCg = 48 cm 2 & 12S = 48 & S = A]KDEg = 4 cm 2 bulunur.


& &

Sinüs Alan Teoremi


Bir üçgensel bölgede iki kenar uzunluğu ve bu kenarlar
arasındaki açı biliniyorsa sinüs alan teoremi ile üçgenin alanı
bulunabilir.
A]ABCg = 2 & sin ^V B h = c & h = c $ sin ^V
B h alan formülünde
& a$h h
h
yerine yazılırsa
a $ c $ sin ^V
Bh
A]ABCg =
&
2
A]ABCg = 2 $ a $ c $ sin ^VB h bulunur.
& 1

Benzer şekilde A]ABCg = 2 $ b $ c $ sin ^W


A h ve A]ABCg = 2 $ a $ b $ sin ^V
C h olur.
& 1 & 1

352 | Matematik 9
ÜÇGENLER

ÖRNEK 15:

ABC üçgeninde AB = 6 cm, BC = 12 cm


m ^A V
BC h = 30c olduğuna göre
A]ABCg değerini bulunuz.
&

ÇÖZÜM:
I. Yol:
ABC üçgeninin alanını bulmak için sinüs alan teoremi uygulandığında
A]&
ABCg = 1 b 1l
2 $ AB $ BC $ sin 30c sin 30c = 2
A]&
ABCg = 1 1
2 $ 6 $ 12 $ 2
A]&
ABCg = 18 cm 2
II. Yol:
5AH? = 5BC? olacak şekilde bir 5AH? çizildiğinde 30c - 60c - 90c üçgeni
oluşacağından AH = 3 cm dir.

A]ABCg =
& BC $ AH
2
A]ABCg = 2
& 12 $ 3

A]ABCg = 18 cm 2 bulunur.
&

ÖRNEK 16:

Yandaki şekilde KN = 2 br, NM = 3 br, NH = 2 br, LM = 4 br


N d 6KM@ ve H d 6LM@ olduğuna göre KLM üçgeninin alanını
bulunuz.

ÇÖZÜM:
NHM dik üçgeninde sin Y
2
M = 3 olduğundan KLM üçgeninin alanını bulmak için sinüs alan teoremi
uygulandığında
A]KLMg = 2 $ KM $ LM $ sin Y
& 1
M

A]KLMg = 2 $ 5 $ 4 $ 3
& 1 2

A]KLMg = 3 br 2 bulunur.
& 20

Matematik 9 | 353
ÜÇGENLER

ÖRNEK 17:
Ahmet, iki kenarının uzunluğu 3 m ve 6 m olan bir üçgen oluşturmak istiyor. Bu üçgenin alanı en çok kaç
olabilir?

ÇÖZÜM:
İki kenar uzunluğu 3 m ve 6 m olan üçgen oluşturulduğunda ABC üçgeninin alanını bulmak için sinüs alan
teoremi uygulandığında
A]&
ABCg = 12 $ 3 $ 6 $ sin a
- 1 # sin a # 1 olduğundan sin a nın alabileceği en büyük değer a = 90c
için sin 90c = 1 dir.
a = 90c alındığında A]&
ABCg = 1
2 $ 3 $ 6 $ sin 90c
A]&
ABCg = 1
2 $3$6$1
A]&
ABCg = 9 m 2 bulunur.

ÖRNEK 18:
Yandaki şekilde A]&
ABCg = S 1 ve A]ACD
&g
= S2
W h = m ^CAD
m ^BAC W h
AB = 4 br, AD = 7 br olduğuna göre
S1
S 2 oranını bulunuz.
ÇÖZÜM:
İç açıortay teoreminden
A^ABC h S
&
BC 4 4
= 7 & & = S 12 = 7 tir.
CD A^ACD h

ÖRNEK 19:
ABC üçgeninde
AD = 4 br, AE = 2 br
BD = EC = 3 br
A]ADEg = S 1 ve A ]BCDEg = S 2 olduğuna göre S 12 oranını bulunuz.
& S

ÇÖZÜM:
ADE ve ABC üçgenlerinde sinüs alan teoremi uygulandığında
A]ADEg = 2 $ 4 $ 2 $ sin W
A & S 1 = 4 sin W
& 1 b_b
4 sin W
A bb
`b S S+1 S = 35 A 8
W
= 35
W 35 W
A]ABCg = 2 $ 7 $ 5 $ sin A & S 1 + S 2 = 2 sin A bb
& 1 b 1 2
2 sin A
a
S1 8 &
S 1 + S 2 = 35 S 1 = 8S ve S 1 + S 2 = 35S olduğundan S 2 = 27S olur.
S1 8S & S 1 8
S 2 = 27S S 2 = 27 bulunur.

354 | Matematik 9
ÜÇGENLER

Heron Alan Formülü


Kenar uzunlukları a, b, c olan üçgenin çevre uzunluğunun yarısı
a + b + c olmak üzere A]& ABCg = u $ ]u - ag $ ]u - bg $ ]u - cg
u= 2
bağıntısı ile bulunur.

ÖRNEK 20:

ABC üçgeninde AB = 7 cm, BC = 8 cm ve AC = 9 cm


olduğuna göre ABC üçgeninin alanını bulunuz.

ÇÖZÜM:
7+8+9
ABC üçgeninin yarı çevresi hesaplandığında u = 2 = 12 cm bulunur.
ABC üçgeninin alanını bulmak için Heron alan formülü uygulandığında
A]ABCg = u $ ]u - ag $ ]u - bg $ ]u - cg
&

A]ABCg = 12 $ ]12 - 7g $ ]12 - 8g $ ]12 - 9g


&

A]ABCg = 12 $ 5 $ 4 $ 3
&

A]ABCg = 12 5 cm 2 bulunur.
&

ÖRNEK 21:
Yandaki şekilde
AC = 5 cm, AD = 7 cm
BC = 9 cm, CD = 6 cm
olduğuna göre A]ABCg değerini bulunuz.
&

ÇÖZÜM:
5+7+6 &
ACD üçgeninin alanını bulmak için Heron alan formülü uygulandığında u = 2 u = 9 cm dir.

A]ACDg = 9 $ ]9 - 5g $ ]9 - 7g $ ]9 - 6g
&

A]ACDg = 9 $ 4 $ 2 $ 3
&

A]ACDg = 6 6 cm 2
&

A]ABCg 9 & A]ABCg 3 & ]&
& &
= = 2 A ABCg = 9 6 cm 2
A]ACDg
& 6 6 6

Matematik 9 | 355
ÜÇGENLER

İç Teğet Çember Yardımıyla Alan Bulma


ABC üçgeninin çevresi ve iç teğet çemberinin yarıçapı biliniyorsa
üçgenin alanı hesaplanabilir. ABC üçgeninin yarıçapı r olsun.
A]BOCg = 2
& a$r

A]AOCg = 2
& b$r

A]AOBg = 2
& c$r

Bu üçgenlerin alanları toplamı ABC üçgeninin alanına eşit


olacağından
A]ABCg = 2 + 2 + 2
& a$r b$r c$r

]a + b + c g $ r & &
A]ABCg = A]ABCg = u $ r
&
2
ABC üçgeninde O noktası iç teğet çemberinin merkezi ise oluşan
üçgenlerin alanları kenar uzunlukları ile orantılıdır.
A]BOCg A]AOCg A]AOBg
& & &
a = b = c tir.

ÖRNEK 22:
ABC üçgeninin iç teğet çemberinin merkezi I noktasıdır. Buna göre
a) ABC üçgeninin iç teğet çemberinin yarıçapını,
b) BIC üçgeninin alanını bulunuz.

ÇÖZÜM:
a) ABC üçgeninin alanını bulmak için Heron alan formülü kullanıldığında
= 16 A]ABCg = 16 $ ]16 - 9g $ ]16 - 10g $ ]16 - 13g
9 + 10 + 13 &
u= 2
A]ABCg = 16 $ 7 $ 6 $ 3
&

A]ABCg = 12 14 br 2
&
ABC üçgeninin alanı iç teğet çember yardımıyla hesaplandığında

A]ABCg = u $ r & 12 14 = 16 $ r & r = 4 br bulunur.


& 3 14

b) ABC üçgeninde I noktası iç teğet çemberin merkezi olduğundan oluşan üçgenlerin alanları oranı,
kenarlarının oranına eşittir.

A]ABCg = 9S + 10S + 13S


&
A]BICg = 10S & A]BICg = 10 $ 8
& & 3 14
32S = 12 14
A]BICg =
& 15 14 2
3 14
S = 8 br 2 olur. 4 br
bulunur.

356 | Matematik 9
ÜÇGENLER

Eşkenar Üçgenin Alanı


Eşkenar üçgende bir köşeden karşısındaki kenara dik inildiğinde
30c - 60c - 90c dik üçgeni oluşacağından
a 3
a$ 2
A]ABCg =
& a2 3 a 3
AH = 2 bulunur.
2 = 4 olur.

ÖRNEK 23:
Alanı 9 3 cm 2 olan bir eşkenar üçgenin yüksekliğinin uzunluğunu bulunuz.

ÇÖZÜM:
Bir kenarının uzunluğu 2x olan ABC üçgeni çizildiğinde
A]ABCg =
& 2x $ x 3
2
2
9 3=x 3
x = 3 cm
AH = x 3 & AH = 3 3 cm bulunur.

ÖRNEK 24:

ABC eşkenar üçgeninde


5AD? = 5BC?, 5DE? = 5AC?
A]ABCg = 36 3 cm 2
&

olduğuna göre A]DECg değerini bulunuz.


&

ÇÖZÜM:
Bir kenarı a olan ABC üçgeninde
A]&ABCg = 4 & 36 3 = 4 & a 2 = 144 & a = 12 cm olur.
a2 3 a2 3

DEC üçgeni 30c - 60c - 90c dik üçgeni olduğundan


EC = 3 cm ve DE = 3 3 cm olur.
3$3 3
A^DEC h =
&
2
9 3
A^DEC h = 2 cm 2
&

Matematik 9 | 357
ÜÇGENLER

Benzer Üçgenlerin Alanları


Benzer iki üçgenin alanları oranı benzerlik oranının
& &
karesine eşittir. ABC + DEF ve benzerlik oranı k ise
A^ABC h
&
2
& = k olur.
A^DEF h
Bu ifadenin doğruluğu aşağıdaki gibi gösterilebilir.

& &
ABC + DEF ve benzerlik oranı k olduğundan
a h
d = h' = k olur.
a$h
A]ABCg 2 & A]ABCg = a $ h $ 2 & = a $ h
& &
=
A]DEFg A]DEFg
& d $ h' & 2 d $ h' d h'
2 =k $ k
= k2

ÖRNEK 25:
ABC üçgeninde
m^ABC h = m^DAC h , 3 AC = 4 DC
% %

A]ADBg = 28 cm 2
&

olduğuna göre A]ADCg değerini bulunuz.


&

ÇÖZÜM:
ACB açısı her iki üçgenin de ortak açısıdır. Bu durumda
m]ADCg = m]BACg olur. AA benzerliğinden ABC + DAC olacağından
% % & &
benzerlik oranı k olsun.
A]DACg b 3 l2 & S
&
S + 28 = 16 16S = 9 ]S + 28g & 7S = 28 $ 9
9 &
& = 4
A]ABCg
S = 36 cm 2

Sıra Sizde

SORU:

ABC dik üçgeninde


5AB? = 5BC?
5AB? = 5ED?
5BD? = 5AC?
A]BDEg
&
m]BCAg = 67, 5c olduğuna göre & oranını bulunuz.
%
A]ABCg

ÇÖZÜM:

358 | Matematik 9
ÜÇGENLER

ÖRNEK 26:
Yandaki şekilde 5DE? ' 5BC? dir.
AD = 4 BD ve A ]BDECg = 18 br 2
olduğuna göre ADE üçgeninin alanını bulunuz.

ÇÖZÜM:
5DE? ' 5BC? olduğundan
m]ABCg = m]ADBg ve m]ACBg = m]AEDg olur.
% % % %
& &
Bu durumda ABC + ADE olacağından
AD 4x & AD 4
AB = 5x AB = 5 tir.
Benzer üçgenlerde alanlar oranı, benzerlik oranının karesine eşit
olduğundan
A]ADEg b 4 l2 & S
&
16
= 5 = 25 & 25S = 16S + 16 $ 18
A]ABCg
& S + 18
9S = 16 $ 18
S = 32 br 2 tir.

ÖRNEK 27:
5PR? ' 5ST? ' 5BC?
6 PS = 2 AP = 3 SB
A ]BCTSg = 40 cm 2 olmak üzere
ABC üçgeninin alanını bulunuz.

ÇÖZÜM:
PS = k için AP = 3k ve SB = 2k olur. 5AB? ve 5AC?
kenarları eşit parçalara bölünerek elde edilen üçgenlerden küçük
üçgenin diğer büyük üçgenlere olan benzerlik oranları sırasıyla
1 1 1 1 1 1 1 1 1 1
2 , 3 , 4 , 5 , 6 olduğundan alanları oranı 4 , 9 , 16 , 25 , 36 olur.
En üstteki küçük üçgenin alanına A denirse diğer parçaların alanları
sırasıyla A, 3A, 5A, 7A, 9A, 11A olur.
A ^BCTS h = 11A + 9A = 20A = 40 olduğundan A = 2 cm 2 tir.
A^ABC h = 11A + 9A + 7A + 5A + 3A + A = 36A = 72 cm 2 bulunur.
&

Matematik 9 | 359
ÜÇGENLER

ÖRNEK 28:
A

ABC ve FDE üçgenlerinin eş açıları şekilde gösterilmiştir.


]&
DFEg
AB = 10 br ve DF = 4 br olduğuna göre A & oranını bulunuz.
F A]ABCg
D

E
B C

ÇÖZÜM:
ABC üçgenindeki eş açılar harflendirilir. Bir üçgendeki iki iç açının ölçü-
leri toplamı, komşu olmayan dış açının ölçüsüne eşittir. Buna göre
m]DFEg = x + y
%

m]EDFg = x + z
%

m]FEDg = y + z olur.
%
& &
Karşılıklı açılar eş olduğundan DFE + ABC dir. Benzerlik oranı
4 & A]DFEg b 2 l2
DF &
= & = 5
AB 10 A]ABCg
A]DFEg
&
4
& = 25 olur.
A]&ABC g

ÖRNEK 29:
Üçgen şeklindeki bir çocuk parkı etrafına renkli taşlar kullanılarak bir bisiklet yolu yapılacaktır. Parkın kenar
uzunlukları 10 m, 17 m ve 21 m dir. Bisiklet yolu yapıldıktan sonra parkın yeni kenar uzunlukları 20 m,
34 m ve 42 m olacağına göre bisiklet yolunun alanını bulunuz.

ÇÖZÜM:

A
D

F
E C
B

Parkın çevresine yapılacak bisiklet yolunun alanı DEF üçgeni için


A]ABCg - A]DEFg dir. Ç]DEFg
& & &

u2 = & u 2 = 10 + 17 + 21 & u 2 = 24 m
ABC üçgeni için 2 2
Ç]ABCg A]DEFg = 24 $ 14 $ 7 $ 3
& &
u1 = & u 1 = 20 + 34 + 42 & u 1 = 48 m bisiklet yolunun alanı,
A]DEFg = 84 m 2
2 2 &
Heron alan formülü uygulandığında
A]ABCg - A]DEFg = 336 - 84
& &
A]ABCg = 48 $ 28 $ 14 $ 6
&
= 252 m 2 bulunur.
A]ABCg = 336 m 2
&

360 | Matematik 9
ÜÇGENLER

ALIŞTIRMALAR

1 ABC üçgeninde 4
AB = AC = 4 5 cm
BC = 8 cm ise
A]ABCg ini bulunuz.
&

ABC üçgeninde m]BACg = 90c olmak üzere A


%
noktasının 6BD@ na göre simetriği K ! 6BC@
noktasıdır.
AB = 6 cm
BC = 10 cm
olduğuna göre A]BKDg ini bulunuz.
&

2 Yandaki şekilde 5BD? 5


açıortay
5BA? = 5DA?
BC = 12 cm
A]DBCg = 48 cm 2
&
olduğuna göre
ABC üçgeninde m]BACg = 90c B, D, C
%
AD kenarının
doğrusaldır. G noktası ABC üçgeninin ağırlık
uzunluğunu bulunuz.
merkezi olduğuna göre A]ABCg ini bulunuz.
&

3 ABC üçgeninde B, E, C 6 ABC üçgeninde


m]ABCg = 60c
doğrusal %

5AB? ' 5DE? m]BACg = 75c ise


%

A]ABCg ini bulunuz.


BE = EC &
A]APDg = 20 cm 2
&

olduğuna göre taralı


alanı bulunuz.

Matematik 9 | 361
ÜÇGENLER

ABC dik üçgeninde


7 9
AD = AB
5AB? = 5BC?
5BH? = 5AC?
BC = 12 cm
Yukarıdaki şekilde 5BE? ' 5CD? HD = 4 cm
5BA? = 5AD? ise A]&BCEg ini bulunuz. olduğuna göre taralı
alanı bulunuz.

8 Yandaki şekilde 10 ABC üçgeninde


m]DABg = 60c
% AD FC BE 1
AB = AC = BC = 4
AC = 2 cm, CD = 6 cm olmak üzere
AF = 3 cm, FB = 5 cm
A]DCEg = S 1
&

A]EFBg = S 2
&

Verilenlere göre S 1 - S 2 değerini bulunuz. taralı bölgenin alanı 36 cm 2 olduğuna göre ABC
üçgeninin alanını bulunuz.

11 Melis ve ailesi, ormandaki piknik alanında piknik yapmaktadır. Piknik bitiminde Melis’in babası piknik
ateşini söndürdüğünü düşünerek piknik alanından ayrılıyor. İki saat sonra piknik alanının kenarındaki
ormanda yangın çıktığı haberi geliyor. Saatler süren söndürme çalışmaları sonucunda yangın söndürülüyor.
Gün ağardığında yanan orman alanı hesaplanmak isteniyor.
Yanan alanın çekilen görüntüsü aşağıda verilmiştir.

a) Görüntüdeki ölçeği kullanarak yanan orman


alanının yaklaşık değerini hesaplayınız.

b) Haritaya göre yanan orman alanı tüm


ormanın yaklaşık yüzde kaçıdır?

c) İki fidan arası 3 m ve bir fidanın dikim maliyeti


4 TL dir. Buna göre yanan orman alanına
dikilecek fidan sayısı ve dikim hangi aralıkta
olacağını hesaplayınız.

Haritada 1 cm 500 m ye karşılık gelmektedir. Fidan Sayısı Fidan Dikim Maliyeti (TL)
50 000-100 000 200 000-400 000
100 001-200 000 400 004-800 000
200 001-300 000 800 004-1 200 000
300 001-400 000 1 200 004-1 600 000

362 | Matematik 9
ÜÇGENLER

ÖLÇME VE DEĞERLENDİRME -1

1 ABC üçgeninde 4
AB = BC
AD = DC
m^ABC h = 30c
%

m^DAC h = 40c ise


%

m^BCD h + m^BAD h toplamı kaç derecedir?


% %
Yukarıdaki şekilde A, E, C doğrusaldır.
A) 70 B) 60 C) 50
m^DBA h = m^DBC h, m^CAD h = m^BAC h = 60c
% % % %
D) 45 E) 40
olduğuna göre BDC açısının ölçüsü kaçtır?

A) 70c B) 60c C) 50c D) 40c E) 30c

2 Yandaki şekilde 5
m^GAF h = b
%

m^FBHh = a
%

m^DEA h = c
%

m^KDC h = d
%

ABC üçgeninde AB = ]3x - 5g cm


olduğuna göre a + b + c + d toplamı kaç dik AC = ]5x + 7g cm, BC = 26 cmise üçgenin
açıya eşittir? çevre uzunluğu en az kaç cm dir?
A) 3 B) 4 C) 5 D) 6 E) 7 A) 47 B) 53 C) 62 D) 70 E) 84

3 6

ABC üçgeninde
AD = AC = 4 cm ABC dik üçgeninde 5AE? kenarortay,
DC = 6 cm ve BD = 8 cm 5AE? = 5ED?, AD = 9 br, ED = 17 br
olduğuna göre ABC üçgeninin alanı kaçtır? olduğuna göre BC kenarının uzunluğu
aşağıdakilerden hangisidir?
A) 4 B) 8 C) 4 7 D) 12 E) 8 7
A) 16 B) 15 C) 14 D) 12 E) 10

Matematik 9 | 363
ÜÇGENLER

7 ABC üçgeninin ağırlık


ABC üçgeninde 10
merkezi G, ABG
5AB? = 5BC?, AD = DC üçgeninin ağırlık
ED = BC , m]BAC
%g
= 40c merkezi P noktasıdır.
olduğuna göre EDC açısı PG = 10 br ise CD
kaç derecedir? kenarının uzunluğu
kaç br dir?

A) 25 B) 30 C) 35 D) 38 E) 45
A) 60 B) 70 C) 80 D) 90 E) 100

8 11
ABC üçgeninin diklik
merkezi üçgenin içindeki
M noktasıdır.
AB = 3 cm, AC = 5 cm
olduğuna göre BC = x
in alabileceği kaç tam
ABC üçgeninde sayı değeri vardır?
EC = 3 br, BD = 3 br, BF = 4 br,
FC = 5 br, AD = ^x + 2h br, AE = x br A) 1 B) 2 C) 3 D) 4 E) 5
olduğuna göre AP değeri kaçtır?
PF
9 8
A) 7 B) 6 C) 5 D) 2 E) 3

Yandaki şekilde ABC üçgeninde


9 12
m]ABDg = m]DBCg
% %
AD = 3 cm
m]DCEg = m]ECBg
% %
DB = 3 cm
EC = DC = 12 cm
AE = 2 cm
ED = 6 cm ise ABC
EC = 7 cm
üçgeninin çevresi
kaç cm dir? BC = 12 cm
A) 60 B) 56 C) 48 D) 44 E) 36
olduğuna göre DE = x değeri kaçtır?
A) 4 B) 5 C) 6 D) 7 E) 8

364 | Matematik 9
ÜÇGENLER

13 Yandaki şekilde
5DC? = 5BE?, 16 ABC eşkenar
m]DACg = 30c
% üçgeninde
BD = 12 cm
AP = 5 br
m]DCAg = m]ACEg, AC = x kaçtır?
% % BP = 12 br
CP = 13 br
A) 6 B) 7 C) 8 D) 9 E) 10 olduğuna göre a açısı kaç derecedir?

A) 120 B) 135 C) 150 D) 155 E) 165

14 O noktası ABC 17 Yandaki şekilde,


çevrel çemberinin 5DC? ' 5AB?
5AD? = 5DE?
merkezidir.
OH = 3 HC
CE = EB
AB = 6 cm
DE = 7 cm
AC = 6 3 cm ise
AD = 12 cm
BC kenarının uzunluğu
kaçtır? olduğuna göre A(ABCD) kaçtır?

A) 3 B) 4 C) 3 3 D) 6 E) 6 3 A) 88 B) 84 C) 75 D) 72 E) 60

15 18

ABC üçgeninde m ^BCD


%h = 15c,
ABC eşkenar üçgeninde DB = EA ve
CD = 2 AB , AD = 4 br,
m]DFCg = 80c, m]BECg = x kaç derecedir?
% %
olduğuna göre AB = x = ?
A) 50 B) 65 C) 80 D) 85 E) 90
A) 5 + 2 3 B) 3 + 3 3 C) 2 + 2 3
D) 3 + 5 E) 2 3 - 1

Matematik 9 | 365
ÜÇGENLER

ÖLÇME VE DEĞERLENDİRME -2

1 4

Yukarıdaki şekilde 5OB ışını AOD açısının, 5OC


ışını AOE açısının açıortayıdır. m]BOCg = 35c
%
ABCD karesinde
ise DOE açısının ölçüsü kaç derecedir? AE = EC = BD
olduğuna göre m]DCEg = x açısının ölçüsü
%
A) 70 B) 60 C) 50 D) 40 E) 30
aşağıdakilerden hangisidir?

A) 10c
B) 15c
C) 18c
D) 20c
E) 25c
2

Şekilde verilenlere göre m]GBFg = 18c


%
AB = AG = BF = FC = CE = ED
olduğuna göre m]EDCg = x açısının ölçüsü
%
5
kaçtır?

A) 10c B) 12c C) 14c D) 15c E) 18c

Yukarıdaki şekilde m]CBAg 1 90c, BC = 8 br


%
AD = 10 br, AC = 13 br dir. Kenar
uzunlukları tam sayı olan ABCD dörtgeninin
çevresinin alabileceği en küçük değer kaçtır?
3 A) 29 B) 30 C) 33 D) 34 E) 35

Şekilde m^ABE h = m^EBC h, m^BCE h = m^ECD h


% % % %
verilenlere göre a açısının ölçüsü kaçtır?

A) 125c B) 120c C) 118c D) 115c E) 110c

366 | Matematik 9
ÜÇGENLER

6 Yandaki şekilde 9
AB = AC
BC = 2 CD
s]ABEg = 30c
%
ABC bir üçgen,
AB = 6 br, BC = 8 br , m]ABC
%g
= 2 $ m]ACBg
%

olduğuna göre s]CEDg = x kaç derecedir?


% ise AC kenarının uzunluğu aşağıdakilerden
hangisidir?
A) 50 B) 45 C) 40 D) 35 E) 30
A) 2 7
B) 29
C) 3 11
D) 2 21
E) 5 5

ABC üçgeninde
m]BADg = 30c ve 7 BD = 3 DC
%
olduğuna göre AB kenarının uzunluğu kaçtır?
10
A) 9 B) 10 C) 11 D) 12 E) 13

ABC üçgeninde 5AS? açıortay, 5BP? kenarortay,


8 AB = 15 br, BS = 10 br, SC = 8 br
olduğuna göre AR = x kaç birimdir?

13 45 9 34 18
A) 2 B) 7 C) 2 D) 9 E) 5

AD = x + 3, FC = x + 1 olduğuna göre
Ç ^ABC
&h kaçtır?

A) 36 B) 38 C) 40 D) 45 E) 50

Matematik 9 | 367
ÜÇGENLER

11 13

Yukarıdaki şekilde ABC üçgeninde K ağırlık merkezi olmak üzere


5AB? = 5BD? 5AB? = 5BC?, m]%
EAFg = m]%FACg
AE = EF = FC KH = 1 cm, EF = 3 cm, FC = 4 cm
AB = 12 cm olduğuna göre DC kenarının uzunluğu kaçtır?
CD = 6 cm
A) 3 B) 4 C) 5 D) 6 E) 7
olduğuna göre taralı alan kaç cm 2 dir?
A) 12
B) 18
C) 24
D) 30
E) 36

12 ABCD karesinde 14
DP = MN
5AP? = 5PN?
olduğuna göre
PAN açısının ölçüsü
kaç derecedir? ABC bir üçgen, 5BN? iç açıortay, AB = 13 br
MC = 10 br, BP = PC , 5BM? = 5MC? ,
BN = 2 NM olduğuna göre NP kenarının
A) 30 B) 40 C) 45 uzunluğu aşağıdakilerden hangisidir?
D) 50 E) 60
A) 29
B) 6
C) 41
D) 3 5
E) 7

368 | Matematik 9
VERİ, SAYMA VE OLASILIK
9.6. VERİ

Neler Öğreneceksiniz? Veri-Grafi̇k Hazırlama ve


Yorumlamayı Öğrenmek Neden
Önemlidir ?

• Veri kavramını, sürekli ve kesikli veri • Günlük hayatı anlamak veri grafik
çeşitlerini, hazırlama ve yorumlama bilgisine sahip
• Merkezi eğilim ölçülerini (aritmetik olduğunuzda daha kolaydır. Örneğin sağlık
ortalama, ortanca ve tepe değer) ve sektöründeki muayene ve tahlil sonuçları,
hesaplamalarını, seçim analizleri, ekonomik gelişmeler
neticesinde elde edilen fiyat endeksleri,
• Merkezi yayılım ölçülerini (en büyük ve
büyüme, enflasyon, işsizlik, ihracat ithalat
en küçük değer, açıklık, alt ve üst çeyrek,
vb. verilere ait grafikler bunlar arasında
çeyrekler açıklığı ve standart sapma) ve
sayılabilir.
hesaplamalarını,
• Yaşamın her anı bir veri olarak ifade
• Merkezi eğilim ve yayılım ölçüleri
edilebilir. Bazen bir grafik, dosyalar dolusu
yardımıyla gerçek yaşam problemlerini
bilgiyi özetler. Yaşamı kolaylaştırmak;
yorumlamayı,
verilerin çözüm ve sonuçlarını elde
• Farklı değişkenler üzerinden veri etmek için bu grafikleri yorumlayabilmek,
toplamayı ve tablo oluşturmayı, aralarındaki ilişkiyi belirleyebilmek
• Ölçme sonucu elde edilen verilerin gerekir.
çizgi, sütun (çubuk), daire (pasta dilim), • Sonuç olarak veri ve grafikler sayesinde
serpme, histogram ve kutu grafiklerini ulaşılan bilgiler günlük yaşama dair
oluşturup yorumlamayı ve bu grafiklerden bilinçlenmeyi sağlar ve hayatı kolaylaştırır.
sonuç elde etmeyi öğreneceksiniz.

Matematik 9 | 369
VERİ

İ
statistik kelimesinin “durum ve devlet” anlamına gelen Latince
“status” kelimesinden geldiği bilinmektedir. Eski yıllarda insanlar,
devletin çeşitli yönlerden durumunu ifade edebilmek için birtakım
bilgiler toplayıp bunların değerlendirmesini yaparak istatistiğin temelini
atmıştır.
Belli amaçlar için araştırma yapılarak verilerin toplanması, toplanan
verilerin tasnif edilmesi, çözümlenmesi ve değerlendirilmesi ile ilgili
yöntemleri inceleyen bilime istatistik denir.
Modern istatistiğin gelişmesindeki en büyük etken, 16 ve 17.
yüzyıllarda bazı bilim insanlarının olasılık teorisi üzerine çalışmaya
başlamaları ile olmuştur. Bu çalışmaların öncüleri arasında Cardano
(Kardano), Pascal (Paskal) ve Fermat (Ferma) vardır.
İstatistiksel problemlerin analizine yönelik önemli çalışmalardan birisi
de 1766 yılında Daniel Bernoulli (Daniyel Bernuli) tarafından yapılan
çiçek hastalığı ve bu hastalıktan ölen kişilerin ölüm oranları bilgilerinin
analizidir. Bu analiz, aşının faydasının gösterilmesi amacıyla yapılmıştır.
İstatistik dendiğinde neredeyse bütün bilim dallarına yardımcı olan
ve sorunların çözümünde uygulanan yöntemler topluluğunu kapsayan
bir bilim dalı akla gelmektedir. Her beş yılda bir 20 Ekim tarihi Dünya
İstatistik Günü olarak kutlanmaktadır.

Etkinlik Elif Öğretmen, Biyoloji Olimpiyatları’na öğrencilerinin katılmasını


istemektedir. Bu amaçla öğrencileri arasında bir seçme sınavı yapmaya
karar verir. Sınava farklı şubelerden 10 öğrenci katılır. Sınav sonucunda
katılan öğrencilerin puanları 10 tam puan üzerinden
"5, 7, 8, 7, 9, 7, 8, 9, 7 , 8"
şeklinde açıklanıyor. Açıklanan puanlara göre
• Sınavın puan ortalaması kaçtır?
• Puanların hangi değer etrafında yığıldığı söylenebilir?
• En çok tekrar eden puan kaçtır?
• En düşük ve en yüksek puanların farkı kaçtır?
• Sınav puanları göz önüne alındığında öğrenci başarı seviyeleri için
ne söyleyebilirsiniz?
Cevaplarınızı, bölüm sonunda öğrendiklerinizle ilişkilendirerek
gözden geçiriniz.

370 | Matematik 9
VERİ

Günlük yaşamda sağlık, ekonomi, eğitim ve iş gibi birçok alana


yönelik konularda ölçüm, sayım, deney, gözlem veya araştırma yoluyla
elde edilen toplanmış ve çözümlenmiş bilgilere veri adı verilir. Sayısal
değer bildiren veriler nicel, sayısal değer bildirmeyen veriler de nitel
veriler olarak adlandırılır.
Matematikteki veri ise işlenmemiş ham bilgidir, istatistiksel amaçlar
için kullanılır. Elde edilen veriler kullanılarak grafik çizimleri yapılır.
Böylece bilgiler görsel hâle getirilerek daha kolay yorumlanabilir.
Matematiksel veriler sayılarla belirtilir. Sayısal veriler iki bölüme ayrılır:
Kesikli Veri: Belirli bir aralıktaki tüm reel sayı değerlerini alamayan
veri türüdür. Örneğin
• Bir kümesteki tavuk sayısı,
• Bir mobilya mağazasındaki sandalye sayısı vb. gibi.
Sürekli Veri: Ölçümler sonucu elde edilen, belirli bir aralıkta bütün
gerçek sayı değerlerini alabilen veri türüdür. Genellikle ondalık sayılarla
belirtilir. Örneğin:
• Aile bireylerinizin boyları
• Arkadaşlarımızın kilo değerleri vb. gibi.
Bu veriler kullanılarak yapılan işlem ve çizilen grafikler sayesinde
birçok konuda insanlara kolaylık sağlayacak bilgiler sunulabilmektedir.
Aile sağlığı merkezi ve aile hekimliklerinde bebek gelişiminin takibini
sağlayan veri tablosu, aşı takvimi (Tablo 9.6.1), nüfus artış ve azalışını
gösteren sayısal sonuçlar ve ekonomik durum grafikleri bunlara örnek
olarak gösterilebilir.

İlköğretim

İlköğretim
Doğumda

12. Ayın

18. Ayın

24. Ayın
1. Ayın

2. Ayın

4. Ayın

6. Ayın

1. Sınıf

8. Sınıf
Sonu

Sonu

Sonu

Sonu

Sonu

Sonu

Sonu
Aşılar

Sarılık (Hepatit B) I II III


Verem I
Difteri-Boğmaca- I II III R
Tetanoz
Zatürre-Menenjit I II III R
Kızamık-
Kızamıkçık- I R
Kabakulak
Difteri-Boğmaca R
Çocuk Felci I II
Erişkin Tipi R
Difteri-Tetanoz
Sarılık (Hepatit A) I II
Suçiçeği I

Tablo 9.6.1: Sağlık Bakanlığı aşı takvimi

Bulunduğunuz bölgeden seçtiğiniz dört ilin son üç nüfus


sayımına ait bilgileri tablo yapıp yorumlayınız.

Matematik 9 | 371
VERİ

9.6.1. MERKEZİ EĞİLİM VE YAYILIM ÖLÇÜLERİ


1. Merkezi Eğilim Ölçüleri
Bir veri grubunun hangi değer etrafında toplandığını gösteren sayısal değerlerdir. Bunlar aritmetik
ortalama, ortanca (medyan) ve tepe değer (mod) olarak adlandırılır.

Aritmetik Ortalama
Veri grubunda bulunan verilerin toplamının veri sayısına bölünmesi ile elde edilen değere aritmetik
ortalama denir. X sembolü ile gösterilir.

X= formülü ile hesaplanır.

ÖRNEK 1:
Bir kreşte 6 gün boyunca oyun grubuna katılan çocuk sayıları
sırasıyla 3, 4, 2, 8, x, 5 şeklindedir.
Bu kreşte oyun grubunda bulunan çocuk sayısının günlük ortalaması
5 olduğuna göre 5. gün oyun grubuna katılan çocuk sayısının kaç
olduğunu bulunuz.

ÇÖZÜM:
Verilerin aritmetik ortalaması 5 ise
3+4+2+8+x+5 22 + x
X=5= 6 = 6
22 + x = 30 ise x = 8 olarak bulunur.

ÖRNEK 2:
Babası, Can’dan 1 hafta boyunca her gün, günün aynı saatinde hava sıcaklığını ölçmesini ve bir hafta
sonunda ortalama sıcaklığı hesaplamasını istiyor. Can, tüm ölçümlerini yaptıktan sonra günlük ortalama
sıcaklığı 22 derece bulmuştur. Sıcaklığın 14 ve 25 derece olduğu iki günde ölçüm yapmamış olsaydı, oluşacak
yeni aritmetik ortalama ne olurdu?
ÇÖZÜM:
7 verinin toplamı :x
Veri sayısı :7
x
Aritmetik ortalama 22 olduğundan 22 = 7 & x = 154 bulunur.
Bu durumda
Yeni veri toplamı : 154 - (14 + 25) = 154 - 39 = 115
Yeni veri sayısı : 7-2 = 5
115
Yeni aritmetik ortalama : 5 = 23 olarak bulunur.

372 | Matematik 9
VERİ

ÖRNEK 3:
Tabloda 2 öğrencinin biyoloji sınavından aldıkları puanlar verilmiştir. Bu puanlara göre hangi öğrencinin
biyoloji dersinde daha başarılı olduğunu bulunuz.
1. Sınav 2. Sınav 3. Sınav
1. Öğrenci 90 65 55
2. Öğrenci 76 100 46

ÇÖZÜM:
1. öğrencinin aritmetik ortalaması X 1 , 2. öğrencinin aritmetik ortalaması X 2 olmak üzere
90 + 65 + 55 _b
X1 = = 70 bb
3 b
`b X 2 2 X 1 olduğundan 2. öğrenci biyoloji dersinde daha başarılıdır.
76 + 100 + 46 bb
X2 = 3 = 74 b
a

Ağırlıklı ortalama: Veri grubu içindeki her bir verinin; veri ağırlığı ile çarpıldıktan sonra alınan
toplamın, ağırlıklar toplamına bölünmesi ile elde edilen ortalamadır.

ÖRNEK 4:
Aşağıdaki tabloda, bir sınıftaki öğrenci sayıları ve bu öğrencilerin matematik dersinden aldıkları puanlar
verilmiştir. Buna göre öğrencilerin matematik puanlarının ortalamasını bulunuz.

Öğrenci Sayısı 3 4 5 8
Matematik Puanları 90 85 60 30

ÇÖZÜM:
3 $ 90 + 4 $ 85 + 5 $ 60 + 8 $ 30 = 1150 =
Ağırlıklı ortalama = 8+5+4+3 20 57, 5 bulunur.

Ortanca (Medyan) Değeri


Bir veri grubunun medyanını bulmak için veriler önce küçükten büyüğe ya da büyükten küçüğe
doğru sıralanır. Buna göre
Veri sayısı tek ise ortadaki veri ortanca (medyan) dır.
Veri sayısı çift ise ortadaki iki verinin aritmetik ortalaması medyandır ve Q 2 ile gösterilir.

ÖRNEK 5:
15, 20, 25, 10, 15, 10, 15 veri grubunun medyanını bulunuz.
ÇÖZÜM:
Veriler küçükten büyüğe sıralandığında

10, 10, 15, 15 , 15, 20, 25 sıralamasının ortasındaki veri 15 olduğundan veri grubunun medyanı
. Q 2 = 15 olur.
Medyan

Matematik 9 | 373
VERİ

ÖRNEK 6:
Ali’nin kütüphanesinde bulunan farklı türdeki kitaplarının sayıları 5, 10, 8, 16, 4, 13 şeklindedir. Bu veri
grubunun medyanını bulunuz.
ÇÖZÜM:
Veriler küçükten büyüğe doğru sıralandığında 4, 5, 8, 10 ,13 ,16 sıralaması elde edilir. Veri sayısı çift
olduğundan ortadaki iki verinin aritmetik ortalaması aranan medyan değeri olur.
Veri grubunun medyan değeri:
8 + 10
Q2 = 2 = 9 olarak bulunur.

ÖRNEK 7:
x, 4, 5, 8, 12 veri grubunun medyanı 6 ise grubun aritmetik ortalamasını bulunuz.
ÇÖZÜM:
Medyan 6 ise veri sıralaması 4, 5, x, 8, 12 şeklinde olur. Bu durumda

4 5 x 8
_
12 bb
bb
. `b X = 4 + 5 + 65+ 8 + 12 = 7 olarak bulunur.
bb
6 b
a

Tepe Değer (Mod)


Ölçümler sonucu elde edilen verilerden en çok tekrar eden
veriye grubun tepe değeri (modu) denir. Veri grubundaki sayıların
tekrar sayılarına frekans adı verilir. O hâlde en yüksek frekans
moddur.
• Bir veri grubunun birden fazla modu olabilir.
• Veri grubunda tekrar eden sayı yoksa grubun modu verilerin
tamamıdır. Veri grubunda tüm farklı verilerin tekrar sayıları
eşit ise tekrar eden her bir veri grubun modudur.

ÖRNEK 8:
Aşağıdaki veri gruplarının (varsa) modunu bulunuz.
a) 3, 11, 4, 12, 4, 2
b) 9, 15, 6, 11, 6, 17, 15, 23
c) 2, 2, 5, 5, 8, 8, 4, 4
ç) 1, 1, 1, 1

ÇÖZÜM:

a) 2, 3, 4, 4, 11, 12 veri grubunda en çok tekrar eden veri 4 olduğundan mod değeri 4 olur.
b) 6, 6, 9, 11, 15, 15, 17, 23 veri grubunda en çok tekrar eden veriler 6 ve 15 olduğundan veri grubunun
modları 6 ve 15 olur.
c) 2, 2, 4, 4, 5, 5, 8, 8 veri grubunda tüm veriler eşit sayıda tekrar ettiği için veri grubunun modu 2, 4, 5 ve
8 dir.
ç) 1, 1, 1, 1 veri grubunda tüm veriler eşit olduğundan grubun modu 1 dir.

374 | Matematik 9
VERİ

ÖRNEK 9:
8, 6, 9, x, 9, 7 veri grubunun iki farklı modu olduğuna göre x in alabileceği değerler toplamını bulunuz.
ÇÖZÜM:
Veri grubunda 9, iki kez tekrar ettiği için birinci mod 9 olur. İkinci mod ise 8, 6 ve 7 verilerinden herhangi
birinin 2 kez tekrar etmesi sonucu oluşur. Bu durumda x, 3 farklı değer alır. Bunlar 6, 7 ve 8 dir.
Buna göre x değerleri toplamı: 6 + 7 + 8 = 21 olarak bulunur.

ÖRNEK 10:
Bir deneme sınavına katılan öğrenci sayıları ile yaptıkları doğru cevap sayıları aşağıdaki tabloda verilmiştir.
Öğrencilerin doğru cevap sayılarının modu 19 olduğuna göre x in alabileceği en küçük tam sayı değerini
bulunuz.

Öğrenci Sayısı 6 5 7 x 8 9
Doğru Sayısı 18 17 16 19 20 15

ÇÖZÜM:
Mod 19 olduğundan x sayısı, diğer öğrenci sayılarından daha büyük bir değer olmalıdır. Bu durumda
x > 9 olur. 9 dan büyük olan en küçük tam sayı arandığı için x = 10 olarak bulunur.

Sıra Sizde

SORU:
Tam sayılardan oluşan 4, 7, x, x + 3, 10, y, 2y - 5 veri grubunun aritmetik ortalaması 10,
modu 7 ise medyanını bulunuz.

ÇÖZÜM:

Seçtiğiniz bir konuda (sınıf arkadaşlarınızın burçları, tuttukları futbol takımları vb.) tüm sınıf
arkadaşlarınızın bilgilerini alınız. Aldığınız bilgilere göre bir veri grubu oluşturunuz. Veri
grubunun merkezi eğilim ölçülerini bulunuz.

Matematik 9 | 375
VERİ

2. Merkezi Yayılım Ölçüleri


Bir veri grubundaki verilerin birbirine yakınlık veya uzaklığı hakkında bilgi veren ölçülerdir. Bunlar açıklık,
alt çeyrek, üst çeyrek, çeyrekler açıklığı ve standart sapma olarak adlandırılır.

Açıklık
Veri grubundaki sayılar küçükten büyüğe doğru sıralandığında elde edilen en büyük sayıya veri
grubunun en büyük değeri, en küçük sayıya ise veri grubunun en küçük değeri denir.
En büyük değer ile en küçük değer arasındaki fark açıklık olarak adlandırılır.

ÖRNEK 1:
3, 5, 7, 11, 13, 16, 20, 25, 30 şeklinde verilen veri grubunun açıklığını bulunuz.

ÇÖZÜM: 3 , 5, 7, 11, 13, 16, 20, 25, 30


. .
En küçük değer En büyük değer
O hâlde veri grubunun açıklığı: En büyük değer - En küçük değer = 30 - 3 = 27 olarak bulunur.

ÖRNEK 2:
8, 10, 6, 12, 26, 10, 5, 21, x şeklinde verilen veri grubunun açıklığı 22 olduğuna göre x in alabileceği
değerler toplamını bulunuz.
ÇÖZÜM:
Veri grubunda x dışındaki herhangi iki verinin farkı 22 olmalıdır. Açıklık 22 olduğuna göre x ya en küçük ya
da en büyük veri olmalıdır. x en küçük veri ise verilenlere göre en büyük veri 26 olur. Bu durumda
22 = 26 - x olduğundan x = 4 bulunur. ...(1)
x en büyük veri ise en küçük veri 5 olur. Bu durumda
x - 5 = 22 olduğundan x = 27 bulunur. ...(2)
Buna göre (1) ve (2) den x değerleri toplamı: 4 + 27 = 31 olur.

Sıra Sizde

SORU:
Bir halk oyunları yarışması 5 kişiden oluşan bir jüri tarafından Seyyah 92 86 95 85 75
değerlendirilmektedir. Puanlamaya jüri üyelerinin verdiği en
Ekin 90 80 93 87 85
düşük ve en yüksek puanlar dâhil edilmeyip kalan puanların
aritmetik ortalaması alınarak ekiplerin toplam puanları hesaplanacaktır. Bu yarışmaya katılan“Seyyah”
ve “Ekin” adlı halk oyunları ekiplerine jüriler tarafından verilen puanlar yukarıdaki gibidir. Buna göre
1. Ekip puanları ile kalan verilerin medyanlarını ve açıklıklarını hesaplayınız.
2. Ekiplere verilen jüri puanlarının (uç değerler çıkarılmadan) medyan ve aritmetik ortalama değerini
hesaplayınız.
3. Bu iki durum incelendiğinde aritmetik ortalamanın ve medyanın uç değerlerden ne şekilde
etkilendiğini yorumlayınız.
ÇÖZÜM:

376 | Matematik 9
VERİ

Alt Çeyrek, Üst Çeyrek, Çeyrekler Açıklığı


Bir veri grubunun alt çeyrek, üst çeyrek ve çeyrekler açıklığını bulmak için veriler, küçükten büyüğe
sıralanır. Elde edilen veri grubunun medyanı bulunur. Medyandan küçük olan verilerin oluşturduğu
gruba alt grup, büyük olan verilerin oluşturduğu gruba üst grup adı verilir. Alt grubun medyan
değerine alt çeyrek, üst grubun medyan değerine üst çeyrek denir. Üst çeyrek ile alt çeyrek arasındaki
fark çeyrekler açıklığı olarak ifade edilir.
Alt çeyrek Q 1 , üst çeyrek Q 3 ve çeyrekler açıklığı da Q sembolü ile gösterilir.

ÖRNEK 3:
Bir hastanenin iç hastalıkları polikliniğine 11 gün boyunca gelen hasta sayısı aşağıdaki gibidir.
35, 42, 46, 38, 33, 42, 48, 52, 30, 55, 41
Bu veri grubunun alt çeyreğini, üst çeyreğini ve çeyrekler açıklığını bulunuz.

ÇÖZÜM:
Veriler küçükten büyüğe doğru sıralanırsa 30, 33, 35, 38, 41, 42, 42, 46, 48, 52, 55 bulunur. Bu durumda
Alt grup Üst grup
6444444447444444448 6444444447444444448
30, 33, 35, 38, 41, 42 , 42, 46, 48, 52, 55
. . .
Alt çeyrek Medyan Üst çeyrek
( Q 1) ( Q 3)

Çeyrekler açıklığı: 48 - 35 = 13 olur.


ÖRNEK 4:
Salı günleri kapalı olup çalışmayan bir bayan kuaförüne 6 gün
boyunca gelen müşteri sayısı 18, 27, 10, 36, 30, 45 ise bu veri grubunun
alt çeyrek, üst çeyrek ve çeyrekler açıklığını bulunuz.

ÇÖZÜM:
Veriler küçükten büyüğe doğru sıralandığında 10, 18, 27, 30, 36, 45 olur. Veri sayısı çift olduğundan
Alt grup Üst grup
6444447444448 6444447444448
10 18 27 30 36 45 Çeyrekler açıklığı = 36 - 18 = 18 olur.
. .
Alt çeyrek Üst çeyrek

ÖRNEK 5:
Ömer Bey, çiftliğinde organik yumurta üretmektedir. 12 gün boyunca elde ettiği yumurta sayıları 14, 11,
10, 13, 8, 12, 9, 15, 11, 14, 12, 14 tür. Bu veri grubunun alt çeyrek, üst çeyrek ve çeyrekler açıklığını bulunuz.
ÇÖZÜM:
Veriler küçükten büyüğe doğru sıralandığında 8, 9, 10, 11, 11, 12, 12, 13, 14, 14, 14, 15 olur. Veri sayısı
çift olduğu5dan
Alt grup Üst grup
6444444444447444444444448 644444444444474444444444448
8 9 10 11 11 12 12 13 14 14 14 15

Alt çeyrek ^Q 1 h =
10 + 11
2 = 10, 5
Çeyrekler açıklığı = 14 - 10, 5 = 3, 5 olur.
Üst çeyrek ^Q 3 h =
14 + 14
2 = 14

Matematik 9 | 377
VERİ

ÖRNEK 6:
A ve B araba galerilerinin 9 hafta boyunca sattığı araç sayıları aşağıda tablo hâlinde verilmiştir. Bu verilere
göre 9. hafta sonunda daha istikrarlı satış yapan galeriyi bulunuz.
1. Hafta 2. Hafta 3. Hafta 4. Hafta 5. Hafta 6. Hafta 7. Hafta 8. Hafta 9.Hafta
A Galerisi 4 0 3 2 2 3 2 3 0
B Galerisi 2 2 1 4 2 0 3 4 1

ÇÖZÜM:
A ve B galerilerinin haftalık satış adetlerinin oluşturduğu veri grupları için
A galerisi B galerisi
Alt grup Üst grup Alt grup Üst grup
64444744448 64444744448 64444744448 64444744448
0, 0, 2 , 2, 2 , 3, 3, 3 , 4 0, 1, 1 , 2, 2 , 2, 3, 4 , 4
. .
Medyan Medyan
alt grup, üst grup ve medyan değerleri bulunur. Buradan
Alt çeyrek (A) = 0 + 2 = 1 1+1
Alt çeyrek (B) = 2 = 1
2
3+3 3+4
Üst çeyrek (A) = 2 = 3 Üst çeyrek (B) = 2 = 3, 5
Çeyrekler açıklığı ^A h = 3 - 1 = 2 Çeyrekler açıklığı ^ B h = 3, 5 - 1 = 2, 5 elde edilir. Bu durumda A
galerisinin çeyrekler açıklığı, B galerisinin çeyrekler açıklığından küçük olduğundan A galerisi en başarılı
araba galerisi olarak bulunur.

ÖRNEK 7:
Bir çini tabak üretim firmasının sahibi olsaydınız çalışma günlerine
göre boyadıkları tabak sayıları verilen ustalardan hangisinin daha
istikrarlı olduğunu düşünürdünüz?
A ustası: 7, 9, 6, 5, 3
B ustası: 4, 8, 5, 5, 6, 8
C ustası: 5, 4, 1, 11, 7, 6, 9, 5
ÇÖZÜM:
Ustalardan hangisinin daha istikrarlı olduğunu bulmak için çeyrek açıklığı hesaplandığında
A ustası B ustası C ustası

3 5 6 7 9 4 5 5 6 8 8 1 4 5 5 6 7 9 11
Alt çeyrek (B) = 5 Alt çeyrek (C) = 4 + 5 = 4, 5
Alt çeyrek (A) = 3 + 5 = 4 2
2 Üst çeyrek (B) = 8
7+9
Üst çeyrek (A) = 2 = 8
9
Üst çeyrek (C) = + 7 = 8
2

Çeyrekler açıklığı Çeyrekler açıklığı Çeyrekler açıklığı


^Ah = 8 - 4 = 4 ^Bh = 8 - 5 = 3 ^C h = 8 - 4, 5 = 3, 5

B ustasının çeyrekler açıklığı değeri en küçük olduğu için daha istikrarlı bir çalışma yaptığı görülür.

378 | Matematik 9
VERİ

Standart Sapma
Bir veri grubundaki her bir verinin aritmetik ortalamadan ne kadar uzaklaştığı standart sapma ile
belirlenir. Başka bir ifadeyle grup içindeki farklılaşmaya standart sapma adı verilir ve S ile gösterilir.
• Veri grubunun standart sapması yüksek ise veriler aritmetik ortalamadan uzak, standart
sapması düşük ise veriler aritmetik ortalamaya yakın demektir.
• Standart sapmanın düşük olması veri grubunun homojen, yüksek olması ise heterojen
olduğunu gösterir.
• Yapılmakta olan ölçme değerlendirme sınavlarında standart sapmanın yüksek olması seviye
farklılığının yüksekliğini, standart sapmanın düşük olması bu sınavlarda seviyenin birbirine
yakınlığını ifade eder.
Bir veri grubunun standart sapmasını bulmak için aşağıdaki adımları izleriz.
1. Adım :
Veri grubunun aritmetik ortalaması bulunur.
2. Adım :
Her bir verinin aritmetik ortalama ile farkının kareleri toplamı bulunur.
3. Adım :
Bulunan toplam, veri sayısının bir eksiğine bölünüp karekökü alınır.

S = Standart Sapma, X = Aritmetik Ortalama, n = Veri Sayısı

^ X 1 - X h2 + ^ X 2 - X h2 + f + ^ X n - X h2
S= dir.
n-1

ÖRNEK 8:
Beş lise öğrencisinin öğrenim süreleri boyunca aldıkları derslere ait
puanların aritmetik ortalamaları ve bu puanların standart sapmaları
aşağıdaki tabloda verilmiştir.
Aritmetik Ortalama Standart Sapma
Ali 90 2,1
Ayşe 87 3,5
Sena 90 1,9
Yusuf 85 4
Esra 87 3,8

Bu beş öğrenciden en başarılı olan üçüne başarı belgesi verilecektir.


Buna göre belge verilecek öğrencileri bulunuz.

ÇÖZÜM:
Aritmetik ortalamaların eşit olduğu durumlarda, karşılaştırma yapabilmek için standart sapmaya bakılır.
Standart sapması küçük olan ortalama daha istikrarlı olacağından başarı belgesi verilecek öğrenciler sırasıyla
Sena, Ali ve Ayşe’dir.

Matematik 9 | 379
VERİ

ÖRNEK 9:
Aşağıdaki tabloda Efe ile Erol’un son 5 futbol maçında attıkları
gollerin sayıları verilmiştir. Bu iki sporcudan daha istikrarlı olan tercih
edilecektir. Bu kişiyi belirleyiniz.

1. Maç 2. Maç 3. Maç 4. Maç 5. Maç


Erol 6 1 2 2 4
Efe 2 3 4 4 2

ÇÖZÜM:
6+1+2+2+4 15
a) X = 5 = 5 =3
b) ^ 6 - 3 h + ^ 1 - 3 h2 + ^ 2 - 3 h2 + ^ 2 - 3 h2 + ^ 4 - 3 h2 = 9 + 4 + 1 + 1 + 1 = 16
2
Erol:
16
c) S = 4 =2
2+3+4+4+2 15
a) X = 5 = 5 =3
Efe: b) ^ 2 - 3 h2 + ^ 3 - 3 h2 + ^ 4 - 3 h2 + ^ 4 - 3 h2 + ^ 2 - 3 h2 = 1 + 0 + 1 + 1 + 1 = 4
4
c) S = 4 = 1
Erol ve Efe’nin aritmetik ortalamaları aynıdır. Bu durumda attığı gollere göre standart sapması küçük olan
Efe’nin tercih edilmesi daha uygun olur.

ÖRNEK 10:
Bir ilaç firması; bir hastalığı tedavi amaçlı A, B, C ilaçları için
üretime başlayacaktır. Üç ülkede kobay fareler üzerinde yapılan
denemeler sonucunda hastalıktan kurtulan fare sayısı aşağıdaki tabloda
belirtilmiştir.

2012 2013 2014 2015 2016


A İlacı 9 16 20 8 12
B İlacı 4 17 14 8 22
C İlacı 14 15 10 20 6
Bu verilere göre ilaç firmasının öncelikle hangi ilacın üretimine başlaması gerektiğini bulunuz.

ÇÖZÜM:

9 + 16 + 20 + 8 + 12 65
XA = 5 = 5 = 13
^ 13 - 9 h2 + ^ 13 - 16 h2 + ^ 13 - 20 h2 + ^ 13 - 8 h2 + ^ 13 - 12 h2 100
SA = 5-1 = 4 =5
4 + 17 + 14 + 8 + 22 65
XB = 5 = 5 = 13
204
SB = 4 = 51 . 7, 14
14 + 15 + 10 + 20 + 6 65
XC = 5 = 5 = 13
112
SC = 4 = 28 . 5, 29
Standart sapma küçüldükçe başarısızlık riski azalır ve tedavide başarı şansı artar. Bu sebeple standart
sapması küçük olan A ilacını üretmeye öncelik verilmesi uygundur.

380 | Matematik 9
VERİ

ÖRNEK 11:
a) Bir öğrenci grubunun A ve B derslerinden aldıkları puanların homojenlik durumlarının kıyaslanması,
b) Bir sınıftaki öğrencilerin bir ay boyunca en çok dinlediği müzik türünün belirlenmesi,
c) Derya’nın bir haftada çözdüğü 40 soruluk matematik testindeki 32, 35, 30, 31, 38, 34, 40 olan
net sayılarını temsil eden değerin bulunması işlemlerinde merkezi eğilim ve yayılım ölçülerinden
hangisinin kullanılması gerektiğini belirleyiniz.
I. Standart sapma
II. Medyan
III. Mod
ÇÖZÜM:
a) İki farklı veri grubunu karşılaştırmak için en uygun değerlendirme ölçüsü standart sapmadır. (a→I)
b) Bir ayda en çok dinlenen müzik demek, hem matematiksel işlem gerektirmeyen hem de en çok tekrar
eden sayı demek olduğundan en uygun değerlendirme ölçüsü mod olur. (b→III)
c) Matematik net sayıları küçükten büyüğe doğru sıralandığında 30, 31, 32, 34, 35, 36, 40 tır. Kırk hariç
diğer verilerin birbirine yakın olduğu görülmektedir. Bu sebeple en uygun değerlendirme ölçüsü
medyan olur. (c→II)

Sıra Sizde

SORU:
Bir apartmanda bulunan 9 ve 22 no.lu iki dairenin günlük doğal gaz tüketim değerleri metreküp
cinsinden tablo hâlinde
gösterilmiştir. Pzt. Sal. Çar. Per. Cum. Cmt. Paz.
Tabloya göre her bir dairenin
9 no.lu daire 14 13 12 10 8 9 11
günlük doğalgaz tüketiminin
22 no.lu daire 12 13 11 10 9 10 12
a) Aritmetik ortalamasını,
b) Standart sapmasını,
c) Açıklığını bularak yorumlayınız.

ÇÖZÜM:

Türkiye İstatistik Kurumu (TÜİK) aylık olarak Türkiye ile ilgili farklı konularda yapılan
araştırma sonuçlarını açıklamaktadır. Bu araştırmalardan birini seçiniz. Seçtiğiniz
araştırmaya ait veri grubunun merkezi yayılım ölçülerini bularak yorumlayınız.

Matematik 9 | 381
VERİ

ALIŞTIRMALAR

1
Aşağıdaki tabloda verilen terimleri tanımları ile eşleştiriniz.
En büyük değerden en küçük değerin çıkarılması ile elde edilen
.....
1. En büyük değer sayısal veridir.
2. Medyan ..... Belirli bir aralıktaki tüm gerçek sayı değerlerini alamayan veri türüdür.
3. Alt çeyrek
4. Açıklık Veriler sıralandığında veri sayısı tek ise ortadaki veri, veri sayısı çift ise
.....
5. Kesikli veri ortadaki iki terimin aritmetik ortalamasıdır.
6. Mod Ölçüm sonucunda elde edilen veri dağılımındaki her bir verinin
7. Sürekli veri ..... aritmetik ortalamadan ne kadar uzaklaştığını, grup içindeki
farklılaşmayı ortaya koyan ölçümdür.
8. Standart sapma
9. Çeyrekler açıklığı Veri grubunda bulunan veri toplamının veri sayısına bölünmesi ile
.....
10. Aritmetik ortalama elde edilen değere denir.
..... Ölçümler sonucu elde edilen verilerden en çok tekrar eden veridir.

2 Aşağıdaki tabloda Eray, Emre ve Mert’in son 5 sezonda attıkları gol sayıları verilmiştir. Santrfor transfer
etmek isteyen A takımının teknik heyetinde olsanız bu 3 oyuncudan hangisini tercih edersiniz? Neden?

1. Sezon 2. Sezon 3. Sezon 4. Sezon 5. Sezon


Eray 12 13 16 13 11
Emre 12 13 12 13 10
Mert 13 9 11 7 15

3 Aynı telefon operatörünü kullanan 8 kişilik gruba bir aylık aynı Genel Ağ paketi hediye edilmiştir.
Bu 8 kişinin Genel Ağ paketini bitirme süreleri gün olarak aşağıdaki gibidir.
15, 18, 12, 14, 18, 17, 15, 19
Bu veri grubunun
a) Tepe değerini,
b) Medyanını,
c) Açıklığını,
ç) Çeyrekler açıklığını,
d) Aritmetik ortalamasını,
e) Standart sapmasını bulunuz.

382 | Matematik 9
VERİ

x, 2x+2, 3x+4 sayılarının aritmetik ortalaması 8


10, 14, 12, 20, 16, 12, 17 veri grubunun modu
4
38 ise x değerini bulunuz. x, medyanı y ve çeyrekler açıklığı z olduğuna
göre x, y ve z arasındaki sıralamayı bulunuz.

5 Bir öğrencinin öğrenim gördüğü matematik, 9 Aşağıdaki tabloda belli yıllara ait belediye
fizik, kimya ve biyoloji derslerinin haftalık nüfusunun genel nüfusa oranları verilmiştir.
ders saatleri ve bu derslerden alınan puanlar Genel nüfus belediye nüfusu ve köy nüfusu
aşağıdaki gibidir. olarak ikiye ayrılmaktadır. Belediyelik
olamayan yerin köy olduğu bilindiğine göre
Dersler Ders Saati Ders Puanı köy nüfusunun yüzdelik oranlarının aritmetik
ortalamasını bulunuz.
Matematik 6 72
Fizik 2 82 Belediye Nüfusunun Genel
Sayım Yılları
Nüfusa Oranı
Kimya 2 75
2000 %78
Biyoloji 3 85
2008 %82
Bu öğrencinin yukarıdaki derslerden aldığı
puanların ağırlıklı puan ortalamasını bulunuz. 2009 %83
2010 %83
2011 %83
2014 %93

6 Bir otobüs durağında bekleyen öğrencilerin 10 7, 8, 9, 10, 3, 12, 5, m, 15 veri grubunun


yaşları 10, 7, y, 12, 13, 10, 12 şeklindedir. aritmetik ortalaması 8 ise standart sapmasını
bulunuz.
Öğrenci yaşlarının oluşturduğu veri grubunun
modu 10 olduğuna göre medyanını bulunuz.

7 3, 37, 10, x, 15, 23, 30, 8 11 A: 20, 30, 34


Veri grubunun açıklığı 35 ise x in alabileceği B: 12, 10, 18, 12
değerlerin toplamını bulunuz. C: 15, 15, 24
Verilen A, B ve C veri gruplarının standart
sapmalarını küçükten büyüğe sıralayınız.

Matematik 9 | 383
VERİ

9.6.2. VERİLERİN GRAFİKLE GÖSTERİLMESİ


Grafik 9.6.1 de; TÜİK verilerine göre 2015-2016 yılları aylık değişimler göz önüne alındığında tüketici
fiyatlarının (TÜFE) hangi yılda daha stabil (durağan) olduğu söylenebilir?

Yıllık Değişim
Oranı (%)

12

9,58
10 8,79
8,78 8,81
8,09 8,05 7,95 8,10
7,46 7,64 7,58
8 8,53
7,91
7,24 7,55 7,61
7,20 6,81 7,14 7,28 7,16 7,00
6 6,57 6,58
2015
4 2016

1 Aylar
1 2 3 4 5 6 7 8 9 10 11 12
Grafik: 9.6.1

Grafik 9.6.2 de, TÜİK verilerine göre, Türkiye ve bazı Avrupa Birliği ülkelerindeki peynir üretiminin
sıralaması verilmiştir. Tarım ülkesi olarak bilinen Türkiye’nin sıralaması hakkında ne söyleyebilirsiniz?

Peynir Üretimi
(1000 ton)

2500

2000

1500

1000
500
0
ya sa ya da ya ye re ka re ya
an Fran İtal llan olon ürki gilte imar İsviç stur
Alm Ho P T İn an
D Av u

Grafik: 9.6.2

Grafik 9.6.3 de; TÜİK verilerine göre, Türkiye'deki emeklilerin yaş dilimlerine göre oranları verilmiştir.
Sizce Türkiye’de çalışanların 45-65 yaş arasında emekli olmasının olumlu ya da olumsuz ne tür sonuçları
olabilir?
35-45 yaş
% 1,2

76+yaş
% 9,6
66-75 yaş 46-55 yaş
% 20,1 % 29,3

56-65 yaş
% 39,8

Grafik: 9.6.3

384 | Matematik 9
VERİ

Etkinlik

Bir telefon satış mağazasındaki 3 farklı telefon markasının 4 aylık telefon satışları Tablo 9.6.2 de verilmiştir.
Telefon markaları ve satışları ile ilgili oluşturulan grafik türlerini inceleyiniz.
Aylar
Markalar Mayıs Haziran Temmuz Ağustos
A 80 95 75 120
B 60 45 30 90
C 25 40 55 60
Toplam 165 180 160 270
Tablo: 9.6.2

1. Üç markanın aylara göre satış miktarlarını gösteren Grafik 9.6.4 teki çizgi grafiğini inceleyiniz. Grafiğe göre

Satış (Adet)

140 a) Dört aylık toplam satış miktarı en fazla olan


120 markayı bulunuz.
100
80 A b) Temmuz ayında en fazla satış yapan marka
60 B hangisidir?
40 C
20
c) Hangi markanın satış miktarındaki değişim en
0 Aylar fazladır?
Mayıs Haziran Temmuz Ağustos

Grafik: 9.6.4

2. Üç markanın aylara göre satış miktarlarını gösteren Grafik 9.6.5 teki daire grafiğini inceleyiniz. Grafiğe göre

A Satış adetlerinin yüzdelik oranlarını gösteren


B daire grafiği verilmiş olsaydı her bir markaya ait
C yüzdelik oranlar ne olurdu?

Grafik: 9.6.5

Matematik 9 | 385
VERİ

Ölçme sonucunda elde edilen verilerin nokta, çizgi, sütun gibi farklı şekillerle ifade edilmesine
grafik denir. Grafikler, sayısal verileri görselleştirerek bunlar arasında karşılaştırmalar yapılabilmesine
imkân tanır. Böylece sayısal veriler daha kolay anlaşılıp yorumlanır.
Yaygın olarak kullanılan grafik türleri şunlardır:
1. Histogram
2. Çizgi grafiği
3. Sütun (çubuk) grafiği
4. Daire (pasta, dilim) grafiği
5. Serpme grafiği
6. Kutu grafiği

1. Histogram Oluşturma
Gruplandırılmış bir veri topluluğunda, verilerin tekrar etme sayılarının bitişik dikdörtgen şeklinde
sütunlar hâlinde gösterimidir. Veri sayılarının çok olduğu durumlarda tercih edilmektedir. Her aralık
eşit olarak bölünür ve gösterilir. Her bir veri değeri için değil, belirli aralıktaki toplam veri sayısı için
yorum yapılmaktadır.
Histogram oluşturulurken aşağıdaki işlemler yapılır:
1. Veri grubunun açıklığı bulunur.
2. Veri grubunun açıklığı seçilen grup sayısına bölünür.
Açıklık
1 Grup genişliği
Grup sayısı
3. Yukarıdaki eşitsizliği sağlayan en küçük tam sayı, grup genişliği olarak belirlenir.

ÖRNEK 1:
19 kişilik bir sınıftaki öğrencilerin fizik dersi sınav sonuçları aşağıda verilmiştir.
43, 22, 86, 77, 24, 37, 59, 30, 64, 36, 48, 74, 41, 90, 53, 27, 40, 50, 31
Sınav sonuçlarını 7 grup olacak şekilde gösteren histogramı çiziniz ve yorumlayınız.
ÇÖZÜM:
Histogramı oluşturmak için veriler küçükten büyüğe doğru sıralanır.
22, 24, 27, 30, 31, 36, 37, 40, 41, 43, 48, 50, 53, 59, 64, 74, 77, 86, 90
Grup genişliğini bulmak için açıklık seçilen grup sayısına bölündüğünde
Açıklık = 90 - 22 Açıklık
Grup sayısı 1 Grup genişliği
= 68 olur.
68
7 1 Grup genişliği Eşitsizliği sağlayan en küçük doğal sayı
9, 7 1 Grup genişliği alınacağından grup genişliği 10 olur.

Veriler, grup genişliği 10 olacak şekilde tablodaki gibi gruplanır.

Öğrenci
Sayısı

Aralık Frekans 5
22-31 5 Oluşturulan histograma göre 9
32-41 4 4
42-51 3
öğrencinin 22-41 aralığında puan
3
52-61 2 aldığı görülmektedir. Sınıfta 52
62-71 1 2 puandan yüksek alan 7 öğrenci,
72-81 2 1 51 puandan düşük alan 12 öğrenci
82-91 2 vardır. Sınıfın yarısından fazlası 51
0 Puan
31 2-41 2-51 2-61 2-71 2-81 2-91 puandan düşük almıştır.
22- 3 4 5 6 7 8 Aralıkları

386 | Matematik 9
VERİ

ÖRNEK 2: Öğrenci Sayısı

Yandaki histogramda bir okuldaki öğrencilerin uyku saatleri 80


verilmiştir. 21.01 ile 22.30 arasında uyuyan öğrenci sayısının okuldaki
toplam öğrenci sayısına oranını bulunuz. 48
40
ÇÖZÜM:
Okuldaki toplam öğrenci sayısı = 14 + 28 + 40 + 48 + 80 = 210 28
21.01 ile 22.30 arasında uyuyan öğrenci sayısı
14
= 40 + 48 + 80 = 168
21.01 ile 22.30 arasında uyuyan öğrenci sayısının okuldaki 0 Saat
168 30 00 .30 .00 2.30
20. 21. -21 -22
toplam öğrenci sayısına oranı 210 olur. Buna göre . 00- 0.31- 1.01 1.31 2.01-
2
2 0 2 2 2 2
168 8 80
210 = 10 = 100 = %80 olarak bulunur.
Öğrenci Sayısı

ÖRNEK 3: 80
Bir okuldaki son sınıf öğrencilerinin günlük toplam test çözme 70
60
süreleriyle ilgili birtakım bilgileri içeren histogram verilerine göre 50
a) Toplam test çözen en çok kaç öğrenci olduğunu, 40
b) Hangi aralıkta en az sayıda kişinin test çözdüğünü, 30
20
c) Hangi aralıkta en çok sayıda kişinin test çözdüğünü bulunuz.
10
0 Süre (Dk)
30-? ?-99
ÇÖZÜM:

a) Açıklık 99 – 30 = 69 bulunur. Grafikteki verilere göre 7 adet grup olduğundan


69
grup genişliği 7 = 9, 85 çıkar. Genişlik 10 alınır.
Buna göre grupların aralık dağılımları: 30-39, 40-49, 50-59, 60-69, 70-79, 80-89, 90-99 olduğundan 60
dakika boyunca test çözen öğrenci sayısı dördüncü sütuna karşılık gelir ve en çok 50 öğrenci vardır.
b) En az kişi sayısına sahip grubun test çözme dakika aralığı 90-99 olur.
c) En çok kişi sayısına sahip grubun test çözme dakika aralığı 50-59 olur.

2. Grafik Türleri
Çizgi Grafiği
Sürekli verilerin yatay ve düşey eksendeki değerleri işaretlenerek bulunan noktaların düz çizgilerle
birleştirilmesi sonucunda elde dilen grafik türüdür.

Bir ilin yılın ilk 5 ayındaki ortalama sıcaklık değerleri Tablo 9.6.3 ve Grafik 9.6.6. da verilmiştir.
Sıcaklık
Aylar Ocak Şubat Mart Nisan Mayıs Haziran
Ortalama 25
8 6 12 15 17 20 Sıcaklık
Sıcaklık °C 20
20
Tablo: 9.6.3 17
15
15 12
10 8
6
5

Tablo ve grafik incelendiğinde şubat ayından itibaren 0 Aylar


Ocak Şubat Mart Nisan Mayıs Haziran
sıcaklıkların artmaya başladığı görülmektedir.
Grafik: 9.6.6

Matematik 9 | 387
VERİ

ÖRNEK 1:
Aşağıda bir fındık üreticisinin yıllara göre fındık üretim miktarlarını gösteren çizgi grafiği verilmiştir.
Üretim (kg)

3000
2500
2500
2000 2250
2000 1750
1500
1500 1250
1000 1000 Yıllık Üretim (kg)
500 750
500
250
0 Yıllar
1 2 3 4 5 6 7 8 9 10

Buna göre
a) En az ve en çok üretim yapılan yılları bulunuz.
b) Son 10 yıldaki ortalama üretimin kaç kg olduğunu bulunuz.
c) Veri grubunun açıklığını bulunuz.
ÇÖZÜM:
a) En az üretim 2. yılda, en çok üretim 10. yılda gerçekleşmiştir.
b) Son 10 yıldaki ortalama üretim:
500 + 250 + 1250 + 1750 + 1000 + 2000 + 1500 + 750 + 2250 + 2500 13750
10 = 10 = 1375 kg
c) Veri tabanının açıklığı: 2500 - 250 = 2250 kg dır.

ÖRNEK 2:
Aşağıdaki grafikte, aylık tüketici eğilim anketi sonuçlarına göre tüketicilerin genel ekonomik durum
beklentileri verilmiştir.
Genel Ekonomik Durum Beklentisi

100,6
96,2
93,2

88,6
88,0

Aylar
Ocak Şubat Mart Nisan Mayıs

Grafiğe göre dağılımın mayıs ayındaki genel ekonomik durum beklentisinin bir önceki aya göre değişim
oranını bularak yorumlayınız.

ÇÖZÜM:
4, 4 $ 100
100, 6 - 96, 2 = 4, 4 &96, 2 . 4, 5
Genel ekonomik durum beklentisi nisan ayında 96,2 iken mayıs ayında yaklaşık %4,5 oranında
artarak 100,6 değerine yükselmiştir. Bu artış gelecek dönemde genel ekonomik durumun daha iyi olacağı
yönünde beklentisi olan tüketicilerin sayısının bir önceki aya göre arttığını göstermektedir.

388 | Matematik 9
VERİ

Sütun Grafiği
Belirli kategorilerin frekanslarını veya yüzdelerini göstermede kullanılan bir grafik türüdür. Ölçme
sonucu elde edilen verilerin gelişimini ya da karşılaştırılmasını yapmak amacıyla sütunlar kullanılır.
Karşılaştırılacak değerler yatay eksen üzerinde eşit aralıklarla belirtilir. Her bir değere karşılık gelen
veri ise düşey eksende işaretlenerek sütunlar karşılık geldikleri işarete kadar uzatılır.

Grafik: 9.6.7 de sınıf başkanlığı seçiminde adayların aldıkları oyları gösteren sütun grafiği verilmiştir.

Oy Sayıları Grafik incelendiğinde şu sonuçlar çıkarılabilir:


14 13 • En az oyu Fatih almıştır.
12 11 • Emre 13 oyla sınıf başkanı seçilmiştir.
10 • Esin ve Nur’un aldıkları oyların toplamı, Erdem’in
8
6 oyuna eşittir.
6 5
4 • Sınıf başkanlığı seçiminde toplam 37 oy kullanılmıştır.
2 2
0 Adaylar
Esin Nur Emre Erdem Fatih
Grafik: 9.6.7

ÖRNEK 3:
A ilinde basketbol, voleybol, futbol, masa tenisi ve satranç
turnuvaları düzenlenmiştir. Her okul bu yarışmalardan sadece
birine katılmıştır. Katılan ortaöğretim okul sayıları aşağıdaki grafikte
verilmiştir.
Okul Sayısı
40 36
35
30
25 22 20
20
15 12
10 10
5
0 Turnuvalar
etbol Futbol Voleybol sa Tenisi Satranç
Bask Ma

Buna göre
a) Futbol ve satranç turnuvalarına katılan okul sayısının bütün okul sayısına oranını bulunuz.
b) Masa tenisi turnuvasına katılan okul sayısının tüm okul sayısının yüzde kaçı olduğunu bulunuz.
c) Turnuvalara katılan okul sayılarının aritmetik ortalamasını bulunuz.

ÇÖZÜM:
a) Futbol turnuvasına 36, satranç turnuvasına 20 okul katılmıştır. 5 turnuvaya katılan okul sayısı da 100
dür. Bu durumda oran
36 + 20 56 28 14
10 + 12 + 20 + 22 + 36 = 100 = 50 = 25

b) Grafiğe göre masa tenisine katılan okul sayısı 22 olduğundan istenen oran:
22
100 = %22 olur.

c) 5 turnuvaya katılan okul sayısı da 100 dür. Bu durumda aritmetik ortalama:


100
5 = 20 tir.

Matematik 9 | 389
VERİ

ÖRNEK 4:
Aşağıdaki grafikte bir fabrikada tüketilen ve israf edilen ekmek sayılarını gösteren sütun grafiği verilmiştir.
1
Aylık israf 3 oranında azaltılırsa yıl sonunda fabrikanın satın alacağı ekmek adedini bulunuz.
Bin
Ekmek Tüketim-İsraf Grafiği
400
360000
350
300
Ekmek Tüketimi
250
200 Ekmek İsrafı
150
90000
100
30000
50
7500
0
Aylık (Adet) Yıllık (Adet)

ÇÖZÜM:
Aylık ekmek israfı 7500 adet olduğundan

7500
3 = 2500 & 2500 $ 12 = 30 000
360 000 - 30 000 = 330 000 adet olur.

ÖRNEK 5:
A, B, C, D ve E ile isimlendirilen beş farklı çamaşır makinesinin kullanım tercihine yönelik kullanıcı anketi
yapılmıştır. Ankete 1000 kişi katılmıştır. Anket sonuçlarının yüzdelik dağılımı aşağıdaki grafikte gösterilmiştir.
Grafiğe göre
Yüzdelik Dağılım

45
40
40
35
30
25
25
20
20
15
10 10
5
5 Çamaşır Makinesi
0 Markaları
A B C D E

a) En çok tercih edilen marka olarak hangisinin seçildiğini bulunuz.


b) A ve E markalarını seçen kişi sayılarının ortalamasını bulunuz.
c) En az ve en çok tercih edilen markaları seçen kişi sayılarının oranını bulunuz.

ÇÖZÜM:
a) D markası yapılan ankette en yüksek orana sahip olduğundan en çok tercih edilen marka olur.
b) A markasını 100 kişide 20 kişi seçtiğinden bin kişide 200 kişi seçmiştir. Aynı düşünceyle E markasını da
100 kişi seçmiştir. O hâlde ortalama:
200 + 100
2 = 150 olur.
c) En çok tercih edilen D markasını 1000 kişide 400, en az tercih edilen B markasını ise 50 kişi seçmiştir.

O hâlde istenen oran: 50 = 1 tir.


400 8

390 | Matematik 9
VERİ

Daire (pasta, dilim) Grafiği


Veri grubunun bütün içerisindeki oranını belirtmek için kullanılan grafik türüdür. Bütünün parçaları
hakkında yorum yapılmasını sağlayan en güçlü yöntemdir. Daire grafiklerine yönelik sorularda sıklıkla
kullanılan merkez açı hesabı için aşağıdaki kural uygulanır.
İstenen veri sayısı Daire diliminin merkez açısının ölçüsü
Tüm veri sayısı 360o

Grafik 9.6.8 de, A köyünde yapılan muhtarlık seçiminde azaların


aldıkları oyları gösteren daire grafiği aşağıda verilmiştir.
Alınan Oy Sayısı
10 5
15 Hasan
60
Hüseyin
20
Emine
Rüstem
Yaşar

25 Sevgi
55 Şaban
Ömer
45
Grafik: 9.6.8
Grafiğe göre en az oyu Ömer, en çok oyu Hasan almıştır. Kullanılan toplam oy sayısı 235 tir.

ÖRNEK 6:
Grafikte bir çiftçinin yetiştirdiği ürünlerin dönüm değerleri Dönüm
verilmiştir. Grafiğe göre aşağıdaki soruları cevaplandırınız. 40 45
Patates
1
a) Ekilebilir arazinin 4 üne ekilen ürünü bulunuz. 20
30 Soğan
b) Buğday ekilen arazinin tüm araziye oranını bulunuz.
Buğday
c) Dönüm başına 750 kg soğan elde edildiğine göre çiftçinin
kaç ton soğan ürettiğini bulunuz. Arpa

ç) Bir dönümden 1,5 ton patates elde edildiğine göre çiftçi 90


Nohut
patatesin kilosunu 75 kr.tan satması hâlinde çiftçinin
kazancını bulunuz. 135 Boş

ÇÖZÜM:
1
a) Ekilebilir arazinin 4 ü, toplam arazinin 90 derecelik kısmı olup bu araziye ekilen ürün arpadır.
b) Buğday ekilen arazi 135 dönüm olduğundan tüm arazinin
135c 3
360c = 8 i olur.
c) Soğan ekilen arazi 30 dönümdür. Dönüm başına 750 kg soğan elde edildiğine göre çiftçi,
750 $ 30 = 22 500 kg = 22, 5 ton soğan elde etmiştir.
ç) Patates ekilen arazi 45 dönüm olduğu için
45 $ 1500 $ 0, 75 = 50 625 TL çiftçinin kazancıdır.

Matematik 9 | 391
VERİ

ÖRNEK 7:
Koyun
Yandaki grafikte bir çiftlikte bulunan hayvanların türlere göre dağılımı verilmiştir.
İnek
Bu çiftlikte 51 keçi bulunduğuna göre 120°
80°
a) Toplam hayvan sayısını bulunuz.
b) Tavuklara ait dilimin merkez açısını bulunuz. Tavuk Keçi
c) Koyunlardan 45 tanesi doğum yaptığına göre doğum yapmayan koyun sayısını
bulunuz.
ç) İnek sayısının tavuk sayısına oranını bulunuz.
ÇÖZÜM:

a) Keçi sayısı; daire diliminde 90 derecelik dilime denk geldiğine göre bu sayı, toplam hayvan
90c 1
sayısının
360c = 4 üdür. Toplam hayvan sayısı 4 $ 51 = 204 olur.
b) Tavuklara ait dilimin merkez açısı x ise
x = 360c - (120c + 90c + 80c) = 70c olarak bulunur.

c) Koyun sayısı daire diliminde 120 derecelik dilime denk geldiğine göre bu sayı toplam hayvan
sayısının 120c = 1 olur.
360c 3 204
Toplam hayvan sayısı 204 bulunduğundan koyun sayısı 3 = 68 olur.
Doğuran koyun sayısı 45 olduğu için doğurmayan koyun sayısı
68 - 45 = 23 olarak bulunur.

ç) Hayvan sayılarının oranı, yüzdelerinin oranı ile aynı olacağından


İnek sayısı 80c 8
= =
Tavuk sayısı 70c 7

ÖRNEK 8:
Grafik-1 bir kitapçıdaki kitap türlerinin dağılımını, Grafik-2 ise bu kitap türlerinden kaçar tane olduğunu
göstermektedir.
Adet

96
Fasikül
80° a
160°
Test Kitabı
Roman b

Kitap Türü
Test Kitapları Roman Fasikül
Grafik-1 Grafik-2

Buna göre a+b nin değerini bulunuz.


ÇÖZÜM:
Roman sayısı daire diliminde 160 derecelik dilime denk gelen 96 sayısını gösterdiğinden 80 derecelik
dilime gelen b tane fasikül, 96 sayısının yarısı olan 48 değerine eşittir. O hâlde b = 4 olur.
Daire grafiğinde a sayıda test kitabına denk gelen dilimin açısı = 360c - (160c + 80c) = 120c olur.
Bir derecelik dilime denk gelen kitap adeti = 96 olur.
160
120°lik dilime denk gelen kitap sayısı = 120 $ 96 = 72 tir. O hâlde a = 72 olur.
160
a + b = 72 + 48 = 120 bulunur.

392 | Matematik 9
VERİ

Serpme Grafiği
Düşey eksendeki verinin, yatay eksendeki veriye göre değişim yönünü ortaya koymak için
kullanılan grafik türüdür. Koordinat sisteminde sıralı ikili olarak gösterilir. Özellikle veri sayısının fazla
olduğu durumlarda kullanılır. Genel olarak üç farklı şekli vardır.

4.1 Pozitif Yönlü Değişim 35


Değişkenlerden biri artarken diğerinin de arttığı değişim şeklidir. 30
(Grafik: 9.6.9) 25
20
15
10
5
0
0 10 20 30 40 50 60
Grafik: 9.6.9

4.2 Negatif Yönlü Değişim 35


Değişkenlerden biri azalırken diğerinin arttığı değişim şeklidir. 30
(Grafik: 9.6.10) 25
20
15
10
5
0
0 10 20 30 40 50 60
Grafik: 9.6.10

4.3 Düzensiz Değişim 35


Değişkenler arasında kesin olarak bir artış ya da azalışın olmadığı 30
değişim şeklidir. (Grafik: 9.6.11) 25
20
15
10
5
0
0 10 20 30 40 50 60
Grafik: 9.6.11

Aşağıda bir ürünün yıllık üretim miktarı ile fiyat ilişkisine ait serpme grafiği verilmiştir. (Grafik: 9.6.12)
Yıllık Üretim (1000 t)

200

150

120
100
80
60
50
35

0 Fiyat Ekseni
0 0,25 0,5 0,75 1 1,25 1,5 1,75 2
Grafik: 9.6.12

Üretim miktarı ile fiyat ilişkisi incelendiğinde üretim arttığında fiyatın düştüğü görülmektedir. Bu durum
negatif yönlü değişime örnektir.

Matematik 9 | 393
VERİ

ÖRNEK 9:
Yanda 16 kişilik bir sınıfta yapılan YGS deneme sınavı matematik Net Sayıları
testi net sayıları ile öğrenci sayılarını veren serpme grafiği verilmiştir. 40
Buna göre 35
30
a) En fazla net yapan öğrenci sayısı ile en az net yapan öğrenci 25
sayısını bulunuz. 20
b) Net sayılarının medyanını bulunuz. 15
10
5
0 Öğrenci
ÇÖZÜM: 0 1 2 3 4 5 6 7 Sayısı

a) En fazla net yapan (35 net) beş öğrenci, en az net yapan (5 net) bir öğrenci vardır.
b) Net sayılar sıralandığında ortadaki terim olan 25, net sayılarının medyanı olur.

ÖRNEK 10:

Aşağıdaki serpme grafiğinde dört ürüne ait yıllık üretim tüketim dağılımları verilmiştir.
Tüketim (On bin ton)
a) Tüketim üretim farkının en fazla olduğu
ürünü bulunuz.
50
A (19,45)
B (33,40) A- yeşil mercimek b) Üretimi tüketiminden fazla olan üründe
40
B- kırmızı merci̇ mek üretimin yüzde kaçı tüketilmemiştir?
C (32,31) C- çavdar
30 D D- kuru fasulye c) Kırmızı mercimek üretiminin tüketim
(23,28)
ihtiyacını karşılayabilmesi için yüzde kaç
20
arttırılması gerekir?
10

Üretim
10 20 30 40 (On bin ton)

ÇÖZÜM:
Ü A: Yeşil mercimek ürününe ait üretim miktarı
TA: Yeşil mercimek ürününe ait tüketim miktarı
a) Yeşil mercimek (A) için: TA - Ü A = 45 - 19 = 26
Kırmızı mercimek (B) için: TB - Ü B = 40 - 33 = 7
Çavdar (C) için: TC - Ü C = 31 - 32 =- 1
Kuru fasulye (D) için: TD - Ü D = 28 - 23 = 5
Çavdar ürününde üretimin tüketimden fazla olduğu görülmektedir. Ürün bir sonraki yılların tüketimi
için saklanabilir ya da ihraç edilebilir. Diğer ürünlerde tüketim miktarının üretim miktarından fazla olduğu
görülmektedir. İhtiyaç, ithalat yoluyla karşılanabilir.
b) Üretimi tüketiminden fazla olan ürün çavdardır.
Ü C - TC = 32 - 31 = 1 tüketilmeyen çavdar miktarıdır.
Tüketilmeyen çavdar miktarı 100 de x olsun.
100 $ 1
x = 32 = %3 Çavdar üretiminin %3 ü tüketilmemiştir.
c) Kırmızı mercimek ürününün üretim miktarı tüketim miktarından azdır. İhtiyacın karşılanabilmesi için
üretim miktarının artması gerekmektedir.
TB - Ü B = 40 - 33 = 7 Üretim miktarı 100 de x arttırılmalı ise
100 $ 7
x = 33 = %21
Kırmızı mercimek üretiminin, tüketim ihtiyacını karşılayabilmesi için %21 arrtırılması gerekir.

394 | Matematik 9
VERİ

Kutu Grafiği
Bir veri grubunun en büyük, en küçük, alt çeyrek, üst çeyrek ve ortanca değerlerinin bir
dikdörtgene bitişik çizgilerle belirtildiği grafik türüdür. Bu grafik türünde belirtilen değerler etrafında
verilerin yayılımı ve veri genişliği daha net gösterilmektedir.
• Veriler küçükten büyüğe doğru sıralanır. En
Medyan
küçük değer, en büyük değer, ortanca, alt
çeyrek ve üst çeyrek hesaplanır.
• Sayı doğrusu çizilir. Sayı doğrusu üzerinde
Enküçük ve en büyük değer işaretlenerek kutu
grafiğinin uç noktaları gösterilir.
En Küçük Değer En Büyük Değer
• Alt çeyrek ve üst çeyrek değerleri kutunun
Alt Çeyrek Üst Çeyrek kenarları olarak alınır. Veri grubunun ortanca
değeri kutunun içine işaretlenir.
Grafik: 9.6.13
• Oluşan grafik verilerin dağılımını gösteren kutu
grafiğidir.

ÖRNEK 11:
Dil ve Anlatım dersi yazılı sınavı sonucunda 10 öğrencinin puanları incelendiğinde medyan değerinin 55,
alt çeyrek değerinin 30, üst çeyrek değerinin 85, en büyük değerinin 100, en küçük değerinin 10 olduğu
görülmüştür.
Bu verilere ait tablo ve kutu grafiğini oluşturunuz.
ÇÖZÜM: Dersler

En En
Alt Üst
Küçük Medyan Büyük
Çeyrek Çeyrek
Değer Değer
A Şubesi 10 30 55 85 100
Puanlar
10 30 55 85 100

ÖRNEK 12:
Bir huzurevinde yaşayan kadın ve erkeklerin yaşları ile ilgili bilgiler aşağıdaki kutu grafiğinde verilmiştir.
Grafiğe göre
Cinsiyet

Kadın

Erkek

Yaş
50 55 60 63 66 72 75 82 91

a) Kadın ve erkeklerin yaşlarının medyanlarının arasındaki farkı,


b) Kadınların yaşlarının çeyrekler açıklığı ile erkeklerin yaşlarının çeyrekler açıklığını karşılaştırıp bunu
yorumlayınız.
ÇÖZÜM:
a) Kadınların medyanı Q 2 K , erkeklerin medyanı Q 2 E olmak üzere Q 2 K = 7 ve Q 2 E = 66 & 72 - 66 = 6 olur.
b) Kadınların alt çeyreği Q 1 K , üst çeyreği Q 3 K ve çeyrekler açıklığı Q K olmak üzere
Q 1 K = 63, Q 3 K = 82 & Q K = 82 - 63 = 19 olur.
Erkeklerin alt çeyreği Q 1 E , üst çeyreği Q 3 E ve çeyrekler açıklığı Q E olmak üzere
Q 1 E = 60, Q 3 E = 75 & Q E = 75 - 60 = 15 olur.
Kadınların çeyrekler açıklığı erkeklerin çeyrekler açıklığından büyüktür.

Matematik 9 | 395
VERİ

ÖRNEK 13:
Bir mağazanın hafta boyunca sattığı ürünlerin sayısı 50, 30, 40, 50, 20, 120, 80 olarak verilmiştir. Bu
verilerin
a) Medyanını,
b) Çeyrekler açıklığını bulunuz.
c) Verilere ait kutu grafiğini çizip yorumlayınız.

ÇÖZÜM:
a) Medyan = 50
b) Çeyrekler açıklığı: 80 - 30 = 50

Medyan
20 30 40 50 50 80 120

Alt çeyrek Üst çeyrek

c) Hafta
Kutu grafiği incelendiğinde
verilerin en küçük değere daha
yakın olduğu görülür. Veri
Satılan Ürün Sayısı
grubunun ortancası üst çeyreğe
20 30 50 80 120 göre alt çeyreğe daha yakındır.

Sıra Sizde

SORU: Komisyon (%)


5
Yandaki grafikte, bir emlakçının inşaat firmasıyla yaptığı 5

komisyon anlaşması verilmiştir. Bu dairelerin her biri 4


4
100 bin lira olduğuna göre emlakçının ayda 65 bin lira 3
3
kazanabilmesi için kaç daire satması gerektiğini bulunuz. 2
2
1
ÇÖZÜM: 1
Satılan
0 Daire Sayısı
1-5 6-10 11-15 16-20 21-...

Son iki yılda Türkiye'ye en çok turistin geldiği 5 ülkeyi araştırınız. Bu ülkelerden gelen
turist sayılarını tablo hâlinde gösteriniz. Tabloya göre verilerin daire grafiğini oluşturup
yorumlayınız.

396 | Matematik 9
VERİ

ALIŞTIRMALAR

1 Melis 100 sayfalık bir hikâye kitabının birinci 4 Aşağıda Spor Toto Süper Ligi 2015-2016
gün 15 sayfasını, ikinci gün 25 sayfasını, Sezonu’nda ilk 8 takımın galibiyet ve
üçüncü gün 20 sayfasını ve dördüncü gün 30 mağlubiyet durumunu gösteren serpme grafiği
sayfasını okuyor. Buna göre her bir günün verilmiştir.
sonunda Melis’in okumamış olduğu sayfaların
Galibiyet
sayısını gösteren çizgi grafiğini çiziniz.
30

25

20

15

10

2
Aşağıdaki histogram grafiği bir gruptaki
bireylerin aylık net gelir durumlarını 5
göstermektedir. Buna göre
Mağlubiyet
Kişi Sayısı 1 2 3 4 5 6 7 8 9 10 11 12

100 Grafiğe göre galibiyet sayılarının oluşturduğu


66 veri grubunun medyanı ile mağlubiyet
50 43 sayılarının oluşturduğu veri grubunun
17
10 aritmetik ortalamasının toplamını bulunuz.
0 Aylık Gelir (TL)
000 500 000 500
5 0 0-2 001-2 501-3 001-3
1 2 2 3

a) Gruptaki toplam kişi sayısını bulunuz.


b) Geliri 2001-2500 TL aralığında bulunan
kişilerin tüm grubun yüzde kaçı olduğunu
bulunuz.

8 kişilik bir öğrenci grubunun matematik dersi


5
puanları aşağıda verilmiştir.
50, 65, 70, 40, 80, 62, 90, 74
Verilere karşı gelen kutu grafiğini çizip
yorumlayınız.
3
Bir öğrencinin ay boyunca girdiği kırkar
soruluk deneme sınavlarındaki matematik ve
Fen Bilimleri ders netleri aşağıdaki tabloda
verilmiştir.
Verilere ait serpme grafiğini çizerek veriler
arasındaki ilişkinin türünü belirtiniz.
Deneme Sınavları
1. 2. 3. 4. 5. 6.
Matematik 25 20 30 27 30 35
Fen
20 17 25 23 26 30
Bilimleri

Matematik 9 | 397
VERİ

6 Aşağıdaki grafikte bir fayans ustasının altı 8


Atık pil toplama miktarı ile ilgili grafik aşağıda
gün boyunca döşediği fayans alanının sayısal verilmiştir.
değerleri m2 olarak verilmiştir. Miktar (ton)

Döşenen Fayans Alanı 600


526
40 500 497
35 451
33 414
30 30 30 400
25 27 325
20 300 252
234
200 199
10
100
31
0 Günler
0 Yıl
Pazartesi

Salı

Çarşamba

Perşembe

Cuma

Cumartesi

5 6 7 8 9 0 1 2 3
200 200 200 200 200 201 201 201 201

Grafiğe göre atık pil toplamadaki artış hızının


en az ve en çok olduğu yıl aralıklarını bulunuz.
Veriler dairesel grafik olarak gösterildiğinde
salı günü döşenen alana karşılık gelen merkez
açının ölçüsünün kaç derece olduğunu
bulunuz.

7
Aşağıdaki grafik, bir marketin hafta boyunca Aşağıdaki grafikte bir ilin kasım ayının ilk
9
sattığı ekmek sayısını göstermektedir. Veriler haftasına ait sıcaklık değerleri verilmiştir.
daire grafiği olarak gösterilseydi cuma günü Sıcaklık
satılan ekmek sayısına ait daire diliminin 30 24
merkez açısı 36c olacaktı. Buna göre cuma 20 18 19
20 15 15 15
günü satılan ekmek sayısını bulunuz.
10
Satış (adet)
0 Günler
Pazar
Pazartesi

Salı

Çarşamba

Perşembe

Cuma

Cumartesi

200

100 80 100
55 50 ? 50 43
0 Gün
Pazartesi

Salı

Çarşamba

Perşembe

Cuma

Cumartesi

Pazar

Buna göre
a) Bu ilin haftalık sıcaklık değerlerinin
ortalamasını bulunuz.
b) Bu ilin haftalık sıcaklık değerlerinin standart
sapmasını bulunuz.

398 | Matematik 9
VERİ

Tuna, ormanda bisiklet kullanmaktadır. Bu esnada önüne bir tavşan fırlar. Tavşana çarpmamak için
10
ani bir fren yapar. Tavşan, ezilmekten son anda kurtulur. Tuna, bir hayvansever olduğundan tavşanı
korkuttuğunu düşünerek üzüntüyle eve döner. Aşağıdaki grafik Tuna’nın hız değerlerini göstermektedir.

Grafiğe göre
1. Tuna, bisiklet kullanırken kaç dakika sabit hızla
yol almıştır?
Hız (km/sa) 2. Tuna, tavşana çarpmamak için fren yapmaya
başladığı ana kadar kaç dakika geçmiştir?
30
3. Tuna’nın hızını en çok arttırdığı zaman dilimi
25 hangisidir?
20
A) 16.05-16.10
B) 16.05-16.15
15 C) 16.10-16.20
D) 16.20-16.40
10 E) 16.40-16.50
5 4. Tuna’nın tavşan ile karşılaştığı yere kadar
0 Zaman
aldığı yol ile dönüşte aldığı yolu karşılaştırıp
16.00 16.30 17.10 yorumunuzu yazınız.

11 Aşağıdaki grafikte bir A şehrine gelen turist sayıları üç farklı ülkeye göre ay bazında verilmiştir.

Turist Sayısı
1500

1600
1400
1150

1000

1200
1000

950
950

1000 X Ülkesi
850

800
800
750

Y Ülkesi
700

675

800
650

Z Ülkesi
550
500

600
450
400

350

400
200
0 Aylar
lül im ım n u z s
Ey Ek as zir
a to
mm us
K Ha Te Ağ

Grafiğe göre
1. Kasım ayında X ülkesinden A şehrine gelen turist sayısı, Z ülkesinden gelen turist sayısının yaklaşık
kaç katıdır?
2. Z ülkesinden gelen turist sayısı hangi ayda Y ülkesinden gelen turist sayısından azdır?
3. X ülkesinden gelen turist sayısı kasım ayından itibaren düşüş gösterdiğine göre eylül ayında bu
ülkeden yaklaşık kaç turist gelebileceğini bulunuz.
4. Bir turist, aylık ortalama 1250 TL harcama yaptığına göre A şehrinin ekonomisine temmuz ayında
giren para miktarını bulunuz.

Matematik 9 | 399
VERİ

ÖLÇME VE DEĞERLENDİRME
I. Aritmetik ortalama Aşağıdaki sütun grafiği, bir giysinin verilen
1 5
II. Medyan aylara göre satış fiyatındaki değişim miktarını
III. Standart sapma göstermektedir. Alış fiyatı 100 TL olan bir giysi,
haziran ayında %10 kârla satılmıştır. Bu malın
IV. Çeyrekler açıklığı
aralık ayındaki satış fiyatı kaçtır?
Yukarıda verilenlerden hangileri birer merkezi
Değişim (TL)
eğilim ölçüsüdür?
25
A) I ve IV B) II ve III C) I ve II 20
20

15
D) I ve III E) III ve IV 12
10 8
5 5
sım lık
Ka Ara
0 Aylar
z os l
-5 mu ğust Eylü Ek
im
Hasan’ın matematik dersinden girdiği 5 sınav Tem A
2 -10
puanının ortalaması 65 tir. Hasan’ın matematik -10
-15
ortalamasını 70 yapması için 6. sınavdan kaç -15
-20
puan alması gerekir?
A) 160 TL B) 150 TL C) 145 TL
A) 97 B) 95 C) 93
D) 130 TL E) 125 TL
D) 91 E) 89

4, 5, y, 8, 13, 15 veri grubunun açıklığı 14 6


Beşiktaş’ın şampiyon olarak tamamladığı
3
olduğuna göre y nin alabileceği değerler 2015-2016 Spor Toto Süper Ligi Sezonu’nda
toplamı aşağıdakilerden hangisi olur? oluşan puan durumunu gösteren histogram
aşağıda verilmiştir.
A) 15 B) 16 C) 17
Takım Sayısı
D) 18 E) 19
5
4
3
2
1
4 Okulun öğrenci kulübünün yönetim kurulu
0 Puan
seçiminde en çok oyu alan yedi öğrencinin -29 -39 -49 -59 -69 -79
20 30 40 50 60 70
oy dağılımı 12, 17, 8, 15, 19, 14, 6 şeklinde
gerçekleşmiştir.
Grafiğe göre 60 puanın altında puan toplayan
Oluşan veri grubunun aritmetik ortalama ile takım sayısı, 59 puanın üstünde puan toplayan
açıklığının farkı kaçtır? takım sayısının kaç katıdır?

A) 0 B) 1 C) 2 A) 5 B) 4 C) 3
D) 3 E) 4 D) 2 E) 1

400 | Matematik 9
VERİ

9
Bir öğrencinin yapılan 5 deneme sınavı
sonucunda yaptığı yanlış sayıları sırasıyla
7 - 8. soruları aşağıdaki bilgilere göre
cevaplayınız. 7, 5, 3, 4, 1 şeklindedir. Bu durumda oluşan
veri grubunun standart sapması kaç olur?
Bir firma ürettiği A ve B isimli 2 farklı el
A) 5 B) 11 C) 3
sabununu test etmek için bir grup seçiyor.
Bu gruptan her iki ürünü de kullandıktan D) 7 E) 5
sonra ürünlere 1-2-3-4-5 puanlarından birini
vermeleri isteniyor. Anket sonuçlarına göre A
ve B sabunlarının puanlarını gösteren grafikler
aşağıda verilmiştir.

Kişi Sayısı 10
Aşağıdaki tabloda A ve B illerinde 7 ay boyunca
düzenlenen sanatsal etkinlik sayıları verilmiştir.
40
36

Temmuz
Haziran
30

Mayıs
Şubat

Nisan
25

Mart
Ocak
20
12 A İli 8 10 11 8 6 12 15
10 10
7
B İli 5 7 6 7 10 9 12
0 Puan
1 puan 2 puan 3 puan 4 puan 5 puan
Tabloya göre aşağıdaki ifadelerden hangisi
A Ürünü Anket Sonuç Grafiği
yanlıştır?

A) A ilindeki verilerin aritmetik ortalaması, B


α/3
ilindeki verilerin aritmetik ortalamasından
40°

büyüktür.
B) B ilindeki verilerin medyanı, A ilindeki

verilerin medyanından küçüktür.
C) A ve B illerine ait verilerin modları toplamı
15 tir.
D) A ve B illerine ait verilerin çeyrekler açıklığı
1 Puan 3 Puan 4 Puan 5 Puan
farklıdır.
B Ürünü Anket Sonuç Grafiği E) A ve B illerine ait verilerin açıklıkları farkı 2
dir.
7 B ürününe 3 puan veren kişi sayısı kaçtır?

A) 35 B) 40 C) 45

D) 50 E) 55

11
2, 5, 8, 5, 2, 7, 5 grubunun modu, medyanı, alt
çeyreği, üst çeyreği ve çeyrekler açıklığından
A ürününe verilen puanlar toplamından B oluşan yeni veri grubunun aritmetik ortalaması
8
ürününe verilen puanlar toplamının farkı kaçtır?
aşağıdakilerden hangisidir?
A) 4,8 B) 3,6 C) 4
A) 9 B) 10 C) 11
D) 3,5 E) 5
D) 12 E) 13

Matematik 9 | 401
VERİ

Bir belediye otobüsünün kasım ayı için bir Aşağıdaki grafikte A ve B bitkilerinin boylarının
12 14
duraktan geçişteki gecikme süresi ile gün sayısı yıllara göre uzama miktarları verilmiştir.
aşağıdaki tabloda verilmiştir. Boy (cm) B A
30
Gün Sayısı 10 5 8 4 3
25
Gecikme Süresi
5 0 3 X 7
(dk) 20

Buna göre kasım ayı gecikme süresi ortalama 15


4,5 dakika ise X kaçtır?
10
A) 15 B) 10 C) 8 5

D) 5 E) 3 4
Zaman (yıl)

Dikildikten kaç yıl sonra bitkilerin boyları farkı


20 cm olur?

A) 18 B) 20 C) 23

D) 25 E) 30

13
Aşağıdaki sütun grafiğinde Duru ile Batu’nun Beş öğrencinin aylara göre okudukları kitap
15 sayıları aşağıdaki tabloda verilmiştir.
3 aylık 120 şer sorudan oluşan matematik
testinden yapmış oldukları doğru sayıları
verilmiştir.
Doğru Sayısı Aralık Ocak Şubat Mart Nisan

120
Fatih 7 3 5 2 3
108 104
100 100 100 Selda 1 3 3 2 5
92
80
80 Ünsal 4 1 3 4 2
Duru
60
Batu Gözde 1 4 3 1 3
40
20 Mehmet 4 5 7 2 5
0 Aylar
Ekim Kasım Aralık
Tabloya göre hangi öğrencinin mayıs ayında
Matematik Testi okuyacağı kitap sayısı daha kolay tahmin
Duru ve Batu ocak ayında yeni bir deneme edilebilir?
sınavına girecektir. Bu sınavla birlikte Duru
ve Batu’nun doğru sayılarının ortalaması
A) Fatih B) Gözde C) Mehmet
hesaplanacaktır. Bu ortalamanın Duru’nun
ilk üç deneme sınavındaki ortalamasına
D) Selda E) Ünsal
eşit olması için yaptıkları doğru sayıları
aşağıdakilerden hangisidir?

A) Duru 100, Batu 104


B) Duru 104, Batu 108
C) Duru 100, Batu 116
D) Duru 110, Batu 108
E) Duru 108, Batu 106

402 | Matematik 9
VERİ

16
Kazım Bey’in ekim ayında yaptığı harcamaların 18 İki çiftçinin yetiştirdikleri ürünlerin 2010-
dağılımını gösteren daire grafiği aşağıda 2016 yılları arasında ekildiği arazilerin dönüm
verilmiştir. değerleri aşağıdaki çizgi grafiğinde verilmiştir.

Dönüm

140
120 120 120
120
100 110
45°
100° 100
80 80
80 A çiftçisi
80° 90°
60 B çiftçisi
40 45 50 50
40
20 30
20
Market Akaryakıt Beyaz Eşya 0 Yıl
Giyim Diğer
2010 201
1
2012 2013 2014 5
201 201
6

Ekim ayı kredi kartı borcu 2700 lira olduğuna Buna göre aşağıdakilerden hangisi yanlıştır?
göre aşağıdaki ifadelerden hangileri doğrudur?
I. Giyim harcaması 300 liradır. A) İki çiftçinin ektiği dönüm değerlerinin
II. Akaryakıt harcaması, giyim harcamasının 2 ortancaları farkı 65 tir.
katıdır. B) B çiftçisi A ye göre daha istikrarlı ekim
III. Market harcaması 750 liradır. yapmıştır.
C) İki çiftçinin ektiği dönüm değerlerinin
IV. Beyaz eşya harcaması tüm harcamaların
1 modları farkı 40 tır.
9 udur. D) A çifçisinin ektiği dönüm değerlerinin
açıklığı, B çiftçisinin açıklığından fazladır.
A) I-II B) I-III C) II-III-IV E) A çiftçisinin ektiği dönüm değerlerinin
ortalaması 45 dönümdür.
D) III-IV E) II-III

17
Aşağıdaki grafikte bir voleybol turnuvasına
katılan oyuncuların yaşları verilmiştir.
Oyuncu Sayısı

19
Aşağıdaki kutu grafiğinde bir poliklinikte
40 muayenesi yapılan hastaların yaş dağılımları
35 35
30 gösterilmiştir. Buna göre veri grubunun
30
25
26 medyanı ile çeyrekler açıklığının toplamı
20 kaçtır?
15 13 A Polikliniği
10 10
5 5
Yaş
15 16 17 18 19 20

Oyuncuların yaşlarıyla hazırlanan bir veri Yaşlar


grubunun sırasıyla modu, medyanı ve 1 15 27 45 75
çeyrekler açıklığı hangi seçenekte doğru olarak A) 45 B) 52 C) 57
verilmiştir?
D) 60 E) 65
A) 17-18-1
B) 16-17, 5-2
C) 18-17-3
D) 18-17, 5-1
E) 17-17-2

Matematik 9 | 403
VERİ

Otoparka giren ve otoparktan çıkan araç


22
verilerinin medyanlarının aritmetik ortalaması
20 - 24. soruları aşağıdaki grafiğe göre aşağıdakilerden hangisidir?
cevaplayınız.
A) 24 B) 21 C) 18
Aşağıdaki grafik, boş bir otoparka belirtilen
D) 15 E)12
saatlerde giriş çıkış yapan araç sayısını
göstermektedir.
Araç Sayısı

Çıkan araç
45
Giren araç
40
35
30 11
25 10 30
24 20
20 8
20 5
15
15 14 15
3
10 12
23
12.00 itibariyle otoparkta bulunan araç
10 9
5 7 sayısının gün boyunca otoparka giren araç
0 Saat
sayısına oranı kaçtır?
09.00 - 10.00

12.01 - 13.00

13.01 - 14.00

14.01 - 15.00

15.01 - 16.00

16.01 - 17.00
10.01 - 11.00

11.01 - 12.00

3 5 1
A) 14 B) 14 C) 2
9 11
D) 14 E) 14

20
13.00’te otoparkta bulunan araç sayısı kaçtır?

A) 14 B) 15 C) 16

D) 17 E) 18

21
09.00-17.00 aralığında, saatte ortalama kaç 24
Hangi saatten itibaren otoparkta en fazla araç
araç otoparka giriş yapmıştır? bulunur?

A) 15 B) 14 C) 12 A) 12.00 B) 13.00 C) 14.00

D) 10 E) 9 D) 15.00 E) 16.00

404 | Matematik 9
SEMBOLLER VE OKUNUŞLARI
Sembol Okunuşu Sembol Okunuşu
p p önermesi 6a, b@ a b kapalı aralığı
/ Denktir ^a, b? a dan açık b den kapalı aralık
7 Bazı, en az bir 6a, bg a dan açık b den kapalı aralık
6 her ^a, bh a b açık aralığı
/ ve ^- 3, 3h Sayı doğrusu
0 veya ;x; x in mutlak değeri
n
& ise x x in n. kuvveti
+ ancak ve ancak n
x m n. dereceden karekök x in m. kuvveti

x in b n l . kuvveti
Q ya da
m m
xn
! Elemanıdır % yüzde
s Elemanı değil a a nın b ye oranı (oran)
b
Q Boş küme a c orantı
b =d
3 Alt küme EKOK En küçük ortak kat
4 Kapsar EBOB En büyük ortak bölen
Y
1 Alt küme değil & ABC üçgeni
ABC
s(A) A kümesinin eleman sayısı % ABC açısı
ABC
, Birleşim 5AB? AB doğru parçası
+ Kesişim ; AB ; AB doğru parçasının uzunluğu
A-B A fark B , Eşlik
&
Al A kümesinin tümleyeni ABC , &DEF ABC üçgeni eştir DEF üçgeni
A#B A kartezyen çarpım B nA A açısının açıortayı
N Doğal sayılar kümesi Va a kenarının kenarortayı
Z Tam sayılar kümesi G Ağırlık merkezi
Q Rasyonel sayılar kümesi ha a kenarının yüksekliği
Ql İrrasyonel sayılar kümesi + Benzerlik
&
R Gerçek sayılar kümesi ABC + &
DEF ABC üçgeni benzerdir DEF üçgeni
Z+ Pozitif tam sayılar kümesi sinx x in sinüsü
Q+ Pozitif rasyonel sayılar kümesi cosx x in kosinüsü
R+ Pozitif gerçek sayılar kümesi tanx x in tanjantı
Z- Negatif tam sayılar kümesi cotx x in kotanjantı

A]ABCg
Q- Negatif rasyonel sayılar kümesi & ABC üçgeninin alanı
-
R Negatif gerçek sayılar kümesi X Aritmetik ortalama
R#R Gerçek sayıların kartezyen çarpım kümesi S Standart sapma
R2 Kartezyen koordinat sistemi Q Çeyrekler açıklığı
1 Küçüktür Q1 Alt çeyrek
# Küçüktür veya eşittir Q2 Ortanca
2 Büyüktür Q3 Üst çeyrek
$ Büyüktür veya eşittir . Yaklaşık

Matematik 9 | 405
ALIŞTIRMA ÇÖZÜMLERİ
MANTIK
ALIŞTIRMA (Sayfa 30) ALIŞTIRMA (Sayfa 42)
1. a) 1, b) 0, c) Önerme değil, ç) 0, d) Önerme değil, 1. a) p: “ 7x ! Z, 3x - 7 = 11 ” olduğundan p / 1 tir.
e) 1 b) q: “ 6x ! Z, x 2 = 36 ” olduğundan q / 0 tir
= n ^m, n ! Z; ^m, n h = 1h ”
m
2. p _ q c) r: “ 6x ! Q, x Y
q_r
s_r olduğundan r / 0 tir.
q/s
p/r p_s 2. Sözel ifade
3. a) pl : “ 5 - 3 Y
= 2” pl] x g: “4 ten küçük olan bazı gerçek sayıların
karesi 16 dan büyüktür.”
b) ql : “316 sayısı 5 ile bölünür.”
ql] x g: “Her gerçek sayının 2 eksiğinin karesi
c) rl : “ 2 4 Y
= 42 ”
sıfır veya pozitiftir.”
ç) sl : “ 16 - 3 4 Y
= 2” rl] x g: “2 nin her gerçek sayı kuvveti pozitiftir.”
4. p / 1, q / 0, r / 0 sl] x g: “Bazı tam sayıların karesi 8 den küçük
5. 1 ise bu sayının 1 eksiği pozitiftir.”
6. 1 3. Değili
7. p pl] x g: “ 6x ! R, x - 1 < 0 ”
9. ^p 0 q h 0 ]r / sg Lamba yanar. ql] x g: “ ^7x ! R, 4x + 3 =- 5 h / ]x Y
=- 2g ”
10. 21 rl] x g: “ ^7x ! N, x < 0 h / ^6x ! N, 2 - x $ 0 h ”
11. a) Karşıtı: “Bir hayvan 2 ayaklı ise tavuktur.” sl] x g: “ ^6x ! R, x 3 $ 0h 0 ^7x ! R, x 2 < 0 h ”
4. a) " x ; x < 4, x ! N ,
Tersi: “Bir hayvan tavuk değilse 2 ayaklı
değildir.”
b) !- 1 +
c) "^1, 0h, ^0, 1h, ^- 1, 0h, ^0, - 1h,
Karşıt tersi: “Bir hayvan 2 ayaklı değilse tavuk
değildir.”
b) ]x Y
=- 1g & ^x 3 Y
=- 1 h 5. a) Hipotez:“ 5x - 7 = 18 ”
Hüküm: “ x - 1 = 4 ”
13. a) “ 6x, 3x 2 - 5 < 2 ”
b) ^6x, x 3 $ 0h 0 ^7x, x 2 < 0h ”
“ b) Hipotez: ABCD dörtgeni bir karedir.
Hüküm: ABCD dörtgeninin her bir açısı 90c dir.
15. a) 1, b) 1, c) 0, ç) 1, 13. 1-1-0-0-1-1-0

KÜMELER
ALIŞTIRMA (Sayfa 61)
1. D, 2. 19, 3. B, 4. I, IV. ve VI., 5. 256, 6. 8, 7. a) 16 b) 8 c) 8 ç) 4 d) 24 e) 8, 8. 9, 9. 32, 10. C ve E, 11. 240
ALIŞTIRMA (Sayfa 77)
1. 7, 2. 95, 3. 15, 4. 17, 5. 20, 6. a) En çok 16, en az 12 b) En çok 17, en az 13, 7. 5, 8. A, 9. 6, 10. 1
ALIŞTIRMA (Sayfa 87)
1. 4, 2. - 2 , 3. 10,
3

4. a) 16 b) c) 215 ç) 3.214

5. 28, 6. 5, 7. a) 256 b) 64 c) 64 ç) 9

406 | Matematik 9
8. a) b)

DENKLEMLER VE EŞİTSİZLİKLER
ALIŞTIRMA (Sayfa 105)
2 - 61
1. Rasyonel: - 3, 1, 976, 36 , 7 İrrasyonel: 2 , 3r, 2 5 - 1 2. a) Tek b) Çift c) Tek 3. 18

4.
4
5. 45
6. En büyük:13, En küçük:1
7. 14, 25, 36, 47, 58, 69

9. a) b)

c) ç)

ALIŞTIRMA (Sayfa 121)


1. 2 , 2. 7, 3. - 1 ve 3 , 4. 5, 5. 30, 6. Ç = ^5, 9 h , 7. Ç = ^3, 3 h , 8. En büyük:12, En küçük: - 26
-1
3

9. 11. Ç = ^1, 3h

12. 3

10. 13.

Matematik 9 | 407
ALIŞTIRMA (Sayfa 136)
1. 2 3 - 3 , 2. - 3 , 3. a) z - y b) 1, 4. 2x , 5. Ç = & 2 , 2 0 , 6. 2 , 7. 7 , 8. Ç = ! - 2 + , 9. Ç = ! 3 +
3 -1 7 -9

10. 2 , 11. 4, 12. 13, 13. 26, 14. 6, 15. 5, 16. Ç = ^- 3, 1? , 66, 3g , 17. 0, 18. Ç = ^- 3, 2 D , 19. - 12
-5 -3

20. Ç = Q , 21. 25, 22. ^1, 2 h

23. 24.

ALIŞTIRMA (Sayfa 145)


1. x, 2. 3 , 3. 56, 4. 400, 5. 2 , 6. a) 2 b) 2, 7. 2 , 8. 54, 9. 9 , 10. - 3 , 11. Ç = ! 1 +
-2 a4 5 8m 3 n
b
ALIŞTIRMA (Sayfa 154)
-5 20 6
1. 6, 2. - 8 , 3. 2b - 3a , 4. 2 , 5. 7 , 6. a) 675 b) 2 c) 2017 ç) 2 + 1 , 7. 7, 8. 6

ALIŞTIRMA (Sayfa 164)


5 1 7
1. 2004, 2. 4 , 3. 7. 3 12 , 4. 3 , 5. 1, 6. 9 , 7. 16, 8. 2, 9. 9, 10. a) 2,75 km b) 12 durak

ALIŞTIRMA (Sayfa 172)


5b - 3a
1. - 8 , 2. 32, 3. 21, 4. 38, 5. 2 , 6. 48, 7. 44, 8. 19, 9. 1552, 10. 20, 11. Değişmez

ALIŞTIRMA (Sayfa 191)


1. 390, 2. 17, 3. 60, 4. 20 000, 5. 4000, 6. 80, 7. 25, 8. 2, 9. 280, 10. 36 km/sa, 11. 6, 12. 72, 13. 180
14. 15, 15. a) 400 b) 288 km/sa, 16. 5, 17. 60 km/sa ve 30 km/sa, 18. 1. 9 ay, 2. 15000, 3. 15 ay
19. 1. 82, 2. 8.30, 3. 50000 adım ile 80000 adım arası

BÖLÜNEBİLME
ALIŞTIRMA (Sayfa 207)
2a + 1
1. b = 4 , 2. 8, 3. 59, 4. 6, 5. 2, 6. 8, 7. 10, 8. 23

ALIŞTIRMA (Sayfa 215)


1. 1, 4, 7, 2. 9, 3. 27, 4. 8 8 , 5. 2, 6. 92, 7. 12, 8. 120, 208, 244, 280
ALIŞTIRMA (Sayfa 229)
2 3 4 5 6 9 10 15
A * * * * * *
B * * * * * *
3A * * * * * * *
A2 * * * * * * * *
A+B * * * * * *
A.B * * * * * * * *
2. a) 270 lt b) 10 lt c) 27, 3. 160, 4. 200, 5. 441, 6. 7, 7. 20, 8. a) 6 b) 33

408 | Matematik 9
ÜÇGENLER
ALIŞTIRMA (Sayfa 263)
1. 46, 2. 56, 4. 111, 5. 48, 6. 9, 10, 7. 63, 8) 9

ALIŞTIRMA (Sayfa 301)


3 14
1. 10, 2. 7 , 3. 20, 4. 10, 6. 36, 7. 2 6 , 8. 2 73 9. 3 17 , 10. 3 6 - 3 2 , 11. 2 3 ,
2
12. 2 17 , 13. 4, 14. 3 , 15. 6, 16. 18 3

ALIŞTIRMA (Sayfa 324)


1. 4 5 , 2. 56, 3. 135, 4. 27, 5. 18, 7. 210m, 8. a) 1500m

ALIŞTIRMA (Sayfa 334)


17 80
1. 10, 2. 3, 3. 2 22 , 4. 2 , 5. 3 , 6. 3 2 , 7. 30 , 8. 25

ALIŞTIRMA (Sayfa 342)


11 8 3
1. sin x = 3 , sin 2y= 2 10 , cos x = 2 10 , cos 2y = 3 , 2. 6 , 3. 6, 4. 4 2 , 5.1, 6. 21 , 7. 5 ,
7 7 7 7
9 2 10 2 10 10
8. a)5,4, b) 4 tan x = 3 10 , tan y= cot x = cot y = , 9. 2 6 + 2 2
20 10 , 3 , 2
10. a)12 b) 16 + 12 3 , 11. 4

ALIŞTIRMA (Sayfa 361)


100 ^ 3 + 3h
1. 32, 2. 8, 3. 10, 4. 9, 5. 12 3 , 6. 3 , 7. 9, 8. 2 3 , 9. 24, 10. 117
2 2
11. 1. 0, 75 km ile 1 km , 2.%45 ile %55 arası, 3.Fidan sayısı: 100 001-200 000, Maliyet:400 004-800 000

VERİ
ALIŞTIRMA (Sayfa 382)
1. 4-5-2-8-10-6, 2. X Eray = 13, X Emre = 12, X Mert = 11 .olup aritmetik ortalaması en büyük olan Eray tercih
edilmelidir.
7 40
3. a) 15 ve 18 b) 16 c) 7 ç) 2 d) 16 e) 7 , 4. 18, 5. 77, 6. 10, 7. 40

49 65
8. z < x < y , 9. 3 = 16, 3 , 10. 2 , 11. S A > S C > S B
ALIŞTIRMA (Sayfa 397)
1. 3. Pozitif bir ilişki vardır. 5.

2. a) 136, b) 12,5 4. 22,875


34
6. 50, 7. 42, 8. 2005-2006 en hızlı artış, 2012-2013 en yavaş artış, 9. a) 18 b) 3
10. a) 30 dk b) 40 dk c) 16.10-16.20, 11. a) 3,3 b) Kasım c) 500-550 d) 2 812 500

Matematik 9 | 409
CEVAP ANAHTARI
MANTIK
ÖLÇME VE DEĞERLENDİRME
1-B 2-D 3-A 4-C 5-E 6-D 7-A 8-D 9-B 10-C 11-D 12-E 13-A 14-C 15-B 16-B 17-E 18-A

KÜMELER
ÖLÇME VE DEĞERLENDİRME -1
1-C 2-A 3-E 4-C 5-E 6-E 7-E 8-B 9-A 10-B 11-D 12-C 13-D 14-B 15-A 16-C 17-B
ÖLÇME VE DEĞERLENDİRME -2
1-D 2-C 3-E 4-A 5-A 6-B 7-C 8-B 9-D 10-C 11-E 12-A 13-A 14-D 15-E 16-B 17-A 18-C 19-E 20-D 21-B

DENKLEM VE EŞİTSİZLİKLER
ÖLÇME VE DEĞERLENDİRME-1
1-D 2-A 3-C 4-A 5-E 6-E 7-B 8-D 9-C 10-B 11-E 12-B 13-A 14-B 15-D 16-C 17-E 18-A 19-D 20-C
ÖLÇME VE DEĞERLENDİRME-2
1-D 2-B 3-E 4-B 5-A 6-A 7-D 8-E 9-B 10-D 11-C 12-C 13-E 14-B 15-D 16-E 17-A 18-E 19-C 20-A
ÖLÇME VE DEĞERLENDİRME-3
1-D 2-D 3-A 4-C 5-B 6-E 7-D 8-A 9-B 10-E11-E 12-A 13-B 14-C 15-C 16-D 17-A 18-E 19-B 20-C

BÖLÜNEBİLME
ÖLÇME VE DEĞERLENDİRME
1-E 2-C 3-E 4-D 5-B 6-C 7-E 8-A 9-E 10-D 11-B 12- B 13-A 14-C 15-D 16-D 17-A 18-C 19-D
20-B 21-C 22-E 23-B 24-A 25-B 26 C-27-A 28-B 29-E

ÜÇGENLER
ÖLÇME VE DEĞERLENDİRME -1
1-E 2-D 3-C 4-E 5-B 6-A 7-B 8-B 9-D 10-E 11-A 12-A 13-A 14-D 15-C 16-C 17-B 18-C

ÖLÇME VE DEĞERLENDİRME -2
1-A 2-B 3-E 4-B 5-C 6-E 7-D 8-A 9-D 10-B 11-A 12-C 13-D 14-C

VERİ
ÖLÇME VE DEĞERLENDİRME
1-C 2-B 3-E 4-A 5-D 6-A 7-B 8-C 9-E 10-D 11-A 12-B 13-C 14-B 15-D 16-E 17-E 18-C 19-C 20-D 21-B 22-E 23-A 24-D

410 | Matematik 9
SÖZLÜK
A
açı : Başlangıç noktaları aynı olan iki ışının birleşimi.
açık önerme : İçerisinde değişkenler bulunan ve bu değişkenlerin alacağı değere göre doğruluğu veya
yanlışlığı kesinleşen önerme.
açıortay : Bir açıyı, ölçüleri birbirine eşit iki açıya ayıran ışın.
ağırlık merkezi (üçgen) : Bir üçgenin kenarortaylarının kesiştiği nokta.
ağırlıklı ortalama : Bir veri grubundaki her bir verinin ağırlık değeri ile çarpımlarından elde edilen sayılar
toplamının, tüm ağırlık çarpanları toplamına bölümü ile elde edilen değer.
aksiyom : İspata gerek duyulmaksızın doğruluğu kabul edilen önerme.
alan : Bir yüzeyin bulunduğu düzlemde kapladığı yer.
alt çeyrek : Küçükten büyüğe sıralanıp iki gruba ayrılan verilerin alt grubunun medyan değeri.
alt küme : A ve B iki küme olmak üzere A nın her elemanı B nin de elemanı oluyorsa A, B nin alt
kümesidir.
altın oran : Matematik ve sanatta bir bütünün parçaları arasında gözlemlenen, geometrik ve
sayısal bir oran bağıntısı.
analitik düzlem : Üzerine dik kesişen iki doğru yerleştirilmiş düzlem.
apsis : Koordinat düzleminde bir noktayı gösteren sıralı ikilinin birinci bileşeni.
aritmetik ortalama : Veri grubunda bulunan verilerin toplamının veri sayısına bölünmesi ile elde edilen
değer.
ayrık küme : Ortak elemanı olmayan kümeler.

B
bağıntı : Kartezyen çarpımın her bir alt kümesi
bağıntının grafiği : Bir bağıntının elemanlarının venn şeması yöntemiyle ya da analitik düzlemde
gösterilmesi.
benzer üçgenler : İki üçgen arasında kurulan bire bir eşlemede, karşılıklı açıları eş veya karşılıklı
kenarlarının uzunlukları orantılı olan üçgenler.
bileşen : Bir noktayı belirten sıralı ikililerden herhangi biri.
boş küme : Elemanı olmayan küme.
bölen : Bir bölme işleminde bölünen sayının kaç eşit parçaya ayrıldığını gösteren sayı.
bölüm : Bölme işleminde bölünen içinde bölenin kaç defa olduğunu gösteren sayı.
bölünen : Bir bölme işleminde eşit bölümlere ayrılması gereken miktar veya sayı.
Ç
çelişki : Doğruluk değeri daima sıfır (0) olan önerme.
çevrel çember : Bir çokgenin tüm köşelerinden geçen çember..
çeyrekler açıklığı : Üst çeyrek ile alt çeyrek arasındaki fark.
çizgi grafiği : Verilerin analitik düzleme aktarılıp oluşan noktaların çizgilerle birleştirilmesi sonucu
oluşan grafik
çözüm kümesi : Denklem veya eşitsizlikleri sağlayan değerlerden oluşan küme.
D-E-F
daire grafiği : Veri grubunun bütün içerisindeki oranını belirtmek için dairenin dilimlere ayrılarak
gösterildiği grafik türü.
denk kümeler : Eleman sayıları aynı olan kümeler.
denk önermeler : Doğruluk değerleri aynı olan önermeler.
denklem : İçinde bilinmeyen bulunan, bilinmeyenin bazı değerleri için sağlanan eşitlik .
dış açıortay : Bir üçgenin bir dış açısını iki eş açıya ayıran ışın.
dış teğet çemberinin Üçgenin iki dış açıortayının kesiştiği nokta
merkezi
diklik merkezi : Bir üçgende yüksekliklerin kesiştiği nokta.
doğruluk kümesi : Açık önermeyi sağlayan değerler kümesi
en büyük ortak bölen : İki veya daha çok sayının ortak bölenlerinin en büyüğü.
(EBOB)
en küçük ortak kat (EKOK) : İki veya daha fazla doğal sayının ortak katlarının en küçüğü.
eşit kümeler : Aynı elemanlardan oluşan kümeler.
eşlik : İki üçgenin karşılıklı kenarlarının uzunluklarının ve açılarının ölçülerinin birbirine eşit
olması.
evrensel küme : Bütün kümeleri içine alan, boş kümeden farklı, en geniş küme.
frekans : Veri grubundaki sayıların tekrar sayıları.

Matematik 9 | 411
G-H
gerçek sayılar : Rasyonel sayılar ile irrasyonel sayılar kümesini kapsayan sayı kümesi.
grafik : Ölçme sonucunda elde edilen verilerin nokta, çizgi, sütun vb. farklı şekillerle ifade
edilmesi.
hipotenüs : Bir dik üçgende, dik açının karşısında bulunan kenar.
hipotez : Varsayım.
histogram : Bir kenarının aralıklara diğer kenarının veri sayılarına eşit birbirine bitişik dikdörtgenlerden
oluşan bir grafik gösterimi.
hüküm : Yargı.
İ
iç açı : Bir çokgenin ardışık iki kenarının oluşturduğu ve çokgenin içinde bulunan açı.
iç açıortay : Bir çokgenin bir iç açısını iki eş açıya ayıran ışın.
: Herhangi iki doğruyu üçüncü bir doğru kestiğinde, bu doğruların arasında ve kesenin her
iç ters açı iki tarafında komşu olmayan açılar.
ikizkenar üçgen : İki kenarının uzunluğu birbirine eşit olan üçgen.
irrasyonel sayı : İki tam sayının birbirine bölümü şeklinde yazılamayan sayı.
: Matematiksel kural, sonuç veya tanımları kullanarak bir yargının doğru veya yanlış
ispat olduğunun gösterilmesi.
K-L
kapsama : Bir kümenin, başka bir kümenin elemanlarının hepsini içermesi.
kartezyen çarpım : Boş kümeden farklı olan A ve B kümeleri için; birinci bileşeni A kümesinden, ikinci bileşeni
B kümesinden alınarak oluşturulan tüm ikililerin kümesi.
kenarortay : Bir üçgende her köşeden karşı kenarın ortasına çizilen doğru parçası.
kesikli veri : Sonlu veya sayılabilir belli bir aralıkta her değeri alamayan veriler.
kesişim kümesi : A ve B kümelerinin ortak elemanlarının alınması ile oluşturulan küme.
komşu açılar : İç bölgeleri ayrık ve birer ışını ortak olan açılar.
Kosinüs teoremi : Üçgenin iki kenar uzunluğu ve bu kenarların oluşturduğu açının ölçüsü ile üçüncü
kenarının uzunluğu arasındaki ilişkiyi ifade eden teorem.
kotanjant : Karşı kenar uzunluğunun komşu kenar uzunluğuna oranı.
kutu grafiği : Veri dağılımındaki en büyük değeri, en küçük değeri, alt çeyrek, üst çeyrek ve ortanca
değerlerini tek bir grafikte belirten istatistiksel grafik türü.
küme : İyi tanımlanmış, birbirinden farklı nesneler topluluğu.
liste yöntemi : Kümeyi oluşturan bütün elemanların küme parantezinin içerisinde, aralarına virgül
konularak gösterilmesi.
M
: Veriler sıralandığında veri sayısı tek ise ortadaki terim, çift ise ortadaki iki terimin
medyan aritmetik ortalaması.
mod : Veri grubunda en çok tekrar eden değer.
mutlak değer : Bir gerçek sayının sayı doğrusu üzerindeki görüntüsünün sıfıra olan uzaklığı.
merkezi eğilim ölçüleri : Tepe değer, ortanca ve aritmetik ortalam değerleri
: Terimlerin yakınlık veya uzaklığını belirten standart sapma, açıklık ve çeyrekler açıklığı
merkezi yayılım ölçüleri değerleri
O-Ö
orantı : İki veya daha çok oranın eşitliği.
ordinat : Koordinat düzleminde bir noktayı gösteren sıralı ikilinin ikinci bileşeni.
orijin : Koordinat eksenlerinin kesim noktası.
orta dikme : Bir doğru parçasına orta noktasında dik olan doğru.
ortak özellik yöntemi : Bir kümeyi elemanlarının taşıdığı koşulları veya özellikleri belirterek ifade eden gösterim
biçimi.
Öklid algoritması : İki tam sayının en büyük ortak bölenini bulmak için yapılan ardışık bölme işlemi.
önerme : Doğru veya yanlış kesin hüküm bildiren ifade.

412 | Matematik 9
P
Pisagor teoremi : Bir dik üçgende, hipotenüs ile dik kenarların uzunlukları arasındaki ilişkiyi ifade eden
teorem.
pozitif yönlü değişim : Değişkenlerden biri artarken diğerinin de arttığı değişim şekli.

S-T
serpme grafiği : İki farklı değişken arasındaki ilişkinin yönünü ve kuvvetini gözlemlemek için verilerin
sıralı ikililer olarak grafik üzerinde gösterilidiği grafik türü.
sonlu küme : Eleman sayısı bir doğal sayı ile ifade edilebilen küme.
standart sapma : Bir sayı dizisindeki elemanların aritmetik ortalamaya yakın olup olmadığı hakkında bilgi
veren merkezi yayılım ölçüsü.
teorem : Kanıtlanabilen bilimsel önerme.
ters orantı : Değişkenlerden biri artarken diğeri azalan orantı.
Thales teoremi : Kesişen iki doğrunun, paralel iki doğru tarafından kesilmesiyle oluşan üçgenlerin
kenarlarının orantılı olduğunu gösteren teorem.
totoloji : Doğruluk değeri daima 1 olan bileşik önerme.
trigonometrik oranlar : Bir dik üçgenin kenar uzunlukları arasındaki oranlar.
tümdengelim : Genelden özele çıkarımlar türeterek bilgi üretme yöntemi.
tümevarım : Özel olandan genel olana varmak için uygulanan yöntem.
tümler açılar : Ölçüleri toplam 90° olan açılar.
tümleyen küme : Verilen kümede bulunmayan evrensel kümede bulunan elemanların kümesi.

Ü
üçgen : Doğrusal olmayan üç farklı noktanın doğru parçaları ile birleştirilmesi sonucu oluşan
geometrik şekil.
üçgen eşitsizliği : Bir üçgende herhangi bir kenarın uzunluğunun, diğer iki kenarın uzunluklarının
toplamından küçük, farkının mutlak değerinden büyük olması.
üst çeyrek : Küçükten büyüğe sıralanıp iki gruba ayrılan verilerin üst grubunun medyan değeri.
V-Y
veri : Belli konularda ölçüm, sayım, deney, gözlem vb. yoluyla elde edilen toplanmış ve
çözümlenmiş bilgiler.
yöndeş açı : Paralel iki doğru, bir kesenle kesildiğinde kesenin aynı tarafında kalan aynı yönlü açılar.
yükseklik : Üçgenin bir köşesinden karşı kenara çizilen dik doğru parçası.

Matematik 9 | 413
KAYNAKÇA
• Adams, Robert A., A Course Calculus, Addision-Wesley Publishers Limited, New York, 1995.
• Ağlı, E., Temel Matematik I, Ankara, 2007.
• Ağlı, E., Uygulamalı Genel Matematik I, Ankara, 2007.
• Balcı, M., Genel Matematik (Cilt I), Balcı Yayınları, Ankara, 2003.
• Çelik, B., Temel Matematik, Dora Basım Yayın, 5. Baskı, 2010.
• Çevik, A. S., Bozacı, E., Genel Matematik 1, Nobel Yayınları, Ankara, 2009.
• Dernek, A., Analiz 1, Nobel Yayınları, 2. Baskı, 2009.
• Dönmez, A., Matematiğin Öyküsü ve Serüveni (Cilt I), Toplumsal Dönüşüm Yayınları, İstanbul, 2002.
• Dönmez, A., Temel Analiz, 1. Baskı, Arıkan Yayınları, 2005.
• Erdoğan, A. Matematik, Nobel Yayınları, Ankara, 2009.
• Ertuğrul, İ., Temel Matematik, Ekin Kitabevi, Ankara,2006.
• Göğüş, M., Koçak, Ş., Tayfur, C., Üreyen, M., Matematik I (Diferansiyel Hesap),Eskişehir, 1985.
• Hacısalihoğlu, H. H., (vd.), Matematik Terimleri Sözlüğü, Türk Dil Kurumu Yayınları, Ankara, 2000.
• Hacısalihoğlu, H. H., Balcı, M., Gökdal, F., Temel ve Genel Matematik, Ankara, 1988.
• Haggarty, R., Funtamentals of Mathmatical Analysis, Addision-Wesley, 1993.
• Kaplan, W., Advanced Calculus, Addision-Wesley Publishing Company, London, 1967.
• Nelsen, R.B., The Mathematical Association of America, Proofs Without Words Washington, 1993.
• Öztürk, F., Olasılık ve İstatistiğe Giriş 1, Gazi Kitabevi, Ankara, 2011.
• Öztürk, F., Olasılık ve İstatistiğe Giriş 2, Gazi Kitabevi, Ankara, 2011.
• Sağel, M.K., Aktaş, M., (Eds). Genel Matematik 1, Pegem A Yayıncılık, Ankara, 2007.
• Sertöz, S., Matematiğin Aydınlık Dünyası, Tübitak, Ankara, 2011.
• Silverman, R. A., Calculus With Analytic Geometry, Prentice-Hall, New Jersey, 1985.
• Stewart, I., Matematiğin Kısa Tarihi, Alfa Basım Yayın Dağıtım San. ve Tic. Ltd. Şti. İstanbul, 2016.
• Struik, D. J., Kısa Matematik Tarihi, Doruk Yayınları, İstanbul, 2011.
• Türk Dil Kurumu, Türkçe Sözlük, Türk Dil Kurumu Yayınları, Ankara, 2012.
• Türk Dil Kurumu, Yazım Kılavuzu, Türk Dil Kurumu Yayınları, Ankara, 2012.
• Türkiye İstatistik Kurumu, Basın Bülteni, 03 Ocak 2017, 22 Mart 2016, 19 Ocak 2016

414 | Matematik 9
GÖRSEL KAYNAKÇA
Görsel ve grafik tasarım uzmanı tarafından düzenlenen görseller: Sayfa 11, 106, 156, 160, 199, 264, 311, 312, 315, 325,
327, 336 (Özgün çizim) , 337, 340, 343 (Özgün çizim), 360,362 (Özgün çizim), 374.
Ağ adreslerinden alınan görseller: Sayfa 62 (http://www.tvf.org.tr/img/uploads/haber_resimleri/2016/06
haber_1465132409_3.jpg (ET: 11 04 2017/16.52)), Sayfa 236 (http://okuma-yazma.meb.gov.tr/www/icerik_goruntule.
php?KNO=118 (ET: 06.06.2017/17.27))
Dreamstime (Telif hakkı ödenerek alınan görseller) Sayfa 12/ ID No: 83180382, Sayfa 13/ID No: 18040989,
Sayfa 14/ID No: 7838054, Sayfa 47/ID No: 17519957, Sayfa 48/ID No: 55819483, Sayfa 49/ID No: 63446559,
Sayfa 59/ID No: 9271642, Sayfa 67/ID No: 33041737, Sayfa 74/ID No: 23598721, 57910693, Sayfa 93/ID No: 33040687,
Sayfa 98/ID No: 45083462, 11864189, 4869251, Sayfa 104/ID No: 19446522, Sayfa 106/ID No: 9380689,
Sayfa 155/ID No: 29681182, Sayfa 156/ID No: 11221791, Sayfa 156/ID No: 21158727, Sayfa 157/ID No: 13698635,
Sayfa 159/ID No: 45395728, Sayfa 160/ID No: 19726310, Sayfa 161/ID No: 11504117,
Sayfa 163/ID No: 200379871, 51042631, Sayfa 166/ID No: 32590012, Sayfa 167/ID No: 25776,
Sayfa 171/ID No: 55569044, Sayfa 174/ID No: 9429115, Sayfa 176/ID No: 225485,
Sayfa 179/ID No: 1205619, 88962620, Sayfa 180/ID No: 19225437, 80741634, Sayfa 186/ID No: 257683,
Sayfa 187/ID No: 3162851, Sayfa 194/ID No: 2608845, 78509162, Sayfa 202/ID No: 21004786,
Sayfa 220/ID No: 21004786, Sayfa 220/ID No: 20874942, Sayfa 221/ID No: 513847682, Sayfa 222/ID No: 5025565,
Sayfa 326/ID No: 7802757, Sayfa 335/ID No: 83808400, Sayfa 345/ID No: 28901340, Sayfa 369/ID No: 20103445,
Sayfa 370/ID No: 63447195, Sayfa 372/ID No: 34730963, Sayfa 377/ID No: 75443213, Sayfa 378/ID No: 69052105,
Sayfa 379/ID No: 57623800, Sayfa 380/ID No: 51778437, 53181223, Sayfa 385/ID No: 18033496,
Sayfa 389 /ID No: 5296688, Sayfa 391 /ID No: 39331377,
Kitap genelindeki diğer grafik, tablo ve şekiller görsel ve grafik tasarım uzmanı tarafından çizilmiştir.

Matematik 9 | 415
416 | Matematik 9

You might also like